Sixty Minutes Australia Story on MH370 is a Sensation

On Sunday night, Sixty Minutes Australia aired an episode on the disappearance of MH370. Included was a panel of five experts, consisting of Canadian crash investigator Larry Vance, US air safety expert John Cox, British airline captain Simon Hardy, former ATSB commissioner Martin Dolan, and Australian oceanographer Charitha Pattiaratchi. Before airing, the episode was heavily promoted with teasers claiming there would be “groundbreaking revelations”, the “passengers’ final seconds”, and a “forensic twist”.

Unfortunately, after watching the episode twice, I found nothing in the way of new evidence or insights. What I did see were some of the experts confusing speculation with facts, and cherry-picking evidence to support their pet theories while carefully omitting contradicting evidence.

Despite the obvious shortcomings of the episode, the mainstream media is covering it extensively with headlines like “Experts Have Finally Solved the Mystery Behind the MH370 Disappearance”. The sensational nature of the story makes it attention-grabbing, and hiding behind the Sixty Minutes brand name, there is little or no attempt to fact-check.

A large part of the episode was devoted to Larry Vance’s theory that the captain hijacked the plane and flew it to the SIO to hide it. That plan included a successful ditching with the engines running and the flaps extended, leading to the sinking of the aircraft with the fuselage intact. (This “new” theory was already presented by Mr Vance in a Sixty Minutes story that aired in July 2016.) This ditching would produce only a small amount of floating debris. Mr Vance also refers to the damage to the trailing edges of the right flaperon and right outboard flap and the lack of damage to the leading edges of those parts. He believes this pattern of damage conclusively shows that there were hydrodynamic forces as those parts were dragged across the water surface during the ditching.

What was omitted is that we do have pieces of evidence that refute some of Mr Vance’s claims, and should at least cast a shadow of doubt on many of his conclusions. Briefly,

  • Crash investigators at the ATSB have examined the right flaperon and the right outboard flap that were recovered and determined that some damage on both parts was caused by mutual contact, and the location of that contact could only occur with the flaps retracted.
  • Recovered parts from the passenger cabin show deformation from a high energy impact and not a successful ditching.
  • The final log-on of the SATCOM at 00:19 suggests there was a disruption of electrical power to the SATCOM, which is consistent with fuel exhaustion of both engines, and not a ditching with the engines running. It’s not clear in his scenario what caused the log-on.
  • The damage to the trailing edges of the flaperon and flap could have been caused by aerodynamic forces occurring during a high speed descent. The lack of damage to the leading edges can be explained by separation of these parts from the aircraft prior to impact with the ocean.

Strangely, in the episode, Martin Dolan does not challenge Mr Vance’s scenario with the contradictory evidence published by the ATSB. Perhaps those challenges were made, and they were not included in the episode. Or, perhaps Mr Dolan is not sufficiently familiar with the technical analyses of the ATSB where he could confidently refute some of Mr Vance’s claims.

The theories of Simon Hardy also were featured in the episode. Mr Hardy, like Mr Vance, believes that the captain hijacked the plane, but he believes the plane glided a long distance after fuel exhaustion rather than a ditching with the engines running. The possibility of a glide suggests a crash location at a distance from the 7th arc that is well beyond what was searched. His claim that military data shows that MH370 was flown along the borders of Malaysia and Thailand is presented as shocking new evidence, when in fact the turnback flight path across the Malay peninsula has been known to the public within weeks of the disappearance, and the implications have been widely discussed. (The precise flight path flown as captured by civilian radar has only been recently published, and was the subject of the preceding blog post.) Mr Hardy also demonstrated on a flight simulator that it is possible for a skilled pilot to recover from a high speed descent that matches the satellite data, which was not in dispute, although he does downplay the importance of gently working the controls and applying speedbrakes to help arrest the descent and prevent overloading of the lift and control surfaces. (Why a pilot would first enter into a steep descent, then recover and maximize the gliding distance, was not explained.) At another point, he claims to know exactly where MH370 crashed, although he neglects to state that all drift models suggest a crash point much further north.

In the episode, Mr Hardy once again promotes his theory that MH370’s flight path as it flew south of Penang Island shows indication that the captain turned to the right, lowering the right wing, and allowing the captain to have a final, sentimental view of Penang before leaving Malaysia forever. In fact, using the recent radar data, we can deduce that at the point of closest approach to Penang, MH370’s wings were either level or only slightly banked. After passing Penang, there was a turn to the right followed by a turn to the left, but to conclude that this was an emotional farewell is pure speculation, and weakens his theories.

Although I disagree with some of Mr Vance’s and Mr Hardy’s conclusions, I am in general agreement that the disappearance was likely an intentional diversion and not likely the result of a series of mechanical failures. After reviewing many accident scenarios proposed by some very bright minds, I have yet to see an accident scenario that did not require a sequence of very unlikely events. On the other hand, a deliberate diversion requires no unlikely events, even if we might not understand the motivation for many of the intentional actions.

If the diversion was intentional, the captain becomes the likely suspect, as he had the skill and the best opportunity to divert the aircraft. In addition, as discussed and analyzed in a previous blog post, the incriminating evidence found on his home computer of a simulated flight to the Southern Indian Ocean would be an extraordinary coincidence if the captain was not somehow involved in the disappearance.  There is certainly not enough evidence for a legal determination of guilt. However, I believe there is sufficient evidence to make him the prime suspect.

Perhaps the Sixty Minutes episode did have value in that it did not shy away from presenting what many believe is the most likely scenario, even if some of the conclusions from the experts were either unfounded or premature.

The episode comes at a time when Ocean Infinity is in the final weeks of the seabed search for MH370. If not found, and if there is a willingness to conduct additional searches next year, a decision has to be made whether to prioritize areas along the 7th arc that are further north, or to revisit previous latitudes but search further away from the arc, or to revisit areas that might have been insufficiently scanned previously.  A strong case for the possibility of a glide after fuel exhaustion would support searching wider (+/- 100 NM) from the 7th arc.  Unfortunately, the size of the search becomes unreasonably large unless there is rationale to support a narrow range of latitudes along the 7th arc.

On a final note, I have been asked whether the defeat of the incumbent party in the recent Malaysian elections could lead to a more thorough investigation of the events surrounding MH370. Although it is possible, the winning candidate and former Prime Minister, Mahathir Mohamad, has previously supported the unlikely theory that MH370 was diverted remotely using secret Boeing technology embedded in the flight controls. While this might indicate his willingness to challenge the official narrative, it also might demonstrate his willingness to use the MH370 for political gain rather than seek the truth. Meanwhile, his heir-apparent, former Deputy Prime Minister Anwar Ibrahim, had family and political ties to MH370’s captain, and those ties might taint future investigations. On a positive note, it is possible that any whistleblowers that were previously reluctant to come forward might now feel less threatened.

820 Responses to “Sixty Minutes Australia Story on MH370 is a Sensation”

  1. Richard Godfrey says:

    @Gerald, you asked about my tracking of SC in the previous post.

    I download data from a subscription web site from a company called Big Ocean Data, which tracks all ships, including Seabed Constructor, via satellite data they transmit to Inmarsat, giving their latitude, longitude, speed, course, etc. on average 360 times per day.

    Here is an example Excel file for the current tour:

    https://www.dropbox.com/s/hc614lw3alzahj4/SC%20Track.xlsx?dl=0

    I then create a .kmz file to import the track into Google Earth.

    I then look for the points where SC is stationary at a location for a while and work out whether an AUV launch, AUV check or AUV pick up is taking place. To remain in the same position, SC is usually at a speed below 1 knot, depending on the weather. If SC is stationary for longer than around 6 hours, then a ROV deployment could be underway. The ROV winch runs at a speed of 0.5 m/s, so a depth of 4,000m would take 2h 13m just to lower an ROV to the sea floor and another 2h 13m back to the surface.

    An AUV launch is usually followed by a period of 2 to 3 hours, depending on operating depth, where SC follows the AUV until it reaches the assigned operating depth. The normal operating speed of an AUV is around 3.6 knots, so it is easy to see, when SC keeps to a speed around 3.6 knots for a period of 2 to 3 hours.

    The AUV battery lasts around 60 hours, so an AUV is usually picked up after 55 hours. This can be shorter depending on the search pattern. For a search width of 22 NM, you can expect that the AUV goes out and back for 44 NM and repeats the cycle for 88 NM, 132 NM or 176 NM. After 176 NM at 3.6 knots, you can expect the AUV to be picked up after 49 hours at operating depth, plus 2 to 3 hours down and another 2 to 3 hours up, totalling up to 55 hours. It then usually takes at least 12 hours before the AUV is ready for the next launch. Depending on the sea floor terrain, the AUV speed can vary from 3.2 to 3.8 knots, so the deployment cycle shortens or lengthens.

    Ocean Infinity normally leave a few hours between successive AUV launches and may delay further launches if the weather is bad. Ocean Infinity also usually check on an AUV once or twice during the nominal 55 hour cycle. It is interesting to see where SC stops to check and then try to work out which AUV is at that point after say 24 or 36 hours. From this information, you can then work out the search pattern currently adopted.

    Recently we have seen the search pattern change a few times, depending on the sea floor terrain. Sometimes AUVs are launched at the 7th Arc, sometimes at the 22 NM outer bound S.E. or inner bound N.W.

    My apologies for a long answer to a simple question.

  2. Jerry M says:

    I have followed this mystery from the beginning because it is so compelling and hopefully will be solved one day. However, I only follow it via this blog anymore because it is so well moderated and the technical discussions interest me even if I don’t understand it many times. In this day and age of media sensationalizing everything, your measured, careful and fact based wording is remarkable and appreciated (and yes, I know you are not media). Thank you for the good work.

  3. DennisW says:

    @Ge Rijn

    Some of you are starting to look like frustrated bad losers.

    Being a bad loser is a normal human personality trait. Kahneman and Tversky were able to show that the joy of winning is not symmetrical with the pain of losing (which is greater). I have applied this principle to my investment decisions and personal relationships with great success.

    As far as MH370 is concerned I have no emotional involvement with the disappearance itself or the search strategy. No one has asked me for any advice on where to search. If I was asked, I would advise not to search at all as I have stated here many times. Of course, that is based on hard and cold decision theory as applied by a coin-operated person like me.

  4. Gysbreght says:

    Victor Iannello said: “Strangely, in the episode, Martin Dolan does not challenge Mr Vance’s scenario with the contradictory evidence published by the ATSB.”

    Martin Doland was ‘sidelined’ by the program producers. Found this analysis interesting (Suzie Crowe on the ‘other’ blog):

    http://flightlevel42.co.za/blogs.html#60_Minutes:_How_to_create_your_own_facts_before_the_real_ones_are_at_hand

  5. TBill says:

    @Victor
    The 60 Minutes Aussie episode probably stands the test of time, not for the questionable opinionated details, but for finally giving the public the message that MH370 was an apparent intentional diversion by the pilot. It would have been nice to embrace this apparent accident cause before a suicidal Germanwings pilot took down another load of innocent passengers, but we as a society seem to demand 100.0% proof before changes are made.

    Yes if we ever get more information, we can reassess if the preliminary likely cause finding holds up. But that should not be an excuse for letting more cases happen.

    Safety does not require waiting for an accident to happen. In this case, a HAZOP review before the flight could have shown the vulnerability to rouge pilot. Now we can do a HAZOP review after the MH370 accident, we can clearly see the evidence (not final proof) but we can clearly see many pilot actions were taken consistent with a rouge pilot hijacking the aircraft. Why the heck do we need more proof at this point to take corrective actions?

    Unfortunately there is a reason we need more proof, and I cannot quite put my finger on it, but basically the aviation industry gets a lot of support from the public and other professionals. If this was a petrochemical plant that exploded and unintentionally killed 239 people, that plant manager would be in jail and the company would probably be put out of business. Here we have an apparent *intentional* act, and the public is ambivalent.

    I think it probably has to do with human risk perception, basically we are outraged about risks we do not agree with, but happy to accept certain risks. At the moment the airline industry enjoys support, in part because of comparison with with the awful death toll of auto accidents. But that is probably not the correct way to look at it.

    It will probably not be possible to refute the 60 Min Aussie story with an alternate cause theory, because there is no apparent alternate cause explanation that fits the facts very well. Yes we can argue some facts given were inaccurate.

  6. Victor Iannello says:

    @Jerry M: Thank you for your comments.

  7. Victor Iannello says:

    @TBill: Even if we were 100% certain that the captain diverted and hid the plane, what can be done to prevent a future incident?

  8. Ge Rijn says:

    @DennisW

    Thanks. I’ll look those names up. Sounds interesting.
    Sometimes I regret I did not take your position. But this is impossible for me to do I know.

  9. formula says:

    I fully echo Jerry M’s remarks second above. I value this blog for its incisive commentary and quality information. My thanks @ Victor and also to the many regular contributors.

  10. TBill says:

    @Victor
    After Germanwings of course, more global airlines went to two-in-cockpit rule to help prevent the lock-out scenario. That is an example of a “procedure” solution.

    Many feel the radar transponder should not be designed allow turn off during flight, or as I would say, at least not allow a secretive turn off of the transponder. That would be an example of a “design” change.

    Presumably there could be a long list of proposed procedure or design changes. Controversial of course since industry is not receptive. Some things are already in progress like 24-hr voice recorder, and tracking technology.

  11. Victor Iannello says:

    @TBill: As you know, there are safety ramifications if the crew cannot disable the transponder. I don’t think the aviation industry would allow it.

  12. Don Thompson says:

    @TBill,

    Would it be possible that you retrain your device’s spelling auto-correct to prioritise ‘rogue’ over ‘rouge’?

  13. TBill says:

    @Don
    …sorry I try to fix in the future

    @Victor
    I don’t want to give a long list right now, but the way I look at it, we probably cannot make it impossible to intentionally do the bad things: dive into the ground, depressurize the aircraft, etc.

    But we do not have to ignore the possibility either. Remove some of the window of opportunity and temptation. Next time, make it be that we knew what was being done intentionally, not a temptation to allow secretly carrying out such a plan. Something in the human spirit takes satisfaction in “hacking” the system to show the obvious weaknesses we let happen due to laziness, so at least make it be more difficult.

  14. Sabine Lechtenfeld says:

    @TBill, even if the transponder couldn’t be turned off – how could that prevent pilots from going rogue (I turned autocorrection off btw, so all mistakes are mine alone)? It would only prevent pilots from hiding the plane. And I don’t think that this specific scenario deserves a lot of preventive measures.
    The case of Andreas Lubitz bothers me personally much more than ZS’s case. There were tons of red flags that Lubitz was losing his mind. He had visited a string of doctors who had attested that he was unfit for flying for various reasons. But no one intervened, since the confidentiality between doctor and patient is taken very seriously in Germany. Lubitz tore the reports up and had maneuvered himself into a corner. Sooner or later this would’ve been noticed, and he would’ve lost his job as a pilot for good. And being a pilot was all he ever wanted from life since he was a kid. He had no Plan B for his life.
    The almost unsolvable conundrum is of course this: if doctors start to report pilots who are their patients, the pilots won’t go to a doctor anymore. Or they will go as privat patients and never tell about their job. We have probably to accept that the rogue-pilot problem cannot be totally eliminated. The best preventive measures are probably thorough and regular psychological evaluations conducted by the airlines, which wasn’t done by Germanwings. But even that isn’t foolproof because it’s no problem at all to hide psychological or psychiatric problems. Fortunately pilot suicides are very rare.

  15. Sabine Lechtenfeld says:

    @TBill, I agree that the aviation industry shouldn’t just ignore the problem, even if it is very rare. Reckognizing psychological problems or conditions amongst their employees is one aspect. And technically it should be as hard as possible to take over an aircraft. But it’s tricky, Some rules which have been put into place – like the rule that there should be at least two persons in the cockpit at all times – have serious drawbacks. Many airlines have abolished this rule already because the constant necessity to open the cockpit door makes it easier for outside hijackers to take over the plane. And there are more potential hijackers than suicidal pilots.

  16. airlandseaman says:

    Victor:

    I agree with your comments in the new blog. I would add the following:

    1. There were fewer factual errors in this 60 minutes show compared to the one in 2016. That said, the first show was so full of factual errors, it was a low bar to overcome.

    2. The claim that Malaysia first discovered the Inmarsat data (on days 1) was totally false, and nearly everyone else on earth knows that. And everyone on the set surely knew that too. So why was it allowed to end up in the final cut?

    3. They repeated the false claims that the handshakes were data transmitted from the engines. Geeezzz! When will they ever get that right!

    4. To me, the greatest flaw in the whole show was the decision to let Larry Vance on stage. At the very least, they should have invited someone else on to present the alternative end of flight scenario. Larry’s analysis of the flaperon was 100% wrong from the start. I had a lot of correspondence with Ross C right after the first 60 minutes and explained why Larry’s theory was wrong. So 60 minutes was well aware of the holes in Larry’s theory, but chose to ignore them (again). I also had a lot of correspondence with John Cox, who is advising Larry on his new book soon to come out. He also failed to acknowledge the alternative explanation for the flapern damage. It went mostly unchallenged, even though Dolan knew that ATSB found very strong evidence the flaps were retracted, the 0019 logon was due to MEFE and the small debris from inside the plane indicated a high energy crash, not a “water landing”. And Dolan said as much, but did not challenge Vance as strongly as he should have. (Note that Simaon Hardy said that the plane went supersonic in his simulated “uncontrolled EOF scenario”, consistent with flap and flaperon separation in flight.)

    Fortunately, this 60 minutes episode will have little if any influence on future search decisions. OI will probably continue the search next year, if 370 is not found in the remaining days this season. Hopefully, by next season, there will be new evidence to consider. Maybe the election will result in some of the people coming forward with new details.

  17. Mick Gilbert says:

    @Sabine Lechtenfeld

    Sabine, thank you for the follow up on the Indonesian airspace issue. It should be noted that the incident involving USAF Special Operations Command Dornier Do-328 11-3075 was not an intercept. The aircraft was detected on radar, quite probably SSR via its transponder, asked for its clearance details and then asked to land at Banda Aceh.

  18. Mick Gilbert says:

    @Victor Iannello

    I have been in contact with someone involved in the production of the 60 Minutes episode. The ‘situation room’ round-table discussion was filmed on Friday, 4 May. There was well over six hours of technical discussions and interviews that were then cut substantially. Apparently the producers had their view, mostly aligned with Larry Vance’s theory. That alignment may have had something to do with Vance’s previous involvement with 60 Minutes. There was certainly no general consensus among the panel on Vance’s theory but the dissenting discussion was left on the cutting room floor.

    While the conclusion of the panel was that the event was a deliberate act, highly likely by the captain, there was no agreement on the final few minutes of the flight … controlled ditching (powered according to Vance, unpowered according to Captain Hardy) vs. high speed uncontrolled descent.

    As was explained to me the challenge for the producers was trying to distil a lot of very technical content down to just 40 minutes and then present it in a fashion such that the average television viewer could understand it. Such an endeavour was always going to fall short of the standard of technical discussion that we expect.

  19. Mick Gilbert says:

    @airlandseaman

    Mike, as alluded to above, there were plenty of dissenting views regarding Vance’s theory, including his misinterpretation of the flaperon damage and his failure to account for the final log-on, but it was all cut by the producers in the interest of presenting a coherent story to the viewers. Sadly, they chose the wrong story to tell.

  20. airlandseaman says:

    Mick Gilbert:

    Thanks for the report. That’s as I expected. The producers knew the story they wanted to tell, and cherry picked clips from the interviews to tell that story, while all but hiding the true level of disagreement about the EOF scenarios. Dolan probably did push back more, but that was left on the floor. Cox probably was more balanced, built that also did not make the cut. I know Chari was gritting his teeth.

  21. Andrew says:

    @Gysbreght

    Thanks for posting Mark Young’s thoughts on the 60 Minutes program. I think it’s an excellent description of the way these so-called ‘current affairs’ programs manipulate facts to present a pre-determined conclusion.

  22. Brian Anderson says:

    I have commented back to 60 Minutes previously, but of course I realise that it is a waste of time. They are not interested in any facts that get in the way of the story they want to tell.

    If you look at some of the comments on the 60 minutes facebook page or twitter feed it seems that people are so uninformed that they will believe almost anything. Hence the 60 minutes propaganda is quite effective in swaying peoples’ opinions.

    Trouble is, this is no different from the daily barrage of so called “news” stories we get from mainstream media. The stories are all constructed to present a predetermined viewpoint. We are totally swamped with this form of propaganda on virtually any subject you like to name. It is hard to know what to believe any more. My solution [well, I think it works for me], is to search out alternative views and alternative news sources in order to construct some sort of balanced viewpoint. It takes time to figure out which sources have any credibility.

  23. TBill says:

    @Sabine
    “…like the rule that there should be at least two persons in the cockpit at all times – have serious drawbacks. Many airlines have abolished this rule”

    Wow, really? Can you name names? Do USA airlines require 2 in cockpit? Who had discontinued the practice?

  24. Andrew says:

    @Sabine Lechtenfeld

    RE: “Some rules which have been put into place – like the rule that there should be at least two persons in the cockpit at all times – have serious drawbacks. Many airlines have abolished this rule already because the constant necessity to open the cockpit door makes it easier for outside hijackers to take over the plane.”

    I think it’s worth bearing in mind that much of the opposition to the ‘two persons in the cockpit’ rule came from various pilot associations that objected to having their ‘territory’ invaded, particularly in Europe. Many airlines can’t choose to follow the rule or not, because it has been mandated by their government regulator.

    At the airline where I work, the rule was mandated when reinforced cockpit doors were introduced shortly after the events of 9/11. I have not encountered any problems with the rule while flying wide-bodied aircraft and it works well provided the airline has established procedures for entering and leaving the cockpit, and crews follow the procedures. However, I understand it is a little more difficult for narrow-bodied aircraft, simply because there is less room for people to manoeuvre in and out of the cockpit.

    I think many of the criticisms that have been put forward can easily be mitigated. Those who object to the rule also tend to overlook its biggest safety benefit, ie there is someone in the cockpit who can open the door if the remaining pilot goes ‘rogue’ or becomes incapacitated.

  25. ST says:

    Agree with Jerry M. Though the discussions are at times highly technical, it is still presented in a very concise and clear manner and thanks to Victor for the wonderful work in the analysis and the moderation. Thanks to all the other experts and common folks as well for a thought provoking discussion on this and other blogs.

  26. Andrew says:

    @TBill

    RE: “Wow, really? Can you name names? Do USA airlines require 2 in cockpit? Who had discontinued the practice?”

    The 2-persons-in-the-cockpit rule was mandated by the FAA for all US airlines when reinforced cockpit doors were introduced following 9/11. It was also mandated in some other jurisdictions, but it wasn’t widely adopted until the GermanWings accident in 2015. Some jurisdictions have more recently relaxed the requirements, allowing a pilot to remain in the cockpit alone when the other pilot leaves. The rule is still mandated in the US, Australia, New Zealand and Hong Kong (and probably others). The notable exception is Europe, where the requirements were relaxed by EASA in 2016. All Lufthansa Group airlines, including Lufthansa, Swiss, Austrian, Eurowings and Brussels Airlines scrapped the rule in 2017.

  27. David says:

    @ALSM. ‘Simon Hardy said that the plane went supersonic in his simulated “uncontrolled EOF scenario”……’

    This may be beyond the simulator’s fidelity boundaries I think. With the Silk Air 737 Boeing had to resort to tunnel tests and calcs for analysis. The accident report states that in its engineering simulator, “the nonlinear mathematical software has been validated up to Mach 0.87, and extrapolated using computational data up to Mach 0.99.”

    In the Garuda full motion training simulator, also utilised for part of the Silk Air investigation and I imagine akin to Simon Hardy’s, the software was just validated, “up to the flight operations envelope”.

    From my reading of the report, initially in the dive it needed full manual nose down trim. Without that a control force of 50 lb would have been needed. (When transonic it would have tended to pitch down, the centre of lift shifting aft. When air speed became supersonic locally at control surfaces the aircraft would have been difficult to control.)

    For Simon Hardy to observe so casually that the 777 went supersonic in unmanned simulation suggests either that two aircraft are very different and his simulator extends its services beyond the 737 training simulator, or he is unaware of its limitations.

    If the aircraft are similar, MH370 would have tended to pitch up and if not in a spiral, recover, maybe to a phugoid and its subsequent climbs and dives. Quite possibly it would not reach the same Mach or even the flutter boundary though it might exceed its structural strength including if in a spiral.

    The 737 had engine thrust yet flutter occurred at a few thousand feet, the dive having started at 35,000. No flaps or parts thereof detached in flight, just empennage components.

  28. Sabine Lechtenfeld says:

    @TBill
    @Andrew, you answered TBill’s question already quite precisely. It’s indeed the case that Lufthansa and Germanwings have scrapped the rule. It has been widely reported in Germany. The given reasons were that because of the rule the cockpit door had to be opened constantly for one or another reason, and that it turned out to be rather unpractical and potentially unsafe. Also, flight attendants aren’t pilots and therefore not qualified to stop anything initiated by the pilot. They would be powerless to stop immediate suicidal actions like a nose-down dive. There are many sides to that conundrum. Andrew is of course right when he points out, it might be important that someone can open the door if the remaining pilot passes out or becomes otherwise incapacitated.

  29. Andrew says:

    @Sabine Lechtenfeld

    RE: “Also, flight attendants aren’t pilots and therefore not qualified to stop anything initiated by the pilot. They would be powerless to stop immediate suicidal actions like a nose-down dive.”

    Frankly, I think that argument is one of the biggest fallacies promoted by the nay-sayers. The flight attendant is absolutely NOT expected to occupy a control seat, or to stop the actions of a rogue pilot. He or she is only there to make sure the door can be opened to allow the other pilot to re-enter the cockpit and take action. Some have also argued that simply having another human being in the cockpit might be enough to stop a pilot taking suicidal action.

  30. Sabine Lechtenfeld says:

    @Mick Gilbert
    Mick, yes, you are right, the American transport machine wasn’t actually intercepted, but if I remember correctly, the Indonesians told the pilots that they would intercept them if they wouldn’t comply and land in Banda Aceh. I’ve read several accounts of that story and need to check them again for all the details.
    The upshot is that the Indonesians actually monitor their airspace and react if they detect unauthorized and unaccounted for planes. But the big question is, if that happens at night, too, and of course how well informed a rogue pilot may have been.

    I always wondered why a pilot who ultimately wanted to head towards the final destination SIO, took the trouble to ride up the Strait and then head towards the Andamans into a northwestern direction. That’s a big detour and I doubt somehow that the Indonesians, even if they had detected the plane, would’ve had sufficient time to scramble jets. The plane would’ve crossed Sumatra quickly and been out of Indonesian airspace in no time. And the narrative of a runaway plane in trouble would’ve been far more credible if the plane had continued to fly a straight route after crossing the peninsula. The pilot could then have turned southward unobserved after he was out of Indonesian radar range. So, what’s up with the detour towards the Andamans? Did the pilot just want to avoid Indonesian airspace or could there have been other reasons?

  31. Sabine Lechtenfeld says:

    @Andrew, I have been repeating the reasons which have been given to the public for discontinuing the two-person rule. I agree with you that the mere presense of another person might well inhibit spontanoeus actions. Even just talking to a suicidal person can change the course of events. But a pilot who is hell bent on destroying himself and everybody else onboard probably can’t be stopped easily.
    The subject seems to be very dividing even amongst experts. Here in Europe we have been presented with many arguments pro and con.

  32. Ge Rijn says:

    [Ge Rijn is banned.]

  33. Richard Godfrey says:

    SC is continuing to make good progress up the Broken Ridge plateau area and has reached 27.9499°S. 3 AUVs were collected and 1 AUV launched in the last 24 hours.

    The weather is holding, there is good visibility, with a 11 knot wind, a combined swell and wave height of 2.0 m. There are no tropical storms in the region.

    https://www.dropbox.com/s/a4e2nvq23h2lqwt/SC%20Track%2017052018.pdf?dl=0

  34. Sabine Lechtenfeld says:

    To continue my musings re: invading Indonesian airspace:
    If the pilot had followed a straight route after crossing the Malaysian peninsula in order to simply cross Sumatra instead of flying a detour around the Northern tip, he would’ve been heading towards Medan – a 2-million city with the military airbase Soewondo. If the Indonesians had been monitoring their air space at night, they might’ve been worried about a big unidentifiable and unresponsive jet heading towards a large city…
    Then there was an incident on February 7, 2014 – just one month before MH370 vanished – where Singaporean fighter jets had apparently crossed into Indonesian airspace and were detected over Batam. There have been apparently tensions between Singapore and Indonesia, and the Indonesians have been monitoring the eastcoast of Sumatra well.

  35. Tanmay S says:

    @60 minutes….
    1) First of all its very cruel on part of the experts to put the blame on the pilot without locating the wreckage, FDR, CVR etc. There is not enough substantial evidence to blame the pilot beyond reasonable doubt that Captain is the culprit. He may have done it, but as of now, there is not enough evidence that nails it on him.

    2) Even if pilot is considered to be the culprit, then what was the co-pilot doing all that time? Even if we assume the pilot locked himself in the cockpit, then there are 238 other people on the other side of the locked door and they had around 7 hours to do something. Couldn’t they do anything?

    3) What is the motive behind suicide? Captain ZS life has been through intense microscope all these 4 years by different investigative agencies and there is nothing abnormal found. His personal life, professional life, health, finances everything is found normal. Just one thing though is the simulated flight on his home simulator. Can that be just a mere co-incidence. Also not much details are revealed about it.

    4) Why would a person plan his suicide so meticulously? Anyway if he wants to commit suicide, then switching off the autopilot and putting the plane in a nosedive would be the most simple way. Everything would be over in 2 to 3 minutes. What seen here is, pilot plans a region along SIO, hijacks his own plane, disengages all communications with outside world, takes a long detour to SIO, waits for fuel exhaustion, then probably even glide the plane further before finally ditching towards his destiny (all the time risking being intercepted or even chased by fighter jets). Sounds so crazy and abnormal. Its almost like he is a suicide artist and takes pleasure in the way he has committed his suicide.

    5) I even doubt the timing of this 60 minutes episode. ATSB gave a 85% chance of finding the plane in the priority search area. On this recommendation, SC was hired and now when it has been revealed that there is no plane there and SC is nearing its end of the contract in June, just piece together an episode with global experts, put the blame on pilot and close the case. This seems to be the inside agenda.

    One expert (cant recollect his name), even said that the pilot lowered the plane, banked it a bit, so that he can take one last look at Penang and bid an emotional farewell. I think aviation analysis just died at that moment. It so absurd to even assume such things without any evidence and to say it on a TV show that would be watched the entire world really needs guts. Hats off to that guy.

    At the end, it would have been much better if these experts would have just put their hands up and admitted that as of now, we dont know what really happened. There is a bit here part there evidence pointing towards a human intervention involved (I dont say the pilot), but not enough to say it was suicide by ZS.

  36. Mick Gilbert says:

    @Sabine Lechtenfeld

    Sabine, you raise a very good point about the efficacy of simply flying straight across the Indonesian island of Sumatra rather than dog-legging it up the Malacca Strait and then around the north-western tip before proceeding down into the SIO. Had the airplane simply continued heading south-west after transiting the Malaya Peninsula then crossing Sumatra would have relatively straight forward. At a ground speed of 500 knots it would have taken less than 8 minutes to get from the Kuala Lumpur – Jakarta FIR boundary to the north coast and then a further 12 minutes to transit the island (at that point it is not as wide as the Malay Peninsula is from Kota Bharu to Butterworth). Back in March 2014 I don’t think that there was even a TNI-AU fighter or fast jet on the island of Sumatra, certainly none that were based there permanently although I’m happy to be corrected on this point. Soewondo Air Force Base at Medan is home to a handful of CASA/IPTN CN-235 twin turbi-prop surveillance aircraft and Roesmin Nurjadin Air Force Base at Pekanbaru didn’t become active until late 2014. The other active fighter/fast jet bases were Supadio Air Force Base, West Kalimantan (over 700 nm away), Iswahyudi Air Force Base, East Java (over 1,000 nm away) and Sultan Hasanuddin Air Force Base, South Sulawesi (nearly 1,400 nm away). For a bloke who had ostensibly thought nothing of flying straight past two RMAF fighter bases and their accompanying air defence radars, getting across Indonesia should have been a doddle.

    I think that it was ventus45 who noted that a direct crossing of Sumatra allows the airplane to hit the 1825 ping ring without any issues but the BFOs are all wrong.

  37. Victor Iannello says:

    @Tanmay S: Some of your statements suggest to me that you did not read the article before submitting your comments and have not been following this blog. As for nothing incriminating in the captain’s personal life, we only know as much as is in the leaked report from the RMP, which has been used in the past to whitewash embarrassing deeds, such as the 1MDB scandal. (The former head of the RMP is now under investigation for his involvement.) The RMP report is also missing important information, such as the recently fractured relationship with a family the captain had grown close to. As for the simulator data, there were not a lot of details in the 60 Minutes story, but it has been analyzed in great detail by me and others.

    As I said in the article, there is not enough evidence for a legal determination of guilt. However, many of us do believe there is enough evidence to make the captain the prime suspect.

  38. Victor Iannello says:

    @Mick Gilbert: I’ll say that as they were planning the episode, the producers of 60 Minutes contacted me and asked for my participation. I declined, with the explanation that I had little hope that the format of the show would allow proper treatment of the subject matter. That was not a hard judgment call to make.

    After publishing the blog article, I tried to contact the five guests on the show to solicit comments that I offered to append to the article. I’ve received private comments from two of the guests, but nothing I can share. I also know that Larry Vance’s book will be released on May 23, and the ATSB is already in possession of it. I haven’t seen it, but it will be interesting to see how he traverses some of the conflicting evidence that is available.

  39. Mick Gilbert says:

    @Victor Iannello

    Victor, I can understand your situation. While it would have been terrific to have had someone like yourself on the show to reinforce the scientific basis of the basics of the search effort regrettably it would have been a fool’s errand. It is a sad indictment of the appallingly low standards of contemporary media; they believe that the audience is incapable of making their own mind up even if facts are presented clearly and dispassionately. I can remember when 60 Minutes first aired in Australia. It was largely modelled on the American programme and the standards were exemplary; it was must see TV. These days it’s largely big budget drivel.

    You’re probably aware that Larry Vance’s book and theory are being flogged unashamedly by The Australian newspaper down here, ably supported by the usual peanut gallery complete with a new guest supporting artist. Vance himself is promoting his theory as the only one that explains all the evidence. His impression of a just landed bass when he was corralled on the electrical interruption occasioning the final log-on was a bit of a give away that he’s not even aware of what constitutes all the evidence. (Amazing how none of the ensuing discussion on that topic made it to air.) He goes so far as to conclude one article with ‘That is what happened, and that is a fact.‘ Do they have a Nobel for Arrogance?

  40. Victor Iannello says:

    @airlandseaman, @David: I would not put too much weight on the simulations that Simon Hardy created. @Don Thompson has determined that they were performed using FlightCity’s B777-200LR simulator based at Jandakot Airport. This is not a Level D simulator. Rather, it is PC-based using Prepar3D software from Lockheed Martin. Although it is probably more accurate than the PMDG777 FSX model, it certainly wouldn’t have the fidelity to recreate transonic flight. In addition to using a different 777 version, the conditions were also different from the Boeing simulations. For instance, Hardy manually shutdown the engines, one at a time, using the fuel cutoff switches. There was about 40.6 MT of fuel left (about 20.3 MT in each tank). This means there was essentially unlimited fuel for the APU. You can see when he effortlessly pulls out of the dive that there appears to be no backdrive on the column. The large amount of slip after the second (left) engine shutdown, which initiates yaw-induced bank, is also unexplained, as that did not occur in the Level D simulations witnessed by Mike, even with 0.5 units of rudder.

  41. Julia says:

    @TanmayS. The other crew and passengers, all of whom were in the cabin where emergency oxygen is limited unlike in the cockpit, were most likely deprived of oxygen early on in the flight so they did not have 7 hours to take remedial action.
    It is assumed that the captain did not want the plane to be found which is why he planned the suicide mission so meticulously. If it had all been over in few minutes, this appalling mystery would have been concluded within a few weeks. I find it so very cruel if in fact the Captain did plan it all like this as it demonstrates the complete disregard he had for his passengers and their families. The Germanwings suicide was appalling,terrifying and unforgivable but at least the families have closure. I find it very difficult to imagine how callous the Captain was if indeed it was his intention that the plane would never be found.

  42. Don Thompson says:

    It’s clear that Larry Vance is intent on making his opinion the accepted truth about MH370.

    That intent egregiously contradicts the conduct of accident investigations, set out in Annex 13 that states: it is not the purpose of this activity to apportion blame or liability.

    His bio at Amazon states he was investigator-in-charge for the Swissair 111 accident. Another contradiction of reality, the TSB records that Vic Gerden was the IIC for Swissair 111.

  43. airlandseaman says:

    Victor: re: “@airlandseaman, @David: I would not put too much weight on the simulations that Simon Hardy created. ”

    I don’t put ANY weight on Hardy’s simulation. That was not the point I intended. My point was that Hardy made the statement that the plane went supersonic in his simulation, yet no one connected the dots wrt the flaperon and flap evidence. Given that Hardy reported the simulation went supersonic, and Cox knew about the inflight separation theory, he should have said something. But he didn’t. Or, he did, but it was edited out because that might confuse the audience.

  44. Victor Iannello says:

    @airlandseaman: John Cox consistently has said that Larry Vance’s theories have merit, but there is conflicting evidence. He at least tries to reconcile the evidence. But as you say, Larry’s theories for the most part went unchallenged, at least in what remains after editing.

  45. TBill says:

    @Sabine Lechtenfeld
    @Andrew
    Thank you very much for the 2-in-cockpit discussion. I was going to ask Andrew that a few months ago but I thought maybe it was too sensitive info.

    I am shocked! But actually I was superficially involved in TWA800 fuel fix, so I know that there is always more than one way to solve a aircraft safety problem. So I must hope those airlines have thought about the problem and done something. But Germanwings, Lufthansa, Austria…holy mackerel.

    I assume there may be some secret things. Like in the USA we have air marshals, who knows what capabilities they have? re: cockpit door, maybe sat phone with window antenna.

    I agree with Andrew strongly that 2-in-cockpit is not fool-proof, but it removes the temptation for many cases. That’s what I am looking for in general.

  46. Victor Iannello says:

    It was recently reported that “Vance said he can’t say for certain if the pilot or co-pilot was the perpetrator, although it’s his belief that it’s more likely Shah was responsible because he had ordered two extra hours of fuel.”

    I’ve provided Larry with the evidence that if he said this, it is false, in that the amount of fuel that was loaded is exactly equal to that required by the flight plan, in consideration of required reserves and the alternate airports. I’ve asked him for any contradictory evidence he may have, as it would be quite damning. I’m interested to see his response.

  47. Victor Iannello says:

    There is a new story from Bernama citing a report from the official Chinese news agency that there will Senate hearings in Canberra on how the ATSB considered the available evidence:

    CANBERRA: The Australian government’s transport safety investigator will be subject to parliamentary scrutiny over its handling of the US$150 million (RM595 million) hunt for missing Malaysia Airlines flight MH370, reports Xinhua news agency.

    MH370 disappeared on March 8, 2014 on a flight from Kuala Lumpur to Beijing with 239 people on board.

    Radar and satellite tracking indicated it went down in the southern Indian Ocean.

    An Australian Senate estimates hearing next Tuesday will be held as pressure grows for a full investigation into the handling of the two-year search by the Australian Transport Safety Bureau (ATSB).

    The bureau is accused of ignoring evidence that debunked its theory of how MH370 went down, with some airline pilots demanding a government-appointed Royal Commission.

    Center Alliance Senator Rex Patrick told the Australian newspaper on Wednesday that he would be asking questions of the ATSB about the MH370 search.

    In any circumstances where A$200 million (RM595 million) of taxpayer money has been spent and credible sources raise questions as to the approach or efficacy, some form of inquiry is worthy, Patrick said.

    The probe of the bureau’s investigation follows a forensic analysis of MH370 wreckage by Canadian air-crash investigator Larry Vance.

    Vance claims in his book that damage to the right flap and flaperon of MH370 found on islands off Africa indicated the pilot ditched the aircraft in a controlled manner.

    He dismisses the ATSB’s theory of a high-impact, pilotless steep dive.

    MH370’s location might well depend on which scenario is correct, with Vance and several veteran pilots adamant it was flown outside the 120,000 sq km search zone designed by the ATSB.

    The bureau assumed the pilots were incapacitated at the end of the flight.

    Pilots have called for a fresh search south of the southern border of the ATSB’s original target zone.

    They claim Captain Zaharie Ahmad Shah could have flown the Boeing 777 further than would have occurred on autopilot.

    Vance said the ATSB had failed to properly re-evaluate its search strategy after the flaperon was found in July 2015.

    Mike Keane, the former chief pilot of Britain’s largest airline easyJet, said this week that if the ATSB had knowingly ignored evidence, its search strategy was flawed and the bureau would be complicit in covering up the mass murder of people on board.

  48. TBill says:

    @Victor
    About a year ago, I shared with you my personal email to 60 Minutes (USA) requesting an MH370 story. I did not get exactly what I wanted. I would have asked for Greg Feith vs. Larry Vance. But on the other hand, overall I got what I was looking for.

    my email-
    “60 Minutes (USA) Proposal – Lessons of Malaysian Flight MH370

    I would like to propose a 60 Minutes story on the Loss of Malaysian Flight MH370.

    Although we do not yet know the definitive cause of the accident, intentional diversion, probably by the pilot, is the only scenario that currently fits the known facts of the case. From a flying safety perspective, we should not have to wait for actual examples of commercial airliner crashes to conclude that current aircraft design – which gives pilots complete freedom to turn off all aircraft systems and fly off undetected – needs review.

    Possible positive outcomes :
    (1) Improved air travel safety via improved security and improved aircraft design to help prevent rouge pilot and/or hijacker takeover;
    (2) Encourage continued deep ocean search for the MH370 wreckage to define cause(s) of the disaster, which quite possibly included rouge (sp-rogue) actions such as intentional depressuring to control passengers;
    (3) Hopefully free up undisclosed MH370 information or causation opinions from FBI/US Gov’t
    (4) Help to progress US Congress proposed air safety legislation, which has apparently been stalled by the industry

    I see an opportunity for 60 Minutes to cut through the confusion on MH370, and identify the key messages that are being lost in the media coverage to date.

    Regards, (TBill)
    Concerned citizen – follower of MH370”

  49. Sabine Lechtenfeld says:

    @Mick Gilbert
    Mick, thanks for the additional information re:the Indonesian capacities for scrambling some jets. My next question would’ve gone into that direction. I think we can safely conclude that it wouldn’t have happened even if the radar facilities were operating at night and the plane had been detected in real time. The plane would’ve crossed Sumatra and left Indonesian airspace before any serious action could’ve been even contemplated.
    So, I think it’s a legitimate problem which needs to be (and has been) addressed: why would a pilot who wants to head deep into the SIO and who hadn’t hesitated to cross the Malaysian peninsula without performing any radar evasive maneuvers whatsoever, take a huge detour up the Strait and towards the Andamans instead of just crossing Sumatra and then turn southwards as soon as he was out of radar reach?
    I have always believed that ZS went up the Strait and towards the Andamans because it was an integral part of his plan. If he wanted to negotiate about something, it would’ve made sense to circle just out of radar reach near the Andamans but still in strike distance of Kuala Lumpur if he wanted to create additional leverage by establishing a legitimate threat that he could weaponize the plane. But without going into details here, I’ve developed more and more doubts that ZS really wanted to negotiate seriously about something.
    But if he just wanted to vanish with the plane deep into the SIO, why the detour? Did he want to plant a false hint by going northwestwards while still being within radar reach before turning southwards? If so, it has worked to a certain degree because after Malaysia had finally admitted that the plane had turned around and re-crossed the peninsula, the Strait and the Bay of Bengal was searched extensively.
    I know that there is a school of thought which believes that the plane did indeed cross Sumatra and then turned southwards, and that not only the Lido picture but also the last military radar capture at 18:21 doesn’t show MH370. I don’t know if anyone has ever constructed a viable route across Sumatra which satisfies the BTOs as well as the BFOs. But Indonesia has of course denied that the plane ever invaded their airspace.

  50. Victor Iannello says:

    @TBill: It don’t think it is possible for 60 Minutes or any other TV show that has a similar format and audience to properly treat this material. For the most part, those shows thrive on one-sided presentation of facts and embarrassing who they decide are the bad guys.

  51. AC says:

    From the beginning I felt this 60 Minutes show was more about the issue of Australian taxpayers’ money and scrutinising Australian public officials than about MH370 itself.

    My concern now is that if nothing is found in the next couple of weeks, and the Malaysian Authorities go ahead with their final report this year, will there be any more searches? I got the impression from the recent “Inside The Situation Room” documentary/PR exercise that the Malaysian authorities wanted it to know they had taken this investigation as far as it will go. Of course things could change with a new government, but it could be a lot cheaper to simply release a final report than to commit a pot of money for another ‘no find, no fee’ arrangement.

  52. Victor Iannello says:

    In a piece for The Australian, written by Larry Vance, he says:

    “The flaperons are on the trailing edge of the wings and can be moved up and down. They are part of the flight-control system, helping the ailerons and spoilers to control the rolling movements of the aeroplane. On takeoff and landing, the ­flaperons also function as part of the flap system. They deflect down when the pilot selects the flaps to “down”. In normal circumstances, the flaps are partially extended (down) for takeoff, and fully ­extended (down) for landing. They are fully retracted (up) during cruise flight.”

    The problem with his statement is that for a landing with full flaps down (Flaps 30), the flaperons move so that they are parallel to the trailing edge of the wing. They do not droop parallel to either the inboard or outboard flaps, as this image clearly shows.

    I sent an email to Larry asking for clarification, as this point seems central to his thesis.

    [I have updated the image to more accurately depict the position of the flaperon with full flaps. The previous image showed that the droop was removed due to Landing Attitude Modification, which only should occur at higher landing speeds.]

  53. airlandseaman says:

    Victor Iannello: You are right about the flaperon position at the time of a landing. Larry Vance is wrong…as he is with most everything else related to MH370.

    Actually, the flaperon is not parallel to the wing all the time. It moves with the ailerons up and down as needed to maintain level flight. IOW…it acts like an aileron, not flaps (on approach). In a hard left turn, the right flaperon trailing edge moves down. But in level landing approach flight, it remains in line with the wing.

  54. Victor Iannello says:

    For those that prefer photographs, look at these images extracted by Mike from a video link supplied by Ed Baker that shows the position of the flaperon for a B777-300ER during a Flaps 30 (full flaps down) landing.

    https://twitter.com/Airlandseaman/status/993242276901015552

    Here’s another of 9M-MRO configured for landing:

    https://twitter.com/Edward_767/status/994976186466951168

  55. Victor Iannello says:

    @airlandseaman: Of course. We are talking about a full flaps down (Flaps 30), wings-level landing.

    The position of the flaperon would be different at Flaps 25, as the flaperon droops. You can see in this video at 8:38 the way the flaperon moves parallel to the wing when full flaps are deployed.

    https://youtu.be/Xtwne9UbH8o

  56. Richard Godfrey says:

    @Sabine Lechtenfeld

    You stated “I don’t know if anyone has ever constructed a viable route across Sumatra which satisfies the BTOs as well as the BFOs.”

    I have constructed a long curving flight path from Penang across Sumatra, which satisfies all the BTO and BFO data but only by varying the speed along the way, which is marked in light blue in the link below.

    By comparison Simon Hardy’s flight path is marked in yellow, but ends beyond the maximum fuel range at 38.3°S.

    Also by comparison a flight path to the current Ocean Infinity search area, which requires a loiter over the Andaman’s is marked in orange.

    https://www.dropbox.com/s/adxe94isez3j852/Navigation%20of%20all%20Arcs.pdf?dl=0

  57. Victor Iannello says:

    @Richard G.: That’s interesting! What was the speed profile and track over Sumatra?

  58. Sabine Lechtenfeld says:

    @Richard Godfrey, thanks a lot. Very interesting.

  59. ErikN says:

    Please see exchange in the video at 3:40-4:12. Link: https://youtu.be/Cm1j1fpldkc

    –My transcription with errata–

    Vance: The logic that was used is based on evidence that’s not correct.
    Dolan: So which parts of the evidence are not correct.
    Vance: The fact that the airplane ran out of fuel. It didn’t run out of fuel. The fact that uh…
    Dolan: Oh so your evidence for that is?
    Vance: The fact that the flaps were down and it conducted a controlled…
    Dolan: The fact, the speculation the flaps may have been down.
    Vance: Well, now it’s speculation to you; to me there’s evidence to support it. So…
    Cox: Ok, but, the SDU…so we know that there’s electricity back in the airplane after an interruption.
    Vance: And the…and the cause for that interruption is?
    Cox: That’s my question.

  60. Victor Iannello says:

    @ErikN: That is a great example of what could have been a productive exchange but just confused everybody that saw the program. That’s why I say this kind of format cannot properly treat the subject matter.

  61. Sabine Lechtenfeld says:

    @Erik N
    @Victor
    That’s exactly the part of the dialogue which positively infuriated me. They constantly skated around really interesting points, but it just went nowhere. And the uninitiated viewers were probably marveling what they were talking about.

  62. Richard Godfrey says:

    @Victor

    You asked “What was the speed profile and track over Sumatra?”

    The speed over Sumatra was 494 knots on a track of 218°T.

    This flight path crosses the N.E. coast just west of Lhokseumawe and crosses the S.W. coast near Meulaboh.

    The distance over Sumatra is 85 NM or around 10 minutes flying time.

  63. TBill says:

    @Richard
    How did you get a curved path?
    I believe a very slow slightly curved path (semi-circle segment) fits the rings and ends somewhere around 23-25S. The problem is I cannot see how to fly a path of that shape, except some kind of manual flying mode with a slight curve.

  64. Andrew says:

    @Victor

    RE: ‘It was recently reported that “Vance said he can’t say for certain if the pilot or co-pilot was the perpetrator, although it’s his belief that it’s more likely Shah was responsible because he had ordered two extra hours of fuel.”

    I’ve provided Larry with the evidence that if he said this, it is false, in that the amount of fuel that was loaded is exactly equal to that required by the flight plan, in consideration of required reserves and the alternate airports.’

    Vance isn’t the only so-called ‘expert’ who mistakenly believes that Shah ordered truckloads of additional fuel. Byron Bailey, in yesterday’s The Australian said: “Zaharie, who ordered far more fuel than he would need to fly to Beijing with a reasonable safety margin”.

    As you mentioned, the fuel analysis in MH370’s Operational Flight Plan shows the aircraft was only carrying the legally required amount of fuel, plus an additional 30 minutes of holding fuel. That extra 30 minutes of fuel was a company requirement for Beijing flights; it was not discretionary fuel chosen by Shah.

  65. Andrew says:

    @Victor
    @ALSM

    RE: “The position of the flaperon would be different at Flaps 25, as the flaperon droops. You can see in this video at 8:38 the way the flaperon moves parallel to the wing when full flaps are deployed.”

    From the AMM:

    “The flaperons droop to 10 degrees TED when the flaps are at the 5 position. They droop to 20 degrees TED when the flaps are at the 15 or 20 position. They droop to 31 degrees TED when the flaps are at the landing position (25 or 30).”

  66. Andrew says:

    Further to my previous comment:

    In a textbook ditching, the flaps would be selected to 30°. In that position the trailing edge of the flaps is much lower than the trailing edge of the drooped flaperon.

  67. ventus45 says:

    @Andrew.

    Thanks for that.

    Now, go back to EDWARD’s photo of 9M-MRO landing posted by ALSM above.
    Note that although the starboard flaperon is faired, the port flaperon is clearly down (look near the tailcone under the port tailplane).

    My take is there was right control wheel input at the instant the photo was taken, thus the starboard flaperon was ordered up. The port flaperon remained down at 31 degrees TED.

  68. ventus45 says:

    I might also add that in my ditching I used flap 20 – not 25 or 30.
    The issues are two. First, body angle (pitch at ditch) because you don’t want to exacerbate initial tail suck on entering the water (google for many studies of this phenomenon) in the water. Second, ground effect and drag re rate of speed bleed-off in the final seconds, simply put, a ditching (or ALIGHTING” a PBY, requires “flying on”, at absolute minimum ROD. From a glider pilot’s perspective, a hangar landing profile.

    Now watch a real PBY doing a “touch and go” on Lake Illawarra, my playground as a child, half a century ago.
    https://www.youtube.com/watch?reload=9&v=gF7JB2eNc5o

  69. Pilatus says:

    @Victor,

    In the Normal Mode; The flaperons droop to 10 degrees TED when the flaps are at the 5 position. They droop to 20 degrees TED when the flaps are at the 15 or 20 position. They droop to 31 degrees TED when the flaps are at the landing position (25 or 30).

    Ed’s photo’s are not what they seem; have a close look at the 9M-MRO photo, left flaperon.

  70. Pilatus says:

    @Andrew/Ventus45,

    Pays to refresh the browser now and then.

  71. Andrew says:

    Here’s another video, this time of a B777-200ER wing during an approach and landing into SFO.

    https://www.youtube.com/watch?v=PxBEHTAqtcM

    Note, the flaperon droop is removed when the speedbrakes extend on landing.

  72. Victor Iannello says:

    @Andrew: I think we are seeing differences in flaperon positioning between the -200ER, -200LR, and -300ER versions. It would be interesting to see the corresponding AMM section for the -200LR and -300ER.

  73. ventus45 says:

    @Pilatus,

    Just curious, did you ever fly a B-4 (glider), or were you a power pilot ?

  74. Don Thompson says:

    @Andrew

    Concerning fuel load: subsequent to the initial flight plan submitted to DCA/ATC early on 7th March, the alternate destinations were revised and a final flight plan was agreed at 15:05. The fuel load was based on the revised alternates.

    The original alternates were ZBTJ (Tianjin) and ZBSJ (Shijiazhuang), the revised were ZSJN (Jinan) and ERA:ZSPD. Jinan implied a longer diversion if a landing at ZBAA was denied, say, when holding in descent for approach.

    The revision implied maybe 150nm additional range?

  75. Pilatus says:

    @Ventus45

    The B4 is a great machine! I used to be an aerobatics nut many moons ago.

  76. Don Thompson says:

    @Victor,

    Flaperon views

    This landing is described as a -200LR

    and two more -200ER landings, LHR and SFO

  77. Julian Storey says:

    I am with Jerry M, and as an uneducated arm chair enthusiast I think that until MH 370 is found and its remains analysed we will not know what happened. I feel a little uneasy that most favour the actions of a devious and suicidal pilot.

  78. David says:

    @Andrew. My AMM (-300ER) at 27-11-00 page 67 speaks about a Landing Attitude Modification which with flaps at 25 or 30 will remove flaperon droop progressively in an overspeed approach and entirely at 20 knots above flight manual approach landing speed.

    My TM (-200ER) makes no mention of this but then it has nowhere near the detail of the AMM on this topic.

    Perhaps particular to that model but if not it could be a thought for an engines off flaps-mostly-down ditching, even though not at Ventus 45’s flap 20 (and also Sullenberger’s less than full)?

  79. Andrew says:

    @Victor

    RE: “I think we are seeing differences in flaperon positioning between the -200ER, -200LR, and -300ER versions. It would be interesting to see the corresponding AMM section for the -200LR and -300ER.”

    I don’t have the AMM for the -200LR or -300ER, but I’m not aware of any differences in the flaperon behaviour between the different variants. It can be very difficult to determine what the flaperons are actually doing unless you can see the other control surfaces. A small amount of upward deflection of the aileron on the same side will make the flaperon retract towards the faired position. In the ‘LHR’ video that Don posted, the left flaperon appears to be faired for quite a while after Flap 30 was selected (08:25), but you can see that the left aileron is deflected upwards during that period. The flaperon droops again when the left aileron input is removed, although it’s difficult to tell when the flight controls are in constant motion!

    Yet another video, this time of a B777-200LR landing in Lagos:
    https://www.youtube.com/watch?v=MsoWDXNI2po

  80. Andrew says:

    @Don Thompson

    RE: “Concerning fuel load: subsequent to the initial flight plan submitted to DCA/ATC early on 7th March, the alternate destinations were revised and a final flight plan was agreed at 15:05. The fuel load was based on the revised alternates.

    The original alternates were ZBTJ (Tianjin) and ZBSJ (Shijiazhuang), the revised were ZSJN (Jinan) and ERA:ZSPD. Jinan implied a longer diversion if a landing at ZBAA was denied, say, when holding in descent for approach.

    The revision implied maybe 150nm additional range?”

    Yes, the ATC plan was originally filed with ZBTJ and ZBSJ as the two destination alternates. The alternate selection was revised later in the day, possibly because the later TAF for ZBTJ showed the visibility reducing to 2000m in mist, with a TEMPO period of 1400m in light snow/rain during the likely time of use.

    ZSJN is further away from ZBAA than ZBTJ. The difference is 128 nm as the crow flies, but probably a fair bit more than that when the likely airways routing is taken into account. I don’t see anything unusual in the planned fuel load.

  81. airlandseaman says:

    All the videos confirm that the flaperons are moving up and down like an alieron. They are clearly not fixed like a deployed flap.

  82. Andrew says:

    @ALSM

    RE: “All the videos confirm that the flaperons are moving up and down like an alieron. They are clearly not fixed like a deployed flap.”

    Who said they were fixed? The flaperons move in conjunction with the ailerons to assist roll control, even when they are drooped. If there is no aileron input and the flaps are extended, both flaperons will droop to the position associated with the flap position. If there is an aileron input, then the flaperon on the down-going wing will retract either partly or fully to assist roll control.

    Victor previously said “…that for a landing with full flaps down (Flaps 30), the flaperons move so that they are parallel to the trailing edge of the wing. They do not droop parallel to either the inboard or outboard flaps, as this image clearly shows.” In reply, you said “…But in level landing approach flight, it remains in line with the wing”. Those statements are not correct. The image that Victor posted appears to be computer generated, possibly by FSX or something similar. The flaperon position in the image is not correct.

  83. Shadynuk says:

    @Andrew It seems that the flaperon is retracted as soon as the spoilers are deployed at touchdown. Would that be correct? Would the spoilers be used for a ‘water landing’?

  84. Andrew says:

    Yet another one, with an excellent view of the flaperon position in relation to the flaps during approach and landing:
    https://youtu.be/iT5jnL30aW8?t=549

  85. Andrew says:

    @Shadynuk

    RE: “It seems that the flaperon is retracted as soon as the spoilers are deployed at touchdown. Would that be correct? Would the spoilers be used for a ‘water landing’?”

    Yes, as I said in my earlier comment, the flaperon droop is removed when the spoilers extend on landing. They droop again as soon as the spoilers are stowed after landing, as shown at 11:44 in the video above.

    The spoilers would not be used in a ditching.

  86. Bruce Robertson says:

    @TBill
    @Richard
    “How did you get a curved path?”

    One means to achieve a constantly curving flight path is to have an engine shut down. If MH370 had the left engine inop, the B777 will end up in a slightly curving flight path to the left for two reasons. First, the thrust asymmetry compensation is just a model that uses certain system inputs (e.g. – throttle setting) to calculate expected thrust rather than any direct measurement of trust. The unequal thrust between left and right engine is then compensated via rudder trim only. Without also including a slight roll to the right, the aircraft will fly slightly to the left due to force vectors.

    Secondly, an inop left engine will mean additional unburned full in the left main tank as the automated fuel transfer to the right side will only occur sporadically as the fuel levels become unbalanced beyond, I recall, about 3000 pounds.

    And this is how you arrive at Zenith Plateau.

  87. Shadynuk says:

    @Andrew. Thanks. Sorry I had missed that comment.

  88. Andrew says:

    @Bruce Robertson

    Pardon me for butting in, but:

    1. If the autopilot is engaged, the AFDS will follow a ‘straight’ path, either a constant heading, a constant track, or a great circle track between two waypoints, depending on the AFDS mode. The AFDS will command an angle of bank if necessary, to make that happen, regardless of the rudder position. That might mean the aircraft will fly slightly unbalanced, but it will still fly a straight path.

    2. The B777 does not have an ‘automated fuel transfer’ from one side to the other.

  89. Andrew says:

    Carrying on, in HDG mode the path over the ground will obviously depend on the wind. If the wind is changing, the path will curve. The path will also curve as the magnetic variation changes, assuming the heading/track reference is magnetic.

  90. Bruce Robertson says:

    @Andrew

    1. My premise is the flight envelope protection effected a recovery from an aircraft upset. As such, there is no heading or track selected — no waypoint either, so no banking to maintain a straight path.

    2. Correct warning only. The pilot(s) will have needed to have manually selected fuel crossfeed.

  91. Don Thompson says:

    @Andrew,

    Thank you for the weather detail in your reply, above.

    The change to the diversion airports cannot be conflated with “extra two hours fuel” and the fuel loaded on the aircraft, confirmed by ACARS reports and the despatcher’s records, is consistent with the flight plan. All sets the “extra two hours fuel” aside as another myth.

  92. Andrew says:

    @Don Thompson

    Yes, it’s utter bollocks, spread by people who obviously haven’t bothered to do their homework.

  93. Richard Godfrey says:

    SC is continuing to make good progress up the Broken Ridge plateau area and has reached 27.7475°S.

    Ocean Infinity has completed the 4th cycle of AUV deployment, with 7 AUVs in each cycle.

    The weather is holding, there is good visibility, with a 12 knot wind, a combined swell and wave height of 2.1 m. There are no tropical storms in the region.

    https://www.dropbox.com/s/l07zo6zu83zxulb/SC%20Track%2018052018.pdf?dl=0

  94. TBill says:

    @Bruce Robertson
    At least until about 18:25 flying beyond waypoint NILAM, MH370 appears to be at very high speed with 2 engines, based on the known information (radar/cell phone “connect” at Penang/sighting above Pulau Perak). So any engine failure/limp mode would have to be after that, or you have to provide an credible explanation for why the known information is false.

    Probably the “limp” mode would have to start shortly after Penang to curve to Zenith/Batavia area. This is similar to Mike Chillit’s path – a semi-circle curving around to Batavia.

    Instead of a random curved path with a failed engine, it could be thought of as a slow flight going from ISBIX south turning onto or near vector L894. I find the BFO’s are all off by about +10 so it might make some sense if the 18:25 SDU reboot shifted BFO’s by 10 units evenly.

  95. David says:

    @Victor. Whether or not there was an active pilot at the end remains open. I think it likely that if there had been, in planning he would have ensured there was no ELT transmission, most probably intending a high speed impact to obviate one. Yet that could have been at any time consistent with the aircraft reaching the 7th arc, but without awaiting fuel exhaustion.

    Locating the wreckage is independent of this because the final descent rates indicated the aircraft most likely would have crashed within the current search width either way. Still, should the wreckage not be located in this current search I do not think there will be more searches in the near future since there will be insufficient justification, success probabilities being insufficient: the aircraft quite likely will lie in the vast low probability zone. So the focus of any continuing investigation most probably will shift to pilots and who knew what about their doings, if not more scrutiny of potential hijackers. I have mentioned the desirability of a Royal commission previously.

    The pilot-at-the-end question is part of that general topic and in turn I think the below is relevant to that. It questions again whether the final log-on is more likely explained with a pilot though in far more detail than I have put into this subject over the last couple of years.

    The APU fuel flow on which the ATSB based its 2015 assessment that there would be enough fuel for the 2 mins starting and running, needed to power the final log-on at an APU- loaded rate of 2 lb in 55 secs or 131 lb/hr, generator loading unstated.
    If that was based on installed APU fuel flow direct measurement, under typical load and ambient conditions, that would have required a modification since there is no flow meter, cockpit readout or maintenance recording of that fuel flow. I note also it is a singular figure without conditions specified.

    Test bed fuel flows indicate at sea level the fully loaded APU would consume 620 lbs/min and a likely minimum of 300 lb/hr under no load, at idle. These and FCOM fuel flows can be related to MH370, at 30,000 ft as an example, using approximate methodologies. The indicative outcomes are 343 lb/hr and 350 lb/hr (80 kVA generation, 431 with 120kVA), with the FCOM figure 354 lb/hr (possibly plus likewise).

    Beneath 22,000 ft the fuel consumption could rise again by 41 lbs/hr by my estimate. As I have said, the ATSB has just the one figure for all circumstances.
    Part A of the below goes into all that comprehensively.

    https://www.dropbox.com/s/sdjseaxvwz8tr38/MH%20370%20end-of-flight%20auxiliary%20power%20unit%20fuel%20availability%20and%20accessibility.docx?dl=0

    Even so, fuel might well be sufficient, potentially, even for such consumptions. How much of that would be accessible though I address in Part B.

    The ATSB assessment of fuel availability was reported on in the Definition of Underwater Search Areas, December 2015. The availability was described thus, “In a standard flight attitude (1° pitch), the difference in location between the left engine fuel inlet and the APU fuel inlet would result in approximately 30 lb of fuel being available to the APU after a left engine fuel exhaustion. From this information, the APU had a maximum operating time of approximately 13 minutes and 45 seconds. The pitch attitude would have an effect on the usable fuel for the APU……” and, “In-flight acceleration forces could also affect the distribution of fuel in the tanks.”

    Over a year and a half later in the Search and Debris Update of August 2017 a steep descent became integral to the end-of-flight though that and subsequent reports made no mention of whether these descents affect the access to the fuel supposed earlier.

    I think they would and that it is quite possible as per Part B that there would be insufficient to power the final log-on. That would make a piloted descent the most likely possibility as to the log-on, he shutting down engines with some fuel remaining, consequent APU auto-start in an induced dive leading to the final BFO’s and log-on.

    Personally I doubt that a recovery from the dive would have been planned, in which case what I raise would not affect the search unless such a recovery was spontaneous (unlike his German Wings, Egypt Air and Silk Air predecessors). However this would be contrary to his plan so I think that is unlikely.

    (An unpiloted pitch up, recovery and series of phugoids unpiloted (I mentioned above, May 17th 1:27 AM) would make for an interesting proposition. I do not think it likely, supposing relight-induced bank caused the descent in the first place, that the aircraft would have continued other than in a spiral. That scenario though would be the only one I know of which is consistent, reasonably, with a rapid descent followed by a pull out. Oddly it might increase the chances of a pilotless aircraft flying further than (I believe) a pilot would.)

    The findings remain preliminary since they include various judgements but I think they offer enough substance to warrant attention. Of course also, I might have it wrong.

    I explain that Honeywell will not provide the requisite information on the APU to me, even once the investigation is concluded. Even so the most important judgements are to do with fuel access, more a subject for Boeing to look into, should a need be raised by investigators.

  96. Victor Iannello says:

    @Andrew said: The image that Victor posted appears to be computer generated, possibly by FSX or something similar.

    Yes, you are correct. The PMDG777 model includes “Landing Attitude Modification” which seems to be a little-talked about feature that Boeing implements on the B777 whereby the flaperon droop is progressively reduced at higher speeds to increase the pitch by reducing lift. Unfortunately, the LAM is not accurately replicated in the PMDG777 model, as the droop is completely removed at Vref+5. I have replaced that image with the configuration at Vref+4, which shows the position of the flaperon at full flaps and the correct droop.

  97. Victor Iannello says:

    @Richard G said: The speed over Sumatra was 494 knots on a track of 218°T.

    In the interest of not confusing people, I’ll add that in order to satisfy the BFO criterion at 18:28, a plane on this track and speed would require a concurrent climb. Without the climb, we are left with tracks around 296°T and speeds around 500 knots, which places the plane traveling northwest over the Malacca Strait rather than across Sumatra, consistent with the military radar data.

  98. Victor Iannello says:

    @David: The possibility of a deliberate shutdown of the engines by cutting off the fuel at high altitude and then using the APU for a flaps-down ditching after a long glide cannot be ruled out. In fact, I am surprised that Larry Vance did not propose this as his baseline scenario as it explains the SATCOM log-on at 00:19 and is consistent with what he believes is the cause of the flaperon and flap damage. However, the reason for the steep descent at 00:19 is not explained, and of course, this scenario would run counter to the other evidence suggesting a high speed impact with retracted flaps.

  99. Tim says:

    Perhaps the aircraft IS flying a phugoid, with climbs and descents. And it is this that is confusing everyone’s BFO calculations.
    It seems there may be many possible routes if you assume an autopilot off flight. Is this what is making it so difficult to find a route that we are all in agreement with?

  100. Richard Godfrey says:

    @Victor

    You stated “In the interest of not confusing people, I’ll add that in order to satisfy the BFO criterion at 18:28, a plane on this track and speed would require a concurrent climb.”

    You are correct that the BFO fit at 18:28:15 UTC is only with a climb of around 1,500 fpm. Such a climb however, could have led the Indonesians to believe the aircraft was not a threat.

    There are several other problems with such a flight path:
    1. Discard the Beijing Lido radar trace.
    2. Discard the final Malaysian Military Radar data point at 18:22:12 UTC.
    3. Indonesia’s claim that they never picked up MH370.
    4. What caused the 6 hour curved flight.
    5. Inconsistency of a turn back at the FIR border between Singapore (delegated to Kuala Lumpur) and Ho Chi Minh, flight across Malaysia following the FIR border between Kuala Lumpur and Bangkok, but then flying straight across Indonesian airspace.

    Below is a summary table of the flight path:
    https://www.dropbox.com/s/fc4romqxvt4e62r/MH370%20Curved%20Flight%20Path%20Model%20V17.0%20no%20FMT.png?dl=0

    I note that I incorrectly stated the average ground speed across Sumatra as 494 knots, it should be 457 knots. My apologies for the mistake.

    I did not spend a lot of time on this curved path from Penang over Sumatra to the 7th Arc. I was just interested to see, if there is such a path, that does not require a FMT. I need to redo the flight path properly, as I was only averaging the curve, with straight line segments.

  101. Andrew says:

    @Victor

    RE: “The PMDG777 model includes “Landing Attitude Modification” which seems to be a little-talked about feature that Boeing implements on the B777 whereby the flaperon droop is progressively reduced at higher speeds to increase the pitch by reducing lift. “

    Thanks. The B777 LAM feature barely rates a mention in the Boeing manuals. It’s designed to increase the nose gear clearance on landing if the approach speed is high by killing some of the wing’s lift, resulting in a higher pitch attitude. As @David mentioned in an earlier post, it only operates at landing flap settings, ie Flap 25 & 30. I think you can see it operating when landing flap is selected at 08:15 in Don’s LHR video:

    https://youtu.be/tWKR4tEAeCA?t=495

    I understand the upward deflection of the outboard ailerons is also a feature of the LAM.

  102. Don Thompson says:

    @Andrew,

    My information on alternates, above, was incomplete.

    The final Flight Release included a second destination alternate of Hangzhou was declared, requiring 1h45m/7700kg.

    So…

    ERA at ZSPD/Shanghai-Pudong, allowed 3% contingency: 1200kg
    1st destination alternate at ZSJN/Jinan, allowed 0h46m/4800kg
    2nd destination alternate at ZSHC/Hangzhou, allowed 1h45/10700kg.

    In the Flight Release, the ZSHC allowance comprised the allowance for 1st dest alt, plus “Comp Fuel” = 0h30m/3000kg, plus “Final Resv” = 0h30m/2900kg.

    The selection of ZSHC as 2nd destination alternate, 620nm south of ZBAA, seems extreme by comparison with the original two destination alternates. However, I am aware that it’s been suggested that re-routing well away from weather diversions in Chinese airspace is a good thing.

  103. Sabine Lechtenfeld says:

    @Richard, thanks for revealing more details about this hypothetical flight route across Sumatra. I guess, you were just curious if there existed a route which would satisfy the known data.It’s also interesting to see a route without a fmt.
    You listed the problems with such a scenario yourself. My main question would be indeed what could’ve caused such a curious flight path. Crossing Sumatra on the ZS’s way into the SIO instead of taking the detour around the northern tip of Sumatra only seems to make sense if ZS would’ve crossed the island fast and on a direct route in order to leave Indonesian airspace behind as fast as possible and before any actions could’ve been taken.
    The question remains nevertheless why ZS most likely didn’t choose this option. It would’ve saved him a lot of fuel. As we have discussed before, even if the Indonesians would’ve spotted the plane in real time – and they do indeed seem to monitor their airspace, at least during the day – there wouldn’t have been enough time or resources for actually scrambling any jets. I think the danger of actual interference by the Indonesian airforce would’ve been minimal. Therefore I believe that going up the Strait and northwestwards towards the Andamans was an integral part of ZS’s overall scheme. Going up the Strait and towards the Andamans wasn’t just about avoiding Indonesian airspace. It would make sense however if ZS stayed out of Indonesian radar range once he reached the location of his fmt and started his final leg southwards. His goal was most likely to turn southwards unobserved after all.

  104. Victor Iannello says:

    @Andrew: Thanks. In that video, not only do we see LAM operating at 8:15, but also we see droop progressively added as the speed during the descent reduces to the landing speed. I think with the LAM explanation, we have now reconciled the videos, the simulation, and the AMM.

  105. Andrew says:

    @Don Thompson

    The Alternate fuel that was specified for ZSHC was 10,700 kg. That figure includes the fuel required for a missed approach at ZBAA; climb, cruise and descent to ZSHC; and an approach and landing at ZSHC. The aircraft was also required to land with the Final Reserve fuel (2,900 kg) intact. If the aircraft were to divert to ZSHC from ZBAA, it would need a minimum of 10,700 + 2,900 = 13,600 kg at the commencement of the missed approach at ZBAA.

    Although two alternates were specified in the flight plan, the aircraft was only carrying sufficient fuel to divert to the first alternate, ZSJN. I can’t explain why two alternates were specified on the plan. A second alternate is normally only required when the destination weather is forecast to be below landing minima, which wasn’t the case for ZBAA that day.

    RE: ”I am aware that it’s been suggested that re-routing well away from weather diversions in Chinese airspace is a good thing.”

    It certainly doesn’t hurt to carry extra fuel when flying through China. The country is notorious for delays and sudden airspace closures, especially if there is any bad weather around. When we operate to Beijing from Hong Kong, we normally nominate ZBTJ as the destination alternate, but we carry sufficient fuel to fly to Beijing and all the way BACK to Hong Kong!

  106. Don Thompson says:

    @Andrew, @Victor

    Following the Vance theory, with power available all the way to contact, would the best option be higher airspeed, pitch up, while approaching that carefully identified crest of a swell?

  107. Victor Iannello says:

    @Don Thompson: I’ll defer to @Andrew. Certainly, having engine thrust and not having a limited runway length provides more options to anticipate the swell.

  108. airlandseaman says:

    And having engine thrust available also means you can land a little slower using a higher angle of attack and offsetting thrust. That said, I don’t believe it happened. Too many independent facts line up to support the theory that MEFE occurred circa 00:17:30 at high altitude. There are no facts that support the theory that the plane was landed under power.

  109. sk999 says:

    R. E. fuel, we can compare MH370 to MH371 from the 1st ACARS progress reports for each.

    MH371 ZFW 175.7 tons, total fuel 47.4 tons
    MH370 ZFW 174.6 tons, total fuel 49.2 tons

    So MH370 carried enough fuel for an extra 20 minutes or so.

  110. TBill says:

    @airlandseaman
    ” There are no facts that support the theory that the plane was landed under power.”

    Correct me, but I am thinking the key evidence is the flaperon trailing edge damage. There are 2 options: (1) water damage vs. (2) air damage. If you say water damage, Vance perhaps has an argument.

    I do not see why flaps have to be down though, could be up and still get the trailing edge damage in the water.

  111. DennisW says:

    @TBill

    There are 2 options: (1) water damage vs. (2) air damage. If you say water damage, Vance perhaps has an argument.

    The French know. I cannot believe that they would not share a water damage scenario with the Malaysians and the ATSB. The fact that search areas close to the arc are being prioritized suggests to me that the flaperon damage was not produced by contact with the water.

  112. Kenyon says:

    The trailing edge of the Flaperon is the most fragile feature of the sub-structure.

    It makes no sense to make simple observation of trailing edge damage the seed of a theory that extrapolates (using imagination as evidence) to a full fledge story of who did it, how, and why.

    If someone wants to propose a controlled, SIO, ditch theory it must focus on a plausible explanation of how the Flaperon’s primary attachments failed.

  113. Gysbreght says:

    Kenyon says: “If someone wants to propose a controlled, SIO, ditch theory it must focus on a plausible explanation of how the Flaperon’s primary attachments failed.”

    When a structural element is subjected to loads that exceed its strength, it fails.

    The strength of a component is defined by the loads it is designed for, so it fails when subjected to loads that exceed the design loads. At the same speed the hydrodynamic loads in water are about eight hundred times greater than the airloads the flaperon was designed for. The direction of the load from impact on water is similar to the direction of the aerodynamic loads the flaperon is designed for. As far as can be seen on the available photographs, the flaperon’s primary attachments failed under overload.

    The trailing edge damage on both aileron and flap strongly suggest an impact on water.

  114. Bruce Robertson says:

    @TBill

    “MH370 appears to be at very high speed with 2 engines” Or, it was flying on one engine and lots of gravity, that is, it was in a descent. Once that descent took it to its pre-selected altitude, say 10k feet, altitude was held and airspeed decayed to near stall speed.

    I originally proposed this curved path on March 31, 2014, on Duncan’s site but it died due to lack of interest. In 2015, I put numbers to it and finalized on 21S 104E.

    Mike Chillit was looking at equatorial waters when I pitched him with the idea of Zenith. He adopted it for awhile but then moved south due to less AIS-identified ship traffic. After I sent him some links to animated drift patterns, he took it further with drifter studies.

  115. airlandseaman says:

    TBill: Re your comment: “Correct me, but I am thinking the key evidence is the flaperon trailing edge damage. There are 2 options: (1) water damage vs. (2) air damage. If you say water damage, Vance perhaps has an argument.”

    You are conflating two independent questions: (A) possible evidence of (1) in-flight separation vs. (2) separation at impact, conflated with (B) possible evidence of a ditching attempt (1) with vs. (2) without engine power at the time of the attempted landing.

    All I said was ” There are no facts that support the theory that the plane was landed under power.” Even if there was an attempt to ditch the plane, there could be damage with or without the engines running at the time. So the flaperon damage tells us nothing about the engines.

  116. Joseph Coleman says:

    @Richard Godfrey

    I’m not suggesting or trying to hint any scenario, but could you if possible with the path over Northern Sumatra shown in one of your earlier today graphics, include if possible any weather data, sat image overlay or information with regards the storm cells west of Sumatra.

    Many Thanks

  117. Andrew says:

    @Don Thompson

    RE: ”…would the best option be higher airspeed, pitch up, while approaching that carefully identified crest of a swell?”

    The Boeing FCTM recommends the following technique for a powered ditching:

    ”Select flaps 30 or landing flaps appropriate for the existing conditions.

    Advise the cabin crew of imminent touchdown. On final approach announce ditching is imminent and advise crew and passengers to brace for impact. Maintain airspeed at VREF. Maintain 200 to 300 fpm rate of descent. Plan to touchdown on the windward side and parallel to the waves or swells, if possible. To accomplish the flare and touchdown, rotate smoothly to touchdown attitude of 10° to 12°. Maintain airspeed and rate of descent with thrust.”

    In other words, the lowest speed possible, using thrust to minimise the rate of descent. The touchdown attitude is obviously important, to prevent the aircraft nosing in. The FCTM doesn’t mention the power-off case, but in that scenario I think a reduced flap setting would be advisable, assuming the flaps were still available. The reduced flap setting would help preserve some of the aircraft’s energy during the flare, make it easier to achieve the desired touchdown attitude.

  118. Andrew says:

    @Don Thompson

    Just to clarify my earlier post regarding fuel, MH370 was planned to arrive at ZBAA with 11,900 kg remaining (49,100 – 37,200 kg). Given that a diversion to ZSHC required 13,600 kg from the missed approach point, the aircraft was not carrying sufficient fuel for such a diversion. That’s no big deal, because the aircraft was carrying enough fuel for the first alternate; the second alternate wasn’t legally required.

  119. ventus45 says:

    @DennisW

    “The French know”.
    Interesting comment DennisW.

    People seem to overlook the fact, that the BEA would have all the details from the 2012 taxi accident, since they were party to the investigation, since the other aircraft was an A340.

  120. ST says:

    @Sabine – You said “So, I think it’s a legitimate problem which needs to be (and has been) addressed: why would a pilot who wants to head deep into the SIO and who hadn’t hesitated to cross the Malaysian peninsula without performing any radar evasive maneuvers whatsoever, take a huge detour up the Strait and towards the Andamans instead of just crossing Sumatra and then turn southwards as soon as he was out of radar reach?:

    The reason for the north west direction after crossing Malay Peninsula may be explained by an intent on the part of ZS to pray in the direction of qiblah before the FMT and no return path towards the SIO.

    It is only a theory but your question provoked more reading and it seems to make sense.

    There is a lot of literature on this available with a simple google search. There are even apps used to determine the exact direction based on latitude/longitude co-ordinates.

  121. airlandseaman says:

    ST: 3 years go I noted the course was a great cicle path to mecca.

  122. Mick Gilbert says:

    @ST

    Interesting perspective on the flight up the Straits of Malacca. However, from what I have read and been told by practising Muslims, they do not pray at night (except for the Taraweeh or Salat Qiyam Allayl, which is only performed during the month of Ramadan and only just after the Isha’a or evening prayer).

    @airlandseaman

    It’s also part of a great circle path to Bangalore; maybe he fancied a chicken curry.

  123. Victor Iannello says:

    @Andrew said: The FCTM doesn’t mention the power-off case, but in that scenario I think a reduced flap setting would be advisable, assuming the flaps were still available. The reduced flap setting would help preserve some of the aircraft’s energy during the flare, make it easier to achieve the desired touchdown attitude.

    I think the high drag at Flaps 30 means the descent angle will be steep (> 6°) with no thrust. Less flap could significantly reduce the vertical speed to deal with at the flare, albeit at higher airspeed.

  124. Andrew says:

    @Victor

    Definitely.

  125. airlandseaman says:

    Victor> FWIIW…many glider pilots prefer a steep approach. A steep approach using flaps and spoilers provides better control of the landing point. Of course, landing a 777 without power may be completely different, and there’s plenty of runway in the SIO, but if it was me, I would use full flaps and flare to touch down with nearly zero vertical speed.

  126. ST says:

    @ALSM, Thanks for the input. Good to see you had analyzed this before.
    There seems to be also a 90 degrees angle from quiblah for burial and a requirement that the point of the direction should be the closest point/free from obstruction which could explain the need to go near Pulau Perak and roughly towards VAMPI.

    @Mick – Thanks for the detailed inputs. Yes – it is true that it is not typically done but this being a different scenario with so many passengers that were also impacted, not sure if the possibility of the direction and its significance can completely be ruled out. The correlation to geometric/geographic co-ordinates related to this belief and a flight path seem significant to be ignored considering we are looking at every possible explanation through the entire path that will help come up with an alternate option if the plane is not located on or near the 7th arc.

  127. Andrew says:

    @ALSM

    In this case I think judging the correct flare point and rate of flare would be a bigger issue. With full flap, the rate of descent would be well over 1,000 ft/min and the speed would wash off very quickly during the flare.

  128. Bruce Robertson says:

    @ALSM
    @Andrew

    “full flaps”: There are two cases where flaps wouldn’t be deployed at all. First, there is no automation facility (that I know of) that will drop the flaps for landing. If, after the APU came on line, flight automation was flying the plane again, flaps would not be deployed.

    Second, if a passenger was pressed into service as pilot-du-jour, dropping the flaps would likely be a task not executed.

  129. Andrew says:

    The following piece was published in today’s (Saturday) edition of The West Australian:

    60 Minutes Australia program on MH370 faces criticism

  130. Andrew says:

    @Bruce Robertson

    The discussion was based on a controlled ditching by one of the pilots, as promoted by Larry Vance and others. Nobody implied the flaps would be automatically deployed or that it was done by a passenger.

  131. Sabine Lechtenfeld says:

    @ST, thanks for your comment re: the intentions of the pilot for taking the detour around the Northern tip of Sumatra on his way to the SIO instead of just continuing in a strait route and crossing Sumatra. I don’t exclude religious/spiritual motives for some of his movements. This might be also important for some end-of-flight-scenarios.
    I hope that my musings inspire some new ideas. The school of thought that the pilot simply wanted to avoid Indonesian airspace at all costs because the Indonesians do indeed watch their airspace, doesn’t convince me entirely. Besides being possibly detected by Indonesian radar there would’ve been most likely no negative consequences for violating Indonesian airspace, since the Indonesian airforce had no rapid response resources in the relevant area, and the crossing of Sumatra would’ve taken less time than the crossing of the Malaysian peninsula. Even if the plane would’ve been detected in real time, it would’ve been gone before the Indonesian could’ve come up with a viable response strategy. And why would the pilot try to avoid being detected by Indonesian radar – if it was even working at night, which isn’t clear? The plane had been detected by several other radar facilities before after all, and there are no indications at all that the pilot tried to conduct any radar-evasive maneuvers while crossing the Malaysian peninsula. The opposite seems to be true: he wanted to be seen. And that might indeed be one motive for riding up the Strait until the plane was lost to Malaysian military radar. He might’ve tried to stay within the Malaysian vicinity as long as possible. By forgoing a strait route towards Medan and going over Sumatra after having crossed the peninsula, but turning northwestwards instead and riding up the Strait towards the Andamans, the pilot also makes it pretty clear that the plane wasn’t a runaway ghost plane going inexorably westwards after turning around near IGARI : no – the plane was consciously piloted, and maybe the Malaysians were meant to know it.
    I really think it hasn’t been sufficiently thought through why the pilot took the lengthy detour beyond the northern tip of Sumatra before making the fmt towards the SIO. If his sole objective was to go as deeply into the SIO as the plane’s fuel range allowed, it was definitely a very counterproductive move.
    I will put up a “brainstorming list” when I find some time over the weekend. I think the question is very important for making out potential motives of the pilot (most likely Zaharie Shah).

  132. Richard Godfrey says:

    @Victor

    The Boeing 777-200ER has the following flaps stages: up, 1, 5, 15, 20, 25, 30.

    Flaps 20 is apparently used in an engine out condition, not full flaps as Larry Vance claims.

    The Airbus 320-214 has the following flaps stages: up, 1 (10), 2 (15), 3 (20), 4 (35).

    When US Airways 1549 ditched in the Hudson, Scully decided to use Flaps2 according to the official report.

    From the official report for the US Airways 1549 accident:
    “During postaccident interviews, the captain stated that he used flaps 2 because there were “operational advantages to using flaps 2.” He stated that using flaps 3 would not have lowered the stall speed significantly and would have increased the drag. He stated that he was concerned about having enough energy to successfully flare the airplane and reduce the descent rate sufficiently. He stated that, from his experience, using flaps 2 provides a slightly higher nose attitude and that he felt that, in the accident situation, flaps 2 was the optimum setting.”

  133. Sabine Lechtenfeld says:

    I should add that the problem has been addressed now and then over the years, but not lately as far as I know. Most of us have just accepted that ZS apparently never entered Indonesian airspace, althought there is a minority who believes that ZS did cross Sumatra after all and never went up the Strait. But as Richard Godfrey’s calculations have shown, this doesn’t seem to be very likely, even if we are prepared to throw out the last radar capture and the Lido image.

  134. ventus45 says:

    @ALSM

    We all know that you are dead against a ditch. You are convinced by the final BFO’s, debris, etc. Fair enough.

    Consequently, (knowing that you are also a glider pilot), I have to assume, that your remark “but if it was me, I would use full flaps and flare to touch down with nearly zero vertical speed”, in a power at idle, or no power at all, glide to ditch, is clearly flippant, and is manifestly reflecting your now obvious frustration with the growing support for serious consideration of any ditch scenario, which, I concede, is perfectly understandable given your convictions.

    But, if it were for real, I can not believe, that you would actually do what you stated. If you were in the left seat, and “set up” for that, and I was in the right seat, I can assure you, and everyone else, that I would not remain a subordinate FO, and regardless of the eventual outcome, those who eventually downloaded the CVR, would certainly know it.

  135. Richard Godfrey says:

    @Victor, @Andrew

    I meant to add that this confirms what you both have stated above, regarding the flap setting in an engine out condition, increased drag of higher flap settings and sufficient energy to flare successfully.

  136. David says:

    Balanced report here by Steve Creedy with more detail as to the Vance view.
    https://www.airlineratings.com/news/mh370-australian-search-defended-after-60-minutes-criticism/

    Vance points at the witness marks inside the flap seal pan as indicating the flap was retracting when they occurred. They do indeed indicate movement while the support track was withdrawing. The ATSB did not mention that.

    In a damage assessment I posted some time ago I described one of these dents by the support track rear, which caused a crack in the flap skin, as being made when the flap was 8 inches aft from housed.

    His interpretation as I understand it is that this dent was the first, as the flap retracted, those with it fully retracted following. Mine was that it was the last, with flap or that end of it moving to the rear having separated and this inboard flap piece having broken from the rest. That supported the ATSB interpretation.

    I note that Creedy says of this ATSB assessment that the flap was housed when separated that, “The ATSB analysis was sent to UK investigators at the AAIB and manufacturer Boeing for verification before it was released.”

    Vance is reported to have said, “It would have been impossible for the flaperon to have maintained its normal curved shape at its leading edge and to have maintained its normal curvature along its upper and lower edge.” He overlooks the even better condition of the MH 17 flaperon after that aircraft had hit solid ground.

    “He also argues that damage around the entry hole for the flap support track — the same one mentioned in the ATSB analysis — was caused when the track and the carriage assembly were pulled out of the hole.” That damage was above the support track and I attributed the same cause. I do not see that says anything other than the support track withdrew and probably the carriage assembly with it. To me that is irrelevant to the flap having been deployed when the flap broke and separated, though of course I have not read his book.

    I will post my earlier assessment for those who would like a ready reference to the pertinent photographs.

  137. David says:

    No, my assessments (two) too detailed for these purposes. Instead, Pages 15 to 18.
    https://www.atsb.gov.au/media/5773389/ae-2014-054_mh370-search-and-debris-update_aug2017.pdf

    Other photos at and towards the bottom of this. They can take time to download. Click on them to expand:
    https://www.atsb.gov.au/mh370-pages/resources/images/

  138. Sabine Lechtenfeld says:

    @ventus45, considering that you believe the flaperon was most likely planted at the beach of Reunion – you have laid out your reasons quite detailed some time ago – you must consider the flaperon useless for constructing any end-of-flight scenarios.Is that correct?

  139. David says:

    Correction. The MH17 ‘flaperon’ I referred to above was in fact a part of an outer flap. Nevertheless its condition is surprising:
    http://joostniemoller.nl/wp-content/uploads/2015/03/P1460386.jpg

  140. Don Thompson says:

    @David,

    You made reference to a flaperon from MH17. Would you cite a source for that please?

  141. Richard Godfrey says:

    SC is continuing to make good progress up the Broken Ridge plateau area and has reached 27.4747°S.

    Ocean Infinity has started the 5th cycle of AUV deployment, with 7 AUVs in each cycle.

    The weather is holding, there is good visibility, with a 14 knot wind, a combined swell and wave height of 2.2 m. There are no tropical storms in the region.

    https://www.dropbox.com/s/ei98pj23hxcbe7p/SC%20Track%2019052018.pdf?dl=0

  142. Don Thompson says:

    @David,

    Correction noted!

    @Sabine

    Regarding the apparent path taken around Sumatra/Indonesia. If the intent was not to be ‘followed’, then doubling back when out of possible view would be a useful tactic. I have some further comments to discuss what might have been generally accepted about the air-ground data comms system before 7th March 2014, need to distil down yet, before returning to discussion of a few days back.

  143. Sabine Lechtenfeld says:

    @Don, that thought would’ve been very prominently on my list, too. Especially since the pilot was last “seen” being on a northwestern route towards the Andamans. Also, this focussed aireal searches after the SCS was abandoned, on the Strait and the Bay of Bengal for a while.

  144. Sabine Lechtenfeld says:

    I should add: it is not surprising that the plane (probably) kept out of Indonesian airspace and (anyone’s) radar range once the pilot decided to perform the fmt and to embark on his trip of no return into the SIO. But he could’ve done that also, if he had just crossed Sumatra and waited with the fmt southwards until he was safely out of Indonesian radar range. So, there must’ve been a specific reason to move within radar range up the Stait and northwestwards at first.

  145. Sabine Lechtenfeld says:

    @Don, and I would indeed appreciate if we could return to our discussion about the air-ground data com system and what ZS could or couldn’t have known about it. Thanks a lot for keeping on the ball 🙂
    I will be monitoring the discussion, but I will be travelling for a few days – family affairs! They don’t always appreciate my continuing interest in a plane, which the majority here has forgotten about…

  146. Don Thompson says:

    @Sabine – I’ll watch out for your next comment & pick up then. Safe travels.

  147. Neville Macaulife says:

    I noticed in the 60 Minutes episode that Larry Vance drew heavily on his experience investigating the appalling Swiss Air 111 crash. I looked it up in the Wiki, and found that a researcher reported that magnesium at ten times normal level had been found among the wiring that generated a fatal arc, suggestive of the possibility that an incendiary device had been present.

    Later a Royal Canadian police official, Thomas Juby, involved in the investigation, wrote a book in which he stated his superiors ordered him to remove all references to magnesium from his notes. CBS’s Fifth Estate series has an episode about all this, Swiss Air 111: The Untold Story, https://www.youtube.com/watch?v=s9rVKWsMv_g&t=1534s.

    The Wiki article led me to a review of Juby’s book, (ref. 35, http://thechronicleherald.ca/books/1463929-book-review-swissair-crash-investigator-believes-there-was-a-cover-up,), which left me wondering if I’d strayed into the twilight zone. It contained the following two paragraphs.

    Inexplicably, on the third day of the investigation, the Transportation Safety Board announced there would be no criminal investigation of the crash.

    As a result, the ongoing profiling of passengers and airline employees was abandoned even though disturbing leads were developing. Juby reports, “Of the employees who serviced the aircraft just prior to its last departure, one was never found again. He worked the one shift to service this aircraft and left. He had provided fake identification to his employer when hired.”

    Can anyone document that this is pure hokum? Because if it is not, something is pretty wrong somewhere.

  148. DennisW says:

    @Sabine

    No one I know has any interest in MH370. My PhD room mate has declared the observables to be non-deterministic (after ten minutes of examination). My colleagues came to the same conclusion in about the same time frame. Basically, we are being very dumb in assuming a search area (my conclusion early on). The IG, the DSTG, … are all very foolish. I would characterize them as “dumbshits’, but I am too polite.

  149. Sfojimbo says:

    @Sabine and the rest

    The reason he wanted to avoid Indonesian airspace is the fact that his intent was to make 9M-MRO disappear without a trace, which would have been much more dramatic that the way things did work out. He would have pulled it off except for the Inmarsat checkin which he obviously didn’t know about.

    Also, everyone please keep in mind that Indonesia never said they hadn’t seen MH-370, what they said was that they hadn’t seen MH-370 over their territory. Indonesia and Thailand are both sitting on their radar information they both have it but neither has released it. Thailand has out and out stated that they saw the flight as it approached Koto Bharu. I assume it is diplomatic courtesy to not tell the world what your radar has seen across somebody else’s border – unless asked. And we can be sure Malaysia never asked for that.

    ZS had been a Malaysian AF pilot early in his flying career, he knew the state of Malaysian radar and he knew that a radar track of his flight would never be reported to the world by Malaysia and he was exactly correct, the Lido image is as phony as a three dollar bill. It is a composite of several images and it was probably edited from there.

    This brings up another point, the 60 minutes program’s assertion that there was a “wing dip” is bogus and can be nothing but bogus – Malaysia has never released the radar tapes from Pulau Penang that night. And in any event, something as slight as a wing dip could not be discerned by primary radar.

    ZS knew what he was doing, he knew that his flight over the Malay / Thai border would never be made public. And he knew better than to enter Indonesian airspace.

  150. Andrew says:

    @Neville Macaulife

    RE: “Can anyone document that this is pure hokum? Because if it is not, something is pretty wrong somewhere.”

    See the following discussion about the SR111 episode of CBC’s The Fifth Estate:
    CBC Fifth Estate owes SR111 families an apology

    Juby’s claims are debunked in the last part of the first post.

  151. Mick Gilbert says:

    Returning to the Kota Bharu primary radar discussion, specifically the accuracy of the contradictory altitude data derived from military radar I have been reading Height Finding and 3D Radar by David J. Murrow. It contains a wealth of information on the history of radar height finding techniques together with a discussion of contemporary 3D radar. Discussing the Martello S731 military 3D radar the author states, ‘A height accuracy of 1,000 ft on a small fighter aircraft at 100 nmi is claimed by the radar manufacturer.‘ The S731 was developed in the late 1970s. It has quite a large array (10.6 m high by 6.1 m wide) for a transportable radar and uses an eight beam stack for height finding. Murrow goes on to discuss the Martello S273 (sic) but I’m sure that he means the S723. The S723 was a mid-80s development of the S713 with a shorter but wider array (7.1 m by 12.2 m) and utilises a six beam stack. The S713 in service with the Royal Air Force was referred to as the Type 91. The quoted height accuracy figure for small fighter aircraft at 100 nm is 1,700 feet. The next development iteration produced the Martello S743, which represented a major update to the S713/723 system. While the S743 uses a different technique for determining target altitude to the S723 it is unlikely that it would be significantly less accurate.

    The Royal Malaysian Air Force acquired two S743s in 1992; one of them was operated by No 321 Radar Squadron located at Gong Kedak (Bukit Puteri, Jertih, Terengannu), some 25 nm south of Kota Bharu.

    The Martello series air defence/surveillance radars were manufactured by Alenia Marconi Systems. Alenia Marconi is now part of Leonardo S.p.A., which is also the parent company of SELEX Sistemi Integrati S.p.A. SELEX are the manufacturers of the RAT-31 series of 3D air defence/surveillance radars. It is worth noting that development of the S7x3 series of radars effectively ceased with the introduction of the RAT-31. The RAT-31 uses a hybrid mix of stacked beams and phase steering known as Multiple Simultaneous Pencil Beams. The manufacturer states that each beam provides monopulse altitude measurements with excellent accuracy.

    The RAT-31 is currently a core component of the NATO Air Defense Ground Environment (NADGE) chain having been purchased by the Czech Republic, France, Germany, Greece, Hungary, Italy, Spain, Turkey, United Kingdom and most recently Poland. The RMAF acquired a RAT-31DL in 2009. It is the radar operated by No 310 Radar Squadron located at Western Hill (Northeast Penang Island District, Penang).

    While I have not been able to find any specific altitude accuracy figures for the RAT-31 it is difficult to envisage that it would be worse than the radar it supercedes. Based on that information and the fact that a B777 (wingspan 61 m) presents as a much larger target than a ‘small fighter aircraft’ (wingspan 10-13 m) I would expect that the military radar data showing the target at between 31,100 and
    33,000 ft approaching Kota Bharu and at 32,800 ft travelling away from KB should be good to at least ±1,700 feet and probably better than that.

  152. Mick Gilbert says:

    One further observation regarding the performance of the Kota Bharu primary radar (Alenia Marconi ATCR-33S); once a target is acquired it seems to be tracked continuously and consistently until loss of contact, high elevation range/azimuth errors notwithstanding. There is no evidence of fade-in on acquisition, there is essentially no evidence of dropped returns during each of the two traces (there may be one dropped return between 17:31:26.00
    – 17:31:38.00
    and another between 17:42:08.00
    – 17:42:14.00) and there is no evidence of fade-out at loss of contact. The traces either side of the cone of silence are clean and solid.

    That might seem to be self-evident but it makes for an interesting comparison with the Lido radar data. The lacuna aside, each of the two segments on the Lido slide show a lot of dropped returns, probably in the order of 50 per cent, perhaps more. If the KB radar data is representative of what a civilian-grade ATC PSR can achieve in terms of acquisition and tracking of an approaching and then receding target from and to the edges of its expected range, it is difficult to reconcile the Lido data with the military-grade air defence/surveillance radar at Western Hill tracking a receding target from around one quarter of out to the edge of its instrumented range.

    I guess one question that needs to be asked (or re-asked) is, could Lido be the product of a sea-based radar and could the lacuna be the CoS? I suspect not but it’s probably worth a thought. If the radar was roughly in the centre of the lacuna the acquisition and loss of contact ranges roughly match that of the Thales Netherlands (Signaal) DA-08 air search radar fitted to the Royal Malaysian Navy’s Lekiu-class frigates and Kasturi-class corvettes. However, if that were the case the CoS seems unrealistically large and the quality of the data unrealistically poor.

  153. Victor Iannello says:

    @Mick Gilbert said: I would expect that the military radar data showing the target at between 31,100 and 33,000 ft approaching Kota Bharu and at 32,800 ft travelling away from KB should be good to at least ±1,700 feet and probably better than that.

    And yet, according the RMP report, the military radar placed the MH370 targets around Penang Island at FL447.

  154. Gysbreght says:

    @Mick Gilbert:
    Thank you for the extensive research regarding accuracy of military radar Height Finding. In the next comment on the Kota Bharu primary radar data on MH370 you say: “there is essentially no evidence of dropped returns during each of the two traces (there may be one dropped return between 17:31:26.00 – 17:31:38.00 and another between 17:42:08.00 – 17:42:14.00) “

    You may wish to consider the following chart. I prepared it because I was interested in whether the timestamps in Mike Exner’s table had been rounded or truncated (I suspect both, at different stages of transcription). The red squares indicate the increment between successive time stamps (time 17:31:32 corrected to 17:31:34). The blue triangles indicate (on the right-hand scale) the difference between those timestamps an a ‘reconstructed’ timescale based on antenna rate of rotation 15.72 rpm and azimuth phase shift assuming the antenna rotates clockwise. On that basis I concluded that there was no dropped return between 17:42:08.00 – 17:42:14.00, but that some values were shifted by 1 second, and at 17:42: 14 by 2 seconds.

    https://www.dropbox.com/s/t3ucq1xrqa3vo7s/ErrorsRoundng8.pdf?dl=0

    P.S. What is the Height Finding accuracy of the RMP radars?

  155. Gysbreght says:

    I’ve edited the formating of the chart, without changing the substance. I hope it is clearer in the new presentation.

  156. Victor Iannello says:

    @DennisW: It’s time to adjust the levels of your medications. Go visit your mental health professional. Seriously.

  157. TBill says:

    @Victor
    If OI does not find aircraft, seems like 2 things we need to know:
    (1) French analysis of flaperon or some summary
    (2) Radar summary (Indonesia, S’pore) summary…we need to know a summary of areas ruled out or ruled in by radar or lack of radar.

    In industry sometimes if there is a patent dispute they hire a trusted retired-expert to review both sides secret info and inform who is right and wrong, without disclosing the secrets.

  158. Victor Iannello says:

    @TBill: The list is much longer than that.

  159. Don Thompson says:

    @Mick Gilbert,

    Concerning the RAT-31DL and the Martello. Both are stated to have a max range in the order of 250NM, and a max altitude for detection in the order of 100k/ft. The RAT-31DL feature of ‘Multiple Shaped Pencil Beams’ is explained as four simultaneous beams, the Martello is similar. Consider the geometry of those 4 beams to detect a target some altitude at 250NM range: the beam elevations must be low and the beam forming must subtend a very narrow angle. To detect a target at 165nm over a range of altitudes, the beam elevations must be quite different and requires a wider beam to be formed while at, say, 80nm the goemetry must change again.

  160. airlandseaman says:

    @Mick Gilbert: Thanks for the military radar report. I cannot explain why the Western Hill military radar, over 150 nm away, derived altitudes near KB are what they are (way too low). But note the Civil PSR data from KB radar comprise observations at a much closer range (<10nm). In general, the closer the target is to the PSR, the more accurate the observations.

    I will publish details soon, but in the meantime, I want to report that I have refined the January 2018 KB calibration flight analysis. After correcting one assumption in the prior calibration analysis (thanks Victor for your catch), the 2 minute average PSR speeds match the ADS-B track and speeds perfectly. In particular, the COS speeds are within 2 kts of the ADS-B speeds at 06:41. The azimuth bias error is +0.70 deg. The measured range error is <0.2nm over 90% of the flight.

    Having validated the math model and the radar accuracy using the Jan 2018 data, I returned to the MH370 data. After correcting for the calibrated azimuth error, the path is rotated -0.7 degrees about the radar antenna location, but the speeds are virtually identical. This means the original estimated altitude I reported on April 13 (43,500 feet) was essentially correct. I ran a sensitivity analysis to see how sensitive the altitude is to the range error assumption and found that the altitude for the case of 0 range error is about 46,000 feet. For -0.3 nm, it is 43500 feet. For +0.3 nm, it was 49,000 feet. While some judgement is involved in picking the altitude from a family of possible altitudes, that judgement adds at most ±2000 feet uncertainty.

    In summary, MH370 was certainly not at any altitude below 40,000 feet, as reported in the FI. It is very obvious from the data that anything below 40,000 feet was not even a remote possibility. Given the estimated performance limits, I think it is very likely that MH370 passed KB at the highest altitude 9M-MRO was capable of given the conditions at that time. It might have been slightly lower than what the radar indicated, but definitely not 31000-33000 feet.

  161. Neville Macaulife says:

    @Andrew

    Andrew, thanks for sending me http://forums.swissair111.org/eve/forums/a/tpc/f/322103945/m/9762938617, which clearly indicates that Thomas Juby’s claims of a Swissair investigation cover-up is baseless. I intend to removed my comment because it seems to be off-topic, but haven’t found a way to do it so far.

    Ciruss’s article was an eye-opener. I’m an electrical engineer who knows nothing about flight systems, but when I read that “2,000 pounds of {entertainment} electronics” prone to overheating, and with no proper shut-off, was placed on the same bus as the aircraft’s instrumentation, all I can think is “Wow!”

  162. Victor Iannello says:

    @Neville Macaulife: Rather than submitting a nonsense URL, please leave this optional field blank.

  163. Mick Gilbert says:

    @Victor Iannello

    Re: ‘And yet, according the RMP report, the military radar placed the MH370 targets around Penang Island at FL447.

    Yes, with a ground speed of 578 knots. Coming around the southern end of Penang Island the target was within 15-18 nm of the radar head, at a high elevation and with a rapidly changing azimuth relative to the radar head. Have we learned anything recently about the accuracy of primary radar when the target is close to the radar head, at a high elevation and with a rapid changes in azimuth?

  164. Victor Iannello says:

    @Mick Gilbert: If the distance is 18 NM from the radar head, and the altitude is 36,000 ft, that’s an elevation angle of 19 deg. So if you believe that elevation angle is too high for an accurate measurement of altitude, you’ll have to explain what elevation angles are better. And rate of change in azimuth should have little effect on altitude measurements.

    I don’t know what “we” have learned, but what I’ve learned is measured speeds over short distances are very inaccurate, and measured altitudes should be viewed with great caution.

  165. Andrew says:

    @Neville Macaulife

    RE: ‘Ciruss’s article was an eye-opener. I’m an electrical engineer who knows nothing about flight systems, but when I read that “2,000 pounds of {entertainment} electronics” prone to overheating, and with no proper shut-off, was placed on the same bus as the aircraft’s instrumentation, all I can think is “Wow!”’

    Section 2.14 (pp.227-230) of the TSB’s accident investigation report into SR111 (available here ) has more information on the integration of the IFE with the accident aircraft’s power supply and the management of that project. ‘Wow’ indeed!

  166. Sfojimbo says:

    Would someone help me out and give the location of the “Kota Bharu” radar antenna?

    All I can find is what looks like a tower (secondary) radar at the Sultan Ismail Petra Airport (6° 9’49.20″N 102°17’36.95″E), this radar probably only has a 20 mile or so range.

    Not far south of Koto Bharu is the Gong Kedak radar which is on a hill and about 560ft high (5°47’8.82″N 102°30’16.66″E) I would like to see the returns from that set, but it’s military, so unavailable I assume.

    I assume everyone knows where the Pulau Penang antenna is (5°25’28.70″N 100°15’2.89″E) here it is apparently called Western Hill.

    I haven’t seen any mention of the Thai radar at Hat Yai (6°50’38.02″N 100°25’11.64″E). That looks like a powerful radar on a high hill, but the Thai’s are withholding data from that radar set.

  167. Mick Gilbert says:

    @Sfojimbo

    The radar you’ve identified near 6° 9’49.20″N 102°17’36.95″E is the Kota Bharu air traffic control radar. It’s an Alenia Marconi ATCR-33S with an ALE-9 secondary surveillance radar antenna mounted atop a G-33 primary radar antenna. It is the radar that we’ve been refering to as the Kota Bharu radar.

  168. Sfojimbo says:

    @Mark Gilbert

    That can’t be the radar that is credited with tracking 9M-MRO from the SCS across the peninsula and almost to Pulau Perak: https://www.dropbox.com/s/ax1qnc057byrb3i/20180412102347-32044-map.kmz?dl=0

    That’s a primary radar track. Do you know where that antenna is?

  169. TBill says:

    @Sfojimbo
    If you go to the prior blog article, Victor discusses the civil radar data at KB and then Butterworth, and there is a spreadsheet link with the data summary by Victor.

  170. Sfojimbo says:

    That’s what has me puzzled.

    Victor said in his blog “The data begins at 17:30:33 when the civilian radar installation at Kota Bharu Airport (WMKC) detected MH370 traveling back towards the Malay peninsula about 58 NM from shore.”

    Mick Gilbert says that’s a secondary radar and I agree, you can get a glimpse of the antenna at the Wikipedia page for Sultan Ismail Petra Airport, that’s not what tracked 9M-MRO, 9M-MRO had its transponder turned off. The track provided in the KMZ file I linked to had to have been a primary radar.

  171. Sfojimbo says:

    Upon a second read I see where Mick Gilbert said that the ATC radar is mounted above a G-33 primary radar. I can’t find any primary radar antenna at Sultan Ismail Petra Airport. Primary radars are fairly large and they usually have domes.

  172. Andrew says:

    @Sfojimbo

    @Mick Gilbert said: “It’s an Alenia Marconi ATCR-33S with an ALE-9 secondary surveillance radar antenna mounted atop a G-33 primary radar antenna.

    The Malaysian AIP also states:

    “In the Kuala Lumpur and Kota Kinabalu FIRs, radar services are provided using the following civil/military ATC Radars:

    …g) A 60 NM Terminal Primary Approach Radar co-mounted with a 200 NM monopulse SSR located to the south of Kota Bharu-Sultan Ismail Petra Airport runway.”

  173. Sfojimbo says:

    And the SSR radar at 6° 9’49.20″N 102°17’36.95″E is indeed south of the runway.

    If one were to go to the Wikipedia page for Sultan Ismail Petra Airport and zoom in on the picture with the two 777s you can see that radar. There’s no primary radar on that tower (red & white stripes).

    The airport is almost in the mudflats at sea level. That’s not a good place to locate a Primary radar, and if you did locate a primary radar at sea level you would want a big tower for it.

    I am going through this because I’m so distrustful of any information that comes out of Malaysia.

  174. Mick Gilbert says:

    @Sfojimbo

    As TBill has alluded to, the Kota Bharu radar tracked MH370 between 1730 – 1744 UTC (with a gap between 1736:43 – 1738:55). MH370 was subsequently tracked by the PSR at Butterworth air force base between around 1746 – 1801 UTC. The Butterworth PSR is a NEC ASR located at 5° 28′ 19.7″ N, 100° 23′ 40.7″ E.

    As to PSR antennas generally being fairly large and generally having domes, well it depends and not always. ATC PSR antennas aren’t necessarily that large; the G-33 is about 2.5 m by 8 m. The RMAF military air surveillance radars at Gong Kedak and Western Hill are domed as are the Royal Thai Air Force radars at Ko Samui, Khok Muang and Phuket; on the other hand the Indonesian air force air surveillance radars at Sabang, Lhokseumawe and Sibolga are not domed.

  175. Mick Gilbert says:

    @Sfojimbo

    This might be useful. Go to page 5. Now go to 6.1626486, 102.2935580 on Google Earth, use Street View, look north and see if there’s anything you recognise.

  176. Sfojimbo says:

    Thanks for all the response, I really appreciate it.

    But there’s nothing on that tower at Sultan Ismail that’s anywhere near 8 meters long.

    At Butterworth the only view I have is from Google Earth, but again there’s nothing 8 meters across.

    I remain skeptical of the radar tracks. The Malaysians tried to pass off the Lido image as being from Butterworth too. I don’t buy that at all.

  177. Sfojimbo says:

    @Mick

    Clever! But trees are in the way, I think the Wikipedia picture is just as good. Nothing close to 8 meters there.

  178. Sfojimbo says:

    OMG I backed up down the road and I can see the antenna rotate as the Google car drove down the road.

    Maybe it is 24 feet across.

  179. Mick Gilbert says:

    @Sfojimbo

    And there you have it. That ALE-9 SSR antenna on top is a bit over 8 metres long. That’s a G-33 PSR antenna underneath.

  180. Sfojimbo says:

    Well I beat that one to death.

  181. DennisW says:

    @VictorI

    and measured altitudes should be viewed with great caution.

    There was no “measured” altitude in the case of the KB radar. Altitude is inferred from other observables.

  182. Victor Iannello says:

    @DennisW: That comment was in reference to the altitudes measured by military radar and reported in the FI and the RMP report.

  183. ventus45 says:

    On the noble art of – ditching.

    First – a few ditching events.
    https://www.barnstormers.com/eFLYER/2009/053-eFLYER-FA01-FlatWater.html

    Now the analytical stuff – very heavy going – very heavy – some of it.

    In short – it (ditching) is not “clear cut” – one size fits all kind of event.
    It is complicated by many factors, particularly suction – caused by the shape of the rear under surfaces of the aircraft.

    To quote from http://citeseerx.ist.psu.edu/viewdoc/download?doi=10.1.1.428.2743&rep=rep1&type=pdf
    Page 140 section 5.2.3 – Problematic of the impact of an aircraft on water involving a forward velocity.
    “It is shown that in some ditching cases a rapid increase in the attitude of the aircraft at the moment of first impact is observed. It is also noticed that this behavior depends on the shape of the airframe at the point of impact and on the impact velocity. This phenomenon is due to suction forces caused by an acceleration of the water flow around the impacted area followed by a drop of the water pressure according to the Bernouilli’s theorem. If the pressure decreases under the static pressure, cavitation occurs. Based on different test observations, these forces can be high enough to change the attitude of the aircraft and therefore its kinematics. In the following, the importance of the suction forces are evaluated by investigating a test case reported in [108].”
    [108] McBride E.E. and Fisher L.J. ‘Experimental investigation of the effect of rear-fuselage shape on ditching behavior, NACA Technical Note 2929, Washington, April 1953
    Available at: https://ntrs.nasa.gov/archive/nasa/casi.ntrs.nasa.gov/19930083664.pdf

    Other references dealing with the suction phenomenon.

    https://www.sciencedirect.com/science/article/pii/S1000936112000325

    http://elib.dlr.de/92192/1/p1325.pdf

    http://journal.multiphysics.org/index.php/IJM/article/download/291/283

    https://fenix.tecnico.ulisboa.pt/downloadFile/563345090413563/dissertacao.pdf

    https://fenix.tecnico.ulisboa.pt/downloadFile/1126295043834233/MSc_Joel_Gomes.pdf

    https://researchbank.rmit.edu.au/eserv/rmit:6137/Shah.pdf

    http://citeseerx.ist.psu.edu/viewdoc/download?doi=10.1.1.428.2743&rep=rep1&type=pdf

    https://www.researchgate.net/profile/Qiulin_Qu3/publication/273188238_Study_of_Ditching_Characteristics_of_Transport_Aircraft_by_Global_Moving_Mesh_Method/links/5622624208aea35f2681caff/Study-of-Ditching-Characteristics-of-Transport-Aircraft-by-Global-Moving-Mesh-Method.pdf

    http://www.tc.faa.gov/its/worldpac/techrpt/tc17-52.pdf
    (contains extensive list of ditch events)

    http://articles.adsabs.harvard.edu/cgi-bin/nph-iarticle_query?2014ESASP.727E.202P&data_type=PDF_HIGH&whole_paper=YES&type=PRINTER&filetype=.pdf

    https://trimis.ec.europa.eu/sites/default/files/project/documents/9846/final1-technical-final.pdf

    3:15am. Time for bed.

  184. TBill says:

    @ALSM
    Do you have any idea how/when we get down from FL430?

    I am doing some FS9 checking on the view out the cockpit window (trying to match the bank of the Penang curve). Tentatively I find hard to see Penang from the pilot seat, much easier from co-pilot seat. Pitch down may help.

    Checking Simon Hardy comments on Penang flyover view, which some have said is incorrect.

  185. airlandseaman says:

    TBill: The BU radar data was too sparse to provide a good altitude signal. MH370 passed 5nm south of the Penang southern shore. The GS at that time was about 520 kts. The turn was very gradual starting on a heading of 242 deg and rounding out at about 292 deg. So, a 50 degree turn to the right with a about a 33 nm radius.

  186. David says:

    @TBill. 520 knots, 33nm radius = 6.8 deg of bank.

  187. Mick Gilbert says:

    @TBill
    @David

    In other words he needed about an extra 15° of bank to be able to see Penang.

  188. TBill says:

    @ALSM
    @David
    Thank you. I will do some more work, but basically I’d say if the objective is to see Penang out the window, you are probably sitting in the co-pilot seat.

    FS9 does not give me a co-pilot view, but I can look out that window from the pilot’s seat and see tip of Penang. If I fly the reverse flight path, I can look out the pilot side window to simulate what the co-pilot seat could see, and Penang is right there. Not sure yet how much 6.8% bank helps but it helps (I use 5% as an A/P bank choice).

  189. ventus45 says:

    @TBill

    I think you are aware of my theory – via Medan.

    I have never been happy with the Penang radar story from the earliest days. In fact, I raised some questions about that from the DSTG’s report, on Auntypru, on 17th December 2015, (http://www.auntypru.com/forum/showthread.php?tid=28&pid=3067#pid3067) which no one responded to, or offered an opinion on, at the time.

    The one thing that struck me about Simon Hardy’s “sentimental” last look at home idea, was that it was a “very British” sentiment. After all, Z would have had a full view of Penang out the front windows as he approached. The man was “on a mission”. Why would he need a lingering last look ? It does not “fit” any “sensible plan”, meticulous or otherwise.

    But if you drop “sentimentality’, from the equation, and if you accept that he was in fact heading for Ache, then there is no reason for him to have flown anywhere near Penang in the first place. He could have, should have, turned off B219 at ITBAR and flown direct VAMPI. That would have been both shorter, and faster, AND would have given him an even better view of Butterworth out his left window (see below).

    Therefore, there had to be a really important reason, a vital reason, for him to fly all those unnecessary “extra track miles” to go via south of Penang.

    Perhaps the real reason, was to be able to observe RMAF Butterworth as long as possible (he must have been worried about the possibility of an attempt to intercept him), to see if it was “lit up”, or worse, suddenly “lit up”, ie became active, or remained dark, ie inactive.

    If either it was lit, or lit-up, he knew that even though he was already at FL400 plus, and 500 knots plus, the Hornets could catch him even before he got to Vampi. Continuing to Vampi and beyond would no longer be possible, no longer a viable option.

    If he thought that an intercept was happening, or was “in the works”, his Escape Plan B, was to get “into” Indonesian Airspace ASAP, ie, he had to head for Gunip and Gotla ASAP, which he could do, he could outrun them to there, and be “safe”, since the Hornets could not enter Indonesian Airspace, and even if they did, they sure as hell could not shoot him down there. If on the other hand, he had passed north abeam Butterworth heading for Vampi, and if an interception was happening, he would have had twice as far to fly to get into Indonesian airspace, and the Hornets might just catch him beforehand.

    There are many reasons why I have never accepted the Ache scenario, and I think he did go via Gunip / Gotla, and I think the DSTG plots have many inconsistencies that support my view that the track post “the Hamid phone gap” was fabricated to hide the fact that he did in fact overfly Sumartra from the Indonesians.

    Perhaps Z really wanted

  190. airlandseaman says:

    Mick/David/TBill: The notion that MH370 headed to and rounded Penang to take a look is baseless. We do not know why the path used was chosen. Only speculation.

    The new radar data does not tell us the altitude at Penang, but the data does tell us several other things. It indicates the altitude at KB was very high, probably 43,000 feet. It tells us there was a climbing left turn after IGARI gaining ~6000 ft and slowing to 430 kts TAS in the process. It tells us the plane accelerated to TAS=500 kts (GS=530 kts) over the next 7 minutes. It tells us the path was not as smooth and straight as the public documents suggest. In fact, the 4 second samples makes it very obvious (to me) that the plane was being flown by hand all the way from 17:30 to 18:01, which means it was probably flown by hand from 17:21 to 18:01. At what point the AP was re-engaged (if ever) is unknown. But it certainly looks like the AP was not used between 17:30 and 18:01.

  191. ventus45 says:

    Perhaps Z also wanted to embarrass the RMAF as a final tribute.

  192. ST says:

    @TBill – The previous blog under Civilian radar analysis was closed before I could respond to say thanks for the inputs on where you were with the simulator testing. I continue to read your posts and updates in this blog.

  193. TBill says:

    @ventus45
    I accept your idea to keep Butterworth in sight. In that case I am sitting in co-pilot seat and should have good view out that way. If there was depressurization at IGARI, not sure I care too much about getting shot down by Malaysia, so I am heading straight to VAMPI. Or maybe getting shot down by Malaysia was Plan A. Hish was not about fall for that, I know.

    Are binoculars handy on aircraft?

  194. Sfojimbo says:

    Remember the facts that Malaysia allowed the search in the SCS to continue for up to a week before it was called off – even though it was later revealed that Pulau Penang radar had tracked the flight across the peninsula to the Malacca strait and past MEKAR? It’s obvious to me that the fact that 9M-MRO had crossed the peninsula was revealed only after it became apparent to Hishammuddin and Razak that the the Inmarsat data wasn’t going to be ignored; so they had to fess up about what they had seen that night. I believe the Inmarsat data tripped up Hishammuddin and Razak as much as it did ZS.

    Now four years later, that colossal discrepancy has been forgotten about and and not factored into evolving theories about the flight. Why were Hishammuddin and Razak trying to hide something that was so likely impossible to be kept hidden?

    IMO, ZS was trying to bring down the government; he didn’t think the government could survive something the disappearance of MH-370 would uncover. But somehow Hishammuddin and Razak kept that thing from seeing the light of day in spite of their early stumble (focusing on the SCS). I appreciate all the dogged engineering work that has been done here and elsewhere, but I can’t help but think that this has become a case of not seeing the forest for all the trees.

    How much faith do you guys have that these radar tracks from Sultan Ismail Petra and Butterworth airports are the real thing?

  195. TBill says:

    @Sfojimbo
    My personal perception would be that USA and others forced Razak to give the truth as the data showed. I feel like Razak handled it quite well as far as being forthcoming about “apparent intentional diversion”.

  196. David says:

    Having watched the Senate Rural and Regional Affairs Committee concluded an hour and a half ago,some points. Please excuse some randomness:
    • The conclusion was warm with the Committee chairman thanking Peter Foley and Greg Hood for the work done. There were no sour notes,implied criticisms or allegations.
    • The Chairman concluded by addressing the speculation that there would be an inquiry into the ATSB work by indicating that was not in mind but that some committee members might appreciate a more detailed briefing by Foley. The ATSB agreed willingly to do that, presumably informally, as sought.
    • Foley described the final descents as emanating from a small amount of rudder trim left on after TAC removal on left engine fuel exhaustion. He added that Boeing had indicated the final descents were in a less stable phugoid. A descent like that he said is not what a pilot would choose.
    • He described an additional 45 nm search to the east in response to Simon Hardy’s input, to 70nm. About Byron Bailey’s claim that pilots had not been consulted and believed there would have been a controlled ditching, he said that pilots were deeply involved via Boeing’s advice and that the AAIB and NTSB participation included pilots. He said the Bailey claim of a “pilot controlled ditch” was unsubstantiated. He had read the Larry Vance book which V had forwarded to him last week and about the Swiss flight 111 investigation. I do not remember a substantive comment on what V had said.
    • He repeated the ATSB thesis that the flap (and implicitly flaperon) were not deployed when it separated from the aircraft which eliminated the potential the aircraft was controlled or likely to be in a ditching (personally I think that it would be more precise to say that the flap and flaperon collided when not deployed).
    • The descent rates indicated it was uncontrolled and it was not a descent a pilot would choose.
    • Question: should the investigation have been conducted differently? Answer: a lot was disclosed during it, altering it as it went along. F. made clear that such as whether the fuel taken on was appropriate was up to the investigators. The ATSB worked with the quantity that was there.
    • ATSB analyst who went to France for the flaperon investigation was unable to do anything active, being constrained by their judicial system. The counsel assisting the French inquiry was particularly slow. However BEA was particularly helpful re AF 447.
    • He noted that flaperon trailing edge missing could have been from causes such as engine separation.
    • The ATSB had not speculated on the speed of impact. Boeing modelling continued to impact but the initial altitude was unknown, though Boeing did model “a couple” that the ATSB sought. He added later that the recovered parts indicated a “significant impact” and if there was a controlled ending it had not been very successful.
    • He made clear that the lack of simulation of engine restarts placed “lots of caveats” on the Boeing descent simulations.
    • Having a pilot involved at the beginning and not the end was a problem for many he said. At one point my memory is that Geoff Hood intervened to mention that “control inputs were made”, presumably to include the possibility that there might be other than pilots involved.
    • At the end F. offered his/their plausible explanation-with-conjecture, that what might have happened was pilot hypoxia or decompression after flying depressurised including at 40,000 ft, depressurisation continuing for an hour or so. He referred to the March 1998 DC8 incident discussed on this blog in which the aircraft was unable to pressurise, the captain persisting in climbing and passing out, the co-pilot taking over, the aircraft reaching FL330. The captain in that and in MH370 were of similar age he said and both overweight.
    • There was a question about whether the co-pilot leaving his ‘phone on was common. Hood indicate it happened and was discouraged because of the distraction.
    • The Chairman described Foley as an impressive witness after what must have been an arduous 4 years. He continues as JACC technical officer.

  197. David says:

    After, ” ..Regional Affairs” first line please add, “and Transport Committee hearing into the ATSB’s underwater search for MH370, concluded…”

  198. Mick Gilbert says:

    @David

    Thank you for the summary, David. You’ve hit all the important points. The Hansard (official) transcript should be available on the Senate website tomorrow with a bit of luck. I’m sure that at least one erstwhile contributor and one current New Corps reporter will have been disappointed with the proceedings.

    Victor and Mike, the good news is that you won’t need to pull out that shoe box of travel receipts.

  199. Sfojimbo says:

    @TBill
    On March 8th at 00:35 MH-370 departed KL, it was tracked by Pulau Penang radar at least by 1:22 when it arrived over IGARI; the radar tapes were viewed by a VIP from Malaysian military later that morning.

    March 12: It is assumed by most, that Malaysia was notified of the Inmarsat data on March 12.

    March 13: Andy Pasztor’s WSJ article told the world that MH-370 kept going for “5 hours after turnaround based on satellite data.” Later that day, Hishammuddin denied the WSJ report.

    March 14: Hishammuddin told journalists the report was “inaccurate”. “I would like to refer to news reports suggesting that the aircraft may have continued flying for some time after the last contact, as Malaysia Airlines will confirm shortly, those reports are inaccurate.”

    On the 15th he threw in the towel.

    March 15, Hishammuddin:
    “based on new satellite communication we can say with a high degree of certainty that the Aircraft Communications Addressing and Reporting System was disabled just before the aircraft reached the east coast of peninsular Malaysia. Shortly afterwards, near the border between Malaysian and Vietnamese air traffic control, the aircraft’s transponder was switched off.”

    “The primary radar target, so far believed but not confirmed to be MH-370, could today be identified as MH-370 with the help of new data received from a satellite data provider.”

    “Due to the type of satellite data we are unable to confirm the precise location of the plane…………………….”

    Later on March 15th the SCS search was cancelled. This is eight days after the Pulau Penang radar data had been known.

    It’s true that both Britain and the US knew from day one that MH-370 had rounded the Sumatra peninsula (based on their ship and aircraft movements), I have no reason to think it was anything except Andy Pasztor’s WSJ article that pulled the rug out from under Hishammuddin, although I suppose it could have been somebody from the US government that tipped Pasztor off.

    Neither Hishammuddin nor Razak were forthcoming.

  200. Don Thompson says:

    @sfojimbo

    Air Accident Investigation Branch (AAIB-UK), is automatically an accredited representative to the Malaysian investigation by virtue of the origin of 9M-MRO’s engines.

    Inmarsat and AAIB-UK were relaying to the responsible authorities in Malaysia, or at least attempting to relay, the information contained in the satcom exchanges on Tuesday 11th March.

    Paul Marks had posted an article at the New Scientist on 13th March (UK Time). Marks’s article was accurate in its description of the purpose and significance of the ‘engine’ data.

    There are quite a few reasons that lead to the realisation that Pasztor’s article was simply a ‘join-the-dots’ effort, scraped from a number of sources such as Mark’s article.

  201. Richard Godfrey says:

    SC is continuing to make good progress up the Broken Ridge plateau area and has reached 26.6541°S.

    Ocean Infinity has completed another cycle of AUV deployment, with 7 AUVs in each cycle.

    The weather is likely to deteriorate, there is currently good visibility, with a 10 knot wind, a combined swell and wave height of 2.6 m. There is a tropical storm S.W. of the search area, moving in their direction, which will bring strong winds up to 30 knots by tomorrow evening.

    https://www.dropbox.com/s/nxac3dezy31rwgt/SC%20Track%2022052018.pdf?dl=0

  202. Victor Iannello says:

    @Mick Gilbert said: Victor and Mike, the good news is that you won’t need to pull out that shoe box of travel receipts.

    In my case, that shoebox of receipts from travel and payments involving Australia and Malaysia is quite empty. I’ve never been to either country, and I’ve had zero financial dealings with either country. Anybody that implies anything else is spreading horse manure, as reasonable people understand.

  203. Mick Gilbert says:

    @Victor Iannello

    Re: ‘Anybody that implies anything else is spreading horse manure …’

    Diplomatic to a fault, Victor. I’m sure that I would have been less restrained under the same circumstances.

  204. TBill says:

    @David
    I am not sure I agree with Foley’s logic on all of his interpretation of the evidence, but doesn’t the comment on FL400 hypoxia sound like a “hat tip” to ALSM?

  205. airlandseaman says:

    I don’t know about a hat tip, but I do know Pete found my KB radar analysis interesting and useful. I’m hoping Boeing will confirm.

  206. TBill says:

    @ALSM
    Seems like Foley could be suggesting as much as 42000-ft real altitude which would be about FL400 pressure altitude. I am not aware of any prior suggestion of more than FL350 from the ASTB or MY. I know there have been outside observers who have suggested the higher altitude for a long time.

    Of course, Larry Vance has good timing in his book, or maybe a little late release, because his book could be overtaken fairly quickly by the MY final report if it comes out soon.

  207. Sfojimbo says:

    @Don Thompson
    I had taken the conservative approach in my earlier post, I believe that the Malaysian Government probably knew about the Inmarsat data on March 11, but there is nothing to provide proof of that, so I assumed March 12. The point is that Hishammuddin knew about the Inmarsat data several days before he said “with the help of new data received from a satellite data provider” on the 15th. There may have been some noise surfacing before Pasztor’s article, but the WSJ was the first documented time anyone claimed that the flight had continued on for hours: credit where credit is due.

    In my earlier post, I stated that the SCS search continued on for eight days, this was based on the fact that India continued searching until the 16th, which I did not mention in my post.

    Thanks for the response.

  208. ArthurC says:

    Just curious… how much longer will OI be able to search this season, based on weather, and what latitude could they potentially reach?
    Area 3 seems rather large in size, is there a chance to have it covered before the winter weather sets in?

  209. airlandseaman says:

    I sent my KB Radar analysis to Pete over a month ago. They have had time to look at the analysis now, so it is not surprising (to me) that Pete mentioned FL400 (42200 ft) as possible. Since then, the SQ7838 calibration confirmed the basic technique I used to estimate altitude and the accuracy of the KB radar range and azimuth observations.

    Dennis is correct to say that the technique is not a closed form solution providing a definitive numerical altitude, but even a casual observer can recognize the pattern. It is clear that the altitude was very high at KB (possibly 46,000 feet), and definately higher than a text book estimate of the maximum altitude possible for the B777-200ER.

    Recall too, there were at least 8 military radar observations documented in the RMP Folder#4 between 17:50:59 UTC and 17:56:59 UTC (passing Penang). The observation at 17:50:59 gave an estimated altitude and GS of FL447 and 528 kts. All 8 observations were between FL423 and FL447 and 507 to 528 kts. These military radar obs are very close to what I estimated from the KB Cilvil Radar 5-10 minutes earlier.

  210. DennisW says:

    @ALSM

    I concur that using the KB radar data as given you are lead to a very high altitude. I get numbers at the high end of your range (46,000′ to 48,000′ geometric). I am not sure that I trust the data, which leads me to believe a range calibration error was present at the time the data was taken.

  211. TBill says:

    @Victor
    @all
    Quite fascinating article in the Guardian, summarizing ATSB hypoxia thoughts discussed at the hearing. Sounds like ATSB is giving their reasoning for ghost flight, which we have never heard before. Their reasoning apparently is that no one could survive a depressurzied FL450, even the pilot.

    So the 60 Minutes show has several effects I was hoping for, which is shaking out more hidden info.

    Per ALSM, starting to look like MH370 may have been very high altitude after IGARI as high as 45000-feet. Perhaps it is true that nobody could survive that altitude in a depressurized cabin. This could indicate all were dead, but then we have SDU logon at 18:25, so perhaps the depressure event happened later, would only take a few minutes at FL450, and the descent at 18:40 was to re-pressure…or the other thing is the perp does not need to go to zero pressure, he/she could hold at FL350 cabin and survive that way. Many options.

  212. Gysbreght says:

    @ALSM: FL’s are pressure altitudes. Are the FL’s you mention pressure altitudes or geometric heights?

    What is the average groundspeed in Victor’ Segment D (17:51:23 – 17:51:47 UTC) according to the Butterworth PSR data?

  213. Victor Iannello says:

    @TBill: I don’t follow the hypothetical scenario that Peter Foley is proposing. First, he suggests that it is possible that the captain was incapacitated within minutes of a depressurization. But later, he says that the turns suggest that the captain or somebody else was in control until 18:25, which seems to suggest there was no depressurization. I must be missing something.

  214. airlandseaman says:

    All 8 military observations around Penang were given as Flight Levels (i.e., FL447). There was no distinction made between Flight Levels and altitude in the RMP Folder #4 Report. These values were taken directly from the radar screen shots in Folder #4 at pdf page 25/26. That said, I don’t see how the military radar could directly observe a true flight level. So they may have indicated FL447 on the screen, but for PSR, meaning 44,700 feet (assumed a Pressure of 29.92″). Either way, MH370 was very high and very fast from 17:37 to the end of the BU data at 18:01. And the Lido photo derived speeds are consistent with that.

    Given the high altitude south of Penang, I put more weight on the likelihood that the BFO data at 18:39-18:40 indicated a descent, not a turn. The reason for the high altitude looks more like a way to avoid other traffic (typically below FL410). If the plane was depressurized, it probably happened starting around 17:22, but the climb was not necessarily done to speed hypoxia. I think it was to avaiod traffic while running without lights or TCAS.

  215. Gysbreght says:

    @ALSM: Thanks for answering my first question. How about the second question?

  216. David says:

    @Victor. “(Foley)suggests that it is possible that the captain was incapacitated within minutes of a depressurization. But later, he says that the turns suggest that the captain or somebody else was in control until 18:25, which seems to suggest there was no depressurization. I must be missing something.”

    Me too.

  217. Niels says:

    @ALSM
    Proper interpretation of the 18:39-18:40 BFOs is crucial. I remember a discussion on the implications of the limited scatter in these BFOs in the light of typical descent modes / vertical speed variations. It would be a major step if this point could be brought forward towards a conclusion.

  218. Nederland says:

    Depressurisation could have happened some time after the final major turn (or shortly before), but probably not at IGARI.

  219. Paul Smithson says:

    @ALSM. I understand that you are now more than ever convinced that the altitude inference from the KB data is real. Your first interpretation of the “validation” flight appeared to cast doubt on this and (by your own report) indicated a speed anomaly of +45kts across the COS. I fail to understand how that problem has now disappeared and the PSR data is, after all, declared to be “spot on” for speed, range, altitude and azimuth (albeit with a 0.7 degree shift) and the altitude inference now beyond doubt. Do you plan to publish these workings at some point to persuade the rest of us?

    I would also like to hear from any contributors who have expertise with primary radar signal processing and recording what form such data is typically captured and store in. From what I have read, it appears to be a recording of the CONSOLE representation (effectively 2-D polar coordinates with horizontal range) and not raw range and azimuth. If that is the case, it would render this whole undertaking a fool’s errand unless you know what altitude had been “assumed” in the topographic representation that was recorded.

  220. Victor Iannello says:

    @David: I think what the ATSB was alluding to is a deliberate depressurization, with the person controlling the plane not immediately becoming incapacitated, but becoming ill, and eventually succumbing to the effects of depressurization, and providing no further inputs after a particular point in time (but before 19:41), eventually leading to an uncontrolled descent after fuel exhaustion.

  221. airlandseaman says:

    Gysbreght:
    Assuming 46,000 at 17:51:34 541 kts GS
    Assuming 44,000 at 17:51:34 548 kts GS
    Assuming 42,000 at 17:51:34 554 kts GS
    Assuming 40,000 at 17:51:34 560 kts GS

    Paul: Victor and I both reported that in the beginning, there was a small bug (bad assumption) in our model used to validate the MH370 KB data using SQ7838 data to calibrate. Initially, we were using the true altitude to reverse engineer the Range and Az values from X and Y radar values. Victor noted that the radar would not know the true altitude, and would probably use the FL reported in Mode C as a proxy (i.e., FL400 = 40,000 feet). After making that change, we get a near perfect match between the radar COS speeds calculated using a 2 minute average and the COS ADS-B speeds. So there was no real COS anomaly after all. We just had a bad assumption in the beginning, now solved.

    Before we received the SQ7838 “calibration file”, my MH370 altitude at KB results (~46,000 feet) raised many doubts, due to the B777-200ER altitude limit. So people started looking for alternate explanations. A large negative range error, for example, was proposed to bring the altitude down to about 40,000 feet. But it turns out there is no evidence of a large range error. The most recent calibration gave a range error of only -0.03 nm, and the errors estimated from the KB SQ7838 radar vs. ADS-B comparison suggest range error <0.2 nm. The bottom line is that the KB PSR data indicates an altitude of 43000 to 46000 ft. That is consistent with the military radar estimates at Penang.

  222. Sfojimbo says:

    @ALSM

    I’ve been trying to analyze the radar returns to better understand how much validity I want to assign them and I came across an anomaly; you may have noticed this before but here it is:

    Between the last two “sets” of data points from Butterworth (57:22 and 59:47) there is a blank gap that correlates to about a 22 minute gap in tracking the flight. When you plot this on GE it very much appears that that gap would correspond to where western hill would have interfered with the signal returns from the flight, except it doesn’t quite line up.

    It looks to me like the plot is about fifteen miles ahead of the actual flight location in that region.

    A few other comments:
    The two big dropouts may have been caused by weather, the tower primary radars are also used for weather info if I am right, they might be designed to favor fog banks over targets.

    Much of the jumpiness in the flight track can be attributed to nonlinearities in the azimuth pots on the antenna. Those are low cost radars, they are not intended for aiming weapons etc.

    The first plot (17:30) lines up pretty well with what the range of the KB radar set while the last plot from the Butterworth set plots about 15 miles beyond that radar’s range.
    The range for both of them is 60nm.
    http://aip.dca.gov.my/aip%20pdf/ENR/ENR%201/ENR%201.6/Enr1_6.pdf

  223. Sfojimbo says:

    Not a 22 minute gap, a 22 mile gap.

  224. Gysbreght says:

    @ALSM: Using Victor’s EXCEL file provided in previous post Update 2, I get for Segment D:

    Assuming 46,000 at 17:51:23-17:51:45 656 kts average GS
    Assuming 44,000 at 17:51:23-17:51:45 665 kts average GS
    Assuming 42,000 at 17:51:23-17:51:45 673 kts average GS
    Assuming 40,000 at 17:51:23-17:51:45 681 kts average GS

    You said at 4:21 pm: “These values were taken directly from the radar screen shots in Folder #4 at pdf page 25/26.”

    Those Google Earth screen shots prepared by SKMM (Malaysian Communication and Multimedia Commission) show “FL447 578 kt” between 17:51 and 17:52 UTC. Victor’s KB radiosonde data for 44700 ft height show 27 kt tailwind, FL426, ISA-7°C.

    Groundspeed 578 kt then corresponds to 551 kt TAS, Mach 0.977.

  225. airlandseaman says:

    Sfojimbo: What do you mean by: “… western hill would have interfered with the signal returns…”?

    Are you referring to the WH radar or the hill itself? The plane was well above the local horizon for BU radar. Several of us have published the terrain mask many times. So that was not the reason. People that know the BU radar well tell me it was not in very good shape compared to the more SOTA radar at KB. Frequent drop outs are said to be common at BU.

  226. Mick Gilbert says:

    @airlandseaman

    Mike, correct me if I am wrong but my understanding of the SQ7838 calibration exercise is that you did not receive the same type of data (viz raw PSR slant range and azimuth) for that flight as the original KB radar data. Now, I understand that practical restrictions mean that it is difficult, if not impossible, to obtain the same raw PSR data again and that you have to work with what you can get. However, in the absence of seeing the raw PSR data, the utility of the SQ7838 exercise must be questionable.

  227. Sfojimbo says:

    The hill itself. There may be a clear path for light to travel from antenna to the plane but the radar sends out pulse lobes as you know, and those lobes can and will bounce off any nearby high ground – it’s called ground clutter. That’s why MTI (moving target indicator) had to be invented, all radars are plagued by ground clutter even though their antennas are pointed upward. It’s counter intuitive it affects short ranges, not long ranges as it would seem.

    The proof for me is that the track extends beyond the radar’s range by the same 15 miles that the blind spot for West hill is offset.

  228. Gysbreght says:

    @ALSM: Apologies, I made an error and got the first part wrong. The correct values are close to yours:
    Assuming 46,000 at 17:51:23-17:51:45 546 kts average GS
    Assuming 44,000 at 17:51:23-17:51:45 542 kts average GS
    Assuming 42,000 at 17:51:23-17:51:45 561 kts average GS
    Assuming 40,000 at 17:51:23-17:51:45 568 kts average GS

  229. Sfojimbo says:

    To add to this conundrum, the dropout alongside of Penang is a similar 15 miles. There is no reasonable explanation to be found for that dropout.

    Methinks data was shifted somewhere.

  230. ST says:

    @Victor/ALSM/Andrew – I read statements by different experts and below are things that I feel might align towards a different possibility –
    Is it possible that due to the older age and habits of the pilot impact of de-pressurization was almost immediate and the plane flew per pre-defined path and way points but closer to Penang, the co-pilot recovered from the ill effects of de-pressurization for a short time and turned his cell phone on to provide a last ditch attempt to communicate to the world below on the location of the flight and attempted a descent which might explain his phone signal connecting to cell towers but was overcome by the effects of the hypoxia and the plane continued on auto pilot?

  231. airlandseaman says:

    Sfojimbo: Sorry, but that is not how antennas work, regardless of the antenna type. In general, if a radar has a line of sight view of a target, then it could detect that target. This is independent of phased arrays, beam forming networks, etc.

  232. Sfojimbo says:

    ALSM

    You’re arguing with the voice of experience. I have spent many hundreds of hours looking at a big yellow gob in the middle of my PPI scope and we were on the top of a hill. MTI helped a lot.

    How do you explain the returns that are beyond the radar’s range?

  233. David says:

    @Victor. Re de-pressurisation. Thanks.

    I notice the theory, including an initial climb and depressurisation, endorses deliberate intent.

    Some problems with the theory of decompression sickness are that:
    . surely an experienced, even inexperienced, pilot would be well aware of that risk? Like that of hypoxia from climbing too high?
    . if it did knock him out though I think he would be aware of its onset, as distinct from hypoxia
    . once he became aware one could postulate that he restored power to the left main AC bus to reduce its effects/restore pressurisation or to regain autopilot, but surely dropping altitude at that point would be a first action (BFOs? Fuel cons.?).
    . he then would need the presence of mind to set a course to the south and hope he recovered. That would suggest that step was spontaneous and not what he foresaw in planning.
    . even so that would not guarantee that he would not have held on to the end.
    . the theory relies on depressurisation for an extended period. If he climbed to reduce hypoxia time surely he would be aware that such an extended time would not be needed?

  234. TBill says:

    @Nederland
    “Depressurisation could have happened some time after the final major turn (or shortly before), but probably not at IGARI…”

    do you have an explanation or hypothesis?

  235. airlandseaman says:

    Mick: Re: “However, in the absence of seeing the raw PSR data, the utility of the SQ7838 exercise must be questionable.”

    I do not agree. If you take the time to understand the differences between KB PSR data and KB Mode-C SSR, you find they are not that different for what we need to calibrate. The azimuth error should be nearly identical. I believe the calibration provides a validation of the math, regardless of the any error differences between Mode-C and PSR. So it has helped to validate the altitude estimate.

  236. airlandseaman says:

    Regarding timing…If the PF depressurized the plane, I think it happened early, like 17:22. If that did happen, then I believe the hull was probably re-pressurized at ~18:23.

  237. airlandseaman says:

    Sfojimbo: I have some experience too, as a radar operator, as an antenna designer and manufacturer (1000’s sold). Frequencies from 13.6 khz to 186 GHz. Antennas do not work as you described.

  238. Sfojimbo says:

    ALSM

    Ok you win if you want.

    But how did you explain the returns that were beyond the radar’s range again?

  239. DennisW says:

    @ALSM

    I am completely on board with your conclusions on altitude, but I have a lingering disconnect on speed. The speed is largely dR/dt at distances beyond the CoS. My speed algorithm is slightly different than the ones presented here, and there is no point in going into it except to say that it is fairly smooth and well-behaved. It shows ~470knots (GS) approaching the CoS, and increasing to 490knots at the CoS entry point. At the CoS exit the speed is around 520knots (GS). The average speed through the CoS comes out around 512knots (GS).

    I’ve checked the numbers pretty carefully, and we have a speed disconnect of some 40knots. Something is still not clean.

  240. Mick Gilbert says:

    @airlandseaman

    Re: ‘If you take the time to understand the differences between KB PSR data and KB Mode-C SSR …

    Absent a set of concurrent raw PSR data and SSR data for the same target I fail to see how anyone can understand ‘the differences’. You can’t draw any firm conclusions from the SQ7838 exercise because you don’t know for certain what you are looking at. You’ve got a set of processed position outputs for SQ7838 that may have been calculated from (a) the PSR data together with the SSR altitude, (b) exclusively from the SSR data or (c) a combination of the two. As you don’t know which data sources were used, what the raw data looks like or what the processing algorithms look like you’ve really got no firm handle on what’s the basis for the relatively good processed output.

    Re: ‘I believe the calibration provides a validation of the math …

    That may well be the case but the maths isn’t the issue, it’s the data.

  241. Sfojimbo says:

    I’ve been afraid of this. Here may be the final nail in the coffin for any hope that more information will ever come out of Kuala Lumpur. The new administration, which associates itself with Anwar Ibrahim, will want this hushed up as much as the last administration did; Anwar Ibrahim and ZS were two peas in a pod in March of 2014.

    “Let’s forget about all that”

  242. Nederland says:

    @TBill

    I have put forward this argument before.

    Even if 100% oxygen is supplied, decompression sickness would normally kick in within an hour, making it impossible to operate the aircraft, unless the pilot has breathed 100% oxygen for an hour before exposure.

    https://www.faa.gov/pilots/safety/pilotsafetybrochures/media/dcs.pdf
    https://www.researchgate.net/publication/11034697_Decompression_sickness_latency_as_a_function_of_altitude_to_25000_feet

    This is assuming that the rise in altitude is gentle, not rapid as envisaged in the deliberate depressurisation scenario. In that case the effects are far worse.

    A person aged 40 to 45 already has a chance of suffering altitude sickness three times higher that a normal person, exponentially rising with older age. For smokers, there is an added 5.000 ft of physiological altitude (Zaharie reportedly smoked a cigarette before boarding the plane). Other risk factor are bodyweight, medical preconditions, and stress.

    http://www.pilotfriend.com/aeromed/medical/alt_phys.htm

    On the other hand, there were 238 further people on board, at least some of them had access to portable oxygen bottles, providing oxygen for an hour or two. The primary purpose of these systems is to keep people alive rather than conscious. It is very unlikely that any pilot would have stayed conscious until such time as is sufficient to ensure that everyone else is in an irreversible state of coma. If the aircraft was repressurised in time, some would have regained consciousness. If it wasn’t, then the pilot would have passed out too (I figure it is grossly inconvenient to throw up in an oxygen mask).

    The early flight path around IGARI and up to the tip of Sumatra was obviously deliberate, and not in reaction to an emergency, and Foley acknowledges this (i.e. the plane was hijacked).

    In my view, deliberate depressurisation could have happened because this was part of the plan (in order to commit murder-suicide before crashing in the SIO on autopilot but after the plane was navigated around Sumatra). Or it could have happened in reaction to a passenger insurrection, or the insurrection itself could have caused depressurisation (e.g. sustained attempt to break into the cockpit, causing structural damage to the airframe).

    If, on the other hand, there were pilot inputs at the end of flight, there probably was no depressurisation at any time (in that case there is the question of why there was no passenger insurrection).

  243. Paul Smithson says:

    @ALSM. Thank you for the reply. I had presumed that this was the explanation. I wish someone could try to replicate in a simulator the acceleration 470->530 at various altitudes above FL400 and see what the profiles look like.

  244. Andrew says:

    @all

    According to Reuters, the new Malaysian Transport Minister, Anthony Loke, stated earlier today that the search will not be extended beyond 29 May:

    Malaysia says search for Flight MH370 to end next week

  245. Richard Godfrey says:

    @Andrew

    A cover up to the cover up?

  246. TBill says:

    @Nederland
    OK thank you. Not sure.
    I guess it is possible the pilot put the O2 mask early to acclimate the body, and gradual pressure drop is of course one way to do it. Definitely we have to be open to, whatever happened, might have been a hijacking gone wrong due to PAX action or risky error with depressuring aircraft.

    I am thinking Kalitta 66 hypoxic pilot was heavy smoker and that somehow may have helped him cope with less O2, but now I cannot confirm.

  247. Mick Gilbert says:

    @Richard Godfrey

    Richard, when should SC complete 90 search days? The 28th?

  248. Shadynuk says:

    @Nederlander wrote: “A person aged 40 to 45 already has a chance of suffering altitude sickness three times higher that a normal person…”

    An interesting comment.

  249. Richard Godfrey says:

    @Mick

    Ocean Infinity has the next contract on 1st June 2018.

    I expect that they will leave the search area around 28th May 2018.

  250. Nederland says:

    @Shadynuk,

    Yes, I noticed that could lead to misunderstandig, especially since I would describe myself (age group) as “not normal” 🙂

    May I suggest replacing “normal” with “young”? I haven’t doublechecked that post before submitting.

  251. Nederland says:

    @TBill

    In theory, the pilot might have locked himself into the cockpit close to IGARI, put on the oxygen mask and breathed 100% oxgyen for an hour before depressurising the aircraft. It is still very unclear if he could remain conscious/functional and/or recover after that procedure.

    The pilot in the case quoted by Foley did not recover even at low altitude and had to undergo hospital treatment. He was also a smoker.

    https://www.ntsb.gov/_layouts/ntsb.aviation/brief2.aspx?ev_id=20001206X00986&ntsbno=NYC94LA062&akey=1

  252. TBill says:

    @Nederland
    I am not immediately accepting the correctness of Foley’s justifications for ATSB logic, but it is interesting to hear it.

  253. airlandseaman says:

    A few reply comments here:

    1. Mick Gilbert: Your speculation about possible big differences between PSR and Mode-C are baseless. You provide no data or proof of any significant flaws in the analysis. Pure speculation. The calibration flight provides added confidence in the MH370 KB PSR data analysis. Even with no calibration data, the original MH370 analysis I published April 13th stands on its own as a reasonable KB speed and altitude profile.

    2. Re O2 systems: There are 3 types of O2 systems on the aircraft. The passengers have access to the “drop down” O2 masks with chemical reaction generators. These provide virtually no help at 45,000 feet. Passengers using these masks would be comatose in a few minutes after depressurization. Then you have 6-8 “walk around O2 bottles” scattered around the aircraft with simple masks not much better than the drop down masks. They too would prove little help at 45,000 feet. Finally, they have pressure masks in the cockpit. The pressure masks are good to 45,000 feet and should work fine for an hour if used properly. Above 45,000 feet, you need a pressure suit, like fighter pilots wear. But removing a pressure mask for even 30 seconds at 45,000 feet could lead to hypoxia. So, if the PF donned a pressure mask and depressurized the plane circa 17:22, he could have remained conscious until 18:23, but restricted to remaining in the cockpit wearing a pressure mask, tightly sealed to his face.

  254. Nederland says:

    @airlandseaman

    Passengers were alive on the Helios flight even after a few hours, although probably in irreversible coma.

    Foley’s argument is that, while the pressure masks would supply sufficient oxygen to stay conscious, they do not alleviate symptoms of decompression sickness (unless they have been employed for an hour before exposure). Decompression sickness in a rapid pressure loss at 45,000 feet would render any pilot unable to operate the aircraft within very few minutes, or less, if risk factors are involved (besides age, smoking, body weight, the source above also mentions spinal cord injury).

  255. ventus45 says:

    I think the transcript of the hearing will be available tomorrow.
    It might be best to wait for it.
    I think it is Reference Number 20562, which is still listed as “pending”.
    https://www.aph.gov.au/Parliamentary_Business/Hansard/Estimates_Transcript_Schedule

  256. TBill says:

    @Ventus45
    @all
    Here is set of FS9 screenshots showing simulated views of Penang and Butterworth from MH370.

    https://docs.google.com/document/d/1N8LL-_-K7WDeUIjBbBY7S4TWD33-hxzFx3sILJLE5eE/edit?usp=sharing

    Summary- Simon Hardy suggests MH370 might have banked to see Penang for sentimental reasons. These FS9 views tends to suggest that is a theoretical possibility, but from the vantage point of the co-pilot seat. @Ventus45’s alternate suggestion is the pilot may have been visually monitoring for a potential military jet interception attempt activity from Butterworth airfield. Butterworth is certainly well within this apparent field of view.

  257. Don Thompson says:

    @TBill,

    Whoever might have been sat in the captain and FO seats got a great view of Penang, out their front windshield, on the way over from Kota Bharu.

    An intercept from Butterworth’s F/A-18 Hornets was more than unlikely. Major Zaidi explained this to Olivier Sibille of Envoyé Spéciale.

  258. Sfojimbo says:

    ZS wasn’t worried about interception by the Malaysian AF. If there’s one thing that stands out it is the fact that he knew what to expect from the Malaysian military, including the radar watch. As he passed over Penang he knew that MH-370 hadn’t been officially reported missing yet. He had gained an extra hour in that regard by staging his takeover at FIR turnover: after he checked out of KLATCC, but before he checked into HCMATCC. He had rounded Banda Aceh before any alarm bells went off.

  259. DrB says:

    @airlandseaman,

    You said: “Even with no calibration data, the original MH370 analysis I published April 13th stands on its own as a reasonable KB speed and altitude profile.”

    I don’t think flying higher and faster than the Boeing published aircraft capabilities qualifies as “reasonable”.

  260. Don Thompson says:

    This morning I acquired a copy of MH370: Mystery Solved by Larry Vance, et al. I have completed a first read of the eBook.

    Essentially, Vance gives primacy to conclusions he, and his colleague Heaslip, made from the visual inspection of ATSB’s wreckage images of the recovered inboard section of the right outboard flap and the flaperon. He has had no physical access to any debris images, working only from photographic records.

    The conclusions drawn from the condition of the right outboard flap section and the flap then support the overall, emphatic, solution to the problem: the inescapable premise is that the aircraft descended under control of a pilot, and with engine power, to an attempted landing on the ocean.

    The condition of the remainder of the debris catalogue is lazily exploited as being consistent with a ditching, thereby confirming the central premise.

    The final Log On at 00:19 is cursorily relegated, by the author, to a possible consequence of a bus switching transient, or similar event, during the final stages of the approach to the water while the engines continued to run. That the switching transient caused the reboot. This forum’s readers should be sufficiently knowledgeable that bus supply transfers between available sources are effected so as systems, such as the AES, ‘hold up’ (are not interrupted). There is even a suggestion that the APU was started prior to ‘touchdown’, and continued to run while the aircraft settled in the water. Now, that is ‘weisian’ nonsense.

    He demonstrates no adequate understanding of the signalling metadata recorded at Perth and, indeed, reiterates oft cited nonsense about engine monitoring, as below:

    “Fortunately, the pilot was not aware that he did not disable one specific ACARS/satellite connection to do with engine monitoring. It was this remaining electronic connection with the satellite that investigators used to figure out that the airplane had flown to the southern Indian Ocean.”

    also:

    “From an overall investigation perspective, it is not necessary to figure out exactly what precipitated the reboot of the communications unit to come to a conclusion about the loss of MH370\[…\].”

    Given that the reboot of the AES is the final record, in any form, that was recorded in real-time I find that assertion quite astounding.

    My overall impression is that the author is making some sort of luddite call that the discipline of physical inspection sits at the zenith of skills necessary for accident investigation, and that modern trends for data analysis and communications intelligence is merely a distraction.

    Ultimately, the author makes no prediction where the wreck of MH370 may be found. So the book is about as useful as a snowball in hell.

  261. ventus45 says:

    @TBill

    Thanks very much for taking the time and trouble for doing that reverse course simulation. I just had a look, but I can’t download it to blow it up larger, since it seems it is “view only”.

  262. Andrew says:

    @ALSM

    RE: “The pressure masks are good to 45,000 feet and should work fine for an hour if used properly. “

    I refer you to a document I posted previously: PRESSURE CHANGES AND HYPOXIA IN AVIATION

    Positive-pressure breathing is not easy, as noted on pages 1003-1004 of the document. PPB is very hard work, even for a short period. It would be difficult for a young, fit, well-trained individual to do it for an hour, let alone an unfit, overweight, 53 year old without proper training. Further, as already noted, there would be a significant risk of decompression sickness during that period, which could result in one or more of the following: joint and limb pain, respiratory disturbances, skin manifestations, visual disturbances, neurological disturbances, or cardiovascular collapse (see pages 996-998). If the captain did depressurise the cabin at high altitude and did not descend, it is highly likely that he would have become incapacitated fairly quickly, even if he was wearing an oxygen mask.

  263. Sfojimbo says:

    There is no reason he could not have restored cabin pressure after 20 minutes or so above 40k. It wouldn’t take more than that.

  264. Mick Gilbert says:

    @airlandseaman

    Mike, let me be very clear on one point please; I haven’t speculated about big differences between the PSR and the Mode-C data. You are claiming that the SQ7838 exercise in some way validates the accuracy of the KB PSR data. I am saying that I don’t think that you can use that exercise in that fashion because you don’t know what the underlying SQ7838 PSR data looks like; you are comparing oranges with fruit punch and on the basis that the punch tastes okay you’re assuming that the oranges that went into making it must be okay also.

    Let me remind you of Victor’s summary of the SQ7838 exercise;

    One problem we have is that the KB radar data we have does not reflect only the PSR measured values. For each time, we have only X,Y, and pressure altitude, but not the underlying range and azimuth data. For some times, the PSR and SSR measurements are combined in order to derive X and Y. (The KB SSR head was operating in Mode C, which provides range, azimuth, and pressure altitude, but not GPS position.) At other times, there is only SSR data available. So, we really don’t know the accuracy of the measurement if only the PSR data is available. But also, the calibration on Jan 13, 2018, might be different than on Mar 7, 2014.

    You have taken data, applied maths and got an answer to fill a gap in the data. The problem is that your answer probably sits beyond the bounds of reality (viz airplane performance) and is contradicted by other data (viz military radar). It’s not a lay down misere.

    Beyond that I’d suggest that you’ve got some serious problems at a scenario level. Somewhat surprisingly you state, ‘The pressure masks are good to 45,000 feet and should work fine for an hour if used properly.‘ Numerous references have been provided by Andrew and me to the contrary. From around FL400 and up, even with a pressurised 100% oxygen supply, positive pressure breathing is necessary to maintain consciousness. As Andrew, a former military pilot, has stated, ‘… positive pressure breathing is hard work and not something to be taken lightly for an extended period, especially by pilots that are not trained in the required technique. Military fast jet pilots are appropriately trained; airline pilots are not. In the airline context, the pressure-demand mask is only intended to avoid hypoxia during the time it takes to complete an emergency descent.

    Andrew also provided the following from Pressure Changes and Hypoxia in Aviation;

    PPB [Positive Pressure Breathing] may be applied to the airway using a tightly fastened oronasal mask, but this is uncomfortable and causes distension of the upper respiratory tract, difficulty with speech and swallowing, and spasm of the eye- lids due to pressurization of the lachrymal ducts. Whether pressure is applied by mask or by other means (see below), PPB distends the lungs and expands the chest. Overdistension can be prevented by training in the technique of PPB, but even so there is a tendency for inspiratory reserve volume to fall and expiratory reserve volume to rise: pulmonary ventilation may increase by 50% when breathing at a positive pressure of 30 mm Hg. The associated fall in PACO2 means that hyperventilation is a feature of PPB, although this too can be minimized by training. The cardiovascular effects of PPB include peripheral pooling, impaired venous return, and reduced central blood volume; if there is a loss of peripheral arteriolar tone, then tachycardia and a gradual fall in blood pressure will lead to a collapse resembling a simple vasovagal syncope.‘ Vasovagal syncope is rapid onset loss of consciousness commonly referred to as ‘fainting’.

    The article goes on to state that, ‘The overall result is that PPB has such severe disadvantages that flyers use it for altitude protection only as an emergency “get-me-down” procedure.

    If that weren’t enough, on top of the complications associated with PPB you also have decompression sickness to contend with. The symptoms of decompression sickness include joint and limb pain, confusion or memory loss, headache, visual disturbances such as scotoma (spots in visual field), tunnel vision, diplopia (double vision) and blurred vision, extreme fatigue, seizures, dizziness, vertigo, nausea and vomiting. I probably don’t need to point out that vomiting under such circumstances would almost certainly be fatal.

    Beyond the matter of the problems associated with simply remaining conscious under those circumstances you propose a scenario that places the pilot in an environment demanding high levels of concentration and motor coordination; hand flying an airplane towards or at the edge of its absolute performance envelope with the engines running beyond their normal limits and with the flight protections disengaged. It’s not necessarily preposterous but we’re probably rapidly approaching that point.

  265. TBill says:

    @Ventus45
    OK you can try again I changed the setting. FS9 actually has Butterworth air strip, but it does not seem to show up in more distant views like the pics I showed.

  266. David says:

    @Andrew.”If the captain did depressurise the cabin at high altitude and did not descend, it is highly likely that he would have become incapacitated fairly quickly, even if he was wearing an oxygen mask.”

    Even if standard training for airline pilots does not include positive pressure breathing, do you think that airline pilots generally would know of what you say above?

    Secondly, if one’s intent were to incapacitate all others on board quickly, yet positive pressure breathing is difficult and risky for some categories, what cabin altitude would be the maximum to aim for, roughly?

    Also, after onset of decompression sickness symptoms would you expect incapacitation to follow quickly?

  267. Sfojimbo says:

    ZS had been a military (fast jet) pilot early in his flying career. Knowing his intellectual curiosity and dedication to aviation knowledge, I think it’s safe to say that he knew more about high altitude effects than anyone posting here.

    ZS wasn’t an ordinary kind of guy.

  268. airlandseaman says:

    Andrew/Mick: I have some direct experience with high altitude pressure breathing. I know what it feels like. I received training many decades ago at the chamber in Colorado Springs. Since then, I have had a number of wave flights above 35,000 feet (one to 39,000 feet). It is very cold up there, but I did not experience the issues you cite. Granted, 39,000 is not 45,000, but I’m just saying, breathing with a pressure mask and regulator (A14) was not a problem for me. I believe the PF could have used a mask for up to an hour. OTOH, I think Foley raises a valid question about the decompression sickness aspect. That needs a closer look.

    Mick: I stand by the KB radar analysis. I do not think your criticism is valid. As I have said all along, I believe the plane was flown to the highest altitude it was capable of (whatever that was). The radar data suggests that was 43,000 to 46,000 feet. You criticize the use of SQ7838 data to calibrate anything, apparently even the azimuth, but show no proof of any specific flaw in the work. Meanwhile, no one has produced any measurements proving what 9M-MRO was capable of doing. The use of one size fits all manuals is not proof. It is general guidance, and it is close, but imperfect when it comes to any specific air-frame and engine set. It’s a double standard. You take the Boeing manuals as absolute knowledge, but reject hard measurements at KB based on a lot of what if’s.

  269. David says:

    @Andrew. My third question would be more sensible if you would add please, “irrespective of any ameliorative steps he could take?”

    @Sfojimbo. No mention of military training of ZS in FI. Still as you imply he could have researched it all, out of curiosity or during planning.

  270. Andrew says:

    @Sfojimbo

    RE: “ZS had been a military (fast jet) pilot early in his flying career.”

    Please provide a reference for that statement. According to the Malaysian Factual Information report for MH370, ZS completed his schooling in 1978 and was accepted into MAS as a cadet pilot in 1981. He was then provided with ab-initio pilot training in the Philippines and joined MAS as a Second Officer in 1983. There is no mention of any military flying experience and, given the timeline, I think it is extremely unlikely.

  271. ventus45 says:

    @TBill
    Got it as pdf – thanks.

  272. Andrew says:

    @ALSM

    Strictly speaking, PPB isn’t required until the cabin altitude exceeds 40,000 ft. Below that altitude, a pilot breathing 100% oxygen at ambient pressure will achieve the equivalent oxygen partial pressure available at altitudes of 10,000 ft or below. Above 40,000 ft, the pilot MUST breathe oxygen under pressure to maintain sufficient oxygen in the bloodstream.

    The problems with PPB occur due to the pressure differential that is required to maintain sufficient oxygen pressure in the lungs. That differential increases with altitude and reaches its maximum at about 45,000 ft, where the differential is the maximum the lungs can withstand without permanent damage. You would not have experienced anywhere near the maximum differential at altitudes between 35,000 and 39,000 ft, so it is not very surprising that you found that PPB ‘was not a problem’. Out of interest, how long did you spend at those altitudes?

  273. Andrew says:

    @David

    RE: “Even if standard training for airline pilots does not include positive pressure breathing, do you think that airline pilots generally would know of what you say above?”

    Those with military experience certainly would, but most others probably wouldn’t have much detailed knowledge unless they had a particular interest in the subject and sought out the information. The ATPL theory syllabus covers the basics of decompression and hypoxia, but it does not cover positive pressure breathing or its associated problems.

    RE: “Secondly, if one’s intent were to incapacitate all others on board quickly, yet positive pressure breathing is difficult and risky for some categories, what cabin altitude would be the maximum to aim for, roughly?”

    No higher than about FL400.

    RE: “Also, after onset of decompression sickness symptoms would you expect incapacitation to follow quickly, [irrespective of any ameliorative steps he could take]?”

    I’m not sure; there is a wide range of symptoms and everyone is affected differently. My guess is that it would largely depend on the health of the individual and the rate of decompression.

  274. Sfojimbo says:

    My apologies.

    I remember reading a bio of him (in 2014)that talked about his dedication to flying and how he owned a sports plane (Pitts?) that he did aerobatics in. I (apparently falsely) remember it saying he had flown for the Malaysian Air Force.

    But as I just researched it, I can see that he joined MAS when he was 20 or 21, so there was no time for him to ever have been in the AF.

    I stand corrected.

  275. DennisW says:

    @rDrB

    I don’t think flying higher and faster than the Boeing published aircraft capabilities qualifies as “reasonable”.

    Yes. I don’t want to challenge ALSM’s conclusions since I don’t want to appear I am I picking on him. Like most IG opinions his is pretty f’ed up.

  276. TBill says:

    Re: O2 Mask
    Don’t forget approx. 42,500-ft was about equal to FL400 on this day. So this gets close to ALSM’s KB altitude, yet not too extreme for PPB O2 mask.

    On FS9 flight sim (which I know has serious shortcomings for depressure) but it takes me a long time to depressure aircraft with bleed air on. Presumably some combination of outflow valve opening and bleed air could hold cabin pressure at FL400 even if aircraft was somehow at higher altitude.

    Quick depressure would require (1) open outflow valves, and (2) cut off bleed air. Joe Nemo has been developing a scenario description. But as has been discussed, a more gradual depressure approach is possible.

  277. David says:

    @Andrew. My thanks.

    My musings:
    . Unless he had done some research he could have aimed for a cabin altitude higher than wise, opening up the Foley possibility but also that of self-hypoxia but in either case it seems unlikely he would last an hour.
    . Had he done some homework on this and aimed, “No higher than about FL400” he might have shut the outflow valves a little lower to allow for slow leakage subsequently. He should have remained resistant to hypoxia so long as he then kept his eye on cabin altitude. However he would have left open the possibility of the Foley de-compression sickness, though he could be expected to last longer than if higher, in which case he might have made it to the power restoration at the first log-on.
    . Had he done enough homework to be aware of the decompression risk his aim point would have been lower.
    . I notice in his simulation he got to 40,000 ft

    I hope the above is a fair distillation.

    @TBill. He could have gone higher then shut the outlows at the desired cabin altitude, which might have led to a faster decompression but that would complicate things for marginal gain I think.

  278. ventus45 says:

    @ TBill
    The simplest way would be to just cut the packs and closing the outflow valves, whichh would lead to a slow depressurization through existing leakages.

  279. Richard Godfrey says:

    @Dennis

    You stated “Like most IG opinions his is pretty fucked up”.

    I am fed up with your insults, against the IG.

    Please show me, where my opinions are fucked up.

    You are sounding more like the mad Dutchman every day.

    Can I arrange a round of Golf for you both with the Seniors at the Golfpark de Teuvart?

    It might help both of you to calm down a bit.

  280. TBill says:

    @Richard
    Some time around 2-weeks after the MH370 accident, my crash theory changed from internal fuel tank fire, to intentional diversion. Thanks to yourself and Victor, there was a like-minded viewpoint to latch onto. Thank you.

  281. ventus45 says:

    For those interested – the transcript is now available.
    Begins half way down page 54 with:
    “Senator CHISHOLM:
    I’m interested in MH370 and wonder if you’ve had any involvement in the investigation in recent months ?”
    http://parlinfo.aph.gov.au/parlInfo/download/committees/estimate/e5833f20-4a8f-43f3-80e0-09ffa20248c5/toc_pdf/Rural%20and%20Regional%20Affairs%20and%20Transport%20Legislation%20Committee_2018_05_22_6176.pdf;fileType%3Dapplication%2Fpdf

  282. TBill says:

    @DennisW
    You have always asked “why MH370? vs. flight to Europe or Jeddah?”

    Strikes me we have a new possible answer: cannot get to FL430 with the fuel load. Keep in mind the sim studies show FL400 at DOTEN.

    …there are some reasons why flight MH150 to Jeddah might not have been the best candidate flight for hijack, including crew of 3 pilots, time of day (radar), and heavy fuel load would have prevented a climb to FL430, that now looks like it may have actually happened at IGARI.

  283. Richard Godfrey says:

    @TBill

    You are very welcome!

  284. Gysbreght says:

    @TBill: “… a climb to FL430, that now looks like it may have actually happened at IGARI.”

    I’ve always thought of that as a possibility, but not at M.977.

  285. TBill says:

    @Gysgreght
    …acknowledged you were active on that, if not a early pioneer. The new data gives us reason to go back and look at it.

  286. David says:

    @TBill. Speculating along Foley lines, with the onset of decompression sickness after a deliberate decompression of an hour or so the NW destination had to be abandoned. Nothing for it but to set a course to the never-never down south.

  287. formula says:

    @ David “…with the onset of decompression sickness after a deliberate decompression of an hour or so the NW destination had to be abandoned. Nothing for it but to set a course to the never-never down south.”

    Had the NW destination been reached after deliberate decompression, how might the perpetrator expect to explain the state of the PAX?

  288. TimR says:

    @All, As DennisW put it recently there are two distinct camps.
    1. “the “mission” was over at the FMT. At that point the primary objective had failed, and the pre-planned “or else” was initiated. …… PIC input was no longer required under any conceivable circumstances.”
    2.”On the other hand, if the flight went toward CI as Jean-Luc et. al. have postulated, then there is no question the PIC would have remained engaged.”

    If the flight was under control it is possible Shah ended up ditching up near Christmas Island.
    MH370 could well be found near the 7th arc to the East of Christmas Island.

  289. airlandseaman says:

    Note the military radar observations ITVO Penang, reported in the RMP Folder #4 at .pdf Pages 25-26. These numbers are in close agreement with the KB Civil radar assessment.

    Time (UTC) Altitude (ft) Ground Speed (kts)
    17:50:59 FL447 528
    17:51:59 FL447 528
    17:52:27 FL447 525
    17:52:59 FL447 520
    17:53:59 FL447 512
    17:54:59 FL447 528
    17:55:59 FL423 507
    17:56:59 FL423 525

  290. Mick Gilbert says:

    @airlandseaman

    Mike, I think you’ll find that the data for the first two registered points are:

    17:50:59 FL447 578
    17:51:59 FL447 578

  291. David says:

    @formula.”Had the NW destination been reached after deliberate decompression, how might the perpetrator expect to explain the state of the PAX?”
    Maybe harm at the far end was in mind.

  292. airlandseaman says:

    Mick Gilbert: I agree it looks that way in the fuzzy pic (Diagram 2). I thought it was obviously wrong, and therefor what looks like a 7 is in fact a 2, due to a poor reproduction of the image. But you are correct to point out it looks like 7s. Given the other 6 observations, I think my assumption is probably good.

  293. ventus45 says:

    Foley is “clutching at straws” with his “decompression sickness” scenario, and it really is quite ridiculous to boot.

    Foley knows that most people accept the very high likelihood of deliberate decompression to neutralize the cabin crew and passengers, (probably the FO as well if he had left the flight deck) at, or even slightly before Igari. This would be both logical and necessary, for any “intended” flight back over the peninsula, and most definitely for the turn at Penang, since many on board, particularly the FO and cabin crew, would easily recognize it, and the two “lit up” bridges to the mainland, both of which are many kilometres long !!

    So even if the cabin crew had not been particularly concerned up until Penang, they sure as hell would have been when they saw Penang Island and the bridges, all of which are utterly unmistakable to “locals”, if they were still “functional”.

    His use of the DC-8 freight flight, in the way he did, was on one hand, quite funny, but on the other, disingenuous.

    His “directed” attention to the DC-8 Captain being similar in age and obesity to the B-777 Captain was very deliberate, and his clearly perfunctory ending of that section, his attempt to present it as a “slam dunk”, and having the Senators accepting it, without comment, seemed almost to be “scripted”. He glossed over the fact that the FO and FE were totally functional at all times, and landed the aircraft normally. Foley’s scenario all centers on the “age” and “obesity” of the Captains (plural), and nothing else. A “slam-dunk” indeed, M’lud.

    If Foley’s point is to be taken seriously, and the Captain of the B-777 was to become incapacitated “within a few minutes” like the DC-8 Captain, then that should have occurred, when, or “within a few minutes” of when, the decompression occurred, ie, near Igari, around 17:25. Yet Foley acknowledges, that he, and everyone else, agree, that “someone” was controlling the aircraft’s flight path, at least until – he said – 18:25.

    That means that IF this obese B-777 Captain was “that someone”, then he supposedly survived in a fully functional state, (medical miracle) on the oxygen mask, for A FULL HOUR, and ONLY THEN, or soon after, scummed to “decompression sickness”.

    If Foley is correct, and the hour long decompression is correct, then Foley is indirectly hinting, that “that someone” was most likely not the Captain. Could it have been someone else directing the plane ? It raises (through the back door – so to speak) the possibility, however unlikely, that the aircraft could have been hijacked, by someone else, not the Captain, and not the FO, probably soon after “top of climb”.

    If you rule out such notions as “out-there”, then Foley is effectively saying (WITHOUT actually SAYING IT) that no decompression occurred before 18:25.

    Look at the timing again. Last supposed radar hit 18:22. First SDU logon 18:25.

    The 18:25 logon requires “someone” to be there, to restore power to the left bus.

    Interesting. Why have an “uncontrolled” flight post 18:25 ?

    Logically, Foley is saying that the flight was definitely controlled by “someone”, at least until the 18:25 logon, but became “uncontrolled”, some time afterwards, but he can give no time frame for that event occurring. He simply leaves the FMT “hanging” as the last logical “controlled” event (without saying so directly). Hardly convincing M’lud.

  294. Mick Gilbert says:

    @airlandseaman

    Mike, I’m quite confident that they’re both 7s. The numeral has a markedly different profile to the 2s (most notably the difference in shape of the upper arm, the 7 is horizontal whereas the 2 is markedly curved and the absence of a lower horizontal arm on the 7) and is not discernibly different to the profile of 7 in FL447, particularly when you look at Diagram 3. There’s also a very neat comparative set on Diagram 2, specifically the stack of data points for (from top to bottom) 1.50.59, 1.55.59 and 1.51.59; the speeds are respectively, 578 knots, 507 knots and 578 knots with the 7s falling in a rough diagonal.

  295. TimR says:

    @All, With the Southern Indian Ocean having been well swept without success along the 7th arc in the locations various authorities and experts suggested MH370 could have ended up it would be valuable to now go back and reiterate what we believe could have happened.
    Sometime ago DennisW suggested that while Shah was airborne negotiations took place on the ground in KL.
    Shah would have ensured as much time as possible would have been given for such negotiations while still intending to land, release the passengers and place himself at the mercy of the authorities.
    It was possible that MH370 could have come round below Sumatra to Cocos Islands and up to a ditching near Christmas Island.

    Two drift studies undertaken by professional organisations in independent countries free of confirmation bias gave a high probability MH370 was between 8°S and 23°S at the 7th arc.

  296. airlandseaman says:

    Mick: You can toss those outliers, and the point I was making remains.

  297. TBill says:

    @Ventus45
    Yes I was just reading your link to the transcript. Foley was doing good up to that point, but he really got down in the dirt there. I am thinking Kalitta 66 flight it was the old guy, smoker pilot who stayed awake. I never thought of ZS as over-weight.

    However, I thought it was important that he acknowledged the intentional depressurization scenario. What we want to know is if someone with planning and experience could pull it off in a B777 with a special B777 pilot O2 mask and cabin altitude control technique. It is almost worth finding the MH370 aircraft just to see how and if this terrible action was done. To me, this is the central darkest issue of MH370.

    The other really amazing thing is he is talking openly about FL400. Up until a few weeks ago, we were all assuming it was FL350 because ATSB had said some years ago that the higher altitude rumors were wrong due to inaccurate radar. Now Mr. Foley is freely talking about FL400, which we only recently are thinking only maybe that happened.

  298. ST says:

    @Andrew – Is there anything that would make a possible depressurization immediately upon reaching cruising altitude impossible? Asking because the more you think about all the aspects you have talked about, it would be easier to get the passengers unconscious earlier on during the flight when oxygen levels for ZS can be suitably maintained/controlled before the deviation at or close to IGARI and the subsequent potentially higher altitude flying?

    That would explain why other crew members and co-pilot may have had no time to react.

  299. airlandseaman says:

    TBill: Re: “The other really amazing thing is he is talking openly about FL400.”

    I’m not surprised. I don’t know it for fact, but I suspect Foley received some confirmation from Boeing about the KB PSR Radar report I sent 5 weeks ago. He may have independent information about the altitude as well. In any event, MH370 was evidently high and fast from KB all the way to at least 18:01, and probably 18:22, after which time we know the left AC Main Bus came back on, and the BFO data at 18:40 looks more and more like a descent, not a turn. Beyond that point, we still have little to guide us on how long the plane remained WNW of KL, before heading to a point somewhere along the 7th arc.

    I have no doubt that, if the plane was depressurized, the PF would have had no significant difficulty breathing for an hour using the company issued Pressure Demand Mask in the cockpit. As for decompression sickness, I don’t see any hard data to support that one way or the other. But it is possible.

  300. HB says:

    @Don RE: Lance book

    I dont generally agree with the scenario postualted by L Vance and unless the out of control spiraling led the plane beyond the 25nm of the 7th arc, the ATSB scenario has been invalidated by the search. Not sure what’s the view of others on this.That of course does not validate Vance s scenario by default.
    Saying that, one of Vance observations about the flap seal pan merrits further attention. The sequence of events is difficult to judge by visual inspection. This is true for both Vance and ATSB. ATSB had of course a closer look but they have not validated the failure sequence by radigraphic inspection, test or modelling as far as I know. Following the senate hearing, P Foley mentioned he read Vance s book and stands to his conclusions but again no validation activities have been done. Also the plane has not been found near the 7th arc. Added to that, neither ATSB or Vance take into account how the load bearing structure and hinges have failed which should be highly indicative of the event sequence (item that has been debated at length).
    I am not saying that Vance is right on the flap seal pan observation but I am not personally satisfied by the evidence presented by ATSB. The lead investigator is also unlikely to do any work unless they are forced to do so as part of ICAO A13 compliance. There is clearly missing analysis on debris that could be extremely useful. my opinion.

  301. ventus45 says:

    @TimR

    Are you seriously suggesting that we should search the arc all the way up to be beaches of Java (at 8 degrees south), seriously ?

    I musty admit, that early on, before I finally dismissed the Lido slide, that I did seriously consider “a negotiation scenario”, and the logic for it, at that time.

    However, two things caused me to reject that idea.
    First, my rejection of the Lido track up the strait, and Second, the reality of international politics.

    Any “negotiation” scenario, Lido slide or not, fails at the second point.

    It is a “fact”, like it or not, that Internation Policy, is that no government will “negotiate” with a highjacker.

    The reason for that is simple. If you do (and we did in the past), all that happens is that it encourages others to do the same. That happened, and that is why the policy was changed, to make it clear, to any would-be hijacker, that they were not going to “win” in the end.

    Z was very well aware of all of this, as is every airline pilot, world wide.

    If Z hijacked his own aircraft, as I think we now all beleive, it is obvious, that he never intended to land it on any runway, anywhere, ever, not in Indonesia, not on Cocos Island, not on Christmas Island, nor anywhere else. He intended to “vanish” period, and for 4 years, he has acheived his goal, both in remaining vanished, and as I am sure history will eventually show, he has in no small way helped preciptate his often and openly stated, primary political goal, of regeim change in Malaysia.

    Flying orbits, or otherwise tracking so as to remain within range of a suitable runway, is pure fantasy, and directly contrary to his objective to vanish, and to stay vanished. You don’t drop it in the ocean where it could be found easily. You pick the most remote and difficult place to search that you can reach.

    For planning, execution, and acheived outcomes, I reluctantly give him ten out of ten, with a gold star, and bar.

    I think it is high time, more than high time I think, that people stopped pussy footing around, with their heads in the sand, to swallow their emotions, and “bite the bullet” on this reality, as unpleasant as it is, to even contemplate, let alone accept. That is what police have to do, when confronted with henious crimes.

    Only then, can we seriously consider the implications, and move forward, towards eventually solving the case.

    There is only one way to do that.

    To paraphrase what my 20 year younger police officer friend of many years advised me, long ago now:-
    “you have to get into the mind of the criminal Ventus”.
    “You have to try and think like he did”.
    “You have to find the primary motive, there may be two or more” (we have – politcs and vanishing).
    “You hace to find the means” (we have – obvious – one B-777 will do nicely).
    “You have to figure out the “best” method – of “best” using those means – to “best” achieve the motive(s)”.
    “Motive, means, and method Ventus.”
    “The time honoured formula, it always works, in the end.”
    “Your case Ventus”.
    “Get on with it”.

    STEP ONE.
    “you have to get into the mind of the criminal Ventus”.
    “You have to try and think like he did”.
    Ok.
    Become Z.
    Become the mission planner.
    Make it “you mission”.
    Figure out how “you” would do it.

    I did.
    The result was via Medan to ditch.

    Anyone else want to have a go ?

  302. TBill says:

    @ventus45
    It struck me ATSB’s openning remarks:
    “At the ATSB, we exercise great care not to engage in conjecture or speculation.”

    But as you say, how can we solve the problem without speculation about what might have happened? That is exactly the missing ingredient to date.

  303. TimR says:

    @All, Shah could have proceeded out into the Andaman Sea then come back and out to BEDAX.
    At BEDAX turned South and at 19:41 crossed the 2nd arc at around 4°30’N in the low 400’s, slowed and settled at a steady loiter and continued round below Sumatra via ISBIX and the Cocos Islands using waypoints.
    At the Cocos Islands turned up towards Christmas Island with a ditching between 8°30’S to 9°30’S near the 7th arc on a flightpath towards an Indonesian airfield.
    As far as BTO’s were concerned arc 2 to 3, 3 to 4 and 4 to 5 fitted accurately while flying at a steady speed despite waypoint turns between 2 to 3 and 4 to 5.
    5 to 6 was interrupted by Christmas Island being considered for a possible landing.
    Something went wrong towards the end of the flight.

  304. Mick Gilbert says:

    @TBill

    Bill, don’t confuse Connie [Kalitta] Flight 861 with Kalitta Flight 66. 861 was the DC-8 flight of 15 March 1994 that Foley referenced, 66 was a Learjet flight of July 2008. The former was investigated by the NTSB (NTSB Identification: NYC94LA062), the latter was not. I’ve never seen any details (age, experience, etc) for the flight crew on Kalitta 66. On the other hand the FAA Airmen Medical Records for the flight crew of Kalitta 861 are referenced in the report.

  305. Sfojimbo says:

    ST raises a possibility that I’ve never seen mentioned anywhere; I think we should give some thought to what went on before IGARI. At 1:07:55 Fariq made his second, unexplained “maintaining level three five zero” announcement to ATC. This sounds like an indication of some confusion on the flight deck, possibly some contention between the flight crew; possibly Z accused F of not reporting FL 350 and demanded he do so and then told Fariq to go take a break in the cabin. Fariq would have been glad to do just that under such pressure.

    If Z found himself alone at 1:09 there are two things I can think of that he might do to prepare for depressurization, which was coming in 13 minutes. One would be to hyperventilate on oxygen and as ST suggests he might lower the air pressure in the cabin slowly so it wouldn’t be noticed, but significantly so as to make those in the cabin more vulnerable for what would be coming next. It also might lessen the shock on his own body to help mitigate decompression sickness problems.

    I don’t know how far he could go along those lines before setting off alarm bells, but it seems worth doing to me.

    Another point I would like to raise is that he would not have had to keep the plane depressurized for more that 20 minutes or so if he got the cabin altitude above 40,000. Maybe double it to make sure, but he certainly wouldn’t have needed an hour at very high altitude. The flight deck would have been freezing cold while vented to 40K +.

    I know nothing about high altitude decompression sickness, but from my SUBA days I know that the early and even moderate symptoms are joint pain
    and that the symptoms are mitigated quickly by depressurization. I don’t think Z faced anything unmanageable from decompression sickness.

  306. Mick Gilbert says:

    @Sfojimbo

    Re: ‘At 1:07:55 Fariq made his second, unexplained “maintaining level three five zero” announcement to ATC.

    According to the Factual Information all nine radio calls after take-off, including the 0107:56 MYT call, were made by the Captain, not the FO.

  307. Sfojimbo says:

    I thought it was the other way but if FI is correct, then he had even more time to hyperventilate and mitigate the effects of altitude sickness.

    I have an MP3 file of the radio traffic that night, I’ll listen again. Four years ago I came to the conclusion that the last voice (1:19:29 “good night Malaysian Three Seven Zero”), was different from the earlier communications.

  308. Sfojimbo says:

    Above I meant to say the symptoms are mitigated quickly by re-pressurization and I left the “C” out of SCUBA.

  309. Nederland says:

    Re hard data on decompression sickness:
    https://www.researchgate.net/publication/11034697_Decompression_sickness_latency_as_a_function_of_altitude_to_25000_feet

    According to this article, it is reasonably safe (for a healthy fighterjet pilot) to fly unpressurised up to FL210 for unlimited times. Above that level, the risk of falling ill rises dramatically (obviously, the single most important risk factor is age and smoking). At FL250, there is a 20% chance to fall ill within 45 mins. Extrapolating the curve the time frame nearly halves with every additional 1000 ft altitude (by the factor 0.6). This means at FL 350-400, that time frame is reduced to very few minutes (There is still an 80% chance of not falling sick, but that chance drops dramatically with prolonged exposure). 100% oxygen supply does not alleviate this (unless breathed in for half an hour or so before exposure to altitude).

    There were 15 portable oxygen bottles on board of MH370 (FI), each lasting for at least an hour (masks dropping down at 13,500 ft, signalling that the flight crew will get a hold of the bottles). Depending on the FL (and actual pressure in the cabin), it is probably easy to pass out despite these bottles. However, while it is easy to pass out from hypoxia, it is also easy to recover. How long would it take until death occurs, if a person, although unconscious, is still being fed with oxygen? The body goes into survival mode, supporting basic heart and brain functions, while switching off all higher functions of consciousness.

    If you think that through, it seems next to impossible to find the right ‘mixture’ to get away with this.

    The DC-8 incident does not come up with a clear time frame, but it very much seems that the crew immediately decided to descend (i.e. within few minutes at FL330, or earlier) before experiencing symptoms.

  310. Andrew says:

    @ventus45

    RE: Peter Foley’s testimony

    You (and others) assume that MH370 was depressurised at or before IGARI, but what if that didn’t occur until some time around 18:25? I also think you overstate the likelihood of passengers or cabin crew recognising the aircraft had turned around and back-tracked across the Malay Peninsular, for several reasons:

    1. It was late at night (after 1 am local time), when many passengers would have been trying to get some sleep. Those that weren’t trying to sleep were probably eating, assuming some kind of refreshment service was offered by the cabin crew. I doubt that many passengers were staring out the windows.

    2. The cabin crew are normally busy in the galley or serving passengers during the early part of a flight. They simply don’t have time to stare out the windows, trying to identify the passing landmarks below. In my experience, the cabin crew usually have very little idea of the aircraft’s location at any given time.

    3. If anyone was looking out the windows, the reported cloud cover at Penang airport at the time was BKN 280. In other words, there was a broken layer of cloud at FL280. ‘Broken’ means that 5-7 eighths of the sky was obscured by cloud. That cloud cover would have made it difficult for anyone on board the aircraft to positively identify landmarks on the ground.

    Consider the following scenario:

    1. The second pilot (presumably the FO) is incapacitated inside the cockpit, some time between top-of-climb and IGARI.

    2. The aircraft’s transponder and ACARS are disabled shortly after passing IGARI. The L AC bus is also de-powered around the same time to disable the SATCOM. Additional loads, including IFE components, are automatically load shed, disabling the moving map displays in the cabin. The IFE/PASS SEATS power switch might also be selected OFF to ensure power is removed from the IFE. The resulting IFE failure is easily explained by a reassuring PA to the cabin, “Ladies & gentlemen, I’m terribly sorry to inform you, blah, blah…”

    3. The aircraft is turned around towards Penang, using HDG SEL mode.

    4. After passing Penang, the FMC is programmed to fly direct to a waypoint at the top end of the Malacca Strait and then to a waypoint deep in the Indian Ocean. LNAV mode is engaged.

    5. The aircraft is depressurised, incapacitating the passengers and cabin crew.

    6. The L AC bus is re-powered around 18:25.

    7. At some point the pilot (presumably the captain) is also incapacitated.

    8. The aircraft continues on its merry way until fuel exhaustion.

    That scenario obviously won’t appeal to those who believe the pilot was in control until the end, but it does satisfy Peter Foley’s suggestion re the Kalitta incident, does it not?

  311. Andrew says:

    @ST

    RE: “Is there anything that would make a possible depressurization immediately upon reaching cruising altitude impossible? Asking because the more you think about all the aspects you have talked about, it would be easier to get the passengers unconscious earlier on during the flight when oxygen levels for ZS can be suitably maintained/controlled before the deviation at or close to IGARI and the subsequent potentially higher altitude flying?”

    I guess it’s possible, but I still think there’s a significant risk of decompression sickness if the cabin was depressurised for a long period.

  312. Mick Gilbert says:

    @Nederland

    The portable ‘walk around’ oxygen bottles carried on 9M-MRO were 301 litres capacity. At 100% flow, the usual setting for an emergency, you get 15 minutes out of a bottle.

  313. Andrew says:

    @Nederland

    RE: “The DC-8 incident does not come up with a clear time frame, but it very much seems that the crew immediately decided to descend (i.e. within few minutes at FL330, or earlier) before experiencing symptoms.”

    The NTSB stated: “According to the printout of the flight data recorder, the airplane had achieved an altitude of FL330, 18 minutes after takeoff. The flight stayed at that altitude for approximately 4 minutes and then initiated a descent to 10,000 feet. After 2 minutes at 10,000 feet, the descent continued to 8,000 feet.”

    https://www.ntsb.gov/_layouts/ntsb.aviation/brief2.aspx?ev_id=20001206X00986&ntsbno=NYC94LA062&akey=1

  314. Richard Godfrey says:

    SC progress up the Broken Ridge plateau area has been slowed by bad weather.

    Ocean Infinity has continued operations, but at a slower pace.

    The weather is poor, there is still good visibility, but with a 23 knot wind and a combined swell and wave height of 5.9 m. There is a tropical storm S.E. of the search area, but moving away. It will take another 24 hours for the high swell to calm down and operations to normalise.

    https://www.dropbox.com/s/6uz8f81higb6ckn/SC%20Track%2025052018.pdf?dl=0

  315. Mick Gilbert says:

    @Nederland

    Apologies, my mistake on the walk around bottles. The SABRE bottles carried on 9M-MRO provide 44 minutes of oxygen at the HI flow rate.

  316. David says:

    @Victor. On your above theme, the “60 Minutes” program, what got my attention was Martin Dolan’s statement to the effect that no more information was needed to establish cause, which was to the effect that there was a piloted hijacking of the aircraft.

    While understandably that may be an insufficient finding for the families from the aircraft future safety aspect it does provide the focus needed for any improvements which might be wrought to prevent repetition, even if the ‘how’ remains unknown. As to ‘why’, that need not be known for this purpose.

    The ‘why’ and who else might have been complicit, or involved in as yet undisclosed ways, should be the subject of further separate inquiry, independent of any more wreckage search.

    I have raised earlier the desirability of a Malaysian Royal Commission but at the least there should be a criminal investigation looking into who knew what about the principal person(s) of interest and into who else might be of interest.

    I doubt that recommending this is the business of the investigation but its findings would I think be hard to ignore if reasonably definitive, along the Dolan lines.

    There are two ways this could happen, the first being a conclusion of the final report that a pilot was very likely involved with cause, even if not naming names. Even should that not transpire, a second could be as with the Silk Air report with its ’cause not established’. It also enclosed dissenting remarks from an accredited investigation representative, in that case the NTSB. I believe that including those would be obligatory in the MH370 final report (in the Silk Air case, the Australian representative advanced views which were excluded from the final report because he was not accredited).

    I hope this will not be over even should the search prove unsuccessful and formal searching be discontinued. Indeed leaving the way open for a new search might delay the final report again. It may prove beneficial to provide this new focus and one which might lead to the closure still being sought for MH17, where again the root cause has been established.

  317. Gysbreght says:

    Suppose there is a connection between MH370, MH17, and 1MDB, will the new government reveal it?

  318. Don Thompson says:

    @HB

    Thank you for the response. I am not recommending that everyone with an interest in MH370 should read MH370:PS (I’ve only read three of the many, many books published on the subject). There are inconsistencies in its thesis that all readers, including those casually informed, should be aware of.

    The author doesn’t make any concerted effort to understand and rebutt the analyses that have been used to inform the search to date, his message boils down to: ‘I, emphatically, know that an attempt was made to land on the ocean, I don’t know where, but I know this because I and my colleagues have a life’s experience in looking at dings & scratches on airplane wreckage.’

    To be so emphatic about the state of the flaperon and flap, might be afforded credibility if the extension mechanism for the flaps was correctly described: the description that is set out in book, for flap extension, is entirely wrong.

    Your remarks concerning the analysis of the damage to the recovered parts have some merit but one has to consider that ATSB has actually had first hand exposure to the flap in its lab, and considering the diagrams used to illustrate its analysis, ATSB understands the mechanism by which it is extended/retracted. I reiterate that the authors of MH370:PS did not have physical access to debris items and there is no evidence they exploited any image enhancement to extract additional available detail in the ATSB images used to draw conclusions.

    I’d encourage you to review ATSB’s image portfolio.

  319. Victor Iannello says:

    @Gysbreght: I’d say, it depends on the connection and how deep the corruption. There are limits to how much of the government, past and present, that Mahathir and Anwar can fire torch. At some point, healing has to be a high priority.

  320. Victor Iannello says:

    @David: Yes, additional comments from others would be welcome in the report from Malaysia. Silk Air 185 was a bit different, though, as there was the implication that there was a mechanical failure, and I think the NTSB and Boeing wanted to “set the record straight” with their version of likely events.

  321. Sfojimbo says:

    @ Victor
    The Mahathir and Anwar government will be just as committed to coverup as the Razak government was. This is evidenced by Mahathir’s announcement that funding for the search will be reviewed.

    @ST
    More important than how long an aviator remains depressurized, is how fast the depressurization process takes. That’s why SCUBA divers ascend slowly and have stops where they pause and let the nitrogen return to its gaseous form.

    @Gysbreght
    I am surprised to find anyone in this forum would try to float an idea that MH-370 and MH-17 are even possibly connected. That’s tinfoil hat kinda stuff.

  322. Gysbreght says:

    @Sfojimbo: Follow the money.

  323. Nederland says:

    @Mick

    I guess it’s also unclear if all flight attendants would switch their bottles to high flow immediately, at a lower rate duration would double or triple, that could still be enough to regain consciousness after the plane was allegedly repressurised. In sum, I don’t think anyone could plan for all eventualities in that particular scenario.

    @Andrew

    Thanks for clarification, 4 mins doesn’t sound much.

  324. Victor Iannello says:

    @Sfojimbo: I don’t think it’s crazy to think that for some yet undisclosed reason, Russia was not happy with the loss of MH370, and the downing of MH17 was in reaction to that loss. That said, I don’t think Russia caused the loss of MH370. It’s also possible that factions in Russia lost money in the 1MDB scandal. With billions of dollars missing, there must have been a lot of people not very happy with Malaysia. Not all of the stolen money was Malaysian money.

  325. ST says:

    Thanks Andrew and Sfojimbo for your inputs.

  326. Dazza says:

    @Victor, Many thanks for your continued professional and reasoned administration of this blog together with your undoubted expertise. Without this moderation of a diversity of views mostly sincere and reasonable, but also expert, inexpert, uneducated and loony-tune (or a combination thereof) we would not have progressed this far. I repeat my (though it is not mine to grant) gratitude that you have persevered under such adversity.
    Many times, I have been prompted to respond to previous posts but reluctantly withheld because of lack of confidence especially with regard to opinions as opposed to your actual competent contributing mathematicians. In particular, a Dutchman has so obscured moving forward that it took me 9 months to realise that his comments were counter- productive and (almost) destructively illogical-but enough of that.
    I now come to the newly (April 2018) released extracted file for the KB PSR data. There is an (obviously?) missing record between 31:26 and 31:32; I have taken the liberty of introducing another at 31:29 with an interpolated distance/azimuth of 50.7/47.6. This still leaves the end of the “A” leg at 36:43.
    The new “B” leg now starts at 39:03 (not 38:55). This is because the intervening period in the COS represents a further two pseudo captures from the radar head.
    There is obviously a zig-jag effect on step by step speed estimations from (mostly) imperfections in azimuth reconciliation but at 36000 ft this gives the following average speeds (on an essentially straight-line basis) from start (30:33) as:
    30:33, ? , capture1
    33:59,470kts, capture55
    36:43,485, capture 98: followed by COS
    39:03,494, interpolated capture135
    42:51,503, interpolated capture195
    44:30,506, interpolated capture221
    All captures are based on 4/3.812 rotation speeds with allowance for azimuth changes. The figures are not too different at 32000 ft. COS has been taken a whole. They do not seem to require top ground speeds above 530 kts although others may disagree.
    At last I will attempt to make my point. There is no indication from the above figures that a geometric altitude of approx. 42000-45000 ft is necessary if the COS time frame is expanded by 2 or 3 missed pseudo-captures. (I must admit that one instantaneous speed has been doctored at the end of the “A” leg where large azimuth changes produce large speed (jig-jag) errors over 4/3.812 seconds. I have also used a spherical earth.
    I offer this only as a suggestion that the B leg is misplaced and do not wish to impugn any IG contributor or their sources. I realise that further analysis is being conducted on possible range/azimuth miscalibrations. If, however I am correct then the error would evidently lie somewhere in Malaysian officialdom technical capabilities in recording and then consolidating the data.
    I expect backlash. Please protect me!

  327. TBill says:

    @Mick
    I am not confusing Kalitta 66 which would have taken off Manassas Va a few miles from my house. I am saying it seems contradictory, but as you say, missing the official report, so I am going from memory of reading the accounts.

  328. Mick Gilbert says:

    @Nederland

    The regulator on the SABRE portable oxygen bottle has separate HI and LO flow outlets and a simple ON/OFF control knob. You get either HI or LO flow by connecting a mask to the relevant outlet. LO flow is for use with children and only supplies 2 litres of 100% oxygen per minute. HI flow provides 7 litres of oxygen per minute. When compared to other portable oxygen systems such as the Scott and AVOX bottles, 7 lpm is nearly double their HI flow rates of 4 lpm but still significantly less than the flight crew oxygen system which delivers 20 lpm. The bottles are stowed with the mask connected to the HI flow outlet. Having earlier checked the specs I noticed that, unlike the masks for the overhead passenger oxygen system, the SABRE masks don’t have inhalation and exhalation check valves meaning that they deliver undiluted 100% oxygen. That makes them far more effective at altitude than the passenger masks.

  329. Nederland says:

    @Mick

    OK, now what do you think would be the maximum altitude at which to use those specific oxygen bottles not just to stay alive, but also to stay conscious? There were more bottles than cabin crew members. Those that retained consciousness would likely have used another bottle, which would then sum up to 1:30 hrs duration (of consciousness). I guess it is impossible to say as it very much depends on individual constitution, but is there even a chance a pilot in the cockpit could have remained functional and or/recover but none in the cabin could do the same?

  330. airlandseaman says:

    Dazza: Here are the KB radar observation times for the MH370 PSR data. The spreadsheet can be used to try out your own assumptions for the antenna rotation rate. The rate I chose minimizes differences between the true observation times computed and the time stamps in the original file. https://goo.gl/q1kv3W

  331. Victor Iannello says:

    @Dazza: Thank you for your thoughts. I am in a holding pattern relative to interpreting the KB radar data. I’m not 100% persuaded by any of the explanations I’ve seen so far, and for that reason, I am listening and thinking more than commenting.

  332. Victor Iannello says:

    @ventus45 said: The result was via Medan to ditch.

    The evidence is most consistent with a high-speed impact close to the 7th arc. I haven’t seen a persuasive reason to abandon that conclusion other than the plane has not yet been found yet. If the current search by OI is completed without finding the debris field, we’ll have to provide an explanation as to why we have the null result, and where to search next, if anywhere.

  333. TBill says:

    @Sfojimbo
    “The Mahathir and Anwar government will be just as committed to coverup as the Razak government was. This is evidenced by Mahathir’s announcement that funding for the search will be reviewed.”

    I am not optimistic about the new government’s handling of MH370. I assume Razak had no particular reason to obfuscate since the suggested perpetrator was an opposition party advocate. Razak only had to be careful not to upset Malaysians who did not like the “Z did it” hypothesis. I know some members of the opposition nonetheless were very angry when Razak originally said the accident was apparent intentional diversion, which could be inferred to possibly include pilot hijacking.

    Just a month ago or so, Mahathir was saying he was thinking someone maybe took remote control of the aircraft using the Boeing patent info. So That does not sound too good to me.

    I hope the final report is left as it is already drafted, but there could be new edits. Just last week the new transport minister said he was not sure when the report would be issued. The prior transport minister said the report would be issued immediately after OI finishes.

    The only silver lining is, if the new gov’t deviates from the apparent truths, perhaps USA or other parties will intervene to give alternate opinions. In short, will be interesting to watch.

    I hope the new gov’t bites the bullet and sticks with the truth as far as we know it, but that may be politically unacceptable to the Malaysian public, not sure.

  334. TBill says:

    @Andrew
    I have no problem with your scenario as one possible passive flight scenario (that has not worked so far to find the aircraft – believe me, I am hoping OI finds it today). I would say the discussion here (@David @Gysbreght et al) has centered on the fact that although we tend to mostly support Arc7 end point (vs. long glide), we cannot convince ourselves that the final descent was unpiloted from the data.

    So I don’t see why we, as a nebulous group of independent investigators, should not be developing both piloted and unpiloted scenarios. Then let the chips fall where they fall.

  335. Richard Godfrey says:

    There is a possible ROV deployment at -26.9534 101.0384 underway for the last 7h 13m.

    This may be a result of analysing the scan data from AUV36, which was picked up this morning at 05:01 UTC.

    The possible ROV deployment was started at 07:58 UTC and ended at 15:11 UTC.

    SC is still in the area and waiting, around 200m from the POI.

  336. MH says:

    @Victor “[I don’t think it’s crazy to think that for some yet undisclosed reason,] Russia was not happy with the loss of MH370, and the downing of MH17 was in reaction to that loss”

    it might be MH370 went missing because Russia was not happy over the 1MDB scandal and they expected a proper response from Malaysia but they stalled too long so Russia shoot down MH17 was another reminder. Shortly afterwards Malaysia made some payment.

    [VI added phrase in brackets so that the quotation is not taken out of context.]

  337. TBill says:

    @Richard
    26.9S looks very good by default (and sim studies)

  338. DennisW says:

    OMG. Fingers crossed.

  339. Gysbreght says:

    An interpretation of civil PSR radar data for MH370

    The following chart has been produced with the spreadsheet that Victor Iannello posted in “The Civilian Radar Data for MH370”, Update 2 on April 12, 2018. Segments A, B, C, D, E, and F are as defined by Victor, except that in Segment B 7.6 seconds (2 antenna revolutions) has been added to the tabulated times. The format of the chart is similar to that of Victor’s comment on May 1, 2018 at 9:12 pm, except that the horizontal scale is distance travelled, where Victor used UTC. The target height is 40500 ft. The green lines in the lower right of the chart show UTC on the right-hand scale versus distance, the full line for MH370 and the dashed line for a constant speed of 530 kts. Above those lines the chart shows the time difference between the two green lines on an enlarged scale on the left. The blue line follows the table values. The red lines bridge the cones of silence and other gaps, with the speeds indicated. The slope of the blue and red lines is upwards when the speed is less than 530 kts, level for speed equal to 530 kts, and downwards for speeds greater than 530 kts.

    Segment D is a bit odd with an average speed of 566 kts.

    https://www.dropbox.com/s/muhs9rkglim06kx/TimeDiff40500ft.pdf?dl=0

  340. David says:

    Anwar, possibility still of a new search:
    https://www.theaustralian.com.au/business/aviation/door-still-open-on-new-hunt-for-mh370-says-malaysias-anwar/news-story/bc824cb3d2f09be0b464e4ecfa2b71aa

    As below:

    The Australian 12:00AM May 26, 2018

    AMANDA HODGE
    Kuala LumpurJakarta
    @hodgeamanda

    Malaysia’s prime minister-in-waiting, Anwar Ibrahim, has left open the door to a new search for missing Malaysia Airlines flight MH370 pending a complete ­investigation into a series of domestic failures that occurred on the night it went missing and in the weeks afterwards.

    Mr Anwar told The Weekend Australian he would ensure his new government conducted a full review of internal documents related to Malaysian radar information on the missing plane’s movements, the failure of three separate monitoring systems — air traffic control, the air force and Malaysia Airlines — to quickly raise the alarm, and discrepancies in MH370’s cargo manifests.

    “This is local information in which we can do further digging. Then perhaps we can reopen communications with the international bodies with our new information,” he said yesterday. “The radar could speak volumes. Was it a failure of the system? Was it a failure of those monitoring the system or was it an intention to ignore or cover up? To me, that is still to be ­determined.

    “I am not ruling out further searches in the future depending on what these domestic investigations bring up.”

    His comments come after Transport Minister Anthony Loke Siew Fook intimated that the current “no find, no fee” search by ocean survey company Ocean Infinity could be the final one for the plane, which went missing on March 8, 2014, 38 minutes into its flight from Kuala Lumpur to Beijing with 239 people on board.

    An earlier $200 million search of a 120,000sq km area of southern Indian Ocean — jointly funded by Australia, Malaysia and China — ended in January last year. The plane’s disappearance remains the world’s biggest aviation mystery.

    Mr Loke told The Weekend Australian the new government had agreed to extend Ocean Infinity’s search to May 29 and would “decide later” on any further action. But, he added, “there would be no withholding of information” and a final report would be made public within three months.

    In an open letter to Malaysia’s Pakatan Harapan (Alliance of Hope) government, which won the election this month, relatives of MH370 passengers urged the new administration to be more transparent than its predecessor and release all relevant ­documents, such as the full cargo manifest. For several weeks after MH370 disappeared, Malaysian authorities refused to hand over the plane’s cargo manifest, fuelling theories the plane may have been taken over remotely to prevent sensitive cargo reaching ­Beijing. When they finally did, there were disparities relating to ­2.2 tonnes of cargo initially ­described as lithium batteries and then as “radio accessories and chargers”.

    Mahathir Mohamad, the former Malaysian strongman who was sworn in as the country’s seventh Prime Minister this month, told The Australian in March that he believed the remote takeover theory was plausible.

    Voice 370 has also called for a probe into possible tampering of MH370 records, or an “act of ­omission that may have impaired tracking, search, rescue and recovery”.

    Mr Anwar said he believed there were “a lot of questionable decisions made, discrepancies in cargo and passenger lists”, as well as in the government’s initial ­assertion that the plane flew over the South China Sea. It changed its version only after Thai authorities revealed radar information showing MH370 flew over mainland Malaysia before heading ­towards the Indian Ocean. “They (the former government) have to explain why they had a different version. Clearly we need to look at such failures before we can have final closure,” he said. “I intend to call the Transport Minister … and ensure these things are being done. I believe the radar in the northeast can tell us more than what was clear then.

    “The cargo was also important because what was described by authorities and what was in the cargo was totally different. We went through the list — which ­included mangosteens and technical equipment — but that equipment did not tally (when cross-referenced with the Penang factory that shipped it).”

    Mr Anwar has given little credence to the theory the plane was hijacked by pilot Zaharie Shah, a member of his People’s Justice Party (PKR) that is one of four parties in the governing coalition. Although the two were distantly related by marriage — a daughter of one of Captain Zaharie’s cousins is married to Mr Anwar’s son — and had met a few times, they did not know each other.

    Zaharie and co-pilot Fariq Abdul Hamid were initially named by Malaysian authorities as suspects in the aircraft’s dis­appearance, and conspiracy theories quickly followed. One had the 53-year-old ditching the plane in despair at the end of his marriage, or at the end of a love affair with a married woman. Another suggested he was so enraged at Mr Anwar’s politically motivated second conviction for sodomy, given hours before MH370 was due to take off, that he hijacked his own plane to destabilise the government of Najib Razak.

    Sivarasa Rasiah, Mr Anwar’s lawyer and a PKR MP who ­befriended Zaharie after he joined the party, told The Weekend Australian the pilot was not in court the evening Mr Anwar was convicted on a second trumped-up sodomy charge because his MH370 flight was scheduled to take off four hours later.

    Mr Sivarasa does not believe the pilot murder-suicide theory and said it was a smokescreen for systemic failures in the hours and weeks after plane’s disappeared.

    “The basic issue is two independent Malaysian agencies, air traffic control and the air force, failed that night. If you include MAS, then that’s three. How do you explain this? Where is the ­inquiry into this? If a military jet had been scrambled (at the first sign of the plane changing course), we would not be having this conversation. That’s why it’s so easy to believe there is much more to this than meets the eye.”

  341. TimR says:

    @All, It is quite a coincidence that MH370, following waypoints round below Sumatra to Christmas Island, flies for several hours at a steady ground speed of 370 kts while meeting all the BTO’s up to Christmas Island.

  342. Mick Gilbert says:

    @Nederland

    Re: ‘… what do you think would be the maximum altitude at which to use those specific oxygen bottles not just to stay alive, but also to stay conscious?

    Short answer, I have no idea. The issue is maintaining a sufficient partial pressure of oxygen in the lungs which becomes problematic above around 34,000 feet. Up to 34,000 feet diluted oxygen at ambient pressure will sustain consciousness, above 34,000 feet you need 100% oxygen at ambient and then above 40,000 you need 100% oxygen delivered at pressure. The usual advertising caveat applies – actual results may vary.

    We know from the Helios 552 accident that a crew member (aged 24) managed to remain conscious in a depressurised airplane at 34,000 feet for two and half hours, presumably by using the walk around bottles. The airplane’s complement of four bottles were found in the wreckage; three showed signs of use. Helios carried Scott bottles with a HI flow rate of 4 litres per minute.

    MH370 carried a very experienced cabin crew but the youngest member was aged 34. I’d say that it would be doubtful if any of them could have maintained consciousness for a sustained period in a depressurised airplane above 40,000 feet.

  343. Sfojimbo says:

    @TimR
    If you plot flight paths at various speeds from POVUS that connect to the ring segments, you can plot the path to about anywhere. But in order to get from POVUS to Christmas Island the path would have to pass over Indonesian airspace at several points. I don’t show a path at 370kts going anywhere as far east as Christmas Island. Are you sure that you’ve plotted your ping rings accurately? Here’s a check, the 19:41 ring should intersect the Equator at about 93:42:36 E; if it doesn’t, something is wrong.

    Also, 370kts is a very unlikely gs for a 772.

  344. TBill says:

    @Mick
    @Nederland
    Mick it looks like you are quoting actual altitude FL340 of Helios aircraft, not the internal cabin altitude. Which to my recollection was closer to FL250 inside the aircraft.

    I am not expecting the O2 bottles to be very effective above FL300 or so (inside the cabin).

  345. TBill says:

    @ALSM
    What are you thinking FO cell connect at Penang?
    FL430 seems a stretch but I am not the expert.

  346. DrB says:

    @All,

    HERE is a table I have compiled comparing four different methods of predicting 9M-MRO crash site coordinates: range limit, BFO bounds, BTO/BFO fits assuming autopilot after 19:41, and debris drift analyses.

    Such a comparison is useful when considering the question of where to search next.

    My predicted range limits (using my fuel model) are 14-39 degrees South, and ATSB/Boeing got 18-38 degrees South. All in all, that is satisfactory agreement in my book.

    ATSB/DSTG figured the BFO bounds to be 24.7-34.5 degrees South (allowing ± 5 Hz mean errors).

    Fits to BTO/BFO, assuming autopiloted flight after 19:41, vary widely from 26.5-39.5 for Great Circles. For my single fit of 181.2 degrees magnetic track, my predicted end point is 31.6S.

    Five forward drift studies have remarkably similar predictions. All have northern limits from 25-28 degrees South, and the southern limits are 34.5-38 degrees South (using the drift model results only). I did not include several reverse-drift analyses since they are nowhere near the 7th Arc.

    The ocean surface search for floating debris is also potentially useful in excluding certain areas from consideration. In my opinion, the lack of any definitive identification of 9M-MRO debris in the surface search has been overstated by two drift modelers (Griffin and Nesterov). I say this primarily because the analysts do not appear to have previous experience in working this particular problem. I know from personal experience that assessment of aerial ocean search performance is a very difficult task, and lots of things can and do go wrong. Besides, the search aircraft did detect some objects which were never relocated and examined by the surface ships. Therefore, I find it a stretch for Griffin to conclude, for instance, that the aircraft could not be north of 32S. I also note that the ATSB recommended to OI search areas extending well north of 32S, so Griffin’s interpretation of the negative surface search was apparently not shared by ATSB. In the table I included three of CSIRO’s predictions, two of which involve additional assumptions unrelated to drift modeling. Their drift-model-only prediction is consistent with the other four drift models.

    In summary, the aircraft performance, BTO/BFO data, and debris drift modeling all consistently indicate a crash site in the vicinity of the 7th Arc between about 28-34S. Yet that area has been searched by Ocean Infinity unsuccessfully within 22 NM. The drift model predictions (and the aircraft range predictions) encompass areas farther than 22 NM from the 7th Arc. The 22 NM search limit chosen by OI may turn out to be inadequate to encompass the actual aircraft track between 00:19 and impact, and expanding the search width would now appear to be the best option for a future search, encompassing 28-34S with an initial preference for the central portion.

  347. Sfojimbo says:

    The raw data is off, there is no doubt. The plot at 18:00:51 is 15 miles beyond the range of the radar.

    The zooming chandelle took place at IGARI, and that would have just barely gotten 9M-MRO up to the region of FL43. There’s no way ZS held it at that altitude for 250 miles.

    These are physical impossibilities.

  348. Mick Gilbert says:

    @TBill

    Yes, you’re quite correct, Bill. Although Helios 552 was at FL340, based on subsequent simulations, the cabin was thought to be the equivalent of around FL260. I had forgotten about that. I suspect that you would get a similar but not as marked differential between external and cabin pressure with a deliberate depressurisation, maybe a couple of thousand feet difference with the outflow valves open and air packs off.

    As to the efficacy of the walk around bottles in maintaining consciousness, there are obviously a range of factors to consider. On the simple metric of providing a sufficient partial pressure of oxygen in the lungs to maintain the required levels of oxygen saturation (SO2) in arterial blood to maintain consciousness then for most people the supply of about 700cc of 100% oxygen at ambient pressure on each inhalation between around 34,000 feet and 40,000 feet should do the job. The SABRE walk around kit on 9M-MRO easily meets that requirement on HI flow due to the flow rate (7 lpm) and the mask design (no dilution). The non-diluting mask comes by virtue of the equipment’s dual purpose role as medical oxygen. All of that needs to be tempered with the understanding that none of this kit, the flight crew emergency oxygen supply system included, is designed for sustained use at high altitude. Its purpose is to provide an emergency ‘get me down’ capability.

    That covers hypoxia; decompression sickness is another matter altogether.

  349. ventus45 says:

    @All

    So much to catch up on !

    I have limited time this weekend.
    I have to wrap up now and move my butt.
    I have only had time to address Tim’s and Victor’s posts.
    sorry, the others will have to wait.

    @TBill – May 24, 2018 at 11:45 pm
    Greg Hood, Chief Commissioner ATSB:
    “At the ATSB, we exercise great care not to engage in conjecture or speculation.”
    TBill:
    “… how can we solve the problem without speculation about what might have happened ?”
    “That is exactly the missing ingredient to date.”

    True, I agree.

    We have M1 = Motive(s).
    We have M2 = Means.
    But we DO NOT HAVE M3 = Method.

    The ATSB has ignored M1 outright, as it is not in their remit.
    The ATSB has ignored M3, as it is not in their remit either.
    The ATSB has only considered M2.
    What they have done, is limit their considerations to “ONLY” what the M2 can do “by itself”, ie, unpiloted.
    That in itself is a major constraint, because it limits their considerations, to only what the fuel and performance limits can define, or “bound”, as reachable areas, and which are consistent with the arc.
    That major constraint, has, so far, only got us a null result.

    @Victor Iannello – May 25, 2018 at 10:17 am
    “If the current search by OI is completed without finding the debris field, we’ll have to provide an explanation as to why we have the null result, and where to search next, if anywhere.”

    Why indeed – do we have nothing but a null result ?
    That is now the BURNING question.
    That is what we have to resolve.

    The ATSB, and all the ping analystists, were all so SURE, and all so CONFIDENT, that they were “on the money”.

    But the sad fact is, that as every day passes (and there a precious few left), with OI continuing to doggedly work their way up the arc (for which we should all be very grateful) it is becoming increasingly obvious, even to “blind Freddie in the street”, that “the experts” have lost their bet. Their odds-on favourite didn’t win, it didn’t run “true to form”.

    They (and all the punters, who accepted their “tip”) have “done their money”.
    The “professional punters” may be philosophical about it, but the “man in the street punters”, are not.
    They are now very “not happy Jan”, and many are now muttering, that is time for a “Stewards Enquiry”.
    This is not a joke, it is real.
    It is telling to note, that at the end of the Senate Hearing, that this very issue does get a mention.
    There have been calls for a “References” inquiry, into the ATSB’s handling of the search, which would be a very serious affair if it occurs.
    “Reference Hearings” are up another level, perhaps two levels, and are much more like an inquisition, than a friendly “chewing the fat type chat”.
    The chair, full well knowing this, clearly indicated that he did not want to precipitate that.
    Further, he clearly stated, that he would be much more inclined to “leave it to the next chair”.
    Why ? Politically, for him, and the Minister responsible, it would be a megga hot potato, that’s why.
    As a fall back “cover his arse” move, he requested a “private briefing”, for himself and the other senators, from Foley and Hood, so as to “better acquaint themselves” with the matter, “just in case” external pressure for such an enquiry persists, to the point that (politically) it can not be avoided, and will have to occur.
    Early days. Watch this space.

    Victor:
    “The evidence is most consistent with a high-speed impact close to the 7th arc.”

    “Most consistent”, does not definitively equate to a definitive “IS”.
    That is now the knub of the issue.
    Up until this point, most (not all) have accepted that linkage.
    It is time to revisit this.

    You used a broad brush with the word “evidence” above.
    Did you mean the ping evidence, or the recovered debris evidence, or both ?

    If just considering the ping evidence – I presume you mean the 7th arc eight second BFO’s.
    I am not able to dispute Holland’s maths on that, but at the same time, I am not convinced that the 7th ping occured in flight at all.
    In fact, I think the 7th pings occured in the water, while floating.
    Consider the accepted “in flight” time line – backwards.
    00:19 – 7th ping – the eight seconds – intermediate altitude – high and increasing ROD.
    00:17 – second (left) engine flameout – APU start – 2 minutes to SDU log-on.
    00:11 – 6th arc – single engine cruise – maintaining speed and heading – slow “drift down”.
    00:02 – first (right) engine flameout – loss of stable high altitude cruise – maintain high TAS/M and GS but now slight drift down commences.
    23:41 – still in normal cruise, all engines running.
    Now – keeping DrB’s fuel model in mind, consider my ditch scenario – forwards.
    23:41 – TOD – reduce to flight idle – commence descent – turn left to 135 true.
    00:00 – BOD – now level at 2,000 amsl – TAS/IAS/GS all about 240 kn – on 135 true.
    00:02 – NOTE: – no flame out – idle descent for 21 minutes has reduced fuel consumption from cruise values – ie, saved fuel.
    00:11 – 6th arc – low – slow.
    00:11 to 0016 – 5 minutes for surveying intended ditch area – clear of ships – maneouvering – left downwind to base then left base to final turn – configuring as you go – preparing to ditch.
    00:16 – configured – powered – short final – heading now 315 – aligned with wave crest – with rising sun over right shoulder – with aircraft shadow closing rapidly out left window – necessary to judge flare in final few seconds in featureless (no other useable visual cues) ocean.
    00:17 – ditch – Trents Drown – APU starts – two minutes to SDU logon.
    00:19 – 7th ping – the eight seconds – wave enters APU intake – APU DROWNS – power lost – end of Inmarsat data.
    The eight second BFO’s (and the way off BTO’s as well) were most probably the result of corrupt position and velocity data from the AIMS being fed to the SDU. Considering the dynamics of the sudden decellearation, possible violent yaw etc, during the approximately 500 feet (or less) “water run”, it is more than likely that the gyros etc were scrambled, and thus AIMS produced junk data on the bus to the SDU two minutes later.

    Just by the way, in case the penny hasn’t dropped for some yet, the course reversal to 315, back towards the 6th arc, produces a position very close to the 6th arc, in fact, nowhere near the accepted 7th arc, an obviously significant matter, if a sensible ditch, was set up, by a pilot.

    If just considering the recovered debris.
    The first, and SCREAMING question is, where is all the detailed forensic examinaion of every single piece ?
    There is not one single “fullsome” report, not one, of evidencial standard, on any, repeat any, individual item, not one.
    We have only been given a few “snippets” of information.
    Why is that ?
    If this was a court case, and those items were deemed to be “materiel evidence”, there would be hell to pay.
    Read, “References” Inquiry.

    Victor:
    “I haven’t seen a persuasive reason to abandon that conclusion, other than the plane has not yet been found yet.”

    I would have though that is a pretty powerful reason, more than sufficient in fact, in and of itself !!

  350. David says:

    @Ventus45. I may have got lost somewhere. The 7th arc pings were at the 6th arc?

    Also, ELT transmission?

    Why an inquiry in Australia into lack of the damage assessments Malaysia should have done? There has been no public disclosure if they have.
    Australia did identity assessments.

    Certainly one would call those on the Malaysian site a mix of identity and initial. They do not address sequence or aircraft attitude and speed implications. The final report will not meet with Malaysian obligations without proper damage assessments.

    (incidentally, two gaps which are likely to remain are inability to access the flaperon (France)and vortex generator (Madagascar)).

  351. Andrew says:

    @ventus45

    In your scenario the fuselage must have remained intact for there to be an autostart of the APU, SATCOM transmissions, etc. How then, do you explain the debris from the cabin that has been found?

  352. TimR says:

    @Sfojimbo, “the 19:41 ring should intersect the Equator at about 93:42:36 E”

    The 19:41 arc is right on 93:42:36 E.

  353. Sfojimbo says:

    @TimR
    OK let’s go one more. Where does your 370 kt plot hit the 0:19 ring?

  354. Richard Godfrey says:

    After a pause due to bad weather, SC is continuing to move further north up the Broken Ridge plateau area and has reached 26.6249°S.

    In the last 24 hours the remaining 2 AUVs in the water were collected, but no new AUV deployments were made.

    SC was stationary at 26.9532°S 101.0386°E for a period of 6h 46m yesterday. This does not appear to have been a ROV deployment however. The location was 21 NM outside the 7th Arc and close to where one of the AUVs was picked up. It is more likely that SC was sitting out the bad weather.

    The weather is improving, there is good visibility, with a 13 knot wind, a combined swell and wave height of 4.1 m. The tropical storm S.E. of the search area has moved away.

    On the graphic linked below, the point were SC was stationary is marked S.

    The location where the underwater beacon detected on 4th April 2014 by the Chinese vessel Haixun 01 at 25.975°S 101.461°E, around 8 NM outside the 7th Arc, is marked H1. The water depth at this location is around 4,500 m.

    https://www.dropbox.com/s/hgn8jghi72pkh6f/SC%20Track%2026052018.pdf?dl=0

  355. Sabine Lechtenfeld says:

    I had decided to abstain from commenting until the search is truly finished and we can move on to either celebration or post mortem procedures. But I decided to chime in when I read about the renewed speculations that Russia may have either shot down MH17 deliberately or at least ordered the shootdown. Personally I believe that Putin is an undemocratic despot without many moral qualms whatsoever, and there’s not a lot I would put past him – not even order the shootdown of a civil airliner if it would further his long term goals. But the idea that MH17 was taken down deliberately by Russia or at least at their order, is plain nonsense for a very simple logical reason:
    The shootdown could never have happened if on that fateful day the Ukrainian ATC hadn’t directed the planes smack over the rebels’ territory on a far too low altitude. This was criminally negligent at best, and several official reports have stated that the Ukraine can indeed be partly blamed for having enabled the tragedy. Therefore a deliberate shootdown ordered by Russia is not possible because they had no way of knowing that there would even be an opportunity for taking down the plane. Apart from that the shootdown proved to be very damaging for Russia, because they have been morally blamed (rightfully) for helping the rebels and supplying them with weapons. This is undeniably true and the immediate international reaction was a moral outcry with very negative consequences for Russia. The Ukraine was supported far more vigorously than before the shootdown by those nations which opposed the Krim invasion. And Russia was hit with very effective long term economic sanctions. Apart from maybe having a grudge against certain Malaysian factions because of their embezzling habits, Russia had no motive whatsoever for shooting down MH17 deliberately. And if the Ukrainian ATC had provided the planes with a sensible route instead of a reckless one, the shootdown wouldn’t have been possible in the first place.

  356. DennisW says:

    @Richard

    Bummer. I had my hopes up. Thx for the update.

  357. Sabine Lechtenfeld says:

    @ventus45, if I understand your scenario correctly, you describe more or less how you would’ve ditched the plane if you had been the pilot. Your procedure may be sensible. Since I’m not a pilot I can’t judge. But the big problem is that there isn’t a scrap of material evidence that it did actually happen that way. And you simply declare all evidence against your scenario as either irrelevant (last primary radar capture over the Strait), badly or not sufficiently assessed (debris) or junk data (last BTOs and BFOs). As Andrew has pointed out, the debris which has been found so far, seems to contradict the notion that the plane remained as intact as it would be necessary for your scenario. This needs to get addressed more specifically by you IMO. And do you have any solid hints whatsoever that the last BTOs/BFOs are junk data which originated after a ditch? Is there anything in the numbers which caused you to question them – or do you just postulate that they must be wrong because your scenario only works if these pesky data can be explained away somehow?

  358. Mick Gilbert says:

    @Sabine Lechtenfeld

    Re: ‘… the Ukrainian ATC hadn’t directed the planes smack over the rebels’ territory on a far too low altitude.

    Would you care to expand upon that? My understanding is that at the time of the shoot down L980 was a well used airway through that part of Europe. There had in fact been 160 other civilian flights pass through that airspace on the day MH17 was shot down. Airlines such EVA, Etihad, KLM, Lufthansa, Singapore Airlines, United Airlines and Virgin Atlantic, all among the top 20 safest airlines in the world, were flying over the eastern Ukraine at the time. On the day of the shoot down the Ukrainians issued NOTAMs 1492/14 and 1493/14 prohibiting civilian air traffic below FL320. MH17’s flight plan and actual flight were compliant with those restrictions. It is a matter of public record that rather than climbing from FL330 to FL350 prior to entering Ukrainian airspace as per its flight plan and as requested by Ukrainian ATC, MH17 elected to maintain FL330.

  359. Mick Gilbert says:

    @Sabine Lechtenfeld

    My mistake, NOTAMs 1492/14 and 1493/14 were issued on 14 July, three days prior to the shoot down.

  360. Victor Iannello says:

    @Sabine: Here is what Wikipedia says:

    According to the original flight plan, MH17 was to fly over Ukraine at flight level 330 (33,000 feet or 10,060 metres) and then change to FL 350 around the Ukrainian city of Dnipropetrovsk. When it reached the area as planned, at 15:53 local time (12:53 UTC), Dnipropetrovsk Air Control (Dnipro Control) asked MH17 if they could climb to FL 350 as planned, and also to avoid a potential separation conflict with another flight, Singapore Airlines Flight 351, also at FL 330. The crew asked to remain at FL 330 and the air control approved this request, moving the other flight to FL 350. At 16:00 local time (13:00 UTC), the crew asked for a deviation of 20 nautical miles (37 km) to the left (north) of course, on airway L980, due to weather conditions. This request was also approved by Dnipro Control. The crew then asked if they could climb to FL 340, which was rejected as this flight level was not available, so MH17 remained at FL 330. At 16:19 local time (13:19 UTC), Dnipro Control noticed that the flight was 3.6 nautical miles (6.7 km) north of the centreline of approved track and instructed MH17 to return to the track. At 16:19 local time (13:19 UTC), Dnipro Control contacted Russian air control in Rostov-on-Don (RND) by telephone and requested clearance to transfer the flight to Russian air control. After obtaining the permission, Dnipro Control attempted to hand off the aircraft to Rostov-on-Don at 16:20 local time (13:20 UTC), but the aircraft did not respond. When MH17 did not respond to several calls, Dnipro Control contacted RND again to check if they could see the Boeing on their radar. RND confirmed that the plane had disappeared.

    Do you agree with this?

  361. HB says:

    @David re “Why an inquiry in Australia into lack of the damage assessments Malaysia should have done?”
    This is an interesting question.

    For sure ATSB responsibility was to define the search area and undertake the search.
    Malaysia lead investigation team was to perform analysis in accordance with ICAOA13 and had overall accountability
    French did their own study on the flaperon as per their regulation requirements.

    However, Malaysia said clearly they dont have the competency to perform such detailed analysis and they will defer that to australia. ATSB was certainly aware of that.
    So the key question is whether ATSB had access to the detailed report of the flaperon?
    Were they instructed to perform the analysis for the other pieces by Malaysia?
    Did they even request this data from the French or from Malaysia to inform the search?
    Did they request Malaysia budget to perform the analysis which could have had implications on search area?
    There is clearly a large overlap in the zone of accountability and responsibility for the search between the parties involved.

    An inquiry will be useful in that sense.
    It will be clear if the issue was with the analysis or with the basis for the analysis.
    I have supported public inquiries before and this is extremely useful. It questions all the basis that lead to such a situation and identifies all the root causes. Not necessarily a blame exercise.

  362. Sabine Lechtenfeld says:

    Yes, Victor, this agrees mostly with my information from German Wikipedia and other sources. And I think, we can isolate two critical and decisive moments: 12:53 UTC when MH17’s crew was asked by the Ukrainian ATC to climb to floor level 350 as planned, but they refused. How could the Russians have known this in advance, even if they had learned about MH17’s flight plan somehow?
    The next decisive moment is at 13:00, when the crew asks for a diversion because of adverse weather conditions, which was granted by the ATC. The crew also asks for a climb to floor level 340, which was denied. None of these events was forseeable by the Russians. So, how can they have planned to shoot down MH17? The actually flown route was the result of critical interactions between Ukrainian ATC and the crew of MH17 on that day. Apparently there is still a question mark concerning the slight deviation detected by the ATC near the end of the flight , which to my knowledge hasn’t been explained. I don’t know if it influenced the eventual outcome.
    Where the Ukraine is at clearly at fault: they maintained that floor level 330 was still safe, although it was only marginally higher than the lowest altitude over the areas of conflict which was still deemed safe. And they maintained that the areas which were actually overflown on that day could be crossed safely at a floor level as low a 330. This has been reiterated in various Dutch reports. I also remember that weeks before the MH17 shootdown happened, it has been warned that it was only a matter of time before a civil airplane would be downed by the rebels.
    The decision of the MH17 crew to refuse floor level 350, when it was available could be questioned, too.
    This does not absolve the Russians from having supplied the rebels with weapons and other aids.

  363. Victor Iannello says:

    @Sabine: OK. I understand better. You’re point of contention is that Russia could not have planned in advance to shoot down MH17 because the actual path taken would not have been known prior to entering the area.

  364. Sabine Lechtenfeld says:

    I remember vividly the day when MH17 was shot down. When I learned that the rebels had downed a civil airliner, I was shocked but not overly surprised. This has been brewing for some time, since the rebels had already shot down a few Ukrainian military planes . When I learned that the downed plane was a MAS 777, I was dumbfounded, though.

  365. Sabine Lechtenfeld says:

    Yes, Victor, exactly. Maybe, I haven’t explained myself very well in my previous comment.

  366. TBill says:

    @DrB
    I appreciate your table, but where to search next is problematic. At the moment I tend to favor going up Arc7 +-25nm to at least 20S. Your table shows that 38-39S is possible re: fuel, and there is a small army of proponents who want to go back there with a wider search.

    Since your orig pin was down around 38S, it would be valuable to hear why you do not think the aircraft is down there. @Globusmax is even citing your earlier work on contrails as supporting evidence in favor of the 38S region.

    I am about ready to toss the drift models out the window, unless Richard or someone can convince me otherwise. Everything floating goes West in the whole SIO. As @PaulS points out, we probably cannot rule out 38S for drift source, although on average some 38S drifters will go to OZ, as you know, on any given Sunday, the Dallas Cowboys can be beat (preferably by the Redskins). In other words, on that day’s currents, the drifters skipped OZ, according to Paul’s interpretation of the currents on that day.

    I feel we need to reassess assumptions, and before we can reassess assumptions, we need to make a list of assumptions, which I am working on (n my head so far). Even if it was a passive flight, why is it so impossible the pilot put in 2 waypoints, or on the really wild side, three?

  367. Sfojimbo says:

    As someone who has actually operated a Sam site (acquisition radar operator on a NIKE site) and as someone who has studied the facts surrounding the MH-17 shootdown in great detail, I find it necessary to ask the moderator of this site if he wishes to have a debate about MH-17 take place on this page. If it is OK with Victor, I’m up for it.

    But for now, I would just like to point out that the BUK TELAR that brought down MH-17, was part of the Russian 53rd air defense Brigade and it is clear as a bell to me that it was manned by the soldiers of the 53rd Brigade; it could be no other way. Moreover I will point out that the firing position selected for the TELAR (47°58’15.21″N 38°45’51.58″E) was almost directly underneath the intersection of three navigational airways (L-69, M-70 and L-980 which MH-17 was traveling on), this is a circumstance seldom found at any point on the globe.

    The TELAR operator had to have known that he was targeting an airliner and it is a certainty that he was ordered to do so by his superiors. I find no way of determining at what level of command the decision to take down an airliner was made, but I am sure it was no accident.

    Attempting to place blame on Ukraine for this incident is misplaced to say the least. Airlines have flown for years over troubles in Yugoslavia, Afghanistan, the Gulf, Central America and other regions with rebels below. Ukraine is no exception.

    Then comes the subject of Russian lies………………..

  368. Victor Iannello says:

    I’ve heard several proposals about where to search next, each discounting the validity of some piece of evidence. Before we search wide, I’d like to better understand how in an uncontrolled EOF, it’s possible to satisfy the final BFO values AND end further than 22 NM from the 7th arc, which is what I think some are proposing. If that’s impossible, that means we have to consider a controlled EOF, which means there is the possibility of pilot inputs at any time during the flight. That in turn means it will be practically impossible to know where to search next without introducing other constraints, such as a particular scenario that favors a particular location. At this point, I’m just not seeing a clear way forward.

  369. Victor Iannello says:

    @Sfojimbo: I’m okay with it for now, especially if it relates back to MH370. If it becomes a distraction, I’ll suggest we move on.

  370. Sabine Lechtenfeld says:

    @Sfojimbo,
    I don’t deny at all that the Russian rebels have downed MH17, and that they were probably aided by Russian military. I’m also not disputing that they have been negligent since civil airliners were still flying over the conflict area at an altitude which was wrongfully still deemed safe by the Ukrainians. That’s not my point of contention. But I think it’s totally misplaced to believe that the Russians had targeted deliberately a civil airliner or even MH17 specifically. They simply couldn’t have known in advance that the machine would fly the route it did on that day.There are however many hints, which were gathered from certain social media and Russian news outlets, that they believed to have shot down an An-26 – as they had done just three days before.

    The Ukrainians are clearly at fault, too, because it was within their authority alone to close the area alltogether for civil air traffic or at least increase the minimum altitude for civil airliners. Eurocontrol had spoken to Ukrainian officials mere days before the shootdown and asked them to close down the airspace of the conflict area, since the situation was obviously dangerous. Around 20 military planes had already been shot down. Three days before MH17 was downed, the rebels had shot down an An-26. The risk for civil airliners was apparently obvious to many experts. The downing of MH17 wasn’t coming out of the blue, but the Ukrainians had refused to implement appropriate safety measures, even after experts had talked to them. That’s what I meant, when I wrote a bit flippantly that the Ukrainian ATC sent the planes “smack over rebel territory at an altitude which was too low”. Sorry for this very imprecise expression.

    I don’t want to start an extensive discussion of MH17, although I think it’s totally legitimate to take a closer look at the circumstances. If MH17 had been shot down deliberately by someone who knew full well the true identity of the plane, then the question if there could possibly be a connection to the disappearance of MH370 would definitely arise.

  371. Sfojimbo says:

    Sabine, you’ve said your piece and I’ve said mine. Unless you want to specifically challenge anything I’ve said, I’m happy to let it rest here.

  372. ErikN says:

    @Victor “At this point, I’m just not seeing a clear way forward.”

    This is, I think, spot on. There is not one I can see either. Lists of absolute facts known (as per what @DrB was working on) and assumptions made driving prior search (as I think @TBill is thinking about) would be somewhat helpful. But I think the point is any search is based on some assumptions. And, with a null result, some important assumptions have been proved false. At this point, there is equal weight, from my perspective, to searching about the whole SIO to a very wide band from the 7th Arc. And that weight is not really very heavy. Some serious open book investigations by Malaysia under the new government would be a start. As time passes and government changes, the sharing of “sensitive” information becomes less cause for concern by Malaysia (capabilities change, etc.). So, let’s hope over the coming months that is what happens.

  373. Victor Iannello says:

    @ventus45 said: The eight second BFO’s (and the way off BTO’s as well) were most probably the result of corrupt position and velocity data from the AIMS being fed to the SDU.

    Even if we accept that the navigation data fed to the SDU was corrupt (which I don’t accept) after a ditching, producing abnormal BFOs, the BTO values do not use the navigation data. In your scenario, the plane should be found on the 7th arc, within the statistical limits of the BTO.

  374. Sabine Lechtenfeld says:

    @Sfojimbo, that’s perfectly ok with me as far as the exchange between you and me is concerned.

  375. globusmax says:

    @TBill

    I never said 38S. I am sure DR B will weigh in, but this is his study that I reference:

    https://drive.google.com/file/d/0BzOIIFNlx2aUWEtvSjBVS2JWX0E/view

  376. Victor Iannello says:

    In a new article in The Times, David Griffin insists that his calculations were right.

    Dr Griffin said he was very disappointed that the aircraft had not been found but that he backed his initial findings. “I can’t help thinking it’s somewhere in the vicinity of [latitude] 35s,” he said.

    He said “a small percentage” of the 35,000 square miles of seabed covered by the new search contained pockets of terrain too rugged for unmanned search craft to enter, making it possible that the wreckage had been missed. It was also possible that the aircraft had flown farther than experts had calculated.

    So if OI is not successful, we will have a long line of people insisting that their suggested point of impact along the 7th arc was correct, except the search was not wide enough, or the debris field was passed over and missed.

  377. airlandseaman says:

    I could agree with David that his short drift analysis was probably correct. But I never had any confidence in the debris being from MH370. The French satellite photos only had a hand full of pixels. No way to tell what the debris was. Aerial photos were much better.

  378. Shadynuk says:

    @Victor You wrote: “So if OI is not successful, we will have a long line of people insisting that their suggested point of impact along the 7th arc was correct, except the search was not wide enough, or the debris field was passed over and missed.”

    Yes, that is almost certainly true. For example, the fact that both Fugro and OI have returned to locations for second looks strengthens the view that it could have been missed. Unlikely, but possible. Tough business – searching with insufficient information.

    Perhaps you or someone can answer this. Were all the radios on the plane working during the flight – at least up to the FMT – assuming there really was an ‘FMT’? I ask because I wonder if there could have been traffic on some frequency that would cause the pilot to take actions such as a turn or alter altitude/speed. I mean traffic from a source such as ATC, SAR, military, radar sites, other aircraft ….

  379. airlandseaman says:

    Regarding radios: There is no evidence of any radio failures. All we know is that no one on 370 answered calls after 17:21. If this was not an accident, then you can be sure the PF was listening on various ATC frequencies, including 121.5 MHz, for traffic related to the diversion.

  380. Mick Gilbert says:

    @airlandseaman

    Re: ‘There is no evidence of any radio failures.

    What would evidence of a radio failure look like?

  381. Shadynuk says:

    @Mick I would take as ‘evidence’ repeated weak or very ‘squelchy’ incoming calls with nothing discernible expect the A/C call sign. Given the suspected intent of this diversion, that likely did not happen. I was asking more because I am not certain if perhaps all the suspected fiddling with breakers and power buses may have left the radios without power.

  382. David says:

    @HB. Re Malaysian responsibility for damage assessment, they might have tasked Australia though there is no sign of that. They held many of the recovered items. The ATSB told me that the part outer flap had been sent up there, together with all else the ATSB had I presume since the ATSB stopped MH370 work some time back.

    Of course the ATSB did look into the flap/flaperon contact, relevant to search width and had biology looked into, relevant to drift.

    I hope to post later today a full assessment of what Larry Vance has drawn from flaperon and flap damage and some other points he made, of interest to you I think.

  383. airlandseaman says:

    Mick Gilbert: My point was that a radio failure and a choice not to answer are indistinguishable. If the latter, you can bet they were monitoring traffic, just not transmitting.

  384. Sfojimbo says:

    ZS’s radio traffic monitoring told him at 18:22 that his plan had worked so far, he had evaded being reported for the two hours that he had given himself by turning around in the middle of FIR turnover.

    If he had his radar data worked out perfectly, at 18:22 he knew that he was running three minutes late; he would pass out of range of Pulau Penang in three more minutes.

    After 18:25 or so, he knew he was home free.

    The big mystery to me is: how did he know that the government was going to hide the radar returns from Pulau Penang? They did hide those returns, they are still keeping them secret, and he obviously knew this in advance.

  385. Mick Gilbert says:

    @airlandseaman

    Thanks Mike, I agree with you that a radio failure and a choice not to answer are indistinguishable.

  386. DennisW says:

    @all

    I feel one of the neglected analytical possibilities is integrated Doppler. I took several passes at it (mostly unpublished). I would encourage others to take a look at it. For example, the integrated Doppler between the satellite and Perth is trivial to calculate. The integrated Doppler to any point on the 7th arc, likewise. The integrated Doppler for the flight path is less obvious. I believe that it is the same as the integrated Doppler relative to the nominal position of the satellite (the SDU compensates for the integrated Doppler to that point). Think about it.

  387. Shadynuk says:

    @Sfojimbo Thanks. That is the kind of info I was trying to get at.

    I was more interested to know if the radios were without power than if they had ‘failed’. There are several ways for a radio to ‘fail’ that are distinguishable from choosing not to transmit. I know, I have my share of weak, scratchy, intermittent ….. radios.

    Sorry I did not come through 5 X 5!

  388. Victor Iannello says:

    @David: I read Larry Vance’s book tonight. He is sure that the witness marks on the flap and flaperon suggest the flaps were not deployed, and he often uses circular logic to show that the other physical evidence is consistent with that. It’s too bad that the ATSB is reluctant to challenge his interpretation of the witness marks, as they are most qualified to do this, having had accident investigators examine the flap first-hand. Larry fails miserably at refuting the conclusion that the final BFOs show an accelerating descent, and makes statements showing that he either didn’t read or understand Ian Holland’s paper.

  389. ST says:

    @Victor -Re:So if OI is not successful, we will have a long line of people insisting that their suggested point of impact along the 7th arc was correct, except the search was not wide enough, or the debris field was passed over and missed.

    Here is one that supports your above statement from former Malaysian Airlines executives.

    https://globalnews.ca/news/4229463/malaysian-airlines-mh370-mystery/

  390. Sfojimbo says:

    @ST
    MAS is owned by the government, as such, “executives” of MAS can be assumed to have held their positions due to loyalty to the regime in power – if they were not direct appointees of the Razak government (aka lackeys).

    These people should not be expected to be unbiased. Their line extolling the idea that “MH-370 may never be found” sounds like the party line to me.

  391. ST says:

    @Sfojimbo – Exactly but my point is that what Victor mentioned in his post on people being in line to say that OI or prior searches missed the flight along the 7th arc is already reflected in several different news articles on MH370 that I have read in the past two days.

  392. TBill says:

    @Victor
    As far as Broken Ridge, the difficult terrain, we have not really been advised how well they were able to search the nooks and crannies. I am satisfied that they gave it a shot, probably gave it the appropriate amount of resources for this phase of the search.

  393. David says:

    @HB, Victor, Don Thompson. I join your assessments of the Larry Vance book having accessed a Kindle copy and attach the outcome. The part about damage analysis is detailed since earlier I had gone deeply into the issues he discusses.

    I do not expect many to read this given its length and detail though I do intend it be on the record. I add some comment on other points he makes, including on a couple which have been addressed or touched on earlier.

    My overall assessment is that the book offers little to support its title, “MH370 Mystery Solved”. If he should prove right it is for the wrong reasons.

    At the same time I have updated what I think happened as a possible help with any future damage assessment thinking, whether or not the sequence I describe survives or not. Also, I am unsure still as to what initiated this sequence.

    https://www.dropbox.com/s/ilpmwzem3q7lmb9/PERUSAL%20OF%20THE%20BOOK%20BY%20LARRY%20VANCE.docx?dl=0

  394. Sabine Lechtenfeld says:

    @Sfojimbo, I like your comment from May 26, at 8:30 pm.
    I have offen followed exactly the same train of thoughts. The reboot of the sat com around 18:25 – mere minutes after the last military radar capture – is uncanny. If this wasn’t a coincidence, how could ZS have known this so exactly? And,yes, how could he have known that the radar sightings of an unidentified big plane wouldn’t be swiftly connected with the plane which had actually gone missing? Well, I think it’s possible that he didn’t anticipate that the Malaysians wouldn’t talk about the radar evidence for days, and would insist instead on searching the SCS. Since the plane was last “seen” travelling into a northwestern direction towards the Andamans, ZS might’ve figured that the SIO would be very low on the aerial search menu, and that the Bay of Bengal would be targeted instead. If that was his calculation, he got that right.

    Btw, we could also asks, how ZS could’ve known that the Malaysians would insist for two long hours after Vietnam had reported flight MH370 missing, that the plane was travelling happily over Cambodia and wasn’t missing at all? This terrible blunder gave ZS a crucial head start. He was either very lucky and profitted from sheer incredible incompetence, or he had inside helpers. Most have concluded that He go

  395. Sabine Lechtenfeld says:

    …that ZS got a lucky break.

  396. Mick Gilbert says:

    @David

    Excellent analysis, David, thank you for compiling that.

  397. Victor Iannello says:

    @David: Thank you for your very thorough review. Whether or not you get a reply, you should forward your thoughts to Larry Vance.

  398. Sabine Lechtenfeld says:

    @David, you said about Larry Vance and his ideas: “If he should prove right it is for the wrong reasons”.
    This is a very important observation, which went through my head frequently in the last few weeks, when we dissected various scenarios which seemed to be riddled with flawed arguments. We should distinguish between certain flawed elements and the merit of overall scenarios. We can probably say about Larry Vance: he didn’t manage at all to make a convincing case for his scenario, and there are a lot of factual errors in his book.
    But we have had far more convincing scenarios – and, yet, they have to be discarded simply because the plane hasn’t been found in the predicted area. And I really hope that people don’t fall into the tempting trap to defend their theories nevertheless with the argument that we have been extremely unlucky, and the plane was just missed or overlooked somehow. This is of course a possibility which cannot be completely excluded, but it should not be used as a carte blanche excuse for salvaging cherished scenarios. We should not discard lightly the one thing we have definitely learned from OI’s search: where the plane is most likely not.
    Has OI ever said something about the probability that the wreckage was missed? I guess it depends on the topography and make up of the various terrains.

  399. Nederland says:

    In response to Vance’s book, Peter Foley has ruled out that the flaps were extended/down, but he did not rule out controlled water landing (without engine power), which was however not successful:

    “There are many scenarios that will damage the flaperon. For example, it sits right over the top of engine and in most crash scenarios you’re going to liberate the engines and they’ll come adrift. If your wings are reasonably level, there is going to be consequential damage … We’ve never speculated on the speed of the impact. … So we have quite a bit of data to tell us that the aircraft, if it was being controlled at the end, wasn’t being controlled very successfully. The flaps weren’t deployed. Given the timing, that first logon request, if you like, is most consistent with a scenario where the aircraft had exhausted its fuel.”

    http://parlinfo.aph.gov.au/parlInfo/download/committees/estimate/e5833f20-4a8f-43f3-80e0-09ffa20248c5/toc_pdf/Rural%20and%20Regional%20Affairs%20and%20Transport%20Legislation%20Committee_2018_05_22_6176.pdf;fileType%3Dapplication%2Fpdf
    pp. 60 and 63

    Now, assuming the final BFOs are correct (or at least the first out of two), it seems odd that a pilot, if still in control, would decide to recover from a steep descent. However, if they were able to see the satcom logon on display, they might have guessed that this could involve some tracking ability and responded accordingly. In that case, the constraint on distance from the arc would be the maximum gliding distance after a period of descent (controlled or uncontrolled), quite probably only to the right of the arc.

  400. Sfojimbo says:

    @Sabine
    It wasn’t the Vietnamese who said he was over Cambodia, it was MAS Hq in Kuala Lumpur:

    1802:15 (from log book) MAS told KLATCC that MH370 was able to exchange signals w/ MAS plane in Cambodia.

    They later gave coordinates for a position in the SCS south of Da Nang.

    This all reeks to high heaven of something more than incompetence; this coupled with the fact that ZS knew he could overfly the Malaysian peninsula without being reported tells me that there is an elephant under the rug. I don’t much care about finding 9M-MRO in the southern Ocean. I want to get a look at that elephant.

    My belief is that at 18:25 ZS restored power to the left AC bus which again powered up his FMS and allowed auto-pilot functions again, but also, unknown to him, restored power to the sat transceiver. Three minutes later came the 18:28 ping ring which gave us a known longitude for the plane. This 18:28 ring lined up very well with the MEKAR + 10 miles location the Malaysians had given.

    In my above post I was an hour off in giving the time he knew for sure that he was “home free” (two hours after IGARI was 19:22 not 18:22 as I said above), at that time he was in the Southern Ocean several hundred miles south of Banda Aceh and clear of all radar.

    @Victor
    Does this site have anything resembling an agreed upon timeline?
    I have one, I could clean it up a bit and post it if you want.

  401. Victor Iannello says:

    In a new Geoffrey Thomas article, Blaine Gibson provides more details about the harassment that he and other MH370 debris hunters are facing.

    He also claims that there is still no official determination of the relevance of personal effect items that were recovered and provided to the Royal Malaysian Police.

    Mr. Gibson also said that there was no report yet on numerous pieces of personal effects that were washed up and handed to the Royal Malaysian Police.

    “In at least six cases these pieces have been matched to CCTV vision of passengers boarding MH370,” by volunteers, said Mr. Gibson.

    “And one relative told us they believe that a bag found in La Reunion belonged to their loved one.”

    “However, they do not wish to be identified,” said Mr. Gibson.

  402. Andrew says:

    The ATSB defined a search area for a controlled ditching, described in pages 99-101 of its report The Operational Search for MH370, but it was ruled out by the BFO analysis and was also not consistent with the drift studies. I suspect there will be pressure to search the area some time in the future.

    ATSB – Controlled Ditch Scenario Search Areas

  403. Victor Iannello says:

    @Sfojimbo: If you want to provide a link to a timeline of events, you should. I’m not sure what value you think that might have.

  404. Victor Iannello says:

    @Andrew: Before there are any further searches, I’d be interested in seeing the ATSB and Boeing explain how with no pilot inputs, we can achieve the two final BFOs and still have the plane travel for an additional 22 NM (or even further at latitudes that were searched wider) before impact. Then, there would have to be some rationale for selecting a latitude on the 7th arc to search wide. I’ve always been fond of the BEDAX-SouthPole path because it easy to program and the BTO and BFO fits are excellent. But I would hesitate at this point to recommend it as a search priority because if there were no pilot inputs, the debris should have been found near 34S latitude, and if there were pilot inputs, there is no reason to believe the plane flew along this or any other autopilot path.

  405. Sabine Lechtenfeld says:

    @Sfojimbo, I know that the idea the plane was flying over Cambodia was coming out of MAS headquarters. We were told that a young and inexperienced guy mixed up a flight progess program with real live data coming from the plane. Personally I was always very skeptic if we were told the truth.

  406. Gysbreght says:

    Nederland says May 27, 2018 at 7:58 am:

    “Now, assuming the final BFOs are correct (or at least the first out of two), it seems odd that a pilot, if still in control, would decide to recover from a steep descent.”

    You have to bear in mind that the ‘steep descent’ took 8 seconds and 1200 feet of altitude.

    From the Australian Senate Committe Report, page 62:

    “Senator PATRICK: I’m just curious. You can’t explain how a pilot might do very strange things at the end of
    a flight, but somehow it’s reasonable that the pilot did some very strange things at the start of the flight.
    Mr Foley: And that is the difficulty that I think a lot of commentators have. The evidence at the end of flight
    is that the aircraft wasn’t being controlled, but certainly there were control inputs early in the flight. Bear in mind
    that it took six hours from that turn at the tip of Sumatra to get to the southern Indian Ocean, where it’s most likely
    to be.
    Senator PATRICK: So I go back to my question: at what point do you think you lost I’m now looking at
    page 9; it’s quite a good little diagram that shows the movement across the Malaysian Peninsula and then into the
    Andaman Sea.
    Mr Hood: We’ve haven’t really got any evidence and we don’t want to get into conjecture. Once again, we’re
    not saying it was the pilot either; we’re saying that control inputs were made, because we’ve got no evidence to
    suggest that it was the pilot.
    Senator PATRICK: Respectfully, control inputs”

    What strikes me in that exchange is that on one hand the ATSB says:
    “we’re not saying it was the pilot either; we’re saying that control inputs were made, because we’ve got no evidence to suggest that it was the pilot.” But when discussing the ‘evidence’ on which they conclude that no one was in control at the end of the flight, they reject all actions with the argument that ‘no pilot’ would do such things. If the person at the controls was someone other than a qualified pilot, he could have done things not expected of a pilot.

  407. Gysbreght says:

    @Victor Iannello: “Before there are any further searches, I’d be interested in seeing the ATSB and Boeing explain how with no pilot inputs, we can achieve the two final BFOs and still have the plane travel for an additional 22 NM (or even further at latitudes that were searched wider) before impact.”

    On the basis of available evidence, there is no way the airplane could have crashed more than 22 NM from the 7th arc (even so it travelled 20 minutes and 90 NM in uncontrolled spirals in some simulations). However, on the same basis, you would not have those two final BFO’s either.

  408. HB says:

    @David,
    It is great insight, though difficult to follow your sequence and reasoning. Maybe graphics could help.
    I am proposing to discuss ATSB scenario as Vance’s scenario has issues as you rightly pointed out.

    Focussing on the seal pan damage and the outer crack on the side of the seal pan, it first appears that at the point of separation, the flap was retracted as per ATSB. The seal pan seems to have been damaged by longitudinal forces (as evidenced by damage to end stiffener Yellow in Fig 16). There were also some up and down movements of the support track relative to the seal pan as evidenced by red stiffener damage in Fig 16). ATSB explanation stands so far. Beyond that it is difficult to conclude the flap position at the point of impact (different to the point of separation) like ATSB did or like Vance did as the dynamic scenario needs to be analysed. I have two observations:

    1)
    I cannot convince myself of the nature of the outer crack failure mode without detailed examination.

    It could be a crack due to single or couple of impacts with the support track (side of the track or end of the track) (appear most likely) but there would be other marks for that (photo does not help). External impact would have other marks too and would be evident on the photo. Could be the result of bending and twisting of flap itself but the crack should be much longer and not going accross the side in Z shape. Or it could be fatigue cracking again Z shape unlikely but not impossible (need microscopic look).

    Here I assume it is the result of impact with the support track but it cannot be concluded from the photo. The crack zone is before the red stiffener and continue in Z shape up to that circular shape (Fig 19) at the location of the first stiffener (front side) (first non-colored stiffener shown in Fig 16).

    Now, if the side track end position was in this zone during the impact (ie Flap down) it may be well possible that the support track may ram towards the aft on impact and impacting first this crack zone then the red and yellow stiffeners on the way and in sequence. Attachements or actuators or hydraulics would have failed first. I am not sure what caused that crack but there needs to be more analysis. I am not sure if this is what Vance describes. Also as you pointed, not sure the supoprt track can go that far on impact if the flap is down at impact.

    In the ATSB scenario, this crack zone could be damaged as the track (eventually moving forward) separated from the flap (not at the point of impact). But looking at the deformation of the red stiffener deforming astern, I have some doubt that this is what has happened. The marks on the inside of the pan (if they exist) could provide more clues on the sequence. ATSB did not really analyse the dynamics of the impact.

    The crack could also be the result of stiffener deforming too (unlikely to me but difficult to see).

    2)
    ATSB postulated the argument that the flaperon and flap were aligned (Figure 22) at the point of impact. But, it has been discussed that following a power loss, the flaperon should be in hydraulic bypass/damped mode while the flap in locked position (if my memory is good). Would the flaperon be in 10 deg up position due to aerodynamic forces? Intuitivelly, if the hydraulics were in bypass/damped, alignment is unlikely. Did I miss somthing here?

  409. Victor Iannello says:

    @Gysbreght: I am not convinced that the values and spacing of the final BFOs are impossible without pilot input, as lateral asymmetry after the second engine flameout could explain this. However, the timing of the lateral asymmetry relative to the flameout has to be explained. I am more convinced that once the plane is in a 0.7g banked descent, it won’t recover without pilot input. In any event, I’ve asked the ATSB for more details about the Boeing simulations so we can make more informed judgments about what is and what is not possible.

  410. Gysbreght says:

    @Victor Iannello: “I am not convinced that the values and spacing of the final BFOs are impossible without pilot input,…”

    The 2016 uncontrolled simulations were obviously requested to obtain evidence in support of the final BFOs but failed to produce it.

  411. TBill says:

    @Victor
    “Then, there would have to be some rationale for selecting a latitude on the 7th arc to search wide.”

    I am thinking out load, but the future search areas need to be accompanied with a proposed flight path and end-flight scenario, and logic how this happened. At this point I am assuming a mix of active and passive flight scenarios. I feel like part of the problem of orig Bayesian analysis was to sanitize the need to actually say what happened for different scenarios.

  412. Gysbreght says:

    I would add: “requested and designed“.

  413. TBill says:

    Correction: thinking out loud

  414. Richard Godfrey says:

    Following on from Bobby Ulich’s helpful summary table, I decided to expand on his idea a little.

    Between the 4 BTO/BFO studies and 3 Drift studies using a forward path method and excluding the CSIRO studies, there is a very good match that MH370 ended on average between 27.5°S and 32.9°S, centred around 30.2°S. The 3 Drift studies using a reverse path method, all tended to higher latitudes on average between 21.0°S and 30.3°S. The 3 Drift studies from CSIRO, all tended to lower latitudes between on average 33.1°S and 35.9°S.

    The summary data is attached below:

    https://www.dropbox.com/s/szowtrcbvcr8oqt/MH370%20Summary.png?dl=0

    The full data is in Excel attached below:

    https://www.dropbox.com/s/6knpnkrl33dmhq1/MH370%20Summary.xlsx?dl=0

  415. Victor Iannello says:

    @TBill: If you include the possibility of pilot inputs after 19:41, and an impact at 100+ NM from the 7th arc, it is impossible to define a searchable area. You need to further constrain the set of possible paths with assumptions about intent. With the knowledge we have today, that’s not possible. You’d end up with paths ending as far south as 39S to as far north as Christmas Island. You can see that just by the opinions expressed here in the last week.

  416. Sabine Lechtenfeld says:

    @Victor, would you think it’s sensible to continue searching a corridor around the 7th arc until all areas within the plane’s fuel range have been searched – even if that means going all the way up north and ignore certain drift studies? The last BFOs seem to suggest after all that the plane must’ve crashed SOMEWHERE near the 7th arc. Of course, the plane might very well have been recovered and turned into a glider. But in this case it’s next to impossible to predict where it might’ve been crashed. Therefore sticking to the 7th arc seems to be a sensible search constraint – even if the search result remains negative. But if the plane can’t be found anywhere near the 7th arc, we would know at least a bit more.
    The problem is that going up all the way north along the 7th arc is still a huge area which needs to be covered. I doubt that anyone would be willing to put in the necessary time and money.

  417. Nederland says:

    @Gysbreght wrote:

    ”You have to bear in mind that the ‘steep descent’ took 8 seconds and 1200 feet of altitude.”

    Still, it is reasonable to assume that the descent rates measured at 0:19 did not come out of nothing but that the descent rates were similar up to that time (in both controlled and uncontrolled scenarios) and that any recovering only happened after 0:19:37. This is a reasonable constraint to put on any controlled end of flight scenario and reduces the maximum glide distance.

  418. TBill says:

    @Victor
    At some point we would have to have a ranking of scenarios and search there. For example, we have Ed baker’s proposal. That case is strong on proposed logic, weak on flight path to get there BFO/BTO. That case would have to ne more fully developed. And then stand in competition with other scenarios.

  419. Niels says:

    @Richard Godfrey
    It cannot be correct that BFO indicates a southern latitude limit of S34.5. Can you please explain / provide the full reference (this seems to be old work).

  420. Gysbreght says:

    @Nederland: “This is a reasonable constraint to put on any controlled end of flight scenario and reduces the maximum glide distance.”

    A steep descent during a short time does not reduce the maximum glide distance. It results in excess speed which can be recovered to restore glide distance. I agree those rates of descent “did not come out of nothing”. As far as we know, they required a human control input.

  421. Andrew says:

    @Gysbreght

    RE: ”The 2016 uncontrolled simulations were obviously requested to obtain evidence in support of the final BFOs but failed to produce it.”

    Remind me how you account for the ATSB’s statement:

    ”Some of the simulated scenarios recorded descent rates that equalled or exceeded values derived from the final SATCOM transmission. Similarly, the increase in descent rates across an 8 second period (as per the two final BFO values) equalled or exceeded those derived from the SATCOM transmissions. Some simulated scenarios also recorded descent rates that were outside the aircraft’s certified flight envelope.”

    MH370 – Search and debris examination update, 2 November 2016, p.8

  422. Gysbreght says:

    @Andrew: Please read those words carefully. These statements have been phrased to create an impression that is not supported by the facts. Please ask the ATSB to provide the time it took in the simulations between autopilot disconnect and those ‘recorded descent rates’.

  423. DrB says:

    @TBill,

    You posed several good questions. Here are my responses:

    1. My original end point prediction done in September 2014 was 40.2S using a 192.1 degree great circle path. This is now understood to be very unlikely for two reasons. First, as my 9M-MRO fuel modeling has subsequently shown, that location is slightly beyond the maximum achievable range. Second, the fit ASSUMES a constant TAS, but in fact there is no autopilot mode for holding a constant TAS.

    2. In April 2015 I presented some images of possible contrails/distrails. These are (mostly) linear features visible in satellite images of the area of interest. Later I found out that a somewhat informal study had been commissioned by the investigation team. One of the investigators was Simon Proud. He later stated that the humidity at the nominal cruising altitudes was too low for normal contrail formation (and on that point I believe he is correct). So, we must conclude that the image features (which are real) are not likely to be aircraft contrails, but rather some other naturally occurring phenomenon. I don’t know if any formal report on those negative contrail findings was prepared, but so far none has been referenced or released. If the actual flight path becomes known in the future, it will be interesting to revisit those images to see if any features are visible at the right time and place. In the meantime, I have discounted 9M-MRO as having caused any of them.

    3. My most recent work (7 March 2018 paper) exploring all possible autopiloted lateral navigation and speed control modes does not provide any acceptable BTO/BFO solution in the 38-40S zone. Therefore, I don’t think this area has a significant probability of success.

    4. There are four reasons why the aircraft is not likely to be north of 25S. Taken, together, the case is so strong that, in my opinion, searching there is very low in the ranking of alternatives. First, the BFO errors are large. It is true that we have no other examples of an in-flight power up from previous flights. Still, I don’t know any reason why that circumstance (at 18:24) would be likely to produce a large bias shift. The DSTG limits based on BFO errors were shown in my table to be 24.7-34.5S. So, on the basis of BFO errors alone, north of 24.7S is unlikely (but not impossible). Second, using the autopilot assumption and no turns after 19:41, there are no acceptable fits to BTO/BFO north of about 26.5S. Third, the five reliable forward debris drift studies I cited all have northern limits of 25S or further south. Fourth, none of the pinger detections (all of which were north of 25S) match the characteristics expected from a ULB on the bottom. I expand on this point in the next item.

    5. Two acoustic detections were made by the Haixun 01 Rescue Boat using a hand-held microphone on the end of a pole lowered over the side of the boat. The detector was fixed-tuned to 37.5 kHz, but the bandwidth is sufficiently large to allow detections at nearby frequencies. Typically, pingers use 27-45 kHz frequencies. The detections were fleeting (one only lasting 90 seconds) and were made at locations about 3 km apart on different days. Unfortunately, no recordings were made. This equipment has a nominal range of 2,000 m, so it is not expected to be able to detect a ULB at the greater ocean depth there of about 4,500 m. In addition, the surface noise is quite high when the detector is submerged just a few meters. That’s why Ocean Shield used a towed detector at a significant depth. That helps in two ways: the surface noise is reduced, and the range to the target is reduced, both of which increase the SNR. So, what we know is that the Haixun 01 Rescue Boat detections were at a nominal pinger frequency (but we don’t know that it was at 37.5 kHz, only that it was close to that), were fleeting in time and variable in location and not persistent as would be expected for a pinger on the bottom, and were much stronger than expected for a location on the bottom and more likely to be at a shallow depth. HMS Echo reported detections, but these were discounted as being due to a problem with the equipment. At least one sonobuoy launched from an aircraft detected acoustic pulses, but these were not at the correct 37.5 kHz frequency. Ocean Shield made multiple detections at about six locations over a period of days. These were not stationary in location, were stronger than expected, at least one of which was detected as soon as the microphone was lowered into the water before it was lowered to normal operating depth, and were at the wrong frequency (33.0 kHz). I have analyzed one of the recordings. I confirmed the 33 kHz frequency, and found the pulse period to be 1.108 seconds. This is higher than the nominal 1 second specification. You may recall the reports that, when a detection was made when towing, Ocean Shield turned around but was unable to reacquire the signal. I submit that none of these acoustic detections were from the ULBs on 9M-MRO data recorders. None of them were at the correct frequency and pulse period, were stationary in space, and were persistent in time. More likely, these pingers were equipment locator acoustic beacons from fishing gear drifting near the surface. An example is given HERE.

    6.A pilot could have commanded turns to occur after 19:41. Allowing such turns at arbitrary times and locations certainly expands the intersections with the 7th Arc and could increase the length of the search area. You will recall that the ATSB knowingly decided to limit the search width along the 7th Arc so as to make the initial search possible in a reasonable length of time. They never said the aircraft MUST be within 25 NM of the arc. They simply said that is a reasonable guess for an initial search (and I think it was). Now that the search has been unsuccessful out to 25/22 NM, it would seem reasonable to expand the search width, perhaps out to 50 NM. In my opinion, the priority zones would be 31.6S +/- 1 first, then the remainder of 28-34S, and finally the remainder of 25-38S.

  424. Victor Iannello says:

    @Andrew: What we need to understand is how those progressive descent rates (inferred from the measured BFO values) in the simulation could have occurred two minutes after fuel exhaustion. I am not aware of an explanation. I am trying to get more information from the ATSB and Boeing. That’s why I said that the timing of the lateral asymmetry relative to the flameout has to be explained. I am not in the camp that it is impossible. I’m in the camp that we have to learn more before making a determination of plausibility.

  425. Richard Godfrey says:

    @Niels

    You stated: “It cannot be correct that BFO indicates a southern latitude limit of S34.5. Can you please explain / provide the full reference (this seems to be old work).”

    I did provide the full reference in my original post, as did Bobby in his original post.

    I think you are missing the point that Bobby and I are trying to make. This is a review of a number of studies over the years. I was trying to point out, where there is alignment and where not.

  426. Victor Iannello says:

    @Sabine: I don’t yet have any opinion about where to search next. I am reluctant to recommend widening the search area for the reasons you mentioned. However, in the time before a potential new search is planned, there is an opportunity to learn more about end-of-flight behavior, the BFO, and other matters. Also, some other information might shake loose that helps us. With the Najib administration out of power, I have new hope for a whistleblower.

  427. Gysbreght says:

    @Victor Iannello: “What we need to understand is how those progressive descent rates (inferred from the measured BFO values) in the simulation could have occurred two minutes after fuel exhaustion.”

    I have demonstrated that those descent rates didn’t occur two minutes after autopilot disconnect (which occurred at the first engine flame-out in four of the five simulations that exhibited those descent rates).

  428. Victor Iannello says:

    @Gysbreght: Why must you argue about everything? Those simulations don’t represent the entire universe of all possible outcomes. For instance, engine re-starts were not modeled. That’s why I said we need to understand what is plausible, not just what was observed. If your mind is already made up, that’s your choice. I’d like to learn more.

  429. Gysbreght says:

    @Victor Iannello: Why must you always change the subject? I replied to your comment where you said what I quoted.

  430. Victor Iannello says:

    @Gysbreght: You argue even when I agree with you. The Boeing simulations don’t show the descent rates with two minutes of fuel exhaustion. That’s why I said What we need to understand is how those progressive descent rates (inferred from the measured BFO values) in the simulation could have occurred two minutes after fuel exhaustion. Notice I said “could have occurred” as opposed to “occurred”, and why I said the timing of the lateral asymmetry relative to the flameout has to be explained. You believe there is no possibility for the descent rates to occur within 2 minutes of fuel exhaustion without pilot input. I am not ready to say it’s impossible. I’d like to learn more.

    I am tired of arguing semantics with you. You should take that as a strong warning.

  431. Gysbreght says:

    [@Gysbreght’s comment deleted for now arguing about arguing.]

  432. DennisW says:

    @VictorI

    If you include the possibility of pilot inputs after 19:41, and an impact at 100+ NM from the 7th arc, it is impossible to define a searchable area. You need to further constrain the set of possible paths with assumptions about intent. With the knowledge we have today, that’s not possible. You’d end up with paths ending as far south as 39S to as far north as Christmas Island. You can see that just by the opinions expressed here in the last week.

    My thoughts exactly as summarized below.

    https://docs.google.com/document/d/1UZW9YXRklZ21_BzFNy9Y6G9kmnpzG5dLBTC_35S8ut8/edit?usp=sharing

  433. Victor Iannello says:

    @DennisW: I think searching near Christmas Island is not completely off the table. If what @TimR says is true, then he needs to provide additional information so a search there becomes more than a distant possibility.

  434. Niels says:

    @Richard Godfrey
    It is clear to me which main point you were making; you stated:
    “Between the 4 BTO/BFO studies and 3 Drift studies using a forward path method and excluding the CSIRO studies, there is a very good match that MH370 ended on average between 27.5°S and 32.9°S, centred around 30.2°S”
    Having worked quite a lot with the BFOs I feel that fig. 27 of the 13/08/2014 ATSB should not be used for the purpose you are using it. With the current knowledge of BFO analysis it is really weird to suggest it sets a southern limit of S34.5 and a central latitude of S29.6.

  435. sk999 says:

    Niels and Richard G,

    The analyses in the June 26, 2014 (updated Aug 18, 2014) ATSB report (aside from Appendix G) were computed using the “Eclipse 2” model for the sat+afc effect (as per the Oct 8, 2014 Update and the ATSB Final Report). This model was constructed at a time when Inmarsat did not fully understand how the Miteq EAFC receiver worked (!) and it had serious errors. Appendix G and the October update both used the “Unified Model” for the sat+afc effect (which does properly model the signal chain), a consequence of which is that the search zone moved South by about 5 degrees of latitude between the June and October reports. This conclusion comes from reading and reverse engineering certain text and figures in the Final Report and from descriptions of the SATCOM system from the JON article. Thus, while the BFO bounds of Fig. 27 in the June/Aug 2014 report might match later analyses, such agreement is specious – those bounds were computed using an invalid SATCOM model.

  436. MH says:

    I remember they did some calibration runs around Christmas Island
    Before heading south to map along the 7th arc.

  437. Mick Gilbert says:

    @Victor Iannello

    Victor, the discussions about what could or could not have occured within two minutes of second engine flame out should really consider a period of two minutes or longer. Two minutes is essentially the minimum period between second engine flame out and SATCOM reboot and assumes a prompt APU auto-start, conceivably it could have been longer. Of course, if it was longer than two minutes that probably increases the likelihood of an impact closer to the arc.

    Perhaps another assumption that is worth revisiting is cruising altitude. Let’s say that Mike’s interpretation of the KB data is correct and the airplane was flown well above FL400 for the entire flight into the SIO; that probably widens the uncontrolled descent search width by at least a few nautical miles.

  438. Andrew says:

    @Victor
    @Gysbreght

    Thanks – got it. I didn’t mean to foment an argument!

  439. Andrew says:

    @Victor

    RE: “I am not convinced that the values and spacing of the final BFOs are impossible without pilot input, as lateral asymmetry after the second engine flameout could explain this. However, the timing of the lateral asymmetry relative to the flameout has to be explained.”

    ‘Lateral asymmetry’ could be caused by residual rudder trim or an engine auto-relight, as previously discussed. With the Trent engine, an auto-relight would have to occur fairly quickly after flameout, otherwise the EEC would shut off the ignition and disable the auto-relight. The FCOM for the Trent-powered aircraft states:

    “When the EEC detects an engine flameout, the respective engine ignitors are activated. If the engine does not recover and continues to run down below 35% N3, the EEC shuts off fuel and ignition and disables the auto–relight function.”

    In the simulator trials I conducted, the N3 ran down below 35% N3 within one minute of flameout.

  440. Victor Iannello says:

    @Andrew: In the past, we have discussed whether there would be enough fuel for a relight to produce sufficient thrust asymmetry to bank the plane, as the spool up is slow. I’m not sure we definitively answered the question one way or another. If there was enough fuel for the engine to produce significant thrust, then this could explain the timing of the BFOs. However, once in a banked descent with a downward acceleration of 0.7g, even if the engine flames out, with no pilot input, the plane would not recover from the steep descent, and the plane would be found close to the 7th arc.

  441. sk999 says:

    Side comment –

    This past week I was at a dinner where the conversation steered to MH370, and a young woman (a grad student) revealed that she was from Malaysia. My first question was, from where, and she said “Borneo”. So I guess that that is how she identifies herself, at least to outsiders.

    I next asked what she though about MH370, and she said, in essence, that she didn’t know what to think, because there were so many stories about what happened.

    No grand conclusions – just some observations.

  442. Andrew says:

    @Victor

    I remember some parts of the previous discussion. Frankly, I doubt there would be enough fuel to sustain a relight long enough to produce a significant thrust asymmetry, but that’s just an opinion.

    The other factor is the rudder trim. Most of the B777s in our fleet need about 1-2 units of right rudder trim in the cruise. The recommended technique is to trim the rudder in the cruise with HDG SEL or HDG HOLD engaged. The rudder is trimmed in the direction of the low side of the control wheel until the control wheel is level. The aircraft is properly trimmed when the index on the top of the control wheel is zero.

    If 9M-MRO was similar to other B777s but wasn’t trimmed correctly, then with no thrust asymmetry the autopilot would need to hold a small amount of right aileron to maintain heading. If the autopilot was disengaged, the aircraft would roll left and enter a descending left hand spiral. Of course we don’t know if that was the case and, to date, the aircraft hasn’t been found close to the 7th arc.

  443. Victor Iannello says:

    @Andrew: The problem with the residual trim theory is that the 0.7g downward acceleration occurs two minutes after the second flame out. In those two minutes, the descent reaches only about 4,000 fpm. Then, in the next eight seconds, it increases to 14,000 fpm. With the engine re-start theory, the delay in the bank and downward acceleration can be explained (assuming there was enough residual fuel).

  444. David says:

    ndrew, Victor. If the amount of residual fuel available to the left engine was the ATSB’s 30 lbs, with the APU pump delivering at 3150lbs/hr, that would take the left engine just 34 secs to consume, supposing it consumed at that supply rate, relight or not.

    However I do not know what the engine demand during relight would be.

    Assuming those 34 secs though, the APU would take little or none during that, needing 30-40 secs for inlet opening before its start cycle commences.

    @Andrew. As I understand you if 9M-MRO WAS trimmed correctly there would be no residual trim to cause bank?

  445. David says:

    @Mick Gilbert, Victor, HB. The Vance book. I appreciate your comments. I have made various editorial corrections and an ‘outboard’ to ‘inboard’ swap. Right now Dropbox seems not to be working though, for me at least.

    I will send a copy to Peter Foley when it revives hoping it gets to him and will give some thought too to Larry Vance.

    HB, Yes graphics would help if I had them.
    On your 2. “…following a power loss, the flaperon should be in hydraulic bypass/damped mode while the flap in locked position (if my memory is good). Would the flaperon be in 10 deg up position due to aerodynamic forces?”
    From my memory, with no APU running the RAT would keep one right flaperon PCU going and between battery power to control one left PCU and left windmilling hydraulics I think the one of the left’s would be kept active too.
    If that is right the flaperons should be where the autopilot last put them, presumably aligned with the flaps.

  446. TimR says:

    @ Sfojimbo, “OK let’s go one more. Where does your 370 kt plot hit the 0:19 ring?”

    Apology for the delay, I had gone walkabout.
    I do not know what happened but I believe I know what was intended to happen.
    Shah loitered at a steady 370kts round below Sumatra to give maximum time for negotiations, intending to finally land and release the passengers.
    Unable to land at Christmas Island for some reason he carried on towards one of the airfields in Indonesia and ended up ditching short at around 8°30’S to 9°30’S near the 7th arc.

    A less likely possibility was disruption on the flight deck before Cocos Islands.
    This could have meant there was no turn at Cocos Islands leading to overflight and continuing at a steady 370kts to a ditching further South at around 22°S near the 7th arc

  447. Andrew says:

    @David

    RE: “As I understand you if 9M-MRO WAS trimmed correctly there would be no residual trim to cause bank?”

    Theoretically, yes. The TAC input would be removed as soon as the PFCS reverted to secondary mode, leaving the rudder at its previously trimmed position. There would be a small time delay between engine flameout and the mode reversion, but the thrust asymmetry would be reducing during that time along with the TAC input, so I doubt it would have much effect.

  448. David says:

    @Andrew. Thanks. I notice the ATSB relies directly on residual trim to cause bank for the descent and spiral, said more than once at last week’s hearing. That was said independently from the Foley comment that there were caveats consequent on the simulations not including relights.

    If it is more likely than not that trim procedure was the ‘recommended technique’ in this flight with an experienced pilot on board, presuming he would trim for economy from habit, the ATSB reliance on this would be tenuous.

    Do you think it probable that the aircraft would not be trimmed to the ‘recommended procedure’ were the pilot to do it? The co-pilot?

    Also, would auto-throttle power adjustments lead to residual trim after being adjusted initially, presumably when setting cruise?

  449. Andrew says:

    @David

    It might be a recommended technique, but that doesn’t mean everyone does it! In my experience, there are a lot of pilots who don’t bother. If the FO was flying the aircraft (initially, at least) then he might not have trimmed it properly after reaching the cruise, given his lack of experience on the aircraft. It would probably depend on what he was taught (or not taught) during his training. If he didn’t do it, then I guess it’s possible that it might have been left that way for the duration of the flight, but who knows?

    Autothrottle adjustments won’t affect the rudder trim, if that’s what you’re asking.

  450. David says:

    @Sabine Lechtenfeld. “Has OI ever said something about the probability that the wreckage was missed?”

    Not so far as I know. It is a good question because while the ATSB supervised searches had careful process, technique and expertise management, allowing quantification of results, so far as I know the details of the OI/Malaysian contract remain unreleased. For all I know OI is obliged to provide similar arrangements; and has seen to them whether obliged to or not.

    However Fugro reported comprehensively in the ATSB search report, the ATSB obviously took quality seriously and we know the outcome as to places missed, steps taken to plug those holes etc.

    Should OI not be required to report on results fully for the Malaysian final report and the Malaysians have not required the same monitoring and information feedback, we will never know. That might require a contractual obligation to do so, but we haven’t seen the contract.

    OI appears to be most careful, enterprising and well organised but they seem to have speeded up recently with Richard Cole I think mentioning the AUV spacings have increased. Also OI has had the AUVs at search speeds which I think few expected. It could be that this is because the terrain has changed and experience has been gained; and the same quality as to “holidays” etc apply as in the past. Yet as to what has been acceptable remains unknown (to me anyway). Indeed David Griffin has wondered about whether some of the earlier mountainous terrain could have concealed the wreckage, according to the Times.

    IF OI were to have optimised their search based on reward possibilities and they had X days left it might be tempting to take some chances with missing a few spots here and there, to maximise area covered. I am not implying this has happened, just emphasising that I think you question has legs.

    Maybe others can throw more light on OI’s quality management and final reporting obligations and how well it has met them?

  451. Richard Godfrey says:

    SC is continuing to make good progress up the Broken Ridge plateau area and has reached 26.3519°S.

    Ocean Infinity has started another 4 AUV deployments in the last 48 hours.

    The weather has improved, there is good visibility, with a 14 knot wind, a combined swell and wave height of 3.3m. There are no tropical storms in the region.

    https://www.dropbox.com/s/fnv0bkdtc270dv2/SC%20Track%2028052018.pdf?dl=0

  452. David says:

    @Andrew. Thanks for giving a feel for that.

    “……. it might have been left that way for the duration of the flight, but who knows?” Well, the ATSB has assumed that would happen it seems and the question now is whether that is valid, particularly if Z was assumed to be flying.
    No residual trim = no early bank = no early high descent rate.

    I think our conclusion here (certainly mine) was different to the ATSB’s. The likely effect of residual thrust would be insufficient and too slow. Thus no relight = insufficent or untimely bank.
    Relight chances are unknown though less likely at altitude. If successful, thrust X time = effect = also unknown.

    Besides I note that were there a successful relight that could counter residual trim and even were there bank one way or the other it would be lessened and slower.

    Thus timely BFO descent rate matching general prospects are unknown, probably low but anyway should not be relied on even if Boeing does agree finally to release the timing, configuration and probability data of the recent simulations.

    “Autothrottle adjustments won’t affect the rudder trim, if that’s what you’re asking.” Maybe I am being pedantic here but what I meant was that change of power from autothrottle adjustments does not affect trim? In other words, trim sensibly is thrust independent?

  453. Richard Godfrey says:

    @David @Sabine

    You asked ““Has OI ever said something about the probability that the wreckage was missed?”

    The ATSB stated in their final report dated 3rd October 2017 that the overall sonar coverage was approximately 98%. OI continue to review both the Fugro and their own coverage and believe it is closer to 99%.

    The Fugro search area was 120,000 km2 and the OI search area when completed is likely to be 115,000 km2 (96,200 km2 completed by the last Malaysian report dated 22nd May 2018, 5,600 km2 in addition up to today 28th May 2018, possibly another 13,200 km2 up to 8th June 2018). A total of 235,000 km2 would mean around 2,350 km2 not covered, if 1% was missed. Unfortunately this relatively small area is split up into probably around 160 points of interest.

  454. Richard Godfrey says:

    @Niels, @sk999,

    Re: Fig. 27 in the June/Aug 2014 ATSB report.

    I accept your point and will remove that entry from my MH370 Summary table.

    I much prefer Victor’s analysis of the possible BTO/BFO ranges in any case:

    https://www.dropbox.com/s/yh9f9ne87qsovjf/Victor%20Constant%20Mach%20vs%20LRC%2032%20%C2%B5s.png?dl=0

  455. Richard Godfrey says:

    @DrB, @Niels, @sk999,

    I have updated my MH370 summary table with your corrections:

    https://www.dropbox.com/s/szowtrcbvcr8oqt/MH370%20Summary.png?dl=0

    https://www.dropbox.com/s/6knpnkrl33dmhq1/MH370%20Summary.xlsx?dl=0

  456. David says:

    @Richard Godfrey. One percent. Thank you. Are the details available publicly?

  457. TBill says:

    @Richard
    “I much prefer Victor’s analysis of the possible BTO/BFO ranges in any case:”

    As we move forward, please check the Inmarsat BFO’s in the spreadsheets for the family of flight paths. I seem to notice the “as-measured” BFO’s might be a bit different that what we normally use, which is: 111 141 168 204 221 252, which in turn would impact the BFO-error calculations used in Victor’s graphic.

  458. Niels says:

    @sk999
    Thank you for the useful information on the development /quality of the BFO models.

  459. TBill says:

    @DrB
    @Nederland
    @all
    RE: BFO/BTO calcs
    For the record, I now use DrB’s speadsheet for my BFO calcs. I found some funny behavior (discontinuity) in the FFYap IG path spreadsheet that impacts the 19:41 BFO by a few units.

  460. Niels says:

    @Richard Godfrey
    Good that you removed the “fig. 27” BFO limits from the summary.
    A question on the file “MH370 summary.png”: are the southern and northern latitude estimates determined by averaging? It looks still strange to me to see a southern latitude related to BTO/BFO analysis set to S33.7 degrees…

  461. Richard Godfrey says:

    @Niels

    My apologies! I made it clear in my original post that these were average values.

  462. HB says:

    @David, correct accoding to Davi’s ref, the outboard actuator of the right flaperon is powered from the C hydraulics which is RAT powered so alignment should not be an issue.

  463. Richard Godfrey says:

    @TBill

    You stated. “I seem to notice the “as-measured” BFO’s might be a bit different that what we normally use, which is: 111 141 168 204 221 252, which in turn would impact the BFO-error calculations used in Victor’s graphic.”

    You are not correct in what you (or the royal “we”) claim to have noticed.

    I have been producing MH370 Flight Path models for over 4 years now. Currently at Version 17.

    I have always used the BFO imported directly from the Inmarsat Log data as provided on 27th May 2014, which is as you state from 19:41 UTC onwards, with a few exceptions.

    You state 221 Hz at the time of the 3rd call.

    Between 23:14:01 UTC and 23:15:02 UTC, there are 29 values ranging from 216 Hz to 222 Hz, which average 217.3 Hz.

    At 00:19 there are also 2 values at 182 Hz and -2 Hz, which you should also not ignore.

  464. Richard Godfrey says:

    @David

    You asked: “@Richard Godfrey. One percent. Thank you. Are the details available publicly?”

    Not yet.

    I hope that OI may kindly publish a report at the end of their search.

  465. HB says:

    @David, thanks for your update.
    you stated in your note “From my earlier study I should add that I think that one of the two support track wing attachments ATSB fig 15 again) broke, leverage by the carriage assembly inside causing the support track to fail inside the seal pan under comparatively little flap inner end movement. This would explain how the stiffener atop of it was struck. He has provided no explanation for how that was reached.”. This is also related to your Point 5.
    But this scenario should have resulted in penetration damage and also the track is relatively strong and the remaining support is hinged.
    I think a possibility is that the support track derailed from the carriage assembly and struck all the parts observed damaged inside the seal pan either moving aft during impact or withdrawing after impact. The carriage assembly could have also broken like you indicated in your subsequent note. This could be valid for both Vance and ATSB scenarios. ATSB did not really assess the large crack.

    On the compression failure Point 5: the cracks on the side are more indicative of impact failure as it appears on photo supporting ATSB assessment.

  466. Richard Godfrey says:

    @Niels

    My apologies again for causing confusion! I hope this MH370 summary table makes thing clearer.

    https://www.dropbox.com/s/szowtrcbvcr8oqt/MH370%20Summary.png?dl=0

  467. DennisW says:

    @Richard

    I find it interesting that the forward and reverse drift studies are so different. Of course (not denigrating your study) the reverse drift does not suffer from selection bias. One could argue that forward studies show what is possible, and reverse studies show what is probable. Forward studies do have the advantage of using actual drifter data.

  468. Richard Godfrey says:

    @Dennis

    A profound insight. Many thanks!

    Ocean Infinity are currently searching around 26.3°S.

    This fits well with the reverse drift studies, which have a central latitude of 27.3°S (Meteo France, CMCC, GEOMAR, NOAA).

    This is within the bounds of the most northern limit from a forward drift study at 25.5°S (Nesterov).

    This is just on the bounds of the most northern limit from a BTO/BFO study at 26.6°S (Iannello).

  469. Niels says:

    @Richard Godfrey
    No problem with the slight confusion!
    The new .png summary is a clearer presentation.
    One last question: is the “favoured latitude” for “drift reverse” the correct value (20.0 vs. 25 in the xlsx summary)?

  470. flatpack says:

    @TimR says on May 28, 2018 at 12:03 am

    “I do not know what happened but I believe I know what was intended to happen.
    Shah loitered at a steady 370kts round below Sumatra to give maximum time for negotiations, intending to finally land and release the passengers.
    Unable to land at Christmas Island for some reason he carried on …”

    It might have been intended to look like this or a similar scenario were possible or even probable.

    If political intent were even a small part of the plan, no actual negotiations would be needed to achieve the goal, merely the simulation thereof, leaving the insinuation of an incompetent and cold hearted regime to do the rest.

    I suggest that there were no negotiations, none were attempted nor even planned.

  471. David says:

    Minister’s recent statement to me implies that the final report will be considered by the Malaysian cabinet and released independent of a decision at the cabinet meeting on new searching.
    Since the report will be released, “…in the next couple of months….” the cabinet meeting might be some way off.

    https://www.nst.com.my/news/nation/2018/05/374226/full-report-mh370-search-will-be-made-public-says-anthony-loke

  472. @DrB, @Victor, @Dennis, @TimR,
    @All,
    Hello to all, I have been remotely following your very interesting discussions.

    Coming to the “what next?” question brought by Bobby: his suggestions are summarised in a nice draft table with some potential options. To our view, also expressed in the blog recently, a search zone in the South East of Christmas Island must be seriously considered (~12S/107E).

    Victor called for evidences that would make this option “less distant” and worth considering. Here they are: the full drift analysis report and video that we just completed.
    (http://mh370-captio.net/wp-content/uploads/2018/05/CAPTIO-Debris-Drift-Analysis-v1c.pdf)
    http://mh370-captio.net/wp-content/uploads/2018/05/Drift-mh370-Captio_2.mp4
    The analysis shows not only that when using the CSIRO model with ACTUAL weather data the computed drift leads on time to La Reunion (and then later to Mozambic and South-Africa), but also that debris would have stayed fully within warm tropical waters explaining the numerous barnacles found on the flaperon.

    If the links above don’t work: All details are gathered in a short report and in a video animation at http://www.mh370-captio.net.

    Thus please, Bobby, may I ask you to reconsider your list of options and would you be so kind as to update your table accordingly ? Inmarsat data fit, fuel exhaustion on the 7th arc, precise operational use of the airspace structure to avoid collision, adequate economic aircraft flying mode leading to a timely crossing of the arcs and now the Debris drift fit. I hope these are sufficient evidences 🙂 for not ruling out this plausible option. Thanks.

    Thanks also for this blog and its interesting, lively discussions. In particular for the various recent analyses of the radar data around KB which appear to confirm that the aircraft was under human control at that time and probably not under any emergency situation.

  473. Victor Iannello says:

    Take an hour and listen to a lecture by accident investigator George Bibel on the subject of mechanical and human failures leading to aircraft crashes. He talks about MH370 twice–once during his presentation (28:18) and in another time in response to a question (59:33).

    Some statements:

    “I don’t know why they think they know anything. Obviously that person tried to disappear, and if I tried to disappear, I’d change course a few additional times, so I can’t see how they’re ever gonna find anything.”

    “I think the entire aviation world understands that it was a criminal act, and the official investigators will not say anything without hard facts. So here they are, I think everyone knows it happened, and that won’t say anything, so it adds to the mystery.”

    Interesting statements.

  474. Andrew says:

    @David

    RE: “Maybe I am being pedantic here but what I meant was that change of power from autothrottle adjustments does not affect trim? In other words, trim sensibly is thrust independent?”

    Yes, assuming there is no thrust asymmetry.

  475. Victor Iannello says:

    @Jean-Luc Marchand: Thank you for all your hard work. The drift video is very helpful.

    The search effort is not likely to reach Christmas Island this season. We will have time to prioritize new areas to search. As I’ve said, I am not ready to make a recommendation, but at this point, based on the downward acceleration of 0.7g at 00:19 inferred from the final BFO values, I am not a fan of searching wider at already searched latitudes. Also, I think we may have to seriously consider that there were pilot inputs after 19:41, because if there were none, the plane likely would have been found.

    I think OI would consider searching again if there is a reasonable case that can be made for a new or extended area. Others here are already making recommendations, but I have not persuaded yet by any of the analyses I’ve seen. (I don’t mean to imply that I have the power or influence to decide anything.) If we are going to consider human factors, then we need evidence related to motivation or intent.

  476. DennisW says:

    @Jean-Luc

    I echo Victor’s statements. The video is, indeed very nicely done.

    I have more invested in the Christmas Island scenario than most posters here :-). It remains, IMO, the only scenario with a plausible narrative to accompany it.

    We shall see how the group thinking evolves.

  477. Victor Iannello says:

    Flight UA83 from Delhi to Newark has detoured after passing near the Afghanistan-Uzbekistan border, flew over Kazakhstan, and is now over Russian and traveling to the north towards the Barents Sea. Any ideas as to what is going on? It looks to be way off course.

    https://www.flightradar24.com/UAL83/11885468

  478. TBill says:

    @Victor
    No idea re: UAL83…the only thing I see is the nearby JAL London flight is getting in 6:30AM at London, so that would seem to imply the approx. 5:30AM Newark arrival time (10:30 AM London time) is out-of-date.

    Did you have a link to the George Bibel lecture? Sounds like I would appreciate that lecture.

  479. Victor Iannello says:

    @TBill: I meant to include the link in my comment. It’s there now.

  480. Victor Iannello says:

    @TBill: There are no ADS-B updates, so we no longer can track it.

  481. David says:

    @Andrew. Trim: thank you

  482. Andrew says:

    @Victor

    RE: “Flight UA83 from Delhi to Newark has detoured after passing near the Afghanistan-Uzbekistan border, flew over Kazakhstan, and is now over Russian and traveling to the north towards the Barents Sea. Any ideas as to what is going on? It looks to be way off course.”

    Saturday’s flight (26 May) took a similar route, according to FlightAware.

  483. David says:

    @Victor. On my Flight Radar UAL83 is still (01:37 UTC) in the Barents Sea heading NW from Severny Island.

    Not actual?

  484. Victor Iannello says:

    @Andrew: Thank you. It certainly looked odd.

  485. Victor Iannello says:

    @David: That’s an estimated track. It’s probably following a path similar to the one on May 25.

  486. DrB says:

    @All,

    I have revised my table of predicted MH370 latitudes based on the last several days of discussions. Thanks to sk999 and Richard Godfrey for additional information.

    It is at the same link HERE.

  487. Andrew says:

    @DrB

    The N & S limits for Item #5 appear to have been transposed. The table shows the N limit as 32.0 and the S limit as 31.2.

  488. DrB says:

    @Jean-Luc Marchand – CAPTIO

    Regarding your proposed Christmas Island route ending at ~12S 107E:

    First, I have some questions regarding the ROCs/RODs you use to make the predicted BFOs match the Inmarsat data.

    1. At 18:27:07 you use ROC = 150 fpm with a 360-degree track while at FL328. Yet the FLs are the same at 18:25, 18:27, and 18:28. How can you climb at 18:27 without changing FL from 18:27 to 18:28?
    2. At 19:41 you use ROD = 400 fpm when at FL150. Yet one hour later at 20:41 the FL is only 140. Thus, you are saying the altitude only dropped by 1,000 feet in one hour, but there was a 400 fpm descent at 19:41, which only needs 2.5 minutes to drop 1,000 feet. How can a brief descent of several minutes coincide in time with the 20:41 handshake?
    3. Similarly, you apply a 500 fpm descent at both 20:41 and at 21:41. How can the altitude only change 1,000 feet if you are dropping 400 fpm at 19:41 and 500 fpm at 20:41 (and then you do it again at 500 fpm at 21:41)?

    The entire sequence of descent rates you need to match the observed BFOs from 19:41 – 21:41 makes no sense to me. The three alignments in time of very brief descents with the then-unknown handshake times is not credible to me. At those low altitudes I don’t think there is any need to “pass under” or “pass over” an airway multiple times. I think it is much more likely that your route has track errors causing the BFO mismatches for which you use varying descent rates.

    Another question has to do with the drift angle w.r.t. the wind you assumed for the flaperon. Nesterov uses 0, 18, and 32 degrees to the left (from the Meteo France report by Daniel), whereas your model uses 16 degrees to the right. How did you determine this value, and why is it different from the Meteo France measurements of the flaperon?

  489. DrB says:

    @Andrew.

    Thanks for pointing out the switched values in my table. A revised version is now at the same link.

    I also added some detail on the latest (Nesterov) drift study. It shows that the best-fit starting latitude is 28.2S using a flaperon drift angle of 0-18 degrees left (per DGA), and this moves to 31.9S using a random drift angle. Thus, different assumptions of flaperon drift angle can change the predicted starting latitude by as much as 3-4 degrees. Unfortunately, the flaperon is quite unstable in its orientation in the water, and this reduces the accuracy with which its drift path can be predicted.

  490. DrB says:

    @Jean-Luc Marchand – CAPTIO,

    Another issue with your proposed route may be fuel. There seems to be enough fuel if the portion after 21:41 is flown at HOLDING at FL90/50, but I think this is considerably slower than the 300 knots ground speed you used. At 300 knots there may not be sufficient fuel. I would be interested in seeing any detailed fuel calculations that have been done, to compare with my back-of-the-envelope numbers, if you have them.

  491. David says:

    @Dr B. “Unfortunately, the flaperon is quite unstable in its orientation in the water, and this reduces the accuracy with which its drift path can be predicted”.

    I think also the assumption that it does not settle in the water needs verification observing the sensitivity of starting latitude to drift that you mention. The barnacles on the broken trailing edge give a clue that it could have settled.

  492. Richard Godfrey says:

    Many thanks to DrB for further comments on my summary of 18 studies regarding MH370.

    I have included to new categories in the MH370 summary of various studies: Satellite Photos and Aerial Photos.

    @Niels: GEOMAR had 2 favoured latitudes of 15°S and 25°S.

    Summary: https://www.dropbox.com/s/szowtrcbvcr8oqt/MH370%20Summary.png?dl=0

    Details: https://www.dropbox.com/s/6knpnkrl33dmhq1/MH370%20Summary.xlsx?dl=0

  493. David says:

    @HB. You quote me as writing, “… leverage by the carriage assembly inside causing the support track to fail inside the seal pan under comparatively little flap inner end movement.” You missed an ‘L’ I think. The ‘fail’ in fact reads ‘flail’.

    See the ATSB’s fig 15 top right. Without the support track pivoting about one of those wing attachments and being levered up by the carriage assembly the track inner end could not have reached all the way up to strike the stiffener above because it would have been prevented by first hitting the stiffener above the flap entrance.

    One of the wing attachment lugs needed to fail in bending due to the bend sideways caused by flap displacement outboard, that stress being supplemented by with the tensile loading in the bottom brace to the wing or compression in the track above it, more likely the former I would have thought.

    Had the track separated from the carriage assembly, though I do not see how since the carriage assembly would be forcing it upwards, had it done so on its way down the track towards its inner end again would been prevented from hitting the stiffener above for the same reason as above: the flap would hit the track above the track entrance first.

    I hope that helps.

  494. Sabine Lechtenfeld says:

    So, the post mortems can officially begin….
    What did we learn from OI’s efforts? Unless the search was very unlucky and missed the plane somehow, the area where the plane didn’t crash, has been considerably enlarged. Therefore it’s wrong IMO to write off the whole search as a total failure. We did learn something.
    The big question is of course: why have certain well reasoned scenarios (especially those which favored crash zones around 30°S near the 7th arc) not panned out, although all available data seemed to point into that direction? That should be a big worry.

  495. Don Thompson says:

    Contrary to the claims from MY that the search is over today: it’s not.

    Ocean Infinity’s release describes that the search “is shortly coming to an end”.

    Seabed Constructor continues to deploy AUVs, the latest launch will take the AUVs on a traverse that crosses the 7th arc north of S26º and recovery of the most recently launched AUV will not be made until 31st May.

    Post mortems are best left until Constructor is making 13kts eastward, from a position more than 50km east of the 7th arc.

    MY has published a further search report today, nothing significant noted.

  496. Sabine Lechtenfeld says:

    @Don, I will happily postpone any post mortem discussions 🙂
    The end of the search has been proclaimed prematurely a few times. The German magazine Spiegel ran an article about the end of the search and OI’s failure a few days ago.

  497. TBill says:

    @Victor
    The lecture by accident investigator George Bibel was interesting, but I did not like some of his logic: that nothing could be done to stop criminal activity. He almost said most air accidents are so rare that there is not much that can be done from engineering perspective.

    Interesting Bibel’s blanket statements MH370 was almost definitely criminal activity as far as airline community is considered. I would point out that was the major thrust of the much maligned 60 Minutes episode, although I know we did not like the news program’s approach and accuracy. And whereas the airline community has an understanding of the likely cause of MH370, the public has not been given this message clearly in part due to the media coverage as a mystery.

    I did not like former MAS CEO Dunleavy (further above) saying everything was speculation. We know enough about the flight from radar and satellite data and other evidence to make a preliminary assessment, of course subject to update and/or change if we ever locate the aircraft or more facts.

  498. Richard Godfrey says:

    SC is continuing to make good progress up the Broken Ridge plateau area and has reached 26.2150°S.

    Ocean Infinity has made another 3 AUV deployments in the last 24 hours.

    The weather is ok, there is good visibility, with a 15 knot wind, a combined swell and wave height of 3.2m. There are no tropical storms in the region.

    https://www.dropbox.com/s/d1gmiw4r6v2fwrk/SC%20Track%2029052018.pdf?dl=0

  499. Sabine Lechtenfeld says:

    @TBill, I fully agree with your latest comment – especially your take on the public not having been given the message that the disappearance of the plane was a criminal act. It’s indeed true that the media often reported very inaccurately, and they were too fascinated by the mystery aspect of the case. I noticed that the media gave every new fad, however absurd it was, it’s five minutes of breathless coverage. But it’s also true that the ATSB shied away from taking a clear position – probably because of geopolitical concerns.

  500. Victor Iannello says:

    @TBill: Whether or not it is impossible to protect against criminal actions of one or more crew, I agree with George Bibel that it is very difficult. As for the media, those that stick to the line of the official investigators are accused of not independently reporting on the case. Those that explore different scenarios are chastised for inappropriately premature statements. It is no surprise that there is so much mystery. The media are in a no-win situation. Groups of people will be very upset no matter what they say.

  501. Richard Godfrey says:

    Ocean Infinity has made the following statement on their website:

    https://oceaninfinity.com/conclusion-current-search-malaysian-airlines-flight-mh370-2/

    It is being widely reported in the media.

    At the same time Ocean Infinity continues to deploy AUVs and is carrying on the search?!??!!

  502. Richard Godfrey says:

    Apologies Don! I did not pick up that you had already linked the OI website.

  503. DennisW says:

    @Sabine

    probably because of geopolitical concerns.

    Yes. I think all of us tend not to appreciate the big picture. The loss of an aircraft with 200+ people is tragic, but it pales in comparison to world politics. Likewise with billion dollar defense systems such as SBIRS. Do I think we know where the plane went down? Maybe. Do I think we would reveal this information? Not a chance.

  504. Don Thompson says:

    I have posted a visual update of the bathymetry presently being surveyed by Ocean Infinity’s AUVs.

    The 2D and 3D rendering of the Batavia area is generated from Geoscience Australia’s published multibeam echo sounder (MBES) data, at high resolution, and GEBCO’s SRTM derived data, at low resolution.

  505. Don Thompson says:

    @DennisW

    SBIRS has been one of those ‘spook’ ideas that is resurrected from time-to-time.

    The role of SBIRS is to observe IR spectrum to detect missile launches. What’s the likely range of temperatures that would be recognisable from a rocket motor plume?

    As I look at Flightradar24 the service is tracking 14,211 aircraft, the vast majority of which are turbine engined and generating some level of moderately high temperature exhaust plume (100s of ºC in proximity to the exhaust duct). Add ships that employ gas turbine propulsion but I would assume that stationary ‘hot objects’ could be filtered out by SBIRS.

    If SBIRS cannot differentiate turbine exhaust plumes from rocket engine plumes there’d be a very significant signal-noise problem.

    Do you really consider it plausible that SBIRS would sense the exhaust plumes from turbine engined aircraft?

  506. Victor Iannello says:

    @Don Thompson: The hope was that SBIRS could detect the burst of energy in a crash.

  507. DennisW says:

    @Don

    I have no idea. I have asked friends who are peripherally involved as suppliers, but got no response at all. I do know that IR cameras (in aircraft) are capable of sensing a grid of PVC used to irrigate marijuana plants in the wild. The previous owner of my ranch was a grower, and I did not remove his system. Before law enforcement decided to look the other way I was visited more than once by camo clad people (calling themselves hunters) carrying scary black rifles.

  508. @DrB
    Thanks for your questions. The answers will come in subsequent separate messages to avoid lengthy posts 🙂

    As a Preamble:
    we followed an operational approach in contrast to a pure mathematical approach or a commercial pilot’s approach in a standard flight :
    1- We assumed that the aircraft wanted to hide and land safely but not to crash
    2- Thus knowing the 7th arc, this reduced the fields for investigation
    3- Knowing the constraints – flight envelop, fuel, usage of way points – the trajectory is deduced by adjusting the parameters in accordance with the Inmarsat data
    4- We did consider Learmonth and other southern and eastern targets: they led to incoherent values of the parameters and the fuel or the arc or speed/timing did not fit. We concluded by elimination.
    5- We put ourselves in People in Command’s place and did not stand on our own point of view: so for example your remark « make no sense » is understandable but it is your personal mathematician perspective. It may make sense if we knew all about the case. We just read the data and conclude out of it and we tried not to impose a way of flying.

  509. @DrB

    DrB “At 18:27:07 you use ROC = 150 fpm with a 360-degree track while at FL328. Yet the FLs are the same at 18:25, 18:27, and 18:28. How can you climb at 18:27 without changing FL from 18:27 to 18:28?”

    At 18:27:07, the aircraft was NOT climbing to our view, this small vertical speed comes from the rotation of the antenna around the longitudinal axis of the aircraft. At that time the aircraft was probably manoeuvring in its strategic lateral offset procedure. It was somewhere between finishing the right turn and beginning the left turn. Its roll from right banking to left banking generated an antenna movement close to 150 fpm. This is the estimation of the rotation of the body of the aircraft with the antenna on top of the roof which I computed from different simulations.

  510. @DrB
    DrB :”At 19:41 you use ROD = 400 fpm when at FL150. Yet one hour later at 20:41 the FL is only 140. Thus, you are saying the altitude only dropped by 1,000 feet in one hour, but there was a 400 fpm descent at 19:41, which only needs 2.5 minutes to drop 1,000 feet. How can a brief descent of several minutes coincide in time with the 20:41 handshake?”

    In our simulations, as from 18:40 approx the FMS automaticaly selected the flying mode « ACT ECON DES 240 kn » as soon as a first new lower FL was validated on the CMD. But this does not mean the aircraft was always descending. The RoD and target Flight Levels were input by the People in Command at different stages of the flight.
    We have some clear reading of some of these inputs while the others are less clear but deduced from the Inmarsat data.

    Remember that we « listen » to the Inmarsat data. It appears that the aircraft descended step by step mainly when needing to pass safely under the crossed airways and to stay hidden from possible detection by radars or other traffic. It used the most economical way to descend i.e IAS=240 kn. At 19:41 (arc 2) it was at the end of the first leg of descent which started just before crossing Route P627 (MFL 260) and finished just before crossing route P756 whose MFL (minimum Flight Level)= 160.
    This corresponds to a continuous descent from FL270 to approx FL150 at ROD=500 fpm during approx 24min. It is not a short time laps. The path of the aircraft is so tangent to arc2 and the arc position is dependent of the exact altitude such that they are almost superimposed, thus being at Arc2 on time is not an issue.

    Having passed under route P756, for us the aircraft stayed levelled at approx FL150 until approaching POSOD which is the entry door to the Australian airspace. In between there was Arc3 at 20h41: it is interesting to note that the distance, ground speed (changing but imposed by KIAS=240) and time coincide very well.
    Still reading the data, the aircraft was starting a second leg of descent before entering to this airspace. Why ? Difficult to say except perhaps that each time it quit a FiR and entered into a new one there was a change of FL and/or of direction, but it is just a guess (and does not change the data ☺).
    If you simulate this flight you will see that the aircraft beautifully and smoothly follows the path on time (even thought it is not exactly the same aircraft type)

  511. @DrB
    DrB : “Similarly, you apply a 500 fpm descent at both 20:41 and at 21:41. How can the altitude only change 1,000 feet if you are dropping 400 fpm at 19:41 and 500 fpm at 20:41 (and then you do it again at 500 fpm at 21:41)?
    The entire sequence of descent rates you need to match the observed BFOs from 19:41 – 21:41 makes no sense to me. The three alignments in time of very brief descents with the then-unknown handshake times is not credible to me. At those low altitudes I don’t think there is any need to “pass under” or “pass over” an airway multiple times. I think it is much more likely that your route has track errors causing the BFO mismatches for which you use varying descent rates.”

    I am not sure I understand your question. CAPTIO trajectory is right on time at all arcs if you fly at KIAS=240 and descend and after 21H41 pass to cruise mode which is at KIAS~285.
    Please could you precise what you call a « track error » ?

    We estimated the BTO and BFOs with Yap Fook Fah’ s model. We don’t have our own algorithm for computing them.
    Do you want me to give you the exact coordinates and speed and ROD/ROC of our trajectory? Would you like to cross check them?

  512. @DrB
    DrB: ” Another question has to do with the drift angle w.r.t. the wind you assumed for the flaperon. Nesterov uses 0, 18, and 32 degrees to the left (from the Meteo France report by Daniel), whereas your model uses 16 degrees to the right. How did you determine this value, and why is it different from the Meteo France measurements of the flaperon?”

    Thanks for detecting an error in the report. Following CSIRO’s report 2, we do have actually implemented 16° to the left and not to the right. Sorry for the mistake in the reporting. We will correct this.

    Why 16° ? Because we preferred to stick to the genuine flaperon experiments results as described in the CSIRO report.
    On page 10 CSIRO states « There is a lot of variability in our observations but it does appear that the leeway angle of the genuine flaperon is non-zero and definitely positive (to the left). The mean value is 16° (including one outlier deployment when the drift was a small angle to the right). »
    The choice had to be done between 10° and 20° and we chose the value closest to the reality i.e. the one coming from their estimation from the genuine flaperon.

  513. @DrB
    DrB:”Another issue with your proposed route may be fuel. There seems to be enough fuel if the portion after 21:41 is flown at HOLDING at FL90/50, but I think this is considerably slower than the 300 knots ground speed you used. At 300 knots there may not be sufficient fuel. I would be interested in seeing any detailed fuel calculations that have been done, to compare with my back-of-the-envelope numbers, if you have them.”

    Here, I would recall our philosophy. We did not try to fly our way of flying but we tried to understand how they could fly properly the way the Inmarsat data tells us as we don’t know who was in command. So the HOLDING might have been a good way to fly, I don’t know. If your computations fit the arcs and timing and fuel then it would be a potential solution. That would be fantastic.
    Thanks to you to have made it publicly available ☺, we have used your fuel consumption model 5.5 to estimate the fuel burn from take-off to the end. The settings were mode=KCAS, PDA = 1.5%, Delta Sat = 13, ACP= On. We did change the speed during the flight i.e your Fixed Mach or speed parameter, the ROC and the FL as well accordingly to the profile of CAPTIO trajectory.
    Should I send you by email the consumption table with our trajectory ?

    In conclusion, the question is: Is this trajectory impossible ?
    Until this is demonstrated, CAPTIO trajectory is possible (human in command) and should stay in the list of potential options ☺.
    Thanks

  514. Victor Iannello says:

    @Jean-Luc Marchand: In your scenario, you essentially forced the flight path to satisfy the Inmarsat data assuming the pilot’s attempt was to get to Christmas Island. But at the closest point along the 7th arc, CI is about 111 NM away, as if the plane flew away from CI and crashed. Even if we accept the strange combination of perfectly timely descents to match the BFO values, you will have a difficult time explaining how a pilot wishing to land on Christmas Island would crash 111 NM further than his or her target runway.

  515. DrB says:

    @Jean-Luc Marchand – CAPTIO,

    You said relative to the 150 fpm ROC at 18:27:07: “This is the estimation of the rotation of the body of the aircraft with the antenna on top of the roof which I computed from different simulations.” I calculate an aircraft roll rate of about 53 degrees/sec is needed to generate that ROC; which obviously is too large to be feasible. I guessed a radius of about 3 m from the roll axis to the antenna.

    I am not questioning the particular BTO/BFO values you predicted. I have also calculated them and find values close to your results. My issue is that you pick a track angle and then allow significant ROCs so that the BFOs can be matched. I can’t prove you are wrong, but there are many combinations of track and ROC that do this, and I don’t see the logic that drives your choice of track angles over other possibilities. I suggest you make a plot of FL versus time for your scenario. That very busy graph will show numerous changes in FL, for which I, at present, don’t see the purpose(s).

    Yes, please send me the fuel consumption table for your scenario (done with my fuel flow model). I should like to look it over. Certainly there is sufficient fuel to fly the total range of your proposed path. My question was whether or not this was feasible using your particular speed/altitude schedule. As you have figured out, it’s quite straightforward to apply any flight scenario simply by editing the speed/altitude/ROC columns at the appropriate times in my spreadsheet. That’s one of the things I had intended when I published it, and I am glad you found it useful.

  516. Sabine Lechtenfeld says:

    The main problems with Jean-Luc Marchand’s CI scenario:

    – Why would the same pilots who after the turnaround near IGARI clearly demonstrated that they knew very well how to fly and navigate a 777, somehow “miss” CI at the end of the flight?

    – Where did these capable pilots come from? The background checks of the passengers didn’t reveal anyone who would’ve been capable of expertly flying a 777. Or do you believe the crew was forced to play along? But in this case it would be even less plausible why CI was missed.

    – Why did they run out of fuel? They had enough fuel onboard for comfortably reaching CI. They just had to fly the aircraft at cruising speed and altitude after having cleared the northern tip of Sumatra. Why flying a slow and curved route instead? If they had flown to CI in cruising mode, but wanted to land after sunrise, they could’ve gone into a fuel saving holding pattern after reaching the island and wait for dawn. Or, if landing on CI was denied for some reason, they may have had still enough fuel for reaching another airport. What could’ve been the reason for creating such a risky “land-or-crash” scenario?

    – Why didn’t they try to contact anyone on CI if they wanted to land there? Even rogue pilots normally try to negotiate a clearance for a landing.

    – Why, if they didn’t intend to kill anyone, didn’t they make an SOS call when the fuel situation got tight?

    – How do you explain the sim data which were found on ZS’s computer? Do you think that ZS made those virtual fuel-exhaustive flights into the SIO just for harmless recreational purposes, and it’s a total concidence that his plane crashed a few weeks later on a southern route because of fuel exhaustion?

    The approach of just following the data without trying to construct a viable narrative goes only so far IMO. In this case the narrative becomes so bizarre that I wonder if it is really worth pursuing it.

    That said, there may be other CI scenarios which could make a bit more sense, especially if you give up the idea that CI was somehow missed and that the final crash was a regrettable mistake. A voluntary crash after failed negotiations would eliminate some but not all of the problems I listed.
    You say that you don’t know who was flying the plane and what the motivation of the abduction was. But you carefully avoid to mention any scenarios where Zaharie Shah was the perpetrator who crashed the plane.

  517. Don Thompson says:

    @Victor,

    I was unaware that impact detection might have been expected from SBIRS.

    @DennisW wrote “I do know that IR cameras (in aircraft) are capable…

    Yes, aircraft and LEO IR sensors are useful for mapping out heat sources. Searching for buildings with high heat loss is a often quoted tactic to find illicit ‘greenhouses’.

    What’s the likelihood that SBIRS consumes ‘bandwidth’ to scan a cold, barren, ocean?

  518. Don Thompson says:

    @DrB & J-L Marchand

    9M-MRO’s satcom high gain antenna comprises two apertures, one located on each side of the fuselage above door 3L and 3R, not the single aperture type that is mounted on the fuselage crown. At 18:25 the L/port aperture would be active & look angle to I3-F1 is 51.5º.

    How would the simulation of roll affect BFO for that configuration?

  519. @DrB

    Flight Profile versus time : a sketchy diagram is given with all FLs in our report and you can find it on the home page at http://www.mh370-captio.net.
    It follows the logic of avoiding radar and avoiding to cross airways for not being detected and safely cross other traffic.

    After having spend much time on this, there aren’t so many trajectories satisfaying all the constraints including standard waypoints.

    @Sabine
    we don’t have all the answers 🙂 We also have questions but we want to consider the data first because if all speculative questions are put altogether they form kind of a decision tree which does not help too much to our view. A narrative is not verifiable at this stage. CAPTIO trajectory is a realistic and feasible path matching the knowm data and this can be verified technically now.
    If CAPTIO trajectory has been flown, it shows a high level of sophistication and knowledge of the system (I did not say of piloting 🙂 but went wrong at a certain point.
    I could give my view on the fuel and the question why it missed Xmas…but it would be pure speculation.
    The known fact is: it did not land on Xmas.

  520. DennisW says:

    @Don

    What’s the likelihood that SBIRS consumes ‘bandwidth’ to scan a cold, barren, ocean?

    I don’t know anything about the SBIRS operational protocols. It is true that there is a scanning sensor which uses the rotation of satellite to do the scanning (six full rotations per minute) so the scanning is never inactive. Whether the data link operates 24×7 is not published. The other sensor is a “staring” sensor which can be pointed at areas of interest (which a cold dark ocean is not).

  521. DennisW says:

    @Jean-Luc

    It is a tough group. The main reasons I abandoned the CI scenario are:

    1> As Victor pointed out, flying past CI is difficult to explain.

    2> As Sabine pointed out, the lack of communication prior to crashing is not consistent with an intended safe landing somewhere.

    3> Don Thompson pointed out much earlier that the fuel remaining instrumentation on the 777 would make a surprise running out of fuel very unlikely.

    4> The CI scenario is not compatible with the sim data.

    As Sabine pointed out above the exercising of the “or else” part of a negotiation could explain all the above except the sim data incompatibility.

  522. formula says:

    Concerning some the points of objection to the CI hypothesis, I speculate with two possibilities, either: –

    (I) Were there negotiations planned with the Malaysian government, CI might have been the intended destination presuming success or delay. If negotiations were assumed to have failed (by lack of contact from negotiators with the aircraft), a change of plan might well have called for a different end of flight, or

    (II) if the pilots were acting under duress, they may have accepted instructions to make for CI but en route determined that an end of flight from fuel exhaustion over the ocean was preferable to conforming further with their captors wishes.

    I acknowledge we have no information or quality evidence pointing to either negotiations or to stowaways or persons known to be on board taking on the character of hijackers.

  523. Sfojimbo says:

    “we have no information or quality evidence pointing to either negotiations or to stowaways or persons known to be on board taking on the character of hijackers.”

    Why do you say “quality evidence”? You have no information or evidence of any kind for any of these theories.

  524. Andrew says:

    @Sabine Lechtenfeld

    RE: “But it’s also true that the ATSB shied away from taking a clear position – probably because of geopolitical concerns.”

    Indeed, the ATSB used such concerns on several occasions to justify withholding ‘restricted’ information from the public. The ATSB is not supposed to make its views known because it is not the investigating authority. If the ATSB were to release information without Malaysia’s consent, its accreditation to the investigation would likely be affected, along with its involvement in future accident investigations where Australia might have an interest. ICAO Annex 13 states: Accredited representatives and their advisers…shall not divulge information on the progress and the findings of the investigation without the express consent of the State conducting the investigation.” Other accredited representatives such as the BEA, NTSB and AAIB are in a similar position. As Dennis said, it’s largely about the bigger picture.

  525. DrB says:

    SBIRS is a ballistic missile detection and tracking system. It incorporates multiple wavelengths of infrared sensors which detect and characterize the exhaust plumes from both solid and liquid fueled rocket engines during boost phase. It is not responsive to smaller and colder exhaust plumes from non-rocket engines. It also discriminates against stationary heat sources since it is expecting high-speed missiles. I suspect SBIRS detected the BUK anti-aircraft missile which destroyed MH-17. I think it is extremely unlikely to have detected the impact of MH370.

  526. formula says:

    @ Sfojimbo “Why do you say “quality evidence”? You have no information or evidence of any kind for any of these theories.”

    I suppose I was trying to indicate that there is insufficient (in the public domain at least) to completely rule out such type of theories. For myself, I have difficulties with both theories and so look to some other explanation.

    The unexplained behaviour of Malaysian officaldom in the immediate hours and days following the disappearance might be taken a evidence that there was knowledge of events beyond what was revealed by radar and perhaps that negotiations were opened if not pursued: clearly, the evidence of that behaviour is not of sufficient quality to permit more than speculation.

  527. Mick Gilbert says:

    @TBill
    @Sabine Lechtenfeld

    Why assertions of criminality have been avoided by the ATSB go beyond the point raised by Andrew earlier and relate to the purpose of safety investigatory bodies such as the ATSB, the UK’s AAIB and France’s BEA. None of them are meant to assign blame. They all draw on Section 3 of Annex 13 (Aircraft Accident and Incident Investigation) to the Convention on International Civil Aviation, specifically that ‘The sole objective of the investigation of an accident or incident shall be the prevention of accidents and incidents. It is not the purpose of this activity to apportion blame or liability.

    Australia’s Transport Safety Investigation Act 2003 reflects the Annex 13, Section 3 objective in its stated functions of the ATSB and the British Civil Aviation (Investigation of Air Accidents and Incidents) Regulations 1996 does likewise for the AAIB. In fact the Australian TSI Act goes even further in stating quite clearly what activities the ATSB must steer clear of. Section 12AA Functions of the ATSB of the Act states that:

    (3) The following are not functions of the ATSB:
    (a) to apportion blame for transport safety matters;
    (b) to provide the means to determine the liability of any person in respect of a transport safety matter;
    (c) to assist in court proceedings between parties (except as provided by this Act, whether expressly or impliedly);
    (d) to allow any adverse inference to be drawn from the fact that a person was involved in a transport safety matter.

    Although we’re talking about a different country, the MH17 incident serves as an excellent example of how the restrictions work in a practical way. The air safety investigation authority, the Dutch Safety Board, made no findings as to blame and made no mention of criminality in their findings. The criminal aspects of the MH17 shoot down were handled by an entirely separate authority, the Joint Investigation Team, which had been formed under the auspices of the Dutch Public Prosecution Service.

    In short, the ATSB has steered well clear of any assertions or suggestions of criminality with regards to MH370 because not only is it not their place to do so but also because they are specifically prohibited from doing so.

  528. Sabine Lechtenfeld says:

    @Mick Gilbert, what you say, may be true. But that doesn’t mean that this state of affair isn’t fraught with problems, which could’ve been handled much better. The Dutch safety board didn’t shy away from clearly addressing the cause of the crash – human intervention in form of a shootdown by a BUK missile. The ATSB could’ve stated equally clearly that MH370 has been most likely abducted by a crew member and that a disaster is highly unlikely. If it is, as you say, their job to prevent future incedents and promote the safety of air traffic, they need to address clearly the most likely cause for the loss of the plane, especially if this potentially affects the search strategy. In both of these cases the lines of inquiry between investigating the loss of a plane and a criminal investigation become blurred. The ATSB choose to hide behind this cloud. The Dutch safety board didn’t.
    That said, the actual criminal investigation whose job it is to identify the perp, is (unfortunately) in the hand of the Malaysians. And they have been more than unwilling to do their job properly. They have been obfuscating instead of illuminating. This is not the fault of the ATSB, although they could’ve put much more public pressure on the Malaysians by stating clearly, the known facts and data indicate that the plane has most likely been abducted by a crew member. Unlike the Dutch safety board the ATSB choose not to do this – probably because of geopolitical reasons.

  529. Andrew says:

    @Sabine Lechtenfeld

    RE: “The ATSB could’ve stated equally clearly that MH370 has been most likely abducted by a crew member and that a disaster is highly unlikely.”

    It was not within the ATSB’s remit to make such a statement; they are not the Annex 13 investigating authority.

  530. Mick Gilbert says:

    @Sabine Lechtenfeld

    Sabine, the ATSB is not the Annex 13 investigative authority on MH370 so there are no lines to blur and no clouds to hide behind. It is not their job to offer prognostications as to cause, period.

    To the extent that it could advise the search strategy they addressed the possibility of a pilot-controlled glide in their first Definition of Underwater Search Areas paper and then in subsequent papers addressing the underwater search strategy. That was all that was reasonably necessary for the formulation of possible search area boundaries (it set the outer bounds at ± 125 NM from the 7th arc).

    Now, while none of that may suit the grab-a-rope lynching brigade, it’s the law.

  531. Sabine Lechtenfeld says:

    @Andrew, in this case the ATSB shouldn’t have made any public statements whatsoever. But they clearly have expressed preferences for certain scenarios.

  532. Mick Gilbert says:

    @Sabine Lechtenfeld

    When you say ‘But they clearly have expressed preferences for certain scenarios.‘ you’re talking about end-of-flight scenarios. The ATSB appraised the likelihood of end-of-flight scenarios (controlled versus uncontrolled) in order to set the search area boundaries. Their preference for an uncontrolled end-of-flight scenario, a preference generally accepted as being supported by the available evidence, says nothing about the possible cause.

  533. Sabine Lechtenfeld says:

    @Mick, yes, this is correct as far as the end-of-flight scenarios are concerned. But what for example about Peter Foley’s newest idea that the pilot might’ve succumbed to decompression sickness? Foley clearly formed an idea about what might’ve happened on this plane, and he said so publicly. But there are no data or facts whatsoever which support or negate the idea that there was a decompression event in the first place. So, shouldn’t he have kept this opinion to himself?
    I agree that the ATSB was stuck from the very beginning between a rock and a hard place. But I think they haven’t handled it very well.

  534. @DrB
    @Don Thompson

    Antenna rotation:
    1) the position on the side is not an issue
    2) assumptions:
    2a)the antenna is rougthly at 4 to 4.5 meters from the rotation axis between the 2 wings around which the a/c is rotating (4m is thus used).
    2b) max banking 25°
    2c) elevation 51.5°
    2d) the angular velocity can be estimated from right max banking to left max banking in about 5 sec (conservative slow value) so approx 10°/s
    3) Back-Projecting the tangent speed to the vertical axis in order to determine the equivalent RoC that would impact the doppler the same way on the Sat-Aircraft vector, the estimation is about RoC~150fpm

  535. Mick Gilbert says:

    @Sabine Lechtenfeld

    Sabine, Foley was being pressed by Senator Patrick to explain how a flight could be controlled at the outset but not at end-of-flight. Foley makes it clear that prolonged depressurisation is what is being speculated by ‘the pundits’ (Vance, Bailey, Keane, etc). He provides the Connie [Kalitta] Flight 861 example to demonstrate that it’s possible for someone who is knowingly flying a depressurised airplane, even after having taken the recommended precautions regarding oxygen, to become incapacitated. It’s simply an example of how a flight that is controlled at the outset might become unintentionally uncontrolled. He concludes that exchange by saying,

    So there’s one possible — I’m not saying that happened and I hate to speculate, but it’s at least one plausible scenario.

    I don’t think that you can infer that Foley has formed a view as to what might have happened, he’s simply providing Senator Patrick with an answer to his line of questioning.

  536. ArthurC says:

    I have no intention of deviating the conversation, but I remember that a long time ago there were speculations about the impact being detected by a microphone system in the SIO.
    I forget the scope of that system and I tried to search the web for it, but came out empty.

    The reason I brought it up was the fact that it wasn’t mentioned a lot during the search.

  537. Sabine Lechtenfeld says:

    @Mick, Foley clearly wasn’t in an enviable situation, but according to the rules which you have cited earlier, he should’ve just said, that it isn’t his job to speculate was was going on in the plane at certain moments during the flight. And this isn’t the only but just the latest instance where they broke away from this rule. Just a few months ago shortly after OI started the new search, they said, they couldn’t imagine that the plane crashed further north than 35°S “because crash areas further north could only be explained if the plane loitered somewhere, which doesn’t make sense”. I paraphrased. With this statement the ATSB dismissed a whole group of scenarios.
    The big conundrum is course: how can the ATSB be expected to develop a viable search strategy and find the plane without looking at the available facts and form an opinion about what on earth happened to that plane?

  538. Sabine Lechtenfeld says:

    @ArthurC, you are probably talking about the so called “Curtin Boom” which was a low frequency underwater sound signal detected by underwater microphones of the Curtin University in Australia, which were designed for monitoring sea quakes. The “boom” happened at the right time, but the area of origin (the Central Indian Ocean) isn’t compatible with the satellite data and the so called 7th arc. This boom has nevertheless intrigued a lot of people, who questioned the validity of the Inmarsat sat data in the first place.

  539. DennisW says:

    @DrB

    I think it is extremely unlikely to have detected the impact of MH370.

    Yes, me too. There is a lot out there (on the WEB) hinting at SBIR capabilities i.e. detecting hot spots in forest fires and combat aircraft in war zones. Clearly the primary role of SBIRS is, as you say, detecting missile launches.

  540. Victor Iannello says:

    @Don Thompson, @DrB: I don’t think SBIRS captured MH370’s impact with the ocean. However, at the time, there was some hope that SBIRS captured either a mid-air explosion or an explosion upon impact, either of which would be a large thermal event of wide thermal spectrum.

  541. Don Thompson says:

    @ArthurC & Sabine

    The Curtin researchers analysed recordings made at the CTBTO HA01 hydrophone array off WA’s Cape Leeuwin and the Australian IMOS hydrophones that were located offshore from WA.

    The CTBTO array is capable of discriminating direction (a triad of hydrophones) whereas a single hydrophone is deployed at the IMOS sites. To complicate matters, the IMOS hydrophones operate on a 33% duty cycle (recording for 5 minutes in every 15) and they are autonomous recording units, not permanently online like the CTBTO arrays.

    An event was detected that originated from a point on a bearing that crossed the 7th arc but that event arrived 50 minutes later than would be expected from a point on the arc, i.e. the event originated further away.

    While there was speculation that a surface impact might generate hydroacoustic waves that would carry through the ocean, it’s unlikely that a surface event over deep ocean will translate into the deep sound channel and propagate through the channel. An event, typically, must originate in the deep sound channel or on a shallow shelf sloping down into the deep sound channel.

    Curtin’s researcher’s placed a point of origination somewhere near the Laccadive Ridge.

  542. Don Thompson says:

    @Victor/Dr B/DennisW

    Unfortunately SBIRS isn’t included in the Disasters Charter (Charter On Cooperation To Achieve The Coordinated Use Of Space Facilities In The Event Of Natural Or Technological Disasters).

    However, classified visible imagery was released to the search authorities.

    I do tend to think the SBIRS sensors weren’t suitable for the problem.

  543. airlandseaman says:

    Jean-Luc Marchand – CAPTIO
    Re antenna pointing: all HGAs (both electronically steered and mechanically steered) are required to point accurately under dynamic conditions, such as turns. The pointing algorithms use rates from the 429 bus to meet those requirements. See Rockwell patent for example.

  544. TBill says:

    I had thought maybe SBIRS could detect engine flame out, but I’ve found very few pictures of flame out. There was a fairly thick cloud layer below about 22S. Ed Baker’s theory is the aircraft was crashed above 22S to keep out of the heavy clouds.

  545. Sabine Lechtenfeld says:

    @Don, re: “Curtin Boom”, hasn’t it been suggested that rather than an impact on the ocean’s surface, an underwater disintegration of the plane’s fuselage or rump might’ve been responsible for the sound which was detected by the hydrophones?
    I remember that at the time the scientists were very eager to promote at least the possibility of such an explanation of the underwater sound event. And they wanted to keep the discussion open for quite a while, although they never denied that the point of origin of the sound wasn’t compatible with the sat data. But the Curtin Boom became lodged in the collective brain of the MH370 community nevertheless.

  546. Don Thompson says:

    @Sabine,

    You are correct, it has been suggested that some component of the aircraft may have survived the impact, slowly descended through the water, and then collapsed under prevailing pressure at some depth. The depth would be within the bounds of the deep sound channel.

  547. Richard Godfrey says:

    SC is continuing to make good progress up the Broken Ridge plateau area and has reached 26.0419°S.

    Ocean Infinity has made another 4 AUV deployments in the last 24 hours.

    The weather is good, there is reasonable visibility, with a 6 knot wind, a combined swell and wave height of 2.7m. There are no tropical storms in the region.

    https://www.dropbox.com/s/3hyunbw33rncvjz/SC%20Track%2030052018.pdf?dl=0

  548. @DennisW
    @Sabine

    Our view it should be the other way around: the finding of the wreck and its analysis will give the clues to understand the hijacking scenario. We would not like to enter speculations.
    In your questions 1 to 4 you made an implicit assumption which is still open: you suppose it is an experienced pilot. This is a perfectly valid view leading to your questions which are effectively open in an operational stand point.
    Please, let’s take another perspective for a minute. For example « people in command with low experience but sufficiently trained to fly via the FMS and A/P», then one can see that questions 1 to 3 will receive valid answers.

    In question 4, I believe you refer to the simulator data retrieved from the Zaccharie’s PC, don’t you?
    My question is : are the sim data a factual part of MH370 flight ? I don’t think so.
    Concerning our simulations with the factory setting as fuel burn is concerned, we always succeeded to land on Christmas Island with 1 ton left in the tanks. But with Bobby Ulich’s refined model, the fuel burn is more accurate and the exhaustion occurs at the 7th Arc.

  549. Sabine Lechtenfeld says:

    @Jean-Luc, I completely agree with you that the main objective is finding the plane, which may then ultimately yield the information of what actually happened. But unfortunately the search is time and money consuming. OI or whoever is willing to take up the gauntlet in the future needs a good reason for taking up the search again in a specific area. Personally I would be more than happy if the 7th arc will be searched up to the Javan coast if necessary. Maybe, I’m totally wrong about this, but I doubt that your flight path and the underlying assumptions will convince them to look specifically into your scenario.
    However, since there are other CI scenarios – maybe, this will be checked out after all in the future if Malaysia permits. And that might be the greatest obstacle to future searches 🙁

  550. Paul Smithson says:

    @Richard G. Interesting that new AUV deployments continue even after the MY announcement of “search over”. Do you have any notion of the likely duration of the current tour of OI and the latitude that they might reach before they finally head for port?

  551. Andrew says:

    @Sabine Lechtenfeld

    RE: ”Foley clearly wasn’t in an enviable situation, but according to the rules which you have cited earlier, he should’ve just said, that it isn’t his job to speculate was was going on in the plane at certain moments during the flight.”

    Perhaps so, but Foley was being pressured to explain the ATSB’s position, so he provided an example that explained how there might have been control inputs early in the flight and none at the end. He very deliberately said “I’m not saying that happened”. Both Foley and Greg Hood said they are not in the business of speculation or conjecture, or words to that effect. Throughout the search, the ATSB has been very careful not to speculate about what might have happened to cause the aircraft’s disappearance. On at least two occasions they said:

    ”For the purposes of its search, the ATSB has not needed to determine – and has made no claims – about what might have caused the disappearance of the aircraft. For search purposes, the relevant facts and analysis most closely match a scenario in which there was no pilot intervening in the latter stages of the flight. We have never stated that hypoxia (or any other factor) was the cause of this circumstance.”

    RE: ”The big conundrum is course: how can the ATSB be expected to develop a viable search strategy and find the plane without looking at the available facts and form an opinion about what on earth happened to that plane?”

    Which ‘available facts’ did they fail to consider in developing their search strategy?

  552. ArthurC says:

    @ST, @Sabine

    Yes, thank you very much for the info, that’s the one I was thinking of.

    Just a thought, the Inmarsat data is a crucial piece of evidence, maybe the only one tangible fact that exists, at least for the latter portion of the flight.
    I hate to say this, but if there is even the slightest deviation, it could position the 7th arc just outside of any existing or potential search area.

    Was that thoroughly analyzed and validated?

    Since the wreck wasn’t yet found, I was trying to clarify in my own mind what is irrefutable fact and what is derived or speculated.

  553. Sabine Lechtenfeld says:

    @Andrew, where did I say that the ATSB failed to consider any available facts? If they have or haven’t is debatable. But that wasn’t at all what TBill and I were originally talking about, and what sparked this discussion.

  554. TBill says:

    @ArthurC
    We have the Inmarsat data at 18:25 to 18:28 which correlates quite well with the last radar contact at 18:22 just past MEKAR. That assumes Lido slide is correct. Much of the search has given preference (greater search distance) to the area outside of Arc7.

  555. Richard Godfrey says:

    @Paul Smithson

    You asked: “Interesting that new AUV deployments continue even after the MY announcement of “search over”. Do you have any notion of the likely duration of the current tour of OI and the latitude that they might reach before they finally head for port?”

    OI plans 6 week tours from port to port. This tour started on 1st May 2018 in Fremantle, and if returning to Fremantle, then they will be back by 11th June 2018. This means leaving the search area at the latest by 8th June 2018.

    By 8th June 2018, OI will have reached 24.7°S at their current rate of progress, weather permitting.

  556. TBill says:

    @Richard
    @Victor
    That’s amazing by OI…I am so glad they were able to check the McMurdo hypothesis…I was feeling a little guilty when they spent much time on BR.

  557. Paul Smithson says:

    @Richard. Many thanks for that and for your prior work to provide us with daily updates of OI’s progress.

  558. Don Thompson says:

    I created some Google Earth animations which include the SLDMB track information received from AMSA, a result is posted at Vimeo.

    The main ‘particle’ field is CSIRO/Griffin’s KMZ depicting a flaperon beginning a drift at S27º.

    The clip spans from 8th Mar 2014 through to early May 2014 when the aerial search ceased. The yellow areas denote AMSA’s estimated air search coverage, it remains my contention that the search lines should be much, much thinner for a purely visual search.

  559. ventus45 says:

    @ Don Thompson
    Vimeo link does not work

  560. Don Thompson says:

    @Ventus45

    Apologies, I obvsly omitted to go back & paste in the URL.

    Clip at Vimeo

  561. Andrew says:

    @TBill

    RE: “But that wasn’t at all what TBill and I were originally talking about, and what sparked this discussion.”

    You were discussing TBill’s ‘take on the public not having been given the message that the disappearance of the plane was a criminal act’. You said the ATSB had ‘shied away from taking a clear position’, probably due to geo-political concerns. I agreed with you, but also pointed out that the ATSB is severely constrained in what they can say in public, because they are not the Annex 13 investigating authority. The ATSB cannot make public statements about what they think caused the aircraft’s disappearance. Period.

  562. David says:

    @Richard Godfrey. “By 8th June 2018, OI will have reached 24.7°S at their current rate of progress, weather permitting.”

    Thanks for providing such estimates.
    Assuming the OI search authorised by Malaysia concluded on 29th May, additional searching may be a rewardless prospect. OIs motives and determination need recognition if so.

  563. Andrew says:

    Oops, my previous post was intended for @Sabine Lechtenfeld, not @TBill.

  564. ventus45 says:

    @Andrew

    AMSA are the primary “search” agency, with significant maritime search and recovery experience, and it is, after all, “their remit”.

    Do you have any thoughts on “why” AMSA was “dumped”, at the time that it was “dumped”, and the job given to the ATSB, an organization with no “remit” for the task, and no previous, even remotely relevant, experience, let alone, capability ?

    Might one reasonably conclude, that it may have been a deliberate move, by the powers that be, to garner the “top cover” secrecy provisions afforded by Annex-13 etc, to best protect those “geo-ploitical sensitivities” ?

  565. Andrew says:

    @ventus45

    I’ll start out by saying that I am not a fan of conspiracy theories; the ‘truth’ is usually something far more simple.

    As I understand it, AMSA was responsible for coordinating the surface search until 28 March 2014, a task for which they were well qualified. When that search failed to find the aircraft, it became obvious that a far more complex technical investigation was needed to define an underwater search area. That work required input from Boeing, Inmarsat, Thales and other outside agencies such as the NTSB and AAIB. I believe the ATSB was the organisation best placed to complete that work in this region. AMSA has considerable SAR expertise, but I doubt it has the ability to manage a complex technical investigation.

  566. Pilatus says:

    @Andrew

    “AMSA has considerable SAR expertise, but I doubt it has the ability to manage a complex technical investigation.”

    Regrettably the same can be said of the ATSB based on some of their previous investigations.

  567. DennisW says:

    @Andrew

    +1

  568. Andrew says:

    @Pilatus
    @DennisW

    Perhaps I should have said ‘theoretically’ to satisfy the naysayers. Complex technical investigations are part and parcel of the ATSB’s work, even if some of the organisation’s past investigations are questionable. Was AMSA really a better choice? If not, who?

  569. DrB says:

    @Jean-Luc Marchand;
    @Don Thompson,

    Don kindly provided me with some dimensions. The antenna is located at an angle of 45 degrees from vertical at a radius of 3.1 m. Near wings level, it takes 20 degrees per second or roll rate to achieve 150 fpm in the vertical direction. At 25 degrees bank to the left, it takes 15 degrees per second, and at 25 degrees bank to the right, it takes 40 degrees per second roll rate to generate 150 fpm vertically. I would guess that all these cases exceed the normal autopilot roll rates.

  570. DennisW says:

    @Andrew

    To be clear, I was agreeing with you. The ATSB was the logical choice.

  571. Andrew says:

    @DennisW

    My apologies. I also believe they were the logical choice, despite some past ‘transgressions’.

  572. Andrew says:

    @DrB

    RE: “I would guess that all these cases exceed the normal autopilot roll rates.”

    Absolutely – it’s a transport aircraft, not a fighter!

  573. ST says:

    @Victor/Andrew –
    Below a snippet from an article in the Australian news –

    After doing his best for four years to avoid the issue, in public at least, the head of the Australian Transport Safety Bureau’s underwater search for MH370, Peter Foley, last week came his closest to saying Zaharie must have hijacked the aircraft. It was flown deliberately for about 1½ hours after it deviated off course, Foley told a Senate estimates hearing, and “it’s absolutely evident … an aircraft doesn’t turn itself”.

    From a pure perspective of your experience it would be good to hear your analysis/opinion on the above statement.

  574. Mick Gilbert says:

    @ventus45

    With regards to the underwater search I wouldn’t say that the ATSB had ‘no “remit” for the task, and no previous, even remotely relevant, experience, let alone, capability‘. Let’s not forget that the A in ATSB doesn’t stand for Air. Unlike the UK’s AAIB the ATSB doesn’t deal exclusively with air accident investigations, rather, like the US’s NTSB they deal with transport accident investigations covering air, rail and, perhaps somewhat relevantly to finding something underwater, maritime.

    While the ATSB’s initial handling of the Norfolk Pel-Air accident attracts a lot of criticism it probably worth remembering that the re-opened investigation involved the ATSB finding the submerged wreck of VH-NGA and recovering the flight recorders. Finding an aircraft wreck in 48 metres of water five years after it sank is certainly not the same as finding one in more than 100 times that depth of water but it probably meets the ‘remotely relevant experience’ threshold.

  575. DennisW says:

    @Mick

    For my part I thought the ATSB did a good job of conducting the search for MH370. The fact that it was not found is no reflection on them. They were looking where the analysts told them to look (which was a logical place considering the data). I think they can hold their heads up high despite the result.

  576. Mick Gilbert says:

    @DennisW

    I don’t disagree, Dennis. However, from my perspective, the deficiency in the ATSB’s management of the search was that there should have been something along the lines of the First Principles Review conducted much sooner; certainly no later than mid-2016 when they still had time (and budget) to incorporate lessons learned into a revised plan. I don’t know the ins and outs of Bayesian theory but from my limited understanding it seems that if you don’t find something in the predicted high probability area then you’re probably not going to; it should be read that you’ve probably got one or more bad assumptions. And there didn’t appear to be any active reassessment of the search strategy to incorporate the drift analyses for the recovered wreckage items. They adopted a post-mortem approach rather than an active management approach to the search.

    Having said all that, logical modifications to the search would probably have fallen into one of two approaches; go longer (further along the arc) or go wider. I get a sense that even after a review that they would have ended up going longer, just as OI are doing, rather than going wider. With the benefit of 20/20 hindsight, it would appear that going wider may have been the way to go.

    I’m also struck by the fact that the difference between release-the-pigeons, pop-the-champagne success and complete, utter and abject failure may be as little as 100 metres. Reality is brutal.

  577. Andrew says:

    @ST

    RE: “From a pure perspective of your experience it would be good to hear your analysis/opinion on the above statement.”

    Peter Foley said: “It’s absolutely evident. We’ve always been in agreement with the notion that an aircraft doesn’t turn itself. I mean, there must have been someone in control of that aircraft, probably until about 18.25 or thereabouts.”. At the time he did not say that ‘someone’ was Zaharie, or indeed that it was one of the pilots. Greg Hood, the ATSB Chief Commissioner/CEO, reinforced that point when he said: “We’ve haven’t really got any evidence and we don’t want to get into conjecture. Once again, we’re not saying it was the pilot either; we’re saying that control inputs were made, because we’ve got no evidence to suggest that it was the pilot.”

    However, a short time later, Foley did refer to ‘the pilot in command of MH370’ when he used the Kalitta Air incident and decompression sickness as an example of what might have happened. He said: “Most of the people sitting in their armchairs looking at MH370 are saying that the aircraft was depressurised for up to an hour. The pilot of this particular [Kalitta] aircraft was 51 years old and overweight, and, if you look at the pilot in command of MH370, he was 53 and overweight. So there’s one possible—I’m not saying that happened and I hate to speculate, but it’s at least one plausible scenario.” Foley was careful not to say “the Captain did it” and I don’t think he came anywhere close to saying that “Zaharie must have hijacked the aircraft”, but he clearly thinks it is a plausible explanation.

  578. ventus45 says:

    @Andrew
    @Mick

    “AMSA has considerable SAR expertise, but I doubt it has the ability to manage a complex technical investigation.”

    Interesting comment. I would love to be a fly on the wall in the AMSA lunch room, when they read that ! Incoming !

    AMSA started the search with little more than the initial baseline of the NTSB’s “two tracks”, supposedly produced from their analysis of satellite data.

    As time progressed, the 6th arc became the interim baseline, with what was later to become the 7th arc (the current baseline), initially not understood.

    AMSA alreay had extensive relations and MOU’s with BoM, CSIRO, Defence, Geoscience Australia, Universities, and a host of others in the Marine Industry to draw on, for both advice and assets, many only a phone call away, and many of them on a first name basis at that, all experience and contacts gained from past searches.

    ATSB is a very small agency which HAD NO “IN HOUSE” EXPERTISE, for any marine search or derivation of search parameters. The ATSB started from square in the search business. They had to “outsource” or “contract in” just about everything they needed.

    As for the electronic and satellite issues, AMSA was in the same, and perhaps an even better position than the ATSB, for taking the same technical advice and inputs from the likes of Inmarsat, Thales etc, as the ATSB subsequently did. I can not see how you could infer that they were inferior to the ATSB in that regard, let alone determine that the ATSB was demonstrably superior.

    When you look back at it, with the benefit of hindsight, it was a “search” then, and it still is a “search” today. There is nothing about “the search”, that is ATSB remit specific, other than the fact that it is an aircraft that is missing, rather than a cargo ship that is missing.

    AMSA is a capable, adaptive, and well regarded organization. On the other hand, the ATSB has not exactly covered itself in glory in recent years, as you appear to concede (“… even if some of the organizations past investigations are questionable”) and (“… they were the logical choice, despite some past ‘transgressions’”) including of course, the massive fail, in the case of the PelAir ditching.

    But we won’t mention that will we.

    Well, perhaps we should, since I see Mick has just bought in on this, since there is a common parameter.

    You will remember (I presume you are a local) that the then Chief Commissioner’s reason for not recovering the boxes (the answer that he gave when questioned on the matter in Senate Estimates) was “value for money for the Commonwealth”, i.e. the cost (in his opinion – and his alone) did not justify any potential derived benefit(s) from recovering and reading them.

    Why did I go to all that trouble on PelAir ?

    @Mick, this bit is just for you.

    Re your post May 31, 2018 at 12:07 am. You virtually make it sound like Dolan had second thoughts, and did the right thing, competently, all on his little lonesome.

    Crap.

    It took multiple Senate Hearings, and a highly critical Senate Report, to virtually goad him into recovering them. He resisted all the way, at every step. He was the Chief Commissioner, and it was his prerogative, and his alone Do you remember the infamous session with Senator Fawcett about which edition of Annex-13 was applicable, (read: put it where the sun don’t shine senator) and what the essential “issue under discussion” was ?

    Oh, and just by the way, he didn’t have to find VH-NGA either. EVERYBODY KNEW EXACTLY WHERE IT WAS. The NSW Police Divers (Water Police) had found it years before, photographed it, even videoed it, and had even put out a wreck site buoy marking the spot. Blind Freddy could find it !!

    I will have to assume that you have little detailed knowledge of the 9 year history of this event. Perhaps you should research the matter more fully than apparently you have.

    You may also remember, that when many people questioned the comparative technical merits of the various tenders for the undersea search for 9M-MRO, and wanted an explanation as to why better technology, which was available, was not employed, he again (in Senate Estimates) gave the same reason for his selecting Fugro as the preferred tenderer as he did for not retrieving the Westwind (VH-NGA) boxes, ie, “value for money for the Commonwealth”.

    Many think that AMSA would have done a better job, beginning with a better definition of the technical capability requirements for tender, and thus, a better selection of initial search contractor.

    There are many today questioning the “management” of the search, by the ATSB, and there are growing calls for an inquiry, on this issue.

  579. Mick Gilbert says:

    @ventus45

    I didn’t mention Dolan or anything that went into reopening the investigation.

    You’ve clearly got your own view on the ATSB, I have mine, others have theirs. I have no interest in trying to disabuse people of their views.

  580. Mick Gilbert says:

    @ventus45

    And since we’re tidying up the facts on Pel-Air, it wasn’t the NSW Police that found the wreckage; it was the ATSB. They initially located the wreckage on 29 November 2009 using a sonar receiver to localise the ULB signal. The following week the Victorian (not the NSW) Police were engaged by the ATSB to send an ROV down to survey the wreckage.

  581. Andrew says:

    @ventus45

    RE Mick’s comment: “You’ve clearly got your own view on the ATSB, I have mine, others have theirs. I have no interest in trying to disabuse people of their views.”

    Ditto. Your severe anti-ATSB bias is clearly evident, both here and at auntypru.com. I do not intend to provide a rebuttal to your rant. Why did you even bother to ask?

  582. David says:

    @Andrew, Victor. In a discussion about the source of final descent rates, at May 27th 8:12 PM Andrew you said, “In the simulator trials I conducted, the N3 ran down below 35% N3 within one minute of flameout.”
    Later you added, “Frankly, I doubt there would be enough fuel to sustain a relight long enough to produce a significant thrust asymmetry, but that’s just an opinion.”
    Victor you commented,“The problem with the residual trim theory is that the 0.7g downward acceleration occurs two minutes after the second flame out. ………. With the engine re-start theory, the delay in the bank and downward acceleration can be explained (assuming there was enough residual fuel)”.

    I go into that further.
    Lacking any other indication of fuel flow at altitude in a relight as an example I use that of sea level idle, about 2400 lb/hr (ICAO figure). The APU DC pump could deliver that. At that rate, 30 lb of residual fuel would last the left engine for 45 secs. If it relit immediately after fuel exhaustion, thrust at idle or more would last for 45 secs.

    Two other considerations are that the prospects of a relight decrease with altitude. Also after a delayed relight, the 45 secs would be reduced by the delay.

    Suppose there were a relight. To bank the aircraft the thrust would have to overcome any residual left trim. Then to realise the BFO descent rates it would have to increase bank just before 2 mins from FE such that downwards acceleration was high then but not, as yet, rate of descent. It then would have to increase the descent rate over the next 8 secs by an average acceleration of two thirds that of gravity.

    The supply and demand parts of the above look incompatible unless;
    • the average thrust were sufficient to overcome left residual trim and,
    • there were a reasonable prospect of a relight and,
    • left engine start time, running time and the net thrust were perfect for initiating a bank by 45 secs after FE which was insufficient to cause a high descent rate by 2 minutes after FE, yet sufficient, with no engine thrust and possibly a countering trim, for the above high average acceleration over the next 8 secs, as above.

    The prospects look to be low, even if the flow rate I have assumed is a fair way out.

  583. Nederland says:

    Final report by OI expected by July:

    [i]Speaking on Wednesday, Loke said the search for the MH370 had cost 500m ringgit (£95m) and Ocean Infinity would be releasing a full report on their findings by July.[/i]

    https://www.theguardian.com/world/2018/may/30/search-for-missing-plane-mh370-heads-for-one-last-spot-of-interest

  584. Andrew says:

    @ventus45

    I take it back, I will respond to one part of your tantrum, but only because you misrepresented my comments.

    You said: “As for the electronic and satellite issues, AMSA was in the same, and perhaps an even better position than the ATSB, for taking the same technical advice and inputs from the likes of Inmarsat, Thales etc, as the ATSB subsequently did. I can not see how you could infer that they were inferior to the ATSB in that regard, let alone determine that the ATSB was demonstrably superior.”

    I didn’t even mention electronic and satellite issues, let alone “infer that [AMSA] were inferior to the ATSB in that regard”. I said “I doubt [AMSA] has the ability to manage a complex technical investigation”. That investigation and the definition of a search area involved far more than just electronic and satellite issues. If you have read any of the ATSB’s reports, you might have noticed that a considerable amount of work was done on aircraft performance analysis, systems analysis, simulator trials and eventually debris analysis. I stand by my earlier comment that the ATSB was the organisation best placed to complete ALL that work, or at least coordinate it with external agencies such as Boeing, NTSB, AAIB, etc.

  585. Don Thompson says:

    @ventus,

    A few days ago you & I exchanged messages on Twitter, the discussion was initiated by your erroneous posts about the dates JACC and ATSB ‘took over’ from AMSA. The record shows that JACC was instituted on 31st March, 2014. AMSA wasn’t stood down, AMSA continued to contribute to the search with tasking the air & surface assets, exploiting their oceanographic knowledge, as they had done since 18th March.

    JACC, Joint.Agency.Coordination.Centre (the clue’s in the name), assumed the role of coordinating the various agencies involved. Not least, JACC took the focus of the media.

    Your assertion that “[m]any think that AMSA would have done a better job, beginning with a better definition of the technical capability requirements for tender, and thus, a better selection of initial search contractor. is unfounded bollocks. You really need to pay attention to detail: ATSB engaged the services of David Gallo, then of Woods Hole Oceanographic Institute Special Projects team to advise on the procurement of the deep ocean search services and Sherrell Ocean Services as a technical advisor for the procurement. The release of the ATSB’s procurement material via the Questions on Notice procedure at the Australian Senate RRAT committee provided much useful information. The released information listed those who were shortlisted and how they were scored. The highest scored shortlisted bids, 1st) Fugro and 2nd) Phoenix International’s team, came very close in scoring, with Svitzer 3rd.

    It’s very worthwhile to note that, during the period that ATSB was evaluating those bids Malaysia announced that it had contracted the Phoenix International team, that team included DOS, therefore taking out tenderer #2. Svitzer had teamed with Oceaneering and Williamson (Williamson, as lauded by the #cartoonographer), Oceaneering and Williamson both declared that their specific equipment necessary for the search had not been used for at least 3 years.

    Additional comment on the Phoenix team and DOS. Later in 2014 it became apparent that Malaysia’s Hishammuddin had ‘mispoken’, and only Phoenix International was to be deployed with their unique ProSAS-60 deep tow. DOS appeared aggrieved, their website hosted a ‘sour grapes’ page about MH370 for some time. However, DOS’ vessel, the John Lethbridge, may not even have lasted the course in the sIO: it was scrapped in England following their work in the Mediterranean Sea to recover MS804’s wreckage. DOS does have significant deep water aircraft search and recovery expertise and its staff are now working with OI on Seabed Constructor.

    That Svitzer disclosed their team members’ equipment had not been used in this type of project for 3 and 7 yrs was likely not confidence building. Further, Svitzer made no reference to employing AUVs in their work, something which proved to be absolutely necessary.

    It’s likely impossible, but do try to make more effort to test out your opinions before foisting them on others and expecting acceptance.

  586. ST says:

    Thanks Andrew.

  587. Andrew says:

    @David

    Thanks for your comments re the engine relight. I meant to reply but got sidetracked running some errands. I’ll get back to you tomorrow.

  588. Richard Godfrey says:

    SC is continuing to make good progress up the Broken Ridge plateau area and has reached 25.9359°S.

    Ocean Infinity has made another 2 AUV deployments in the last 24 hours. The AUV50 deployment appears to be a repeat of AUV42, if I got it right.

    The weather is good, there is clear visibility, with a 19 knot wind, a combined swell and wave height of 3.0m. There are no tropical storms in the region.

    https://www.dropbox.com/s/1w8r8opdvbykmg4/SC%20Track%2031052018.pdf?dl=0

  589. Richard Godfrey says:

    @David, @Paul Smithson

    Many thanks to both of you for your kind words.

  590. Trip says:

    Like many of you I have been following MH370 since the beginning. I worked full time in China, India and Southeast Asia for 7 years (2008-15). I flew from Hong Kong to Mumbai about 2 weeks after MH370 and it was interesting that our flight hugged the coast of Myanmar rather than fly over the Bay of Bengal.

    Many of you have shared compelling arguments regarding the fate of the plane. I think we might want to consider a different approach. In this post I present The Wisdom of Crowds.

    The Wisdom of Crowds says 4 conditions are needed for a crowd to be wise.
    Diversity
    Decentralization
    Independence
    Aggregation

    The first three conditions appear easily satisfied, as we are a group of independent thinkers with diverse backgrounds located across the globe. I’ve wondered about the fourth condition, aggregation. Aggregation in simplest terms is voting. This is what brings new understandings to light and allows a consensus around the key findings. No one individual has the complete answer however I believe the solution may be found among us.

    So far we have relied on the wisdom of the moderators, Victor or Jeff, to select what they see as the truth. If this were a scientific problem no doubt there could be a rationally deduced outcome. However this mystery is centered on human motivation, whether it be the pilots, the passengers or the governments.

    Before we can aggregate we need to have clearly stated and agreed on facts. Aggregation is most effective where we have a range for voting (Likert scale) rather than a yes or no, which is polarizing.

    Questioning bias is always an issue so the samples below are stated in 2 possible formats. Some possible questions might include:

    The plane was actively piloted after the FMT OR
    the plane was not actively piloted
    The Inmarsat data is sound OR
    the Inmarsat data is flawed.
    The Malaysian government is withholding key facts OR
    the Malaysians have shared everything.
    Zaharie was negotiating with the government OR
    Zaharie had no political issues.

    (Voting 0 to 5 where 5=Toal agreement, 0=neutral or disagree)

    The point is to find areas where we agree and then work on the areas where there is limited consensus.

    This is obviously Victor’s forum and I don’t want to presume where his interests lie. However I would suggest a post summarizing the facts as we see them. In this era of fake news we need to be careful and listen to one another.

    After we see the group consensus on the facts we could allow each person to offer a one paragraph summary of their perceived outcome and vote on these in the same manner as the facts. Hopefully this might lead us to new and different insights.

    I appreciate everyone’s incredible work in trying to resolve this mystery.

  591. ventus45 says:

    @Andrew, Mick, Don.

    You accuse me of having an anti-ATSB bias.

    Perhaps you should read the Senate Report yourselves.

    Ben Sandilands dis a reasonable summary of it’s findings and concerns here.

    The matter of the deliberate non-recovery of the boxes (in the report) is here.

    Quote.

    Retrieval of the flight data and cockpit voice recorders
    3.53 In its submission the ATSB informed the committee that ‘work commenced to examine the capability and need to recover the aircraft’s cockpit voice (CVR) and flight data recorders (FDR)’ after an initial interview of the captain on 23 November
    34 Mr Martin Dolan, Chief Commissioner, ATSB, Committee Hansard, 28 February 2013, p. 3.
    Page 29
    2009.35 Retrieval of the recorders, the committee notes, is an important opportunity for safety learning for the aviation sector. From evidence provided, the committee understands that retrieval of the recorders would be particularly useful in this instance, as there may not be another example of a night ditching where all passengers survived.36 Recorded data is less subjective than witness accounts.
    3.54 The committee understands that the ATSB has certain responsibilities, set out in ICAO Annex 13, when it comes to retrieval of aircraft involved in accidents. It is an assumption throughout Annex 13 that, where a FDR exists, the accident investigation body will prioritise its retrieval:
    The aftermath of a major accident is a demanding time for any State’s
    investigation authority. One of the immediate items requiring a decision is where to have the flight recorders read out and analysed. It is essential that the flight recorders be read out as early as possible after an accident.37
    3.55 The committee approached the ATSB on this particular point, asking
    Mr Dolan whether he was comfortable that the agency had complied with the
    requirements of the annex in choosing not to recover the VH-NGA FDR because of the associated cost. The committee received the following response:
    That was why I drew your attention to that paragraph that I just read
    [paragraph 5.4 of ICAO Annex 13]. With the decision I made in relation to
    the value as opposed to the cost of recovering the recorders, I was viewing it in the framework of ‘where feasible.’ I consider cost as opposed to benefit to be relevant to the question of feasibility.38
    3.56 During the course of the committee’s hearing on 28 February 2013, an issue emerged relating to the wording of paragraph 5.4. Mr Dolan, explaining that he was reading from the current version of the paragraph in question, challenged the committee’s reading of the annex, according to which an investigative body would be required to gather, record and analyse all available information on an accident or incident. This would include the flight data recorder.
    3.57 Mr Dolan asserted that the copy of the annex in his possession, being more current and dated 18 October 2010, contained slightly different wording. This version does not say that investigations ‘shall’, but rather ‘shall normally’, gather, record and analyse all available information.39
    35 ATSB, Submission 2, p. 35. FDRs and CVRs are two different types of recorders used for incident and accident investigation purposes. CVRs are typically used to record audio in the aircraft flight deck, while FDRs record parameters such as altitude and airspeed with respect to
    time. The committee uses the term FDR to refer to both.
    36 Mr Bryan Aherne, Committee Hansard, 22 October 2012, p. 12.
    37 See Committee Hansard, 28 February 2013, p. 9.
    38 Mr Martin Dolan, Chief Commissioner, ATSB, Committee Hansard, 28 February 2013, p. 9.
    39 See discussion between Senator David Fawcett and Mr Martin Dolan, Committee Hansard,
    28 February 2013, p. 8.
    Page 30
    3.58 However, although the version of the document Mr Dolan relied upon before the committee to support his decision not to retrieve the VH-NGA FDR may have been more current, it was not the version in force at the time of the accident or its immediate aftermath, when such decisions were being made.
    3.59 Furthermore, the ATSB had no disagreement with the committee’s reading of paragraph 5.7 of the annex, which clearly sets out a state investigation body’s responsibilities in this regard:
    Effective use shall be made of flight recorders in the investigation of an accident or an incident. The State conducting the investigation shall arrange for the read-out of the flight recorders without delay.40
    3.60 Despite this, the Chief Commissioner maintained that, according to his reading of the paragraph, the ATSB was not required to retrieve VH-NGA’s FDR:
    What I read that [paragraph 5.7] in the light of, in the structure of this document [Annex 13], is that 5.4 is a general paragraph setting the context with the others, and so we have the question of whether to retrieve them in the first place—had we retrieved them, we would agree: effective use shall be made, and we have to arrange for the read-out, without delay. As I say, the decision I made was in that general context of feasibility.41
    3.61 The ATSB position remained that the relevant paragraph of Annex 13
    provided the agency ‘the necessary discretion…in its conduct of the investigation.’42
    3.62 The committee does not accept this argument. At the time the decision against retrieving the FDR was made the imperative existed for the ATSB to do so. To ignore this imperative by arguing that the benefit did not justify the cost appears disingenuous. To imply that the revised wording in the current version of Annex 13 was the basis for the ATSB’s decision in 2009/2010, before this version was in force, is even more disingenuous.
    3.63 This is not the only example of a FDR which has been under water for some time being retrieved and useful data being produced. Furthermore, the ATSB appears to be of the view that the data is not worth the cost of retrieval as information could be obtained from the flight crew, both of whom survived the accident.
    Committee view
    3.64 The committee finds the ATSB’s refusal to retrieve the FDR incongruous and questionable. Furthermore, the committee takes a dim view of the ATSB’s reliance on a version of ICAO Annex 13 that only came into force in late 2010, nearly a year after the accident, to justify this decision. Mr Dolan’s evidence in this regard is questionable and has seriously eroded his standing as a witness before the committee. Flight data recorders are routinely recovered around the world despite the existence of surviving crew. They provide objective records of how events transpired, and allow speech
    40 Paragraph 5.7, ICAO Annex 13, emphasis added.
    41 Mr Martin Dolan, Chief Commissioner, ATSB, Committee Hansard, 28 February 2013, p. 9.
    42 ATSB, answer to question on notice 8, 28 February 2013, p. 3.
    Page 31 specialists and psychologists to determine stress levels and what was going on in the cockpit at the time.
    43 This could offer valuable lessons for the whole aviation industry,
    not just about why an accident occurred, but, in this case, how such a successful ditching was executed under extremely difficult circumstances.
    3.65 The committee is of the view that the ATSB is taking a very loose
    interpretation of its obligations under ICAO Annex 13. Furthermore, the committee has evidence indicating that by early 2010 two lines had been attached to VH-NGA which were strong enough to raise the wreckage. This evidence calls into question whether the ATSB’s argument concerning cost or associated occupational health and safety concerns was valid, and reflects the fact that the ATSB was not overly concerned to robustly examine options and costs.44
    3.66 Having received in camera evidence on the likelihood of VH-NGA’s flight data recorder yielding useful information about the accident despite more than three years passing since the event, the committee supports calls for the recorder to be retrieved.
    3.67 The fact is, the primary consumer of ATSB investigation reports is the aviation industry. There is much to be learned about what led to this accident, and how injuries were minimised upon impact.
    Recommendation 1
    3.68 The committee recommends that the ATSB retrieve VH-NGA flight data
    recorders without further delay.

    End Quote.

  592. Victor Iannello says:

    @ventus45: I prefer to not relive the Pel-Air investigation here, as few people doubt that the ATSB bungled that one. You believe that AMSA was more qualified than the ATSB to lead the MH370 search effort. Others believe the ATSB was more qualified. Regardless of your negative feelings about the ATSB, I don’t see how this discussion is going to help solve the mystery. Let’s move on.

  593. Sfojimbo says:

    Why would Sultan Ismail Petra Airport’s tower radar be working at 1:30 in the morning when the airport shuts down at 11pm?

  594. Barry Carlson says:

    @Sfojimbo,

    The radar you have mentioned is located to the south of the runway center and is a combined PSR/SSR, i.e. the separate radar heads use the same antenna platform. The radar is part of the Kuala Lumpur FIR and provides 24 hour ATC surveillance, with coverage extending into the adjacent Thai and Singapore FIRs.

    The PSR data is used when required by the airport ATC as an approach radar for vectoring purposes.

  595. lkr says:

    As we get ready for post mortems on the seabed search, or a rationale for a restricted resumption, here’s my suggestion.

    One of the few ways to restrict the search area is doing something with drift analysis [DA below]. But so far, DA is almost worse than useless. The problem is not just that there are very few pieces to work with, and only a couple of these where arrival time is roughly known. The real underlying problem is that no one was ever interested in constructing a search, not just to find more pieces, but to avail in DA. This should have gotten some attention and a bit of funding as soon as the Reunion piece. It could have been done in 2016 using an approach modelled on Blaine Gibson, eg, ATV with local helpers on a stretch of sandy beach. And pieces were still there in 2017, probably reworked from the original cloud of debris, rather than new arrivals from the gyre.

    My bet is that some fraction of the beach arrivals in 2015 and 2016 are still there, periodically coming to the surface. More weathered and fragmentary, but still the sort of thing that a good eye could spot [perhaps an experienced paleontologist is the right talent.

    What BG did was prospecting — here I’m suggesting that drift models could suggest a limited series of isolated sandy beaches [any rocky shore would chew everything up within a few months of arrival] — from, perhaps Thailand or Sri Lanka, Madagascar, and segments along the African coast from Tanzania to Mozambique and SA. Probably 5 to 7 20 km stretches could be chosen that are generally off coastal roads and away from beach-cleaning resorts… Again, the idea is to differentiate on arrival site along — above a certain point on the 7th arc, SriLanka is possible, just as south of a certain point, WA is expected to be in play.

    So if anyone here is approaching a philanthropist with an oceanographic vessel, you might suggest a structured beachcombing to increase the power of drift analysis. This could be done for the cost of a day of ship time..

    And who knows, perhaps PA would enjoy ATV beach cruising with BG?

  596. Sfojimbo says:

    @Barry

    Are you saying that the SSR is used by both the Tower and ATC (24/7), but the PSR is only used by the Tower (airport ATC) and only in use while the airport is open?

  597. Andrew says:

    @Sfojimbo

    My guess is the radar feeds (PSR & SSR) from the WMKC radar are used 24/7 by Lumpur Control to extend its area control radar coverage. During Kota Bharu airport’s hours of operation, the feeds would also be used by Kota Bharu Approach for controlling aircraft operating at WMKC.

  598. Barry Carlson says:

    @Sfojimbo,

    The KL Air Traffic Control Centre in Kuala Lumpur makes use of, or has available to it, data from both PSR/SSR heads 24/7.

  599. ST says:

    According to this article, ZS wanted to settle in Geelong, Victoria and buy a house there. That should have been top most in his mind from an end point perspective and seems to be at Latitude 38.14 S well within possible options table put forth by Dr.B

    https://au.news.yahoo.com/missing-mh370-pilot-wanted-to-settle-in-geelong-family-member-claims-32569086.html

    Possible that the width needs to be wider than 25NM +/- to allow correlation of motive plus the math and science.

  600. Sfojimbo says:

    I have long been puzzled by the accepted premise, that 9M-MRO flew west from MEKAR +10 where radar coverage was lost. Hishammuddin & company just stated as “10 miles west of MEKAR” at 18:22:12: without coordinates. There was no indication whether it remained on the same course, or deviated to Airway N571, or went southward toward IGEBO.

    After analysis was made available from Zaharie’s computer, we learned that he had apparently plotted a flight path westward in the Malacca strait which continued on westward on airway N571, toward waypoint DOTEN, or thereabout. Thus, thinking seems to have congealed on the idea that 9M-MRO continued westward from MEKAR +10 (wherever that is) and made its turn south somewhere west of 93°Longitude. All following theories and calculations, including fuel flow has stemmed from that assumption. Yet the plots from Zs’s computer were made from a flight that originated at KL and was planned to continue on to Jeddah; there are differences between that flight path and the KL to Beijing route; continuing westward wouldn’t have been a proscribed path on the morning of March 8th.

    There simply was no reason (that I can see) not to turn southward at MEKAR and follow an IGEBO –> POVUS routing. Turning south at POVUS also makes the 19:41 Ping ring location logically coherent; if one travels straight west from MEKAR, the 18:28 ring and the 19:41 ring are only 129 miles apart, while the time between the rings is 73 minutes; 777s don’t fly that slow.

    Why wasn’t Duncan Steel right about the search area back in 2004? Am I missing something?

    Here’s a view of the decision point.
    https://www.dropbox.com/s/sxbq51dzor2xd4d/Screenshot%202018-05-31%2014.56.55.png?dl=0

  601. TBill says:

    @Sfojimbo
    I think you will probably have your work cut out trying to make IGEBO to POVUS consistent with the 18:25 to 18:28 set of BTO/BFO signals, which fairly nicely suggest an offset maneuver from N571.

    We do not really know the waypoints used in the sim studies, unfort that part of the dataset was truncated. Looks like it was heading TASEK to VAMPI then we have a turn near DOTEN on an apparent operating line to NZPG.

    Many of us try to stay away from Indonesian airspace and radar so IGEBO to POVUS is not too common for MH370 paths these days.

  602. MH says:

    Ducan Steel probably was correct with the concept for the northern possible routes back in 2004.

  603. Sfojimbo says:

    #TBill

    My plot works perfect with the 18:28 ring.

    I broke the IGARI –> 18:28 ring journey into two parts:

    The first segment is from IGARI to a point that is defined by ALSM’s first data point (17:30:35). That segment is 74.4 nm long and I calculated its average GS to be about 464kts. This includes the Chandelle zoom, it probably started with a very high speed and ended somewhat slower. It can be off a few kts: its purpose is to allow 17:30:35 to be used as a base for future legs.

    The second segment from 17:30:35 to the 18:28 ring worked out to about 500kts average GS (by accident. I used the flight path described by ALSM’s plots from KL & Butterworth, but I ignored all the timing info except for the first point from KL.

    My assumption is that the plane was flown from 17:30:35 on at steady speeds and altitudes (after recovery from the initial altitude soar). IMO it would not have has the speed and altitude excursions shown by ALSM’s data points.

    Almost coincidentally, the time given very early on for Pulau Perak (1802:59) fits within a half a minute of my calculations.

    You can plot it yourself easy enough. Check for your self.

  604. Sfojimbo says:

    From the 18:22 MEKAR +10 position to the 19:41 ping ring is 208nm; the journey took 79 minutes, that’s a GS of 158 kts.

    Are those the numbers you are using?

  605. Sfojimbo says:

    The above is along airway N571.

  606. Richard Godfrey says:

    @Sfojimbo

    You stated “I have long been puzzled by the accepted premise, that 9M-MRO flew west from MEKAR +10 where radar coverage was lost. Hishammuddin & company just stated as “10 miles west of MEKAR” at 18:22:12: without coordinates. There was no indication whether it remained on the same course, or deviated to Airway N571, or went southward toward IGEBO.”

    Malaysian Factual Information states:

    1. “The tracking by the Military continued as the radar return was observed to be heading towards waypoint MEKAR, a waypoint on Airways N571 when it disappeared abruptly at 1822:12 UTC [0222:12 MYT],10 nautical miles (Nm) after waypoint MEKAR.”

    2. “The primary target (military radar) appeared to track west-northwest direction joining RNAV Route N571 at waypoint VAMPI then to 10Nm north MEKAR”.

    ATSB Definition of Underwater Search Areas states:

    “Primary radar data showed that the aircraft tracked along the Malacca Strait. During this time the aircraft passed close to waypoints VAMPI, MEKAR, NILAM and possibly IGOGU along a section of airway N571.”

    ATSB final report The Operational Search for MH370 states:

    “Radar data shows the aircraft then headed to the northwest, eventually aligning with published air route N571 from IFR waypoint VAMPI. The validity of this section of the radar data was verified using the track of a commercial flight that followed N571 about 33 NM behind MH370. The aircraft continued to the northwest until a final radar position for the aircraft was recorded approximately 10 NM beyond IFR waypoint MEKAR at 1822:12”

    Inmarsat in their paper for the Journal of Navigation states:

    “The last published primary radar location of the aircraft was at 18:22 UTC when it passed close to the MEKAR waypoint at N06 30, E096 30. Five minutes later it crossed the 18:27 arc generating a BFO of 142 Hz. Due to the close proximity of the arc crossing to the last radar fix we have limited uncertainty in the arc crossing points and so the main variables we have to align the predicted BFO with the measured value are the aircraft ground speed and track. Trial and error shows that we obtain the best BFO match for an aircraft track of 300°, which is consistent with the aircraft travelling along airway N571 towards the IGOGU waypoint at N07 31, E094 25 at a ground speed of 480 knots. Such a flight path would be consistent with the route determined by radar prior to 18:22 UTC where the aircraft appears to be moving between waypoints.”

    I agree the last radar point at 18:22:12 UTC was cropped from the Beijing Lido Hotel slide and incorrectly labelled, with a pointer off slide. The answers given to the Next of Kin include both the description of the last radar data point as on Flight Route N571 10 nm beyond waypoint MEKAR and 10 nm North of waypoint MEKAR. There are again many discrepancies in the Malaysian presentation.

    However, the ATSB and Inmarsat statements are consistent with a flight following N571 towards waypoint IGOGU, separately verified using radar and satellite data.

  607. Sfojimbo says:

    The FI never out and out made any claims after 18:22 and while I admit I missed the Inmarsat Journal of Navigation sentence, I am not in acceptance of the premise that BFO can provide heading information; “Trial and error shows that we obtain the best BFO match for an aircraft track of 300°” does not instill certainty for me. That might be milking more information than exists from a BFO of 142.

    Have any of you guys actually plotted the course you assume 9M-MRO flew from the MEKAR +10 position to the 19:41 ring?

    The time is 79 minutes and even using my 500kt speed, that only comes out to 655nm. So even if the next turn is made at NILAM, 655nm is mot enough distance to avoid the Indonesian FIR and get back to the 19:41 ring. 886 nm is needed for that, and 886nm produces a very unlikely flight path.

    We have to accept that 9M-MRO traveled through the Indonesian FIR at some point.

  608. Richard Godfrey says:

    @Sfojimbo

    I have reconstructed over 700 MH370 flight paths in the last 4 years and the results are published on this website.

    My current MH370 Flight Model is at version 17 and has also been published on this web site.

    You state you do not accept the BFO data shows heading. This is correct, it shows track.

    Please consider the possibility of a loiter over the Andaman Islands.

    Aircraft can enter holding patterns and are not predestined to only fly in straight lines.

  609. Interesting figures, thanks. To my view the mathematical precision should not hide the physical reality:
    The 150fpm is a value derived from these estimated situation : aircraft heading to 360° (because it maximizes the BFOs), gs=498kt as derived from the official reports (altitude not of importance). If it was 510kt the BFOs would require an additional 100fpm only, very much in the margins zone actually.

    1- Taking into account, the fact that the aircraft is rolling around the axis of its longitudinal inertial momentum which is much lower than the axis of the fuselage, the antenna distance to this axis is circa 4.3m.
    2- a rolling angular velocity close to 5°/sec is indeed more realistic (I just ran more numerous additional simulations).
    3- In addition, during this maneuvering situation, how much would the aircraft benefit from a transitional better lift during the bank to bank rolling as it passed to the « near wing » level? The generated extra lift would come from the latency of the engines and the a/c attitude and would be transitional until the aircraft banks on the other side and would contribute certainly contribute to a slight climb tendency.
    4- In addition, the jitter on vertical speed has been indicatd last year of about +/- 50fpm instantaneously on Victor’s blog
    5- The actual speed of the aircraft is unknown: is it 498kt as we supposed ? or 510kt as seems to indicate your recent computations.

    Thus the 100fpm or 150fpm are not unrealistic values. The heading at 360° is providing the main clue that the aircraft was in the middle of the turns of the Contingency Procedure.

    So, if indeed the antenna rotation is less contributing than initially thought, I see it bringing its share in a situation where all parameters bring also their contribution to the 100 or 150 fpm which are in fact very close to be marginal.

  610. TBill says:

    @Sfojimbo
    Here is my current approach to 19:41 in flight sim:
    VAMPI MEKAR NILAM 0796E 0894E ISBIX (ISBIX or 180S CTH take your pick)
    I am descending at 18:38 to go under N571 at about FL250-ish
    This is similar to DrB’s newer path and also similar to MH370-Captio.net

    Your 74nm IGARI to 18:30.5 sounds too far…expecting around 66nm as quoted above by @sk999 (might have been the prior thread)

  611. TBill says:

    @Sfojimbo
    PS- When I said you have your work cut out at 18:25 to 18:28, I was talking about matching the BFO sequence at least directionally. If you are ignoring BFO that’s easier but none of us here just ignore BFO to my knowledge.

  612. airlandseaman says:

    Jean-Luc Marchand: Perhaps you missed my previous response about the effect of turns (roll axis). The AES HGA updates the antenna pointing continuously (approx every 300 msec). So there should not be any significant BFO error due to turns.

  613. DennisW says:

    @Mike (ALSM)

    It is not a pointing issue. The antenna is changing position in space when the aircraft rolls (moving up or down relative to level flight). I do agree that the effect is small.

  614. @DrB
    @Sfojimbo
    @all

    For those who want to fly CAPTIO trajectory here is the Excel Flight Plan with additional information like the vertical profile and fuel consumption. Have a good session with your flight simulator 🙂

    http://mh370-captio.net/wp-content/uploads/2018/06/CAPTIO-Trajectory-3.xlsx

  615. @airlandseaman
    @DennisW

    Antenna doppler because of antenna movement.
    Yes it is small but it adds up with the aerodynamic physics small perturbation … to me this explains the “slight” and “transcient” climb before the long descent to FL270 before turning and passing under route N571.

  616. Sfojimbo says:

    @ Richard Godfrey

    “I have reconstructed over 700 MH370 flight paths” I can’t find any plots of flight paths here or anywhere else that contain times, distances along with matching airspeeds.

    Such as this:
    https://www.dropbox.com/s/269ugzegm6n5kgc/MH370.kmz?dl=0
    The actual path you’re looking for is titled: IGARI to 18:22 MEKAR + 10

    “You state you do not accept the BFO data shows heading. This is correct, it shows track.” Splitting hairs? BFO appears to be to be dependent on a number of variables (vertical, horizontal direction and Doppler effect)Sat movement and probably others. I wouldn’t bet a hundred million dollars on a single BFO reading as some are doing.

    “Please consider the possibility of a loiter over the Andaman Islands.”I find that concept ludicrous. Cats wander in circles, greyhounds travel in (somewhat) straight lines: ZS was a greyhound.

    @TBill “none of us here just ignore BFO”
    You won’t be able to say that anymore.
    We have to be able to throw out (esoteric) data that leads to illogical conclusions.

    Load my KNL file and put yourself in the captain’s seat at 18:22 and look at your options. ZS would never have gone into Indonesian territorial airspace (within 12nm of land); but entering their FIR without announcement is done all the time by military’s of various nations: and others. I would consider NOPEK – BEDAX to avoid Sebang tower radar, but flying hundreds of miles off course for no purpose, or flying in circles is a non sequitur for me.

  617. Victor Iannello says:

    Sfojimbo: Whether or not you can find them, many of us have created paths from 17:21 and onwards. You can choose to completely ignore the BFO, but without justification, you are going to receive a lot criticism here.

  618. Sfojimbo says:

    @ Victor

    Again: I can’t find any plots of flight paths here or anywhere else that contain times, distances along with matching airspeeds for the time before the second ring.

    Would you be so kind as to give me a link?

  619. Barry Carlson says:

    “Deep-sea explorer and acclaimed director James Cameron says it’s no surprise missing flight MH370 hasn’t been found”, is the story by-line tagged to an article in news.com.au.

    The above is a demonstration of how a person with high public visibility and kudos can unwittingly portray a message, which when examined carefully, reveals that person’s lack of knowledge around the current technology being used in the search for MH370 / 9M-MRO.

    Hopefully, someone will be able to disabuse him of his antiquated belief.

  620. Don Thompson says:

    @Barry Carlson.

    I wholeheartedly agree about Cameron. His objective was to build a submersible that would take him to the depths of the ocean. A feat that confirmed he, and his cameras, could see little beyond the ‘end of his nose’ but, at least, the view was recorded in high-definition.

    Comparing his accomplishment with state-of-the-art deep ocean survey is equivalent, in an aeronautical context, to Baumgartner’s 128,000ft balloon ascent & freefall vs flying reconnaisance UAVs over conflict zones.

    Might as well ask the former captain of the Costa Concordia for his opinion.

  621. Richard Godfrey says:

    SC is continuing to make good progress up the Broken Ridge plateau area and has reached 25.5553°S.

    Ocean Infinity has made another 3 AUV deployments in the last 36 hours.

    The weather is good, there is clear visibility, with a 14 knot wind, a combined swell and wave height of 2.8m. There are no tropical storms in the region.

    https://www.dropbox.com/s/6hnjvoimpno4aqv/SC%20Track%2002062018.pdf?dl=0

  622. Niels says:

    A question for the Satcom specialists:
    Is it really correct to assume that the BTO bias is a constant for the whole accident flight?
    From “Bayesian methods…” section 5.2:
    “This is due to the
    channel dependent calibration term T
    channel not being stationary. Over the span
    of a day it appears constant but in the context of the 20 flights represented in
    figure 5.2 there is a slow variation. As discussed above, different values were fitted
    for each flight.”

    In particular, I’m a bit worried what happens after a power interruption.

  623. Sfojimbo says:

    Not a Sat expert, but I’m a knowledgeable of electronics, and I can think of no reason to think that the latency the plane’s satellite transceiver would change whenever it powers on/off.

  624. Richard Godfrey says:

    @Sfojimbo

    You stated: “I can’t find any plots of flight paths here or anywhere else that contain times, distances along with matching airspeeds.” and “Would you be so kind as to give me a link?”

    You may find the following links helpful.

    On Victor Iannello’s web site:

    http://mh370.radiantphysics.com/2017/11/25/possible-mh370-debris-seen-in-aerial-search-in-spring-2014/#comment-10052

    http://mh370.radiantphysics.com/2018/01/02/ocean-infinity-will-soon-start-new-search-for-mh370/#comment-10599

    On Barry Martin’s web site:

    http://www.aqqa.org/godfrey-model/

    On Jeff Wise’s web site:

    http://jeffwise.net/2014/12/11/up-close-inside-an-ig-southern-route/

    On Duncan Steel’s web site:

    http://www.duncansteel.com/archives/1874

    On Wikipedia’s web site:

    https://en.wikipedia.org/wiki/Analysis_of_Malaysia_Airlines_Flight_370_satellite_communications

    “27. Godfrey, Richard (20 July 2015). “MH370 Flight Path Model v15.1″. Duncan Steel. Retrieved 2 September 2015.”

  625. Niels says:

    @Sfojimbo
    Can you think of a reason then why different values had to be fitted for each flight?

  626. DennisW says:

    @Niels

    I think it is more instructive to look at figure 5.3 of “Bayesian Methods”. That figure shows little variation of BTO over the 20 previous flights. I would question the use of “had to be fitted” with respect to that data.

  627. flatpack says:

    @Sfojimbo

    “… I can think of no reason to think that the latency the plane’s satellite transceiver would change whenever it powers on/off.”

    Embedded systems these days include so much cruft that you should have no expectation of deterministic behaviour with regard to latency.

  628. TBill says:

    @Victor
    …fyi
    (1) Based on flight path work, right now I feel OI should go up to about 20S before going wide.
    (2) Xmas Island- I am not sure how to prioritize Xmas Island cases, except to say, see if we can find more evidence (below)
    (3) I have an older (less developed) McMurdo-derivative path going POLUM to UXORA that lands in some deep trenches at around 22S, and this path is similar to Ed Bakers end-point, but for different rationale.

    Re: Xmas Island
    I hate to talk about rumors, but to my thinking we have three or four rumors about some kind of known plot:
    (1) We have @TimR account of second-hand knowledge of a ZS flight diversion plan
    (2) I believe there was an unsubstantiated email to the news media about a similar plan
    (3) Recently somebody (here?) alluded to the fact Hish may have privately told the 4 Corners interviewers that there was some talk before the flight that (the military?) needed to keep an eye on MH370

    Even if there was an Xmas Island style plan, we would not know if the plan was aborted or went wrong for some reason.

  629. Niels says:

    @DennisW
    To be 100% certain we would need to know the bias values used in the analysis of 20 previous flights and be able to explain possible variations over time.
    I hope we agree this is a point we have to be 100% certain about and that we are in a situation where we should re-evaluate all our assumptions.

  630. DennisW says:

    @Neils

    Yes, your question is a good one. The DSTG report is less than crystal clear on this point. It would be very good if we had the raw data from the 20 previous flights. Carry on.

  631. Niels says:

    @DennisW
    After re-reading section 5.2 several times, Fig. 5.3 may indeed give an indication, as:
    “Figure 5.3 shows a scatter plot of the BTO errors against time. The
    channel dependent calibration term T
    channel was matched to the final flight before
    the accident flight, MH371 (and the beginning of the accident flight)”

    I agree the DSTG book is not crystal clear regarding some crucial aspects of the BTO and BFO analysis. I will ask the authors to share more data.

  632. Victor Iannello says:

    @TBill: I am working on some more northern paths that should be very interesting. More soon.

  633. DennisW says:

    @Niels

    Yes. After my first post I went back and read the narrative associated with figure 5.3 more carefully. I think the figure was constructed using different values for BTO bias. If so, it does not address your question.

  634. Sfojimbo says:

    @Richard Godfrey
    The most useful plot that I cane across from those links was an old one from Duncan Steel, that’s no doubt because he plots his path with Google Earth (or something very similar) as do I.
    http://www.duncansteel.com/wp-content/uploads/2015/07/V15.1-path.jpg

    But this plot begins at NILAM, thus it’s only a snippet. However it’s a snippet of the part that is left off of everybody else’s plots.

    I would like to see a similar plot from something that is currently being proposed so that I could ask why each zig and zag was made.

    What I’m looking for is the justification for putting so much importance on so little data. As I understand it, the thoughts of most people here (and everywhere else) hang on one BFO reading: at 18:25:34.461 a BFO reading of 273 was calculated; and from that one piece of data that could have been induced by a momentary air pocket encountered while the transmission was taking place the entire flight path of 9M-MRO has been deduced?

    @Niels
    Inmarsat offered no explanation for the “slow variation” but I am left with the impression that it is due to atmospheric conditions. I would think transponder latency to be a separate subject.

  635. TBill says:

    @Victor
    “I am working on some more northern paths that should be very interesting. More soon.”

    Yes OI’s prolific productivity has left us red-handed without enough sample cases to north of 26S.

    I was going to ask Richard to check his portfolio. I know we have Richard’s long search for back-up airport paper to about 22S.

  636. TBill says:

    @Victor
    “I am working on some more northern paths that should be very interesting. More soon.”

    Yes OI’s prolific productivity has left us red-handed without enough sample cases to north of 26S.

    I was going to ask Richard to check his portfolio. I know we have Richard’s long search for back-up airport paper to about 22S.

  637. David says:

    @Andrew. A little way back, “Thanks for your comments re the engine relight. I meant to reply but got sidetracked running some errands.”

    Yes there was action on a couple of fronts. Any progress?

  638. Victor Iannello says:

    @sfojimbo said: What I’m looking for is the justification for putting so much importance on so little data. As I understand it, the thoughts of most people here (and everywhere else) hang on one BFO reading: at 18:25:34.461 a BFO reading of 273 was calculated; and from that one piece of data that could have been induced by a momentary air pocket encountered while the transmission was taking place the entire flight path of 9M-MRO has been deduced?

    No, that’s completely wrong. The first BFO value in the log-on sequence was corrupted by a packet collision. The second one (273 Hz) and the cluster at 18:27 are abnormally high due to the warm-up transient. Only the last cluster at 18:28 should be used (unless you are confident about how to correct for the effect of the warm-up transient). If you want to learn more, you should read the paper on the BFO by Ian Holland of the DSTG.

  639. Andrew says:

    @David

    Again, my apologies. I haven’t forgotten – I’m having a closer look at the engine relights that occurred in ALSM’s simulator trials. Life keeps getting in the way, so it’s taken some time to write up a reply. Hopefully later today or tomorrow.

  640. Sfojimbo says:

    @Victor

    Thank you for the direction to Ian Holland’s paper. I have done a quick read of it and have saved it for future reference.

    I learned from your post that “Only the last cluster at 18:28 should be used”, so it is true that I was off in assuming that the BFO reading: at 18:25:34.461 (273) was the sole data used to predict the entire flight path of 9M-MRO.

    Instead the last cluster has been used for that purpose.

    The first packet of that cluster came at 18:28:11.054 and the last packet arrived at 18:28:10.260.

    Thus, a slice of data from .794 of a second was used to predict the entire flight path of 9M-MRO. Correct?

  641. Victor Iannello says:

    @Sfojimbo: No. There is no value that uniquely defines a path. I suspect you know this and are playing games. If so, stop it now.

  642. Sfojimbo says:

    No, I am not playing a game. I’m trying to understand why anyone would think 9M-MRO would have gone past MEKAR +10, NILAM or for sure past IGOGU. This sortie out into the Andaman Sea seems ridiculous to me.

    I have always assumed his path was MEKAR, IGEBO, POVUS. I’ve never been completely hard set on that, I agree he might have given Sebang a bit wider berth, so I agree it’s possible that he went as far west as IGOGU and then came back to about BEDAX to avoid Sebang tower radar. I believe he intersected the 2nd ring just south of the equator.

    I think the recent fuel usage estimates are off by at least 300 nm.

  643. Trip says:

    A year before the flight vanished, in April 2013, company auditors discovered that the airline was not compliant with its own rules for the frequency of essential information sent in regular bursts of data via satellite from its fleet of long-haul airliners to the airline’s headquarters in Kuala Lumpur. Moreover, the auditors pointed out that because of this lapse, by law the airplanes—including Flight MH370—should not have been cleared to fly.

    From Daily Beast, 6/1/18

    I hadn’t seen this before.

  644. DennisW says:

    @sfojimbo

    Well…construct your flight path along with track angle and GS at the ring crossings so we can consider it. We cannot comment on pure arming waving, which is what you are doing.

  645. Victor Iannello says:

    @Sfojimbo: You are without justification ignoring BFO values, ignoring fuel models, ignoring drift models, and throwing in your bias regarding which paths make sense and which paths don’t. I don’t see how that’s very helpful.

  646. Victor Iannello says:

    @Trip: The implication by Clive Irving is that more frequent reporting would have prevented the disappearance of MH370. It may have made it easier to find AF447, but more frequent reporting does not prevent a crew from disabling the reporting system, which he fails to mention.

  647. Barry Carlson says:

    @Trip,

    … and adding to Victor’s comment.

    The Daily Beast is only regurgitating a story penned by Christine Negroni in her book ‘The Crash Detectives’ which was published 18 months ago.

    Operationally, the airline could obviously live without 15 min SATCOM AOC reports, and such weren’t internationally mandated; so money saved.

    Post factum, this is of no relevance to where the aircraft is now.

  648. Sfojimbo says:

    @Dennis
    This was already posted above. It is a kml file, feed it to Google Earth.
    https://www.dropbox.com/s/269ugzegm6n5kgc/MH370.kmz?dl=0

    @Victor
    I am not ignoring BFO values, my whole point above was to find out why everyone assumes that 9M-MRO went NW into the Andaman Sea. From your information and from viewing the Inmarsat log I see that the only possible BFO data that can be used to predict a path to the northwest after MEKAR is the “last cluster” as defined by you.

    That cluster consists of these six packets with their BFO, BTO values:

    18:28:10.260 148, 7540
    18:28:10.398 148, 7540
    18:28:10.559 147, 7540
    18:28:10.718 147, 7540
    18:28:10.879 147, 7540
    18:28:11.054 147, 7540

    These packets define the 18:28 ping ring.

    I will accept that the BFO values indicate a NW heading. But they only indicate that 9M-MRO was on a NW heading for the three quarters of a second that the cluster took place in. These packet exchanges took place at (very close to) 6° 56.366’N 95° 35.001’E.

    IMO 9M-MRO would have almost certainly made a turn to the left within the next 74nm (at or before IGOGU); this route would have kept it clear of Sebang Tower radar (assuming 60nm range) and put the plane on the intended course —> south. This course would have avoided Indonesian territory although would have led the path into the Indonesian FIR.

    But from what I can see, the projected paths into the Andaman Sea would have brought 9M-MRO into Indonesian FIR on the way south anyway.

    Please load my KML file and look at what I’m saying.

  649. DennisW says:

    @sfojimbo

    The kml file lacks GS and track angle at the ring crossing. It is essentially useless. If you want to be taken seriously here, do the analytics and post those.

  650. Sfojimbo says:

    Dennis you are only telling me of your intellectual limitations.

    There are times for everything, there are ground speeds embedded in the tracks. The Rings are dead on accurate.

    Do you know how to use Google Earth??

    BTW
    Although there is a lot more information contained in the KML file, the only parts I am referring to in this thread is the Part from IGARI to POVUS or thereabout. Anything south of the second ring is extraneous to this conversation.

  651. Brian Anderson says:

    @sfojimbo,

    There is a lot of extraneous clutter in that .klm file and many of the lines [yes, south of POVUS ] do not hit, or cross all the “ping rings”. How can you determine the flight path by looking just at the possibilities up to POVUS. There is very little data to support any path between 18:25 and 19:41. And, how can the segment up to POVUS determine the ultimate end point?

    There is a simple but fundamental test that I do when looking at potential tracks, and that is to check to see if the track crosses just inside the 19:41 ring at the correct time. In fact the track ought to cross that ring twice, from outside to inside at precisely 19:41, and then again from inside to outside about 24 minutes later. All the data we have strongly indicates that the track was closest to [and therefore tangential to] the sub-satellite position at approximately 19:53. Unless a track can meet this test then it very hard to accept that it is realistic.

  652. Sfojimbo says:

    @Brian “And, how can the segment up to POVUS determine the ultimate end point?”

    I am so far not in the business of determining the end point. When you build a house you start with a solid foundation or the rest of your work is for naught. Same thing here. I truly believe that all these math experts have based their predictions on a faulty starting point.

    How did you come to the conclusion that the track intersected the 19:41 ring twice? Be specific please.

    If you will notice, I am probably challenging what your assumptions are about two crossings of the 19:41 ring. I drew my southern paths from the assumption that 9M-MRO turned south at POVUS. I agree it may have continued west for a bit more than that, but none of that is cast in stone yet.

    But I wholeheartedly reject the premise that it flew in circles way up in the Andaman sea.

  653. Brian Anderson says:

    @sfojimbo,

    I think you have a lot of reading to catch up on. You don’t have sufficient information to build a solid foundation in this case. You have to take all the data you can find and see if you can figure out the jigsaw puzzle with a lot of the pieces missing.

    “But I wholeheartedly reject the premise that it flew in circles way up in the Andaman sea.”

    On what grounds? There is a real possibility that the aircraft flew one or more orbits, and it is possible to demonstrate that such a path would fit both the BFO and consume sufficient time to then meet the 19:41 ring intercept.

    “How did you come to the conclusion that the track intersected the 19:41 ring twice”

    That is very basic knowledge. Determined right from very early on from the “fuzzy graph of elevation angles”, and from the BTO numbers. Here is one reference . . https://en.wikipedia.org/wiki/Analysis_of_Malaysia_Airlines_Flight_370_satellite_communications

    32. Anderson, Brian (20 March 2015). “Deducing the Mid-Flight Speed of MH370”. duncansteel.com. Archived from the original on 24 March 2015. Retrieved 24 March 2015.

  654. Sfojimbo says:

    @Brian
    “I think you have a lot of reading to catch up on. You don’t have sufficient information to build a solid foundation in this case. You have to take all the data you can find and see if you can figure out the jigsaw puzzle with a lot of the pieces missing.”

    What a load of gibberish.

    “On what grounds? There is a real possibility that the aircraft flew one or more orbits, and it is possible to demonstrate that such a path would fit both the BFO and consume sufficient time to then meet the 19:41 ring intercept.”

    On the grounds that buzzing around the Andamen sea makes no sense whatsoever.

    I notice that like Dennis, you talk in vague generalities. Let’s get a bit more specific. Which of these BFOs leads you to think Zaharie wanted to fly around in the north a bit before he turned south?
    18:28:10.260 BFO 148
    18:28:10.398 BFO 148
    18:28:10.559 BFO 147
    18:28:10.718 BFO 147
    18:28:10.879 BFO 147
    18:28:11.054 BFO 147

    As I pointed out before they all came within 794 milliseconds of each other, that’s a rather short term trend in an eight hour flight.

    Thanks for the link to Wikipedia. You and Dennis will always share a warm place in my heart.

  655. Brian Anderson says:

    @sfojimbo,

    I’ll leave others who might participate here to decide who is talking a load of gibberish.

  656. Sfojimbo says:

    But Brian, stand on your own feet.

    Which of those BFOs??

  657. DennisW says:

    @sfojimbo

    Dennis you are only telling me of your intellectual limitations.

    I have a lot of that for sure. It would be helpful if you cataloged your path in EXCEL or so other routinely utilized method. Basically, I think you are an idiot.

  658. Sfojimbo says:

    Dennis EXCEL is extremely limited for the purpose of laying out the flight path we are working on. GE already has everything laid out ahead of time. Placement of radars not only can be made, but they can be double checked by zooming in and looking at the antenna in question. The same with cities, airports, land masses or even the view out the window as 9M-MRO passed Penang. With one click you can have a grid with all latitudes and longitudes marked clearly and accurately.

    Moreover I posted the KML file for MH-370, all you have to do is load it; and with it you can measure any angle or distance easily. It even shows ocean depths (and I don’t use Microsoft for anything ever if I can help it). And GE is free.

    I don’t mind if you drop out of the conversation should you find GE uncomfortable to use.

  659. Brian Anderson says:

    @sfojimbo,

    Well OK, it is you who is talking a load of gibberish. Is that clear enough for you. I gave you a link to read but clearly you haven’t done so. You probably haven’t taken advantage of the information in the links posted by Richard a little earlier. Oh well, you will have to figure it all out by yourself buddy.

    The BFO’s are not everything, as most people here know. The mystery is much more like a jigsaw than building a house. A jigsaw that has been left out in the weather for some months. The picture is unrecognisable, and a dog has chewed some of the corners. You just have a small handful of pieces. Your BFO list is but one small piece of the puzzle. Have fun trying to piece it together. Come back and tell us all when you have.

  660. Richard Godfrey says:

    @Sfojimbo

    You stated that you could not find any links to flight paths, so I provided you 24 links.

    You took a look and concluded that the flight paths hang on one BFO reading, you obviously missed the 599 BFO and 519 BTO data points in each Excel.

    You then stated they are not in GE, missing the point that I provided Excel to make the calculations of each Latitude and Longitude from the satellite data transparent, as you stated you did not accept the BFO.

    You then complained the one plot you found only started at NILAM, well I am sure you downloaded my MH370 Flight Model V13.1 from the link I provided earlier at http://www.aqqa.org/godfrey-model/ and saw that both the Excel and the GE file start at Kuala Lumpur Airport. Here is the link to the GE plot starting at Kuala Lumpur:

    https://www.dropbox.com/s/bo067o0l314v0pi/MH370%20Flight%20Path%20Model%20V13.1%20Final%20Route.kmz?dl=0 (first published 11th December 2014).

    You then complained about the “zig zag”. Here is a link to the GE plot of the lateral offset courtesy of Sid Bennett:

    https://www.dropbox.com/s/cfwsjaf7m0hk10d/Sid%20Path%20from%201822%20240415.kmz?dl=0 (first published 24th April 2015).

    You then complained, whether people know how to use GE. You probably did not notice that I publish the current progress of Ocean Infinity in GE every day on this website.

    You tell us “everyone keep in mind” or “remember the facts” or you lecture us on “argument from ignorance” or remind us of “our intellectual limitations” or that we are just talking “a load of gibberish”.

    I find you arrogant, patronising, condescending and rude. I am obviously wasting my time trying to help answer your requests and questions.

    In 57 posts on this current thread, you have ended up arguing to a dead end with Mike, Sabine, Dennis, Victor, Brian and myself.

    As Victor pointed out, I think you are just playing games. As Brian pointed, you are talking a load of gibberish. As Dennis pointed out, basically I think you are an idiot.

  661. formula says:

    @Richard Godfrey – “…I publish the current progress of Ocean Infinity in GE every day on this website”

    These progress reports, comprehensive, clear and timely, are an invaluable resource and so thank you very much for your impressive effort.

  662. Richard Godfrey says:

    SC is continuing to make good progress up the Broken Ridge plateau area and has reached 25.4685°S.

    Ocean Infinity has made another 3 AUV deployments in the last 24 hours.

    I estimate that Ocean Infinity will reach around 24.5°S by 8th June 2018.

    The weather is mediocre, there is poor visibility, with a 16 knot wind, a combined swell and wave height of 3.3m. There are no tropical storms in the region.

    https://www.dropbox.com/s/tg7nuwvyipfn1au/SC%20Track%2003062018.pdf?dl=0

  663. Richard Godfrey says:

    @formula

    Many thanks for your kind words.

  664. Victor Iannello says:

    @sfojimbo: I tried to make sense of your KMZ file, but it’s a mess. If you are proposing the path MEKAR-IGEBO-POVUS around Sumatra, you are already ignoring the BFO at 18:28 without justification. Rather than saying paths proposed by others make no sense, you should document your analysis in an understandable format and submit it for review. Rather than trying to educate this group, you should view this group as a resource composed of many knowledgeable people that have been considering various aspects of this problem for a number of years. New perspectives and insights are welcome, but they should be based on informed analysis.

  665. mash says:

    re: Captio trajectory

    Is it possible that the water landing was intended/planned?

    If so …

  666. Shadynuk says:

    @Richard Godfrey I also thank you for your efforts to present the OI search progress. Well done and much appreciated.

    I think I saw this question some days back but could not find a response. Do you know the basis upon which OI is continuing this work? If they find the wreckage now will they receive compensation from the Malaysians?

  667. Sfojimbo says:

    @Victor
    “I tried to make sense of your KMZ file, but it’s a mess.”

    No Victor my KMZ file is not “a mess” but it’s clear that your understanding of it is hazy at best. There’s nothing I can do about that; I can bring a horse to water…………….

    “If you are proposing the path MEKAR-IGEBO-POVUS around Sumatra, you are already ignoring the BFO at 18:28 without justification.”

    That KMZ was posted before our conversation where you identified the “last cluster” as the basis for your current version of “the northern route”. I have since accepted “last cluster” as being valid and useful. I have clearly stated several times since that I am willing to accept the possibility of routes that pass west of POVUS.

    If the lot of you want to continue living in your Andamen Sea fantasy it’s no skin off my nose; I’ll just point out to you that one definition of insanity is to keep doing the same thing over and over while hoping for a different outcome. I’ll be shocked if you can find anyone else willing to put up millions of dollars to search above latitude 29 or around Christmas Island as current thinking here entertains.

    If I may, I’ll ask you the same question that Dennis and Brian ignored.

    Which of these BFOs leads you to think Zaharie wanted to fly around in the north a bit before he turned south?
    18:28:10.260 BFO 148
    18:28:10.398 BFO 148
    18:28:10.559 BFO 147
    18:28:10.718 BFO 147
    18:28:10.879 BFO 147
    18:28:11.054 BFO 147

  668. Victor Iannello says:

    @Sfojimbo: If you want us to consider your KMZ file, update it with your newly gleaned understanding of the BFO set at 18:28, remove the extraneous information, and explain what you are trying to show.

    The BFO values at 18:28 are consistent with a NW track at that time. The BFO values at 18:40 are consistent with either a southern, level track, or a northern descending track. Over the years, we’ve reconstructed many paths that consider these two cases. We also have search results, end-of-flight analyses, drift analyses, and aircraft performance models to help us reconstruct viable paths. I won’t try to educate you in a single comment with learning that has occurred over four years.

    I will repeat my offer one more time. Rather than saying paths proposed by others make no sense, you should document your analysis in an understandable format and submit it for review by the group. Making comments like “Which of these BFOs leads you to think Zaharie wanted to fly around in the north a bit before he turned south?” is not useful. Nobody is avoiding your question. Rather, the question demonstrates your incomplete understanding of the total set of constraints. Paths are not constructed based on a single BFO value, as was explained to you before. Propose your alternative solution and allow others to review it. Otherwise, I will conclude that you are here only to disrupt, as you are contributing nothing useful to the discussion. I am giving you ample opportunity to make your case.

  669. Richard Godfrey says:

    @shadynuk

    Many thanks for your kind words!

    I guess that the contract between OI and Malaysia has ended and OI continues without any prospect of reward from Malaysia, however if OI find MH370 then they have won the marketing battle.

  670. DennisW says:

    @sfojimbo

    Which of these BFOs leads you to think Zaharie wanted to fly around in the north a bit before he turned south?

    Well. Victor beat me to it. The answer is all of them.

  671. DennisW says:

    @shadynuk

    OI had a large sunk cost at the formal end of the search agreement with Malaysia. The incremental cost of continuing until a resupply is necessary is relatively low.

  672. TBill says:

    @Victor
    @all
    As promised I made an attempt at listing ASSUMPTIONS for the MH370 search to date. The intent was to allow reflection on the assumptions for future search planning.

    >>This is a first rough draft for comments.
    https://docs.google.com/document/d/1C9wsl8bW7tIhH05FzgJ8-TNHt86i7A_YUZP8-wIKum0/edit?usp=sharing

    I also provide some POSSIBLE WEAKNESSES of the ASSUMPTIONS and IMPLICATIONS examples to show the impact of the assumptions on selected future search case proposals.

  673. formula says:

    @TBill “14. Of course the big weakness: No Aircraft Found to date based on the passive flight assumptions”

    A minor point but might there be merit in developing the assumption that the searches would have found the aircraft had it been in the areas covered to allow the point made by Richard Godfrey above (28, May,4.43 am)that the searchers estimate some 1% of the areas designated for search could not be accessed, meaning, as RG states, “…around 2,350 km2 not covered, if 1% was missed. Unfortunately this relatively small area is split up into probably around 160 points of interest.”?

    The average size of a POI would then be some 14.7 km2, perhaps meaning such an area could easily (?) contain an entire debris field and hence the searchers would miss finding any of it.

  674. Brian Anderson says:

    @sfojimbo,

    And for the sake of completeness . . . “Which of those BFOs??”
    Same answer as Dennis. All of them could also fit the aircraft orbiting at that time.

    But you also have a fundamental assumption . . “Which of these BFOs leads you to think Zaharie wanted to fly around in the north”. What evidence do you have that Z was flying the plane at that time?

  675. TBill says:

    @formula
    Thank you for the comment.
    Sounds like you would assign priority to filling in any survey coverage gaps in the already searched areas (if the search were to resume next year)?

    On the side of “glass half full”, I’d say 99% means they did cover 16,000 points of interest only missing 160 POIs.

  676. formula says:

    @TBill – especially if any of those c.160 POIs are proximate to “hot spots”, I suppose it would contribute to a case being made. Whether it should be in preference to any alternatives, I have no view.

    The diligence and thoroughness of the search efforts have been impressive and so it is somewhat frustrating, clearly, that caution dictates that the areas searched cannot with full certainty be said not to contain the aircraft.

  677. Mick Gilbert says:

    @David

    Thanks David. And that’s why I don’t listen to radio.

  678. Barry Carlson says:

    TBill;

    Expanding on the matter that @formula raised, i.e. the ‘holiday spots’ in any of the completed searches to date.

    The likelihood of the aircraft being in one discoverable piece on the sea floor is small, and I’d expect at least one of the engines to have left the pylon attachments on impact. One or both engines will have gone essentially straight to the bottom, slowing from the initial deceleration at the surface with increasing drag with depth. While the remainder of the hull and wings (if still attached) will have taken time and distance dictated by the surface / sub-surface currents encountered and the variable increase to its density as numerous parts of the homogeneous interior of the hull, along with the LG tyres/tires, cargo etc., collapse under increasing pressure with depth. The time taken getting to the sea floor equates to the, “How long is a ball of string?” problem.

    Even if this sort of probability was applied to the various survey ‘holidays’, the possibility of a further search based on this assumption is not good. Though a thorough re-examination of the data surrounding the holiday areas is far cheaper than just mobilizing another search.

    Earlier today I listened to the 2GB radio interview with Byron Bailey mentioned by @David. Nothing new was gleaned from it, other than Bailey took a dig at the credentials of former ATSB chief commissioner Martin Dolan and the MH370 operations manager Peter Foley. I think Bailey and others need to face up to he fact that the search as implemented by the ATSB was done with agreement of Malaysia and China, and the outcome is what it is, and if the rational behind it has failed to find the aircraft, then there is no point in blaming those who implemented the search at the behest of others.

    OI have undertaken their search in good faith that the information they had received was the best available. If that turns out drawing a blank, then it is fairly safe to say that the assumptions made regarding an uncontrolled end of flight scenario were flawed. That doesn’t mean that any of the possible fight paths leading to numerous “hot spots” were wrong, as the likelihood is that at least one of them was right, but needs a non passive end of flight scenario added to it.

    Bailey and @Rob’s promoted glide at the end of their predicted paths, may well be right, though the drift modeling doesn’t sit well with it. So until it can be conclusively proved that the drift modeling is off the planet, the chances of anyone going to their southern spot is remote.

  679. Richard Godfrey says:

    SC is making slower progress up the Broken Ridge plateau area and has reached 25.4514°S.

    Ocean Infinity has made another 2 AUV deployments in the last 24 hours. AUV54 released to survey at 01/06/2018 22:28:00 was recovered early after only 48h 15m. AUV55 released to survey at 02/06/2108 03:50:01 had a scan width of 5.2 km instead of the usual 7.4 km from start to end. This means the overall start to start scan width reduced from the usual 9.4 km to 7.2 km for 4 runs, which in turn implies a run scan width of 900 m down from 1175 m either side of the AUV.

    I estimate that Ocean Infinity will now reach around 24.8°S by 8th June 2018.

    The weather is improving, there is good visibility, with a 20 knot wind, a combined swell and wave height of 3.0m. There are no tropical storms in the region.

    https://www.dropbox.com/s/5h35m4pf1wa5f6l/SC%20Track%2004062018.pdf?dl=0

  680. Andrew says:

    @David

    Thanks – I’ll give him one thing, he is persistent. Eight minutes of my life, wasted.

  681. Victor Iannello says:

    @Barry Carlson said: So until it can be conclusively proved that the drift modeling is off the planet, the chances of anyone going to their southern spot is remote.

    I agree.

  682. David says:

    @Victor. The below looks at how unlikely the ATSB explanation is for the final descent BFOs. This in turn suggests that the current interpretation of one or both of the BFOs of the final transmissions is most likely flawed.

    Similarly, the explanation for the powering of the two BFO transmissions after APU auto-start on fuel exhaustion is dubious. This translates to there being an active pilot.

    Joining an unlikely descent acceleration with an active pilot and the wreckage could be 100 miles distant from the 7th arc, well beyond search widths.

    I draw some conclusions from this.

    https://www.dropbox.com/s/h3vwosq3yfq5pd5/THERE%20IS%20REASON%20TO%20THINK%20THERE%20COULD%20HAVE%20BEEN%20A%20GLIDE.docx?dl=0

    @Andrew. While the above is related to the question you are looking into, thanks, I post this assuming the outcome of that will not affect this much.

  683. Victor Iannello says:

    @David: Thank you for the write up. In planning the next search area, if we accept the possibility of an extended glide (whether or not it was preceded by a steep descent), we next have to ask ourselves what latitude and how wide. I don’t see an obvious way to bound this search area to a manageable size.

    On the other hand, if we accept that the final BFO values accurately depict descent rates (and put aside for the moment that none of the Boeing simulations depicted a similar downward acceleration two minutes after fuel exhaustion), followed by an impact close to the 7th arc, then a search along more northern parts of the 7th arc can be limited to a manageable size.

    I think nobody feels comfortable in defining a new search area based on keeping it to a manageable size, but it has to be a consideration, as it was in the past. Unfortunately, no matter what the strategy to select the next search area, there will be the inherent risk of excluding the location of the debris field.

  684. Tim says:

    If the accuracy of the 7th arc is accepted by everyone as being correct, and we are rightly confident that the aircraft spiralled in close to the arc, then it is just a case of extending the search along the arc.

    What I think is our mistake, is to assume an ‘autopilot on’ flight. This flight displays all the characteristics of a decompression followed by a passive ‘autopilot off’ flight. So can anyone make the arcs flt fo a random route. If not, then it’s going to be difficult to define a crash location.

  685. Richard Godfrey says:

    @Tim

    You have a big problem that both the hourly BTO and BFO, between 19:41 UTC and 00:11 UTC, are constant to a level more than 99%.

    If the autopilot was not engaged, then the pilot achieved an autopilot like performance.

    The flight track was anything except random.

  686. Tim says:

    @ Richard,
    Thanks for the explanation. Is it still possible for the flight before 1941z to have made a few random turns/ phugoids and then settled down to a near level/straight flight after 1941z?

  687. TBill says:

    @David
    Thank you for the work-up. Missing though is IFE strategy.
    Refresh my memory about the missing IFE logon during a glide?
    I guess an active pilot could turn that off, but why do so?
    As someone above said, perhaps it was the SDU logon itself that gave the pilot reason to glide away from an initial dive.

    I am increasingly thinking if ZS was said pilot, he would have been smart enough to be concerned that maybe the SDU pings gave indirect location info. What I cannot figure out if he would have cared or not…if alive seems like he was not shy to indicate he was still flying (evidenced by allowing the 23:14 sat call to come in even though unanswered).

  688. DennisW says:

    @all

    IMO, it is going to be impossible to define a credible search area based on current information (and how we have used that information). We need something new – a whistleblower, Malaysian investigative intelligence, a eureka experience relative to the analytics,…

  689. Sfojimbo says:

    Oh, the irony.

  690. Victor Iannello says:

    @Sfojimbo: Previously, I said to you: Propose your alternative solution and allow others to review it. Otherwise, I will conclude that you are here only to disrupt, as you are contributing nothing useful to the discussion. I am giving you ample opportunity to make your case.

    That offer remains.

  691. Sfojimbo says:

    Solution to what Victor? My analysis ends at (even before) the second ping ring. My views have been made clear.

  692. lkr says:

    sfo must be a real joy at his local incel meetings.

  693. formula says:

    @ TBill – a trivial point, perhaps invalid, but under the “Possible Weaknesses” section at 5.a. “Arc6 is relatively close to Arc5 perhaps consistent with a slow down/descent” could there also alternatively have been an ascent at that point, perhaps in contemplation of a glide that would then be of longer duration than if it started nearer the surface?

  694. Victor Iannello says:

    @Sfojimbo: The only thing you have made clear is that you disagree with a flight path towards the Andamans. The path you propose up until 19:41 doesn’t even satisfy the BFO data at 18:28, which you now acknowledge. I’m not going to continue this discussion because you have nothing to discuss, despite my two invitations for you to present your thoughts.

  695. TBill says:

    @formula
    I do think there could be climb at the very end if an active pilot descended lower – say for visual observation purposes – but wanted to be higher for the final maneuvers.

    I am not a long glide enthusiast. But I’d be open on what kind of landing a pilot was trying for: ditch vs. dive. If I am wrong and the aircraft is still up at FL400, it could be a long glide, but I have trouble seeing going way up there, if already lower.

  696. formula says:

    @Tbill – thanks for addressing my remark: I understand.

  697. Sfojimbo says:

    Victor, these pretensions of incredulity are beneath you. I have brought out that there is noting to support an excursion to the north – and that is not a small thing if one wants to attempt to determine where 9M-MRO’s flight ended, which seems to be an obsession here.

    You can accept or reject what I have brought to your forum, but yours and your forum participant’s tasteless personal attacks have no part in a discussion about the disappearance of the airliner. Yes, I’ve been blunt in stating my opinion; but once you come to realize that there is no reason whatsoever to support a northward deviation for 9M-MRO, you may come to see how counter intuitive such a theory really is.

    You repetitively say how “Sfojimbo has learned about the 18:28 BFO data (presumably from you) as if you or your site has brought me out of the dark and into the light, yet this is not so at all. I dismissed the significance of the BFO data before I ever posted here and I continue to dismiss the relevance of that data now. It is true that I thought the theorists of a northern part were pinning their belief in the 273 BFO reading at 18:25:34.461, and you did inform me that you found the support for your northern theory in the six BFO readings from 18:28:10.260 to 18:28:11.054, but that is an insignificant difference to me. The most that means is that 9M-MRO was on a northwesterly track for 794 milliseconds about 27nm west of NILAM. That and nothing more.

    About four years ago I created the KML file I posted here, I have made many additions and a few modifications (mostly in the part between IGARI and VAMPI), but as a template, something to work from, it has not changed all that much. I have never considered the turning point of IGARI as cast in stone. The KML file is a work in progress, not a final solution. I could have deleted everything south of the 19:41 ring (except the other rings) but there has been no reason to do that, I still can get ballpark information from the various speed plots and snippets of information that have appeared in the last four years.

    If you want to argue with my premise, then argue with my premise. Please cut out the personal attacks.

  698. Victor Iannello says:

    @Sfojimbo: I can’t argue with a premise that has not been presented. I keep asking you to present your case and you don’t. (That’s the third invitation.)

  699. Sfojimbo says:

    Victor, I have repeatedly stated my premise.

    I cannot “prove” there was no sortie to the north, all I can do is state that there is no evidence for a northern detour, and that proposing such an idea is counter intuitive and illogical.

    And I can also point out how including such a path in fuel usage estimates will lead to wrong conclusions about when loss of fuel would have occurred at the end of the flight, which I have done.

    Any flight path I might suggest could and certainly would be attacked as unfounded, yet 9M-MRO surely turned south sometime after 18:28:11.054. I’ll leave any suggestion of where that might have taken place open to discussion.

  700. David says:

    @Victor. Thank you for your response re any more searching.
    “….a search along more northern parts of the 7th arc can be limited to a manageable size.”
    Yes though if it could be though there would have to be good reason to single that out.

    @TBill. Re IFE. If the aircraft had been plunged that does not arise. If a glide, it would depend on APU fuel, though I would agree that that is not a very satisfactory answer. Yes, no reason to suppose he would turn it off, particularly since there was an IFE connection at the first in-flight log-on.

    One difference here is that the engines would no longer be running though I cannot see how that would provide a satisfactory answer either unless the SDU could be powered and other equipments needed for IFE connection not, ie load shed.

  701. Victor Iannello says:

    @sfojimbo said: Any flight path I might suggest could and certainly would be attacked as unfounded, yet 9M-MRO surely turned south sometime after 18:28:11.054.

    You started this thread by saying “This sortie out into the Andaman Sea seems ridiculous to me.” I’ve pressed you many times for your reasoning, never mind “proof”. (Mind you that we have never fully resolved whether the plane was traveling north or south at 18:40.) What we get back from you are statements like “proposing such an idea is counter intuitive and illogical”. I have to conclude that you have nothing more definitive to offer.

  702. Sfojimbo says:

    Victor the idea of a sortie out into the Andaman Sea does seem ridiculous to me, yes it does. Other than that I can only refer you back to my last post at 6:36 pm.

  703. Victor Iannello says:

    QED.

  704. Brian Anderson says:

    Chillit, the cartoonographer, once again displays his total misunderstanding of the physics . . .”have we now managed to break Inmarsat’s ping ring metric? Certainly possible. What many don’t realize is that 97% of the ping ring metric is “hardware bias”. That means the actual BTO value is a minor player.”

    He is still claiming that the official position of the 7th arc is in the wrong place, because his own [GPS] based method comes up with a different answer. It’s unclear if he recognises that the sub-satellite position changes over time, and he seems not to understand that the earth is not a perfect sphere. The “hardware bias” comment is nonsense. The BTO is a readily calibrated function of the transmission times between the Perth earth station, the satellite and the aircraft. It is necessary to understand the transmission protocol timing to see why there is a constant term [which will include some small hardware delay], called K, which can readily be calibrated.

    Strange that he continues to have an avid following.

  705. TBill says:

    @Brian Anderson
    “He is still claiming that the official position of the 7th arc is in the wrong place, because his own [GPS] based method comes up with a different answer.”

    …and that is important point right now, because people are asking if Arc7 is correct or not.

    I asked Mike about it the other day, to remind me his rationale. I think he is calibrating from the MH370 data before ACARS went down, using his own techniques too get a correction factor.

  706. Don Thompson says:

    @TBill,

    Chillit’s peculiar observations are termed cartoonography for a reason.

    There is little rationale, other than to mislead.

    If he were to publish his work it could be quickly reviewed. Sight of that work is unlikely.

    Seabed Constructor is working over a region with only scant high resolution bathymetry derived from single MBES tracks acquired, over time, by transitting research vesses (these can be seen using viewers hosted by UK NoDC, GEBCO, ArcGIS, Google Earth, and NOAA, among others).

    During the previous 12hrs, Seabed Constructor has been closely tracking an AUV that was released to survey 2018-06-04/16:33. It tended that AUV closely on its transit NW across the 7th arc, it then checked in on the AUV working the adjacent block (to south east), and is now returning to tend the -04/16:33. This typical when tracking the AUVs through challenging seafloor terrain.

  707. Brian Anderson says:

    @TBill,

    He explained his methodology a little time back. It is flawed.
    Ask him if he takes into account . .
    (i) the fact that the satellite moves significantly over the period of interest, and hence the sub satellite position is different at every ping time,
    (ii) the accuracy of arcs or circles plotted in GE.

  708. Richard Godfrey says:

    SC is continuing to make good progress up the Broken Ridge plateau area and has reached 24.9290°S.

    Ocean Infinity has made another 3 AUV deployments in the last 24 hours.

    I estimate that Ocean Infinity will reach around 24.5°S by 8th June 2018.

    The weather is fine, there is good visibility, with a 17 knot wind, a combined swell and wave height of 3.3m. There are no tropical storms in the region.

    https://www.dropbox.com/s/k1jd0mb4i905s0b/SC%20Track%2005062018.pdf?dl=0

  709. Victor Iannello says:

    @TBill: The location of the ping arcs is uniquely determined by the satellite orbit, the model for the earth, the assumed aircraft altitude, and an offset adjustment. That offset adjustment (the calibration constant, K) can be determined when 9M-MRO was at the gate at KLIA. Since we all use essentially the same satellite orbit, the same earth model (WGS84), and the same methodology to calculate the calibration constant, all of our ping arcs agree very closely for the same assumed aircraft altitude. For the 7th arc to be wrong, one of those assumptions must be wrong. The only possibility is the calibration constant K was different before and after the log-on. The possibility that the calibration constant changed during the flight is worth considering. (That’s not what Chillit is saying. He’s simply incorrectly calculating the ping arcs with no change in K. Years ago, some of us tried to help him, but he refused to listen, so most of us just ignore him.) Do you really think a dozen or more knowledgeable people all incorrectly calculate the ping arcs and all independently arrive at the same wrong answer?

  710. TBill says:

    @Victor
    “Do you really think a dozen or more knowledgeable people all incorrectly calculate the ping arcs and all independently arrive at the same wrong answer?”

    Nope I do believe the ATSB/IG/Inmarsat Calcs.

    Recently many are suggesting widen the search to +-100nm, so I am trying to pin down the rationale: is that (1) uncertainty in the Arc locations? (2) long glide by active pilot, (3) long glide by inactive pilot, etc

  711. Paul Smithson says:

    @victor. What do we have in the way of independent verification of satellite position/orbital characteristics for this epoch? a) comparison of TLE with the positions per JON? b) observed “delta BTO” from a stationary target during the time period in question to see how well it matches predicted delta BTO per orbital model prediction? How much could uncertainty in satellite position contribute to uncertainty in arc position?

  712. Sfojimbo says:

    @Paul
    The satellite positions are all within about 15nm of 1° 24.038’N 64° 29.830’E, if any given wrong sat position is used to calculate it’s ring position then that ring would be off by the same amount in the same direction.

    Inmarsat was the source of the sat locations.

    IMO You’re not likely to find much of any error in ring positions.

    My KML file has the positions for the satellites noted.

  713. Barry Carlson says:

    @All;

    The question has been raised regarding the uncertainty in the 7th Arc position. It is not the first time this has arisen, and this post by Duncan Steel deals with the problem of accuracy of the I3-F1 orbital ephemeris.

  714. Victor Iannello says:

    @Paul Smithson: To follow-up on Barry C.’s comment, the STK results produced by Duncan Steel, which are based on the JSpOC TLEs, have been compared with the position/speed vectors provided by Inmarsat, and they are very close. Inmarsat believes the position vectors they supplied have accuracy better than 1 km. If there are errors in the position of the 7th arc, it’s not from inaccuracy of the satellite position.

  715. Marijan says:

    @Victor

    Sorry Victor, but I have to ask you few questions about your previos post. I thought that BTO calculation is rock-solid. Can you elaborate more on K-value and influence on Arc position for ones (me included) who didn’t dig deep in the calculations? Can we actually calculate the K-value from the 18:25 ping (first ping after log-on) assuming that the airplane continued on the straight path after the last radar contact?

    Or K-value has to be determined experimentally? I doubt that someone would venture to experimentally determine the K-values for in-flight conditions, although that option can be also open now.

  716. Barry Carlson says:

    @Victor has clarified the situation.

    I should have checked before posting the link, but since doing so I note that the .xlsx files referred to in the article are no longer on the dropbox server. I’ll see if I can find them.

  717. Victor Iannello says:

    @Marijan: Considering that if K is assumed to be constant, and if we accept the last radar data at 18:22, then a maneuver off of airway N571 is required to satisfy the BTO data at 18:25-18:28, and the BTO data is more likely to be used to provide evidence that K has changed.

    @sk999 looked at the BTO data for MH371. Using the same bias offset as we use for MH371, the BTO RMS error was 33 μs, which is only 4 μs more than the 29 μs the DSTG says is the standard deviation for R1200 BTO data. So, this would indicate that K was the nearly the same for MH371 as it was for MH370 at the KLIA gate.

  718. paul smithson says:

    Thanks @Victor and Barry C for your responses

  719. Marijan says:

    @Victor

    Thanks, things are more clear now.

  720. Sfojimbo says:

    I have a new version of the KML file which has included two possible flight paths. The IGARI turning point is gone.

    Both of the new plots start at MEKAR and terminate at the 19:41 ping ring.

    The first shows the minimal path needed to be taken in order to avoid radar coverage from the Sebang Tower radar. It has a 690nm path around the coverage area and is plotted at an average ground speed of 520kts (8.6nm/min), it takes 80 minutes (from 18:21 to 19:41). This plot is colored light red, aka pink.

    It is not a prediction of a flight path, it is only used to help estimate what a flight between MEKAR and the 19:41 actually might have been.

    The second path shows what the minimal path would have been if an attempt to avoid coverage of the military radar at Sebang had been made. In order to avoid the (estimated 240nm range) military radar, the path from MEKAR to the 19:41 ring would have had to have been 761nm in length and as the time available was the same, 80 minutes the required ground speed would have been 570kts, which would have been an impossible speed for a 777 to maintain. Although this plight path is impossible, it gives a good indication of the time constraints faced by the pilot. This plot is colored a darker shade of red.

    You can turn these plots on or off by opening the sidebar (under view) clicking on MH-370 and going to the bottom of the file and adding or removing the checkmarks to the left of “Sebang Tower” and/or “Sebang military”.

    You will have a less cluttered view if you turn off the grid (in view) and turn off Borders and labels etc in the sidebar.

  721. Sfojimbo says:

    It is at the same link as the old plot: https://www.dropbox.com/s/269ugzegm6n5kgc/MH370.kmz?dl=0

  722. airlandseaman says:

    All:

    Just back from a short sailing adventure, now catching up. I’m really shaking my head. What is with all the 7th arc accuracy hand wringing? How did so many miss all the memos?

    As Brian and Victor noted, a dozen or so people independently modeled the problem and found virtually the same result as Inmarsat, ATSB SWG, IG, and others. Moreover, as I reported here several months ago, Boeing recalculated the 7th arc circa January 2018 and their revised results matched the arc I provided to OI in January within 70 meters. (I use the code developed by IG member Barry Martin.)

    Questions raised:

    1. Can K (BTO Bias) change over time? No, it cannot change by SDU design. It is a constant for the life of the SDU within known, verified, repeatable limits discussed here extensively. It cannot drift (like the BFO). It varies only slightly, in a quasi-random pattern, due to SDU clock timing jitter, as documented here:

    https://goo.gl/2GWuuN.

    The SDU clock jitter is <±50µs max for 99% of all observations, including the ±10µs GES/CU quantification error. All other sources of bias error or uncertainty combined are small compared to the jitter limits.

    2. How much uncertainty in the 7th arc results from the jitter uncertainty? A: ±5.3 nm, which is included in the ±22nm search budget.

    3. Is the I3-F1 orbit well known? Yes. Verified by many within 1km.

    4. Is the Perth antenna location verified? Yes, again by many, within 10m.

    5. Can K be calibrated once the plane is off the ground? Yes, but only if contemporaneous GPS positions are available. There were no such GPS positions available after 17:06 UTC, thus it is not possible to calibrate the BFO bias at 18:25.

    6. What about the R600 and R1200 logon special adjustments? Several IG members contributed to deciphering how to interpret the raw BTO logon values, improving on the original Boeing and ATSB values. Victor calculated the 7th arc adjustments based on the MH371 data set, which contained ~30 observations suitable for the calibrations. I checked the results and confirmed them. Then I sent them to ATSB and OI. ATSB sent the work to Boeing, and they came up with virtually identical results (70 m delta).

    My recommendation is to set aside further hand wringing over the 7th arc. It is about the only factual information we have that has been scrubbed to death and survived the test of time. It is deterministic (no assumptions needed), free of drift and subject only to a small, manageable noise level.

  723. haxi says:

    @airlandseaman

    Hi Mike,

    Pardon me for raising this question.

    If the 7th arc calculations are precise to this degree, and since the -2 BFO value highly likely indicates an impact close to the 7th arc, what could be the reasons why we still don’t locate the aircraft?

  724. MH says:

    @Haxi – at this point was hoping for some debris field which could lead to
    The rest of the airframe.

  725. Mick Gilbert says:

    @airlandseaman

    Re: ‘How much uncertainty in the 7th arc results from the jitter uncertainty? A: ±5.3 nm, which is included in the ±22nm search budget.

    Mike, in the section of MH370 – Search and debris examination update 2 November 2016 dealing with End of Flight Simulations the ATSB notes that;

    Simulations that experienced a descent rate consistent with the ranges and timing from the BFO analysis generally impacted the water within 15 NM of the arc.

    So, technically, yes, 15 NM ± 5.3 NM is included in the search band but with very little margin.

    It’s worth noting that in the same section the ATSB go on to say,

    The results of the scenarios, combined with the possible errors associated with the BTO values indicate that the previously defined search area width of ±40 NM is an appropriate width to encompass all uncontrolled descent scenarios from the simulations.

    Now,for one reason or another we’ve ended up with a search area width of about ± 25 NM. One might be inclined to say, well, there’s your problem right there. The good news is that with the productivity demonstrated by OI they could sweep an additional 12 NM or so (but only on one side) across pretty much the entire arc from around 39.5°S up to 25°S in one campaign similar to the one now concluding.

  726. Sabine Lechtenfeld says:

    @haxi, that’s literally the million dollar question, we’re all asking ourselves.
    If we rule out that the 7th arc has been miscalculated somehow, we have to consider several possibilities:

    – the plane didn’t crash near the 7th arc, as the last set of BFO data seems to suggest, but the plane was recovered from a dive and flown away from the 7th arc out of the searched corridor. This would probably be the worst-case scenario for future searches because the plane could be anywhere within gliding distance of the 7th arc. Most proponents of this scenario assume that the plane was flown after fuel exhaustion in a glide as far southwards as possible. But that’s just an assumption. The plane might as well have been flown northwards again or into any other direction, depending on the pilot’s exact motivation. For all we know, he may have turned towards Mekka in his final moments…

    – the plane took a route before the fmt towards the SIO which upsets the accepted fuel range calculations. There is a school of thought which believes that the detour towards the Andamans and the avoidance of Indonesian airspace didn’t happen. In this case the Lido slide and the final military radar capture have to be ignored.

    – the plane crashed further north near the 7th arc, and the area hasn’t been searched, yet. This would include all CI scenarios. Scenarios which try to explain a crash further north near the 7th arc, have varying degrees of plausibility.

    – Jeff Wise is still championing the idea that the BFOs have been spoofed, and the plane didn’t end up in the Indian Ocean at all, but went northwards instead and landed safely. He accepts the accuracy of the BTOs and the 7th arc, though. I’m not going to discuss the plausibility of this theory, which is low, although I have to confess that I used to be attracted to this idea for a while 😉 But I moved on when more and more debris surfaced and when the existence of the sim data on ZS’s comuter has been confirmed. But in hindsight there are many more problems with the idea of a spoof and a northern route. The total lack of any confirmed radar sightings on a northern over-land route is just one of them.

    – the plane took an accepted route and did crash near the 7th arc, but the underwater search was very unlucky and missed the sea floor wreckage somehow. While not very likely, this possibility cannot be completely dismissed.

    All the above explanation attempts assume that the sat data are reliable, although Wise considers that the BFOs may have been deliberately corrupted. Personally I favor the first and the third explanation.

    And then there are of course those who throw out the sat data completely:
    – The French journalist Florence de Changy believes the plane crashed near IGARI into the SCS, and the sat data and radar evidence are a cover-up attempt. The complete lack of debris sightings or retrievals in the SCS – although the aerial searches were conducted there immediately – makes this theory highly unlikely, even if we accept trumped-up sat and radar data.

    – Others believe that the Kudahuvadhooans, who spotted a large and lowly flying airliner over their tiny Maldive island, did indeed see MH370. This last story, which has been investigated extensively by Blaine Gibson, has always troubled me to no end, because none of the official explanation attempts which try to explain the sighting, hold water upon closer inspection. However, I do not believe that the islanders did really see MH370. I haven’t the foggiest idea what they actually saw. But I sure don’t believe the suggestion, that they mistook a comparatively small propeller plane of their own state airline which sports a very distinctive cavorting-dolphins design and also must’ve been off-route quite a bit, for a MAS 777. But I disgress, lol 🙂

    Have I forgotten anything worth mentioning? I guess, that unless the still continuing search finds the plane after all, or new data or a credible whistle blower will surface, the guessing game will be in full swing for a long, long time….

  727. Sabine Lechtenfeld says:

    Since I mentioned the Kudahuvadhooan sighting of a large, low and very noisy plane over the tiny Maldive island on March 8, 2014, just after local sunrise, I may as well point out a very quirky fact: if the sat com hadn’t made a comeback at 18:25 UTC, we wouldn’t have any sat data. And if we wouldn’t have any sat data, the Kudahuvadhooan sighting would’ve been treated very differently. It would’ve been entirely plausible that MH370 might’ve continued on a route westwards after the turnaround near IGARI, even if the available radar evidence seemed to point more into a northwestern direction. But the Central Indian Ocean would’ve been searched high’n low for plane wreckage. Nobody would’ve looked towards the SIO for a long time. And the so-called Curtin Boom would’ve received a lot more attention as well – although I want to point out that the timing and point of origin of the Curtin Boom and the timing of the plane sighting over Kudahuvadhoo are mutually exclusive: the plane over Kudahuvadhoo cannot have been responsible for the Curtin Boom. Unfortunately the two events have become linked forever in the collective MH370 memory archive, and a lot of people believed (still believe) that the noisy plane over Kudahuvadhoo could’ve caused the Curtin Boom.

    Why do I point out the what-ifs without the sat data? For no other reason than that it is another strange set of quirky circumstances and coincidences in a saga rich in strange and quirky coincidences. What is now a fringe theory would’ve become the most plausible scenario 😉

  728. Sfojimbo says:

    @Sabine
    “There is a school of thought which believes that the detour towards the Andamans and the avoidance of Indonesian airspace didn’t happen. In this case the Lido slide and the final military radar capture have to be ignored.”

    That’s not true. I believe the detour towards the Andamans didn’t happen and you may have “Indonesian airspace” mixed up with “Indonesian FIR” which are two different things.

    The plots I posted last night show that the path southward is viable and nothing changes as far as the Lido slide is concerned. My plot uses the same route as everyone else up to 18:22.

    There is no doubt in my mind that the search is being done several hundred miles too far south.

  729. Victor Iannello says:

    @Sfojimbo: You are splitting hairs about the words “the detour towards the Andamans”. If MH370 flew near VAMPI, MEKAR, and NILAM, that would be towards the Andamans, even if there was a turn to the south after NILAM and before 18:40. We’ve reconstructed many paths in this manner, terminating near 37.5S latitude.

  730. haxi says:

    @Sabine Lechtenfeld

    Thanks for the reply.

    Let’s put Wise’s spoof aside, and acknowledge that the Inmarsat data are the only reliable evidence we have so far.

    One issue I want to discuss is about the compass. How would the in-flight power cycle affect 9M-MRO’s magnetic compass? To continue to function accurately, would the compass need to be calibrated somehow after the break? And how would a non-calibrated compass affect its hours-long (autopiloted? MAG selected?) journey to the Indian Ocean?

  731. TBill says:

    @Airlanseaman
    “Short adventure” sounds good

    @haxi
    I would say the answer is there is more uncertainty in the BFO than the BTO. BTO tells us we need to search Arc7. BFO tells us where and how far away…well sort of. BFO is ambiguous at best, possibly drifting values, and who knows what at the catastrophic end of flight.

    @Victor @all
    Strikes me from the recent discussions we have some new sub-options for the flight. We now have what I call the “sleeper” flight which is a passive flight but at Fuel Exhaustion the pilot wakes up and glides the aircraft 100nm off of Arc7.

    MH370 Flight Options
    (1) Passive-Unintentional (Random overflight-pilot unconscious)
    (2) Passive-Intentional-Unconscious (non-random straight flight)
    (a) One single waypoint or A/P heading
    (b) Multiple waypoints or discontinuity (less studied)
    (3) Passive-Intentional-Conscious (pilot awakes to glide 100nm at end)
    (4) Active-Intentional-Pilot is always making some adjustments and could have glided at the end

  732. Sabine Lechtenfeld says:

    @Sfojimbo, believe it or not, but I didn’t have specifically your scenario in mind when I outlined in broad strokes the idea that the detour towards the Andamans never happened. Most people who subscribe to these scenarios, believe that the plane simply crossed Sumatra instead of scooting around it’s northern tip. Therefore they need to ignore the Lido slide and the last military radar capture.
    I have not attempted to make a comprehensive list of every single scenario which has been proposed here or elsewhere.

  733. airlandseaman says:

    Mick Gilbert:

    Basically, I agree with the point I think you were trying to make about the search width margin, but your specific statement:

    “So, technically, yes, 15 NM ± 5.3 NM is included in the search band but with very little margin.”

    …is misleading, or easily misunderstood. In fact, the opposite is true. I estimate ±22 nm covers ~95% of the descent cases. Searching along the arc from S36 to S25 probably covers ~95% of the path cases as well, so the choice of width to search was well balanced with the length, given the total area budget. 100% would be desirable, of course, but not feasible given the technology used and area budget.

    No discussion about the search width is complete without the recognition that there is a trade-off between width and length for any 7th arc area budget. In the OI search, it was estimated in December that the time and season constrained budget would be ~100,000 km^2 (which has turned out to be about right; amazing really!). They started out with a search width of ~±28 nm in the CSIRO area, to be sure they covered the Griffin hot spots well. After Boeing recalculated the 7th arc, and their new arc matched the IG arc to 70m, there was greater confidence in the accuracy of the basic 7th arc position. That in turn probably led to the decision to narrow the width slightly, from ±28 nm to ±22 nm, in order to re-balance the remaining area budget for the season.

    FTR, below follows my summary estimates for width to search, as presented to the group assembled in London in December. Implicitly, these estimates include all descent cases, including long glide cases.

    •Components to weigh:
    –7th ARC BTO Uncertainty ±5.4 nm (>99%)
    –7th ARC Altitude Uncertainty ±1.5 nm
    –Estimated descent path uncertainty adds
    ±10 nm (>80%?)
    ±15 nm (>90%?)
    ±20 nm (>95%?)
    ±50 nm (>99%?)
    •Recommendation: ±20 nm centered on 15,000 ft ARC

  734. Victor Iannello says:

    @Mick Gilbert, @airlandseaman: When the ATSB said that Simulations that experienced a descent rate consistent with the ranges and timing from the BFO analysis generally impacted the water within 15 NM of the arc, we have to remember that the positions in the simulations were all adjusted so that they share a common point two minutes after fuel exhaustion, which was assumed to be on the 7th arc. However, for the simulations that had high descent rates, those descent rates occurred after the two-minute mark. Since the 7th arc is coincident with the high descent rates, the impact in those simulations would in general be less than 15 NM from the point of high descent rate.

  735. TBill says:

    @airlandseaman
    “Searching along the arc from S36 to S25 probably covers ~95% of cases”

    At the moment, that’s a controversial statement. 95% of what cases? I would 95% of preferred passive flight cases, ruling out passive flights below 36S, which by the way might have the best overall fit to BTO/BFO.

  736. airlandseaman says:

    TBill: So what is your estimate?

  737. Gysbreght says:

    @Victor Iannello: “we have to remember that the positions in the simulations were all adjusted so that they share a common point two minutes after fuel exhaustion, which was assumed to be on the 7th arc.”
    Considering passive end-of-flight, perhaps the time between A/P disconnect and SDU log-on could have been 5 – 6 minutes?

  738. Bruce Robertson says:

    Much is being made of search width margins but I’m reminded of the search for AF447, where Woods Hole very simply searched along the known flight path. It took them only a week or so to find the plane as opposed to years of searching by others.

    In the case of MH370, we don’t have a known flight path but we do have the 7th arc. There should be a linear search along the 7th arc, orthogonal to the past methods of cross-wise scanning. This linear search should run at least as far north as 20 deg South latitude.

  739. Don Thompson says:

    @Haxi

    Concerning the ‘compass’. The aircraft uses its ADIRS to maintain heading reference. ADIRS is the Air Data and Inertial Reference System which comprises the primary Air Data Inertial Reference Unit (ADIRU) and the standby Secondary Attitude Air Data Reference Unit (SAARU).

    The ADIRS is initialised when the aircraft is on the ground, typically, using a GPS reference reported via the FMS. While in flight, the ADIRU maintains heading reference using its internal accelerometers & laser ring gyros. During flight, if GPS is available, it is used to minimise the IRU drift.

    The ADIRS computes heading relative to true north but also computes outputs relative to magnetic north using a periodically updated magnetic variation table.

    There is a standby magnetic compass, for purely visual reference, mounted centrally above the flight deck windshield.

  740. Don Thompson says:

    @Bruce Robertson

    The WHOI deep ocean search for AF447 was not guided by the simple idea of following the intended flight track.

    There was not ‘years’ of searching by others in the quest for AF447. Three separate deep ocean search phases took place over a period of 2 years but, during that time, only 88 days were spent on task, searching the sea floor.

    Both the AF447 LKP and its wreckage location were NW of the established track, in descent the aircraft had deviated off track and turned more than 180º.

    Concerning the deep ocean search for MH370, the Fugro and Phoenix International deep tow vehicles followed tracks parallel to the 7th arc. Seabed Constructor’s AUVs have followed paths parallel and perpendicular to the 7th arc. Why do you expect that linear search paths, along the 7th arc, are preferable to tranverse paths?

  741. TBill says:

    @ALSM
    I am outta ammo, working on new ideas.

  742. haxi says:

    @Don Thompson

    Thank you for your detailed explanation!

  743. ST says:

    @TBill – One of the ways to come up with new ideas seems to be to read articles and information in the immediate time frame of the loss of the flight including prior blogs and posts on this forum. Knowing what we know today four years hence, a fresh perspective to old information/an/analysis and theories could suggest possible options for future search.

  744. @Sabine Lechtenfeld

    You wrote: “ – Others believe that the Kudahuvadhooans, who spotted a large and lowly flying airliner over their tiny Maldive island, did indeed see MH370. This last story, which has been investigated extensively by Blaine Gibson, has always troubled me to no end, because none of the official explanation attempts which try to explain the sighting, hold water upon closer inspection. However, I do not believe that the islanders did really see MH370. I haven’t the foggiest idea what they actually saw. But I sure don’t believe the suggestion, that they mistook a comparatively small propeller plane of their own state airline which sports a very distinctive cavorting-dolphins design and also must’ve been off-route quite a bit, for a MAS 777. But I disgress, lol”

    It occurs to me that if the plane was MH370, and the pilot was doing all he could not to leave a trail, he must have blundered into the island by mistake. This might provide an indication of his mental state.

    However, I’ve spent much time investigating UFO reports, because I believe they are rooted in atmospheric phenomena rather than aliens from the planet Zug.

    UFO reports, like the earlier tales of sometimes deadly fairy folk, and misbehaving gods that they’ve put out of a job, show that almost any amount of credible testimony can be assembled to “prove” the existence of things that just plain couldn’t have happened.

    It seems to be endemic to our culture. One can talk of shared delusions or dream states (which could conceivably have an atmospheric electrical origin!) But it seems human testimony needs the surety of physical evidence.

    The eye witness testimony for Maldive Island sounds compelling, but so do some accounts of aliens climbing out of flying saucers to fix things.

  745. Andrew says:

    @haxi

    Further to Don’s comments, the ADIRU and SAARU each have multiple power sources. The failure of a single bus (eg the left main AC bus that powers the SDU) will not affect their operation.

  746. Mick Gilbert says:

    @airlandseaman
    @Victor Iannello

    Gents, we can spend from now till eternity debating the assumptions that went into advising the search strategy and whether or not the ATSB could construct a sentence that accurately conveys their intended meaning, and in the grand scheme of things it matters not a jot. What happens next is more important.

    There’s been some reporting that OI are keen to go again but that Seabed Constructor has an offshore support contract out of Dampier, Western Australia commencing in a week or so. That being the case and with the southern winter upon us there’s obviously going to be a hiatus before the search resumes. The question is what do OI do when they return? In very simple terms it boils down to a case of go wide or go long but whichever way you slice or dice it, based on your original probability estimates, you’re now into that pesky last 5 percent zone. Again, you can argue till the cows come home whether those probability estimates were right or not, it’s not particularly helpful.

    If you’re comfortable that the assumptions that went into the current plan are valid then I’m guessing that OI would return to a ± 22 NM search width and continue north along the 7th arc. If that’s the case, how far further north do they go?

    Given the known uncertainties associated with the BTO and altitude assumptions combined with the uncertainties of the end-of-flight simulations I suspect that search width is the problem (the next 48 hours may prove me wrong). If OI choose a go wide approach then it comes down to how much wider. Back of the fag packet calculations based on their current effort indicate that they could sweep an additional 12 NM or so across pretty much the entire arc from around 39.5°S up to 25°S in one campaign. The comparative efficiency problem with go wide as opposed to go long is in running on ‘both sides of the street’.

  747. @Sabine Lechtenfeld

    You wrote: “ – Others believe that the Kudahuvadhooans, who spotted a large and lowly flying airliner over their tiny Maldive island, did indeed see MH370. This last story, which has been investigated extensively by Blaine Gibson, has always troubled me to no end, because none of the official explanation attempts which try to explain the sighting, hold water upon closer inspection. However, I do not believe that the islanders did really see MH370. I haven’t the foggiest idea what they actually saw. But I sure don’t believe the suggestion, that they mistook a comparatively small propeller plane of their own state airline which sports a very distinctive cavorting-dolphins design and also must’ve been off-route quite a bit, for a MAS 777. But I disgress, lol”

    It occurs to me that if the plane was MH370, and the pilot was doing all he could not to leave a trail, he must have blundered into the island by mistake. This might provide an indication of his mental state. Alternately, perhaps the aircraft could have been making a drug drop.

    However, I’ve spent much time investigating UFO reports, because I believe they are rooted in electrical atmospheric phenomena rather than aliens from the planet Zug.

    UFO reports, like the earlier tales of sometimes deadly fairy folk and misbehaving gods they’ve put out of a job, show that almost any amount of credible testimony can be assembled to “prove” the existence of things that just plain couldn’t have happened.

    It seems to be endemic to our culture. One can talk of shared delusions or dream states (which could conceivably have an atmospheric electrical origin!) But it seems human testimony needs the surety of physical evidence.

    The eye witness testimony for Maldive Island sounds compelling, but so do some accounts of aliens climbing out of flying saucers to fix things.

  748. Victor Iannello says:

    @Mick Gilbert said: What happens next is more important.

    More important than what? The whole purpose of that thread was to think about where to search next. I thought that was obvious.

  749. Sfojimbo says:

    Why not search where the plane might actually be?
    https://www.dropbox.com/s/269ugzegm6n5kgc/MH370.kmz?dl=0

  750. haxi says:

    @Andrew

    Thanks for the reply. So we can safely assume that the ADIRU was operative after the reboot?

  751. Mick Gilbert says:

    @Victor Iannello

    Victor, you seem to be looking for an argument where there’s none to be had.

  752. David says:

    @Victor. For the reasons I have put the uncertainties about the final BFOs and APU fuel access to me indicate that choosing what to do based on current information would be firing blind.

    It may be that the final report helps clarify these uncertainties but if not (likely)there is another way. There are several months before OI’s planning the details of any next search is needed. Assuming they are willing in principle to search informally, without contract, I reckon they should invite Boeing’s participation in a joint venture, Boeing seeing to some further research in the meantime.

    What would be worth doing is more calculating and wind tunnel work on flight envelope exceedance, assessment of the likelihood and effect of left engine relight and simulations as best these can represent the real thing including zero trim (which may have been the case) The results should embrace low altitude and high, assuming there could have been drift down from around the 6th arc.

    Then another look at APU fuel access after fuel exhaustion at different attitudes and altitudes would be wise as would SDU testing, when integrated with other dependent equipments, to establish the effect of ambient temperature and time unpowered on the whole.

    It would be good for Boeing’s PR to be seen to help solve this enduring puzzle though I suppose that they would have to take steps to avoid it being seen as a precedent.

    Being informed by the outcome of this further work, OI would be better informed as to where to direct search effort and how successful that might be.

  753. Andrew says:

    @haxi

    Yes.

  754. Victor Iannello says:

    @Mick Gilbert: I’m glad you are in agreement. That’s not the impression I had when I read your comment, but now I understand.

  755. Victor Iannello says:

    @David: I agree that more work and/or explanation from Boeing would be helpful.

  756. David says:

    @ Victor. Thanks. I meant to add damage assessment supposing Boeing could be provided access to the recovered parts, and the consequences of just the one (APU) AC generator for other than RAT recipients.

    I daresay there is more than that which could be done to clarify search priorities, which others might like to add.

  757. DennisW says:

    @David

    It would be good for Boeing’s PR to be seen to help solve this enduring puzzle

    Any work done by Boeing would be handed over to the Malaysians. That is equivalent to tossing it in the toilet.

  758. David says:

    @DennisW. I was talking about a joint venture quite independent of the formal investigation, though possibly Malaysia and Australia would provide any assistance sought. The aim would be just to find the wreckage at no, or little, public expense.

    Recovery and assessment of that would be in the ICAO/Malaysia domain still.

  759. DennisW says:

    @David

    Why would anyone enter a joint venture to find the aircraft? As a Boeing stockholder, I would be thoroughly pissed. As far as Boeing PR is concerned, I doubt any of us are going to buy a Boeing product any time soon, and the people who are on their customer list could care less about MH370. The dissappearance had nothing to do with the aircraft.

  760. TBill says:

    @Victor
    @all
    I have a new essay, thoughts on the MH370 BFO trends (up to 00:11):
    On the Straightness of the MH370 BFO Data

  761. David says:

    @DennisW. “Why would anyone enter a joint venture to find the aircraft?”

    Why do you join in here?

    Because its there.

  762. DennisW says:

    @TBill

    The 19:41 BFO value can easily be reconciled with a variety of tracks and ground speeds. It is impossible to infer a path characteristic from the BFO data.

  763. DennisW says:

    @David

    Why do you join in here?

    Only to reinforce my opinion (long held) that an underwater search is basically a dumb idea. An opinion I have voiced here since before the Fugro search even started. There is no compelling reason to search anywhere. They data we have simply does not support the expense of an underwater search. It is simple decision theory (Apparently few here have taken any courses in it.)

    Furthermore, what is the value to be gained from finding the wreckage? The VDR will be blank and the FDR will show the aircraft was flown to where it was found. There is little potential value in finding the wreckage. There is much more potential value in investigations in KL to determine why the aircraft was diverted.

  764. irthe turner says:

    I keep reading the posts with great interest even though a lot of it is very technical (but interesting). So please allow me to ask some dumb questions, if only so I can meet my daily quota of them. Unless Fugro didn’t do a thorough search, is it safe to say we know where M9-MRO is not? It is also safe to say that the Inmarsat data allowed us to know the aircraft continued its course but did not provide explicit information about the aircrafts location, is this correct? Is it possible that the last ping ring data, when the aircraft ran out of fuel, was caused by something else? Such as the APU re-starting and not a rapid descent? So either the PIC loitered, flew south much later and M9-MRO is further north than what has been searched? Or ZS glided the aircraft away from the 7th arc between 38.3s – 33.5s making the search area almost infinite or additional fuel allowed ZS to ditch the aircraft much further south? Am trying to understand what scenario’s remain at this juncture.

  765. irthe turner says:

    Also, I do not believe that anyone shot M9-MRO out of the friendly skies. IMHO, the Malaysians were at first ignorant as to what was going on. Once they figured it out and knew it was a deliberate act, they started to withhold information for their own selfish reasons. Also, there is nothing more unreliable than eyewitness testimony and any sightings over the Maldives should not to be taken seriously. For many natives living in these contrails, an aircraft is an aircraft and any day of the week is the same as the one before and after.

  766. DennisW says:

    @irtheturner

    It is also safe to say that the Inmarsat data allowed us to know the aircraft continued its course but did not provide explicit information about the aircrafts location, is this correct?

    Yes.

  767. Bruce Robertson says:

    @Don Thompson
    “Why do you expect that linear search paths, along the 7th arc, are preferable to transverse paths?”

    The wider the search along the 7th arc, the longer it takes to get to the northern reaches of the 7th arc. My work suggests MH370 is on the Zenith Plateau, 21S 104E, where there is little interest in searching — most want to get back down to 30S.

    @TBill
    I like your suggestion of a curved flight path … it fits with a northern 7th arc terminus. The best information we have indicates a 7th arc impact in the SIO and we’ve searched about 90% of the arc so far. I hope we finish the job and motor on up to at least 20S latitude.

  768. irthe turner says:

    Hello Dennis, Thank you for confirming that. Am trying to better understand what alternatives remain based on the data we know we know is correct.

  769. David says:

    @DennisW. “There is much more potential value in investigations in KL to determine why the aircraft was diverted.”

    That may be but one does not exclude the other.

    You treat this all as a quasi business decision. You do not see it as in any way a social or a general aviation one. Just on that, I would have thought firms like Boeing would have a long term interest in seeing aviation flourish, to be seen as a contributor to the public image of it and also as part of the solution to mysteries in which it has an implied social stake, if not responsibility. I doubt their shareholders would judge their management just by this year’s profits.

    Taking your approach to its limits they should not have found it in their interest to assist the ATSB. I doubt that the NTSB could insist they did.

  770. DennisW says:

    @David

    You treat this all as a quasi business decision. You do not see it as in any way a social or a general aviation one.

    Correct.

  771. TBill says:

    @Bruce Robertson
    Well not so fast.
    My BFO analysis so far suggests an uncanny almost exact fit to 180S or 187S flights, especially 187S flights. Admittedly I would need to add more hypothetical flights to see if there are other close fits to the data.

  772. DennisW says:

    @David

    You treat this all as a quasi business decision. You do not see it as in any way a social or a general aviation one.

    Doing stupid things is never a good idea no matter what the motivation.

  773. David says:

    @DennisW. Addressing just your 1:47 AM I hope your short-term business approach to this is not widely shared.

    In doing or organising research like this there may not be much direct expense. Ironically they would not be contributing directly to the new search you decry (and, based on current information I share your reservations).

    Their contribution would be, as with the past search, helping with a decision as to whether and where any new search should be undertaken, at others'(OI’s in this case) expense.

  774. formula says:

    @DennisW “There is little potential value in finding the wreckage. There is much more potential value in investigations in KL to determine why the aircraft was diverted.”

    Although if from the wreckage it could be shown who occupied the cockpit at the end of flight, that may give a focus and spur to investigations in KL. Whether that could be discerned, and worth the cost and effort to find out, may well be doubtful.

  775. formula says:

    @ irthe turner – your thinking may be assisted as mine certainly has been by reading Brian Anderson’s “The Last 15 Minutes of Flight of MH370” found @ http://www.duncansteel.com/archives/1461

  776. ventus45 says:

    @DennisW
    “Doing stupid things is never a good idea no matter what the motivation.”

    Since you are a “data driven” type, I presume that you agree (in principle) with Mike, Victor etc, that the Arc is good, and that the BFO’s confirm within +/- 25nm, and that the area search so far, SHOULD have contained the aircraft, and it SHOULD have been found.

    Since we have not found the aircraft, and setting aside your judgement that we shouldn’t be looking anyway, if you were forced to advise the fools that want to continue the search, what would you suggest ?

    Purely on “unresponsive” pilot post FMT and “data” grounds:-
    (a) widen the search swath, (i) inside or (ii) outside, or both, and if so, by how far in each case, and why.
    (b) lengthen the search swath, (i) north or (ii) south, and if so, by how far in each case, and why.
    (c) combined (a) and (b)
    (d) stop the search, and intensively review the scanned data, and selectively re-survey selected patches where scan confidence (either from Fugro, Phoenix or Ocean Infinity) is (after close scrutiny) less than stellar.
    OR
    Consider piloted deliberate actions post FMT that fits the data, derive tracks that meet those criteria (we have about a dozen so far) and then expand the search width to maximum ideal glide range, say 110nm, only at the respective intersections of those paths with the arc, with say +/- 20nm left and right of that track line.
    That strategy would produce a series of defined “patches” of 40×110= 4,400 square nm each, which could be selected for scanning as weather windows allowed.

  777. Gysbreght says:

    @DennisW: “Furthermore, what is the value to be gained from finding the wreckage? The VDR will be blank and the FDR will show the aircraft was flown to where it was found. There is little potential value in finding the wreckage. There is much more potential value in investigations in KL to determine why the aircraft was diverted.”

    The FDR will show what happened near IGARI, and around the SDU reboot at 18:25, which may contain clues as to why the aircraft was diverted. There is also a possibility that the CVR will not be blanc.

  778. Niels says:

    @TBill
    I’ve also produced/observed path curvature and lower than typical cruise speeds around the 19:41 point, see fig. 3 in:

    https://www.dropbox.com/s/0vt9unc8y0znwj3/Straight%20path_V0_9.pdf?dl=01

    I’m currently looking into this as well, however I’m not sure about the significance. If I modify the 19:41 BFO by something like 6 Hz, which most people accept is consistent with error bounds, the path straightens and the speed profile flattens.
    On the other hand, the 18:40 BFOs suggest (for level, southerly flight) a similar lower speed as well. This could indicate that MH370 was flying with a GS in the 400 – 420 knots range from before 18:40 until after 19:41. Following this line of thought, typical calculated 00:19 latitudes are in the S35 – S36 range.

  779. Sabine Lechtenfeld says:

    @Neville Macaulife at June 6, 8:40 pm:

    What are you trying to tell me?
    That the Kudahuvadhooans have seen a UFO? Technically speaking that’s certainly the case, because they saw an unidentified flying object, although I’m pretty sure that the object was most likely a plane from our home planet, and alien sightings haven’t been reported 😉
    To make it perfectly clear again: I didn’t mention the Maldive sighting because I believe that the Kudahuvadhooans really saw MH370. I don’t believe that for many reasons. But it’s one of the stranger twists of the MH370 saga, that without the sat data the sighting wouldn’t have been considered a fringe theory but a solid lead.
    Personally I’m troubled by the fact that none of the offered explanations for the sighting hold water upon closer inspection. And to this day no one has ever come forward – neither pilot nor passengers – and has confirmed “yes, we can clear this up; this was us on that morning over Kudahuvadhoo”. Which is very strange considering how much media coverage the incedent has received. But that’s just me. I hate loose ends and unsolved riddles, which explains my interest in our unfortunately still elusive Malaysian plane. But clearing up the Maldive sighting won’t help us to find it.

  780. DennisW says:

    @Ventus

    Yes, the last arc is very good. However, distance from the arc is not known, and latitude on the arc is not known with a precision that makes an underwater search practical.

    We all have various “reasons” for chosing a search distance from the arc and a latitude. I am sure that we will hear most of these reasons in the days ahead.

  781. TBill says:

    @Niels
    Thank you for the like-minded paper. Let me know what you find out.

    Of course, I have historically been an active-pilot advocate. So I will be looking to see how well dogleg paths fit. There is quite a lot of spare time after 22:41 and before 00:11 without any Arc data to allow for a turn towards the East. But I looks like starting out 187S is one solution.

    @Bruce Robertson
    I am also curious if there is a low and slow solution like you propose. Have you worked up a flight path with BTO/BFO?

    I looked at it briefly and saw a curved path to Zenith area might work if all of the BFO’s are off by 10, which it is possible the SDU re-start caused an offset.

  782. TBill says:

    @DennisW
    “The 19:41 BFO value can easily be reconciled with a variety of tracks and ground speeds. It is impossible to infer a path characteristic from the BFO data.”

    True re: Item-1 reconciliation. Some paths take freedom to add a 300 ft/min ascent at 19:41 to help match the 111 BFO. For example, FFYap’s IG path spreadsheet does this.

    I am not sure about the Item-2 “impossible” comment. There does seem to be a possible geographic dependence to the BFO. The Inmarsat data seems to show that, and the hypothetical straight flights south seem to show that too. Of course, the inherent assumption I am making is that the reported BFO is very accurate, so we can study it. Take away that assumption, which is the general assumption that these is some scatter in the data, and you have to toss my essay out. I am fundamentally a data centric not model ccentric person.

  783. ventus45 says:

    @TBill

    Re dogleg path fits.

    The unresponsive pilot(s) straight flight to flame-out crowd will deny it, but this concept has been an obvious necessity for any deliberate piloted flight path, and is central to my path, from way back. Think 23:40 for TOD and turn.

  784. DennisW says:

    @TBill

    I am not intending to be critical of your efforts. I have commented many times on the statistical properties of the BFO values. I am going to “give it rest”.

  785. airlandseaman says:

    TBill: Re: “There does seem to be a possible geographic dependence to the BFO. ”

    From the first time I read about this notion, I have wondered what could possibly explain such a dependence. My conclusion was and still is that there is absolutely no basis in OCXO physics to explain a geographic dependence. To the extent that there appears to be some relationship, I suspect it is coincidental with some other driver, not geographical.

  786. airlandseaman says:

    TBill: One possibility is that the AES/SDU Doppler comp algorithm (which uses lat/lon as an input) has some small residual error resulting from simplified math or lookup tables used. Just a WAG.

  787. DennisW says:

    @Sabine

    An excellent source of information on the reliability of eye witnesses is the “Innocence Project”. Close to 400 people have been exonerated. Of these more than 70% were convicted based on eye witness testimony. More than 150 “real” perpetrators have been found after exonerations.

    Like you, I am curious about the Maldives sighting. Seems strange that given the publicity and the number of people involved that it remains in the mystery category. Could simply be a case of “group hysteria” where one or two people had an infectious influence on others.

  788. TBill says:

    @ALSM
    Re: Geographic BFO dependence
    The question I had: is it just the calculated BFO shows this? (in which case we would have to question the BFO spreadsheets we are using…and I do see some calc BFO differences between DrB’s xls and FFYap’s xls. But the real MH370 reported BFO’s seem to show the same behavior.

    In FFYap’s xls I think I see a spot, around 19:51, where the calc BFO makes a +1.5 step change over the course of 1-sec to come closer to DrB’s. Maybe I should check DrB’s spreadsheet for step changes (before 22:41) to explain the “geographic” boost up to 204 BFO.

    Right now I use DrB’s BFO calculator because I noticed it was closer to the numbers that Victor has always reported. Where I notice the biggest difference is FFYap’s 19:41 Arc2 BFO is bit low.

  789. airlandseaman says:

    TBill: Keep in mind that we are discussing the frequency of a 1,600,000,000 Hz carrier. The GES/CU recording resolution is only 1 Hz, and the carrier tracking loop noise can easily result in another 1-2 Hz “noise on the value recorded. Therefore, we should not try to infer much from a single +1.5 step change. I would say that is normal and expected.

  790. Richard Godfrey says:

    SC is continuing to make good progress up the Broken Ridge plateau area and has reached 24.7578°S.

    Ocean Infinity has made another 2 AUV deployments and has collected all the other AUVs in the last 48 hours.

    I expect the search to complete early tomorrow, when the last AUV is collected and the data checked.

    The weather is fine, there is good visibility, with a 15 knot wind, a combined swell and wave height of 3.3m. There are no tropical storms in the region.

    https://www.dropbox.com/s/9j8pwv98k2870a8/SC%20Track%2007062018.pdf?dl=0

  791. Richard Godfrey says:

    The current search by Ocean Infinity has been completed with the remaining AUVs having been collected early.

  792. Bruce Robertson says:

    @TBill
    “I am also curious if there is a low and slow solution like you propose. Have you worked up a flight path with BTO/BFO?”

    Only conceptually with a few rough calculations to check reasonableness. From the 19:41 calculation (done by many) of about 400 knots, the compromised plane continued south and southeast in a large- but diminishing-radius arc due to airspeed decaying as fuel is burned. The curved path makes the math much more difficult than the usual straight path at constant airspeed. Still, I think it could be done with an assumption of a somewhat regular change in speed (linear? logarithmic)?.

  793. Richard Godfrey says:

    Seabed Constructor is heading for N.W. Australia, directly to the next job off Dampier.

  794. DennisW says:

    @Richard

    Thx for the updates. Now for the post mortems.

  795. airlandseaman says:

    A complete 2018 S-AIS track record and associated .kmz files are available here:
    https://goo.gl/bsN8GU

  796. Barry Carlson says:

    @airlandseaman;

    Thanks for the S-AIS files.

    ——

    “One possibility is that the AES/SDU Doppler comp algorithm (which uses lat/lon as an input) has some small residual error resulting from simplified math or lookup tables used. Just a WAG.”

    Could be that the AES/SDU Doppler compensation algorithm is using a simplistic equal semi-major/minor axis when using the WGS84 lat/lon. That would show up on north/south flight as the AES/SAT distance would become less than the true distance when using the WGS84 ellipsoid lat/lon. A bit of a wild guess!

  797. Kenyon says:

    Richard Godfrey, thank you for your efforts in monitoring, analyzing, and reporting on Ocean Infinity progress throughout their search effort.

  798. airlandseaman says:

    Thanks to all those who helped us understand the SC movements, AUV tracks and ROV missions, especially Richard Godfrey, Don Thompson, Kevein Rupp and of course Richard Cole, whose insightful animations were so educational. And special thanks to Victor Iannello, who made this excellent forum available to all interested in finding MH370.

    The search for MH370 is a work in progress. ATSB, DSTG, CSIRO, UWA, Fugro, Ocean Infinity, Blaine Gibson, the IG, and many others have contributed to our collective knowledge base. The more we learn, the better the chances of finding MH370 eventually. In this context, OI’s search was highly successful. The 2018 search did not succeed in finding MH370, but the information gained and technology demonstrated moved the effort much closer to eventual success.

  799. TBill says:

    It was a very exciting search by OI. First they knocked off Victor’s 180S pin, and before I had a chance to study it, they were in the volcanic escarpment Broken Ridge area. Then they knocked off the birthday waypoint, DrB’s new pin, Richard’s 30S spot and then Victor/Richard’s NZPG pins, newer LRC mode then old 26.9 spot, and then they just kept going.

    Right now if OI stops back for a few weeks on their way to the sub, I would ask they continue up to 20S.

  800. DennisW says:

    @TBill

    Right now if OI stops back for a few weeks on their way to the sub, I would ask they continue up to 20S.

    I just don’t know.

    The IG is populated with a lot of smart people, and their original analytics are very well done.

    Likewise, the DSTG work was credible, reinforced the IG work (38S preferred), and had a 99% probability ending at 33S.

    Richard’s drift work was excellent, and I was very fond of 30S.

    Stone et. al. estimated the probability of detection, POD, of the sonar search to be 90%, and the terrain associated with AF447 was not as convoluted as the SIO region along the 7th arc.

    At this point one could argue that the probability of a missed detection is higher than the probabilty of finding the wreckage further North.

    Anyway, I would like to hear what others think. I am not ready to endorse any approach.

  801. DennisW says:

    @all

    BTW, I used “further” above. Ami would be all over my ass for that. When physical distances are involved “farther” is used. Further refers to vague stuff like “further your career”.

  802. sk999 says:

    First, I join with ALSM in thanking all those who provided tracking reports of SC and its AUVs and such. You had my attention.

    Second, DennisW opines, “At this point one could argue that the probability of a missed detection is higher than the probabilty of finding the wreckage farther North” and then askes what others think.

    That is my preferred way forward as well, but just saying so does not carry any weight unless you can back it up. I have been working more on a “smoothness” metric as applied to BTOs and BFOs to see if it can be used to rule out (i.e., assign low probability) to the “farther North” routes. It is somewhat similar to what TBill and Niels have posted here recently, but different in detail. More to follow.

  803. Shadynuk says:

    In response to @DennisW’s remarks:

    What I had been hoping for is some ‘convergence’ of the problem. Yes, the areas searched now carry a very much lower probability than the areas not searched. However, with all those ‘hotspots’ searched without result, has this strengthened or weakened any of the assumptions that were used to define those spots? The set of factual data has not changed much recently and unless new facts come to light, it is the assumptions that will govern a selection of a next best area.

    It seems to me that all the areas ‘near’ the 7th arc and within fuel range carry equal probability unless the assumptions can somehow be prioritized. And I still believe that perhaps the most critical assumptions are those related to motivation or intent. What was he trying to do?

    It seems the wreckage is not in the ‘middle’. There is a significant difference between going farther North or farther South. If he was trying to ‘hide’ the plane, go farther South. If he was he was trying to leverage something, and the crash was the result of a ‘failed plan B’, go farther North.

    Beyond that, pick an area and search until the money runs out.

  804. DrB says:

    @All,

    I have been thinking about the possibility that the 00:19 BFOs might be seriously in error, primarily for two reasons:

    1. The aircraft had not been found near the 7th Arc.
    2. The SDU start-up event then is unique.

    Suppose, for instance, that the 00:19 BFOs were roughly 100Hz (or more) too low. That would drastically reduce the RODs inferred then and be more consistent with an extended glide.

    Recall that the first 18:25 BFO was apparently more than 100 Hz too high after the SDU was unpowered in flight for about an hour. Also recall that a different SDU was tested at ground ambient temperature after being unpowered for varying durations. However, no testing has been done on a SDU in the cruise environment, and there are no other examples of in-flight restarts besides the MH370 pair at 18:25 and at 00:19.

    The 00:19 SDU restart is different from the 18:25 event in two ways: the unpowered time was much shorter (maybe only 2 minutes) and the power source was different (the APU).

    So far, I don’t have a plausible theory that might explain how the BFOs could be roughly 100 Hz low. Assuming the same warm-up transient curve as seen in the other restarts, that could only happen if the 00:19 Log-On Request occurred about a minute earlier in the warm-up curve than the 18:25 Log-On Request did. In other words, the 00:19 LOR would have to occur at a time when the OCXO temperature was rising but still less than the set point temperature. The 18:25 LOR occurred near the peak overshoot in OCXO temperature, at a temperature above the set point, leading to a frequency that was too high by ~130 Hz.

    Is it possible for the LOR transmission to occur after a much shorter delay from the power-up/reboot in the circumstances existing at 00:19? Per DonT there are several conditions that must be met for the first transmission to occur. One of these is to have an acceptably small OCXO temperature error. I am just wondering if the other conditions were met at 00:19 when the OCXO temperature error was negative, but at 18:25 they were met when the temperature error was positive. That would cause different signs in the frequency errors, and this would raise the true 00:19 BFO values and reduce the calculated rates of descent.

  805. airlandseaman says:

    Bobby: As you know, all the OCXO electronics operate from regulated DC. Thus, a change in 115 VAC source could not cause a change in frequency, even if the 115 volt source was 5-10% different.

  806. haxi says:

    @TBill

    OI failed to win the contract to search for the ARA San Juan, as the Spanish company Igeotest Geoscience won the deal.

    Looking for any possibility that OI might be back for MH370 search later this year.

  807. DrB says:

    @airlandseaman,

    You missed my point, Mike. My question had to do with the logical tests done by the SDU after a restart when both IDGs were down and the APU was providing power to the SDU. I was not suggesting any change in OCXO warm-up, just the possibility that the length of time required to complete the other health and status checks (besides the one dealing with the OCXO temperature error) might differ.

  808. Sabine Lechtenfeld says:

    @Dennis, as a criminal psychologist I’m very familiar with the subject of reliability of eyewitness reports. I will check out your hint. Will make an interesting read.
    I’m always weary of the “mass hysteria” explanation, although there are indeed some incedents which are hard to explain any other way. But the parameters don’t fit the case of the Maldive sighting since the elements of fear, mystery, importance or an emotionally charged situation are absent. The witnesses just saw an annoyingly noisy lowly flying large plane which executed a pronounced turn southwards, and it was a totally unfamiliar kind of plane – which makes it highly unlikely that the islanders saw a propeller plane of their own state airline. And if I remember it correctly, the noisy plane has been reported initially even before it became widely known that MH370 had turned around and re-crossed the Malaysian peninsula. That detail is important of course. At the time of the observation this incident wasn’t emotionally charged but just annoying.
    It troubles me that the Maldivian authorities have come up with elaborate but upon closer inspection totally unacceptable explanations. They even invented a flight which never happened. Why? I don’t recall the exact details but Blaine Gibson has done a fairly thorough investigation which shows that the official explanation is nonsense and has been retracted eventually. Why didn’t the Maldivian authorities just say:”we haven’t the foggiest idea what our dear Kudahuvadhooan brothers and sisters saw” and leave it at that?
    I am less convinced by Blaine Gibson’s assertion that many eyewitnesses didn’t hesitate to identify the plane as a MAS 777. While some witnesses may firmly believe that now, all the precise identifications were done in hindsight, and their memory may be corrupted by the media coverage of the MH370 case. The initial descriptions of the mystery plane were much more vague, and a lot of airlines use a color scheme which is very similar to MAS’s design.

    While the sighting is a somewhat baffling riddle, I’m also fascinated by another curious wide spread phenomenon: the irrational but steadfast believe of many MH370 armchair experts (like us 😉 ) that the Maldive plane’s hypothetical crash soon after the sighting may have caused the Curtin Boom, and that the two events form a strong hint towards the CIO. But the timing of the two events tells us that this is not possible. I have argued this point endlessly with Brock McEwen and others. He eventually conceded that I am right but he continues to casually link the two events to this day. Even journalists who have covered both events never bothered to do the exact math and were content that the two events were timed reasonably close – which tells us a thing or two about the current state of investigative journalism.

    Ok, I will leave the subject alone now. It’s highly unlikely that we will ever find out what the Kudahuvadhooans have seen. But I’m fairly sure that the sighting and the Curtin Boom will be revisited by many, since OI has failed to find the plane in the SIO.

  809. Sabine Lechtenfeld says:

    @Shadynuk, at June 8, 12:23 am:
    I agree with you that the areas along the 7th arc aren’t all equally probably if we fact or in motivation and intent of the pilot.
    If he always intended to crashand to hide the plane, the regions within fuel range on the southern end of the 7th arc are more likely. Since these areas have all been searched, end-of-flight maneuvers like an extended glide need to be considered. If the crash was a result of a Plan B – perhaps after failed negotiations – then crash zones further north are far more plausible. In this case an extended glide and an attempt to hide the plane, makes not a lot of sense. If ZS planned to negotiate – or did negotiate – and threatened to crash the plane, it would be perfectly clear that he was the perp.

  810. airlandseaman says:

    DrB: You are right. I missed that distinction…timing vs. AC voltage. But you missed my point as well. The OCXO warm-up timing should not be effected by a change in the 115VAC source…APU vs. ME generator. However, the cabin temp could have been different at 00:19 vs. 18:25, and that could have an effect on the AES warm-up timing.

  811. Victor Iannello says:

    [Comments here are closed. Please continue the discussion under the new article.]